Вы находитесь на странице: 1из 561

BY DR.

MOHAMMED ATIAA KAREEM ALNASHY


-------------------------------------------------------------------

1
BY DR. MOHAMMED ATIAA KAREEM ALNASHY
-------------------------------------------------------------------

CUMMINGS
OTOLARYNGOLOGY
HEAD & NECK SURGERY
FOURTH EDITION
REVIEW

2
BY DR. MOHAMMED ATIAA KAREEM ALNASHY
-------------------------------------------------------------------
CUMMINGS OTOLARYNGOLOGY—HEAD & NECK SURGERY
FOURTH EDITION REVIEW

VOLUME ONE
Part One: General Considerations in Head and Neck
Charles W. Cummings, Editor
K. Thomas Robbins, Associate Editor Part Two: Face
David E. Schuller, Editor
J. Regan Thomas, Associate Editor

VOLUME TWO
Part Three: Nose
David E. Schuller, Editor
J. Regan Thomas, Associate Editor Part Four: Paranasal Sinuses
David E. Schuller, Editor
J. Regan Thomas, Associate Editor Part Five: Salivary Glands
Bruce H. Haughey, Editor Part Six: Oral Cavity/Pharynx/Esophagus
Bruce H. Haughey, Editor

VOLUME THREE
Part Seven: Larynx/Trachea/Bronchus
Paul W. Flint, Editor Part Eight: Neck
K. Thomas Robbins, Editor Part Nine: Thyroid/Parathyroid
K. Thomas Robbins, Editor

VOLUME FOUR
Part Ten: General
Lee A. Harker, Editor Part Eleven: Infectious Processes
Lee A. Harker, Editor Part Twelve: Vestibular System
Lee A. Harker, Editor Part Thirteen: Facial Nerve
Lee A. Harker, Editor Part Fourteen: Auditory System
Lee A. Harker, Editor Part Fifteen: Cochlear Implants
Lee A. Harker, Editor Part Sixteen: Skull Base
Lee A. Harker, Editor Part Seventeen: Pediatric Otolaryngology

Mark A. Richardson, Editor

3
BY DR. MOHAMMED ATIAA KAREEM ALNASHY
-------------------------------------------------------------------

-----------------------------------------------
CUMMINGS
OTOLARYNGOLOGY
HEAD & NECK SURGERY
FOURTH EDITION REVIEW
------------------------------------------------
Charles W. Cummings, M.D.
Distingushed Service Professor
Department of Otolaryngology—Head and
Neck Surgery
Johns Hopkins University School of
Medicine
Baltimore, Maryland

Paul W. Flint, M.D. Lee A. Harker, M.D.


Professor Deputy Director
Department of Otolaryngology—Head and Neck Surgery Boys Town National Research Hospital
Director, Center for Airway, Laryngeal, and Voice Disorders Vice Chairman
Co-Director Minimally Invasive Surgical Training Center Department of Otolaryngology and
Human
Johns Hopkins University School of Medicine Communication
Baltimore, Maryland Creighton University School of
Medicine
Omaha, Nebraska

Bruce H. Haughey, MBChB, FACS, FRACS Mark A. Richardson, M.D.


Professor and Director Professor and Chairman
Head and Neck Surgical Oncology Department of Otolaryngology—Head and Neck
Surgery
Department of Otolaryngology—Head and Neck Surgery Oregon Health and Science University
Washington University School of Medicine Portland, Oregon
St. Louis, Missouri
K. Thomas Robbins, M.D. David E. Schuller, M.D.
Professor and Chair Professor and Chairman
Division of Otolaryngology, Department of Surgery Department of Otolaryngology—Head
Southern Illinois University School of Medicine and Neck Surgery
Springfield, Illinois Executive Director, Arthur G. James Cancer
Hospital
and Richard J. Solove Research Institute
J. Regan Thomas, M.D. Deputy Director, Comprehensive Cancer Center
Francis L. Lederer Professor and Chairman The Ohio State University
Department of Otolaryngology—Head and Neck Surgery Columbus, Ohio
University of Illinois
Chicago, Illinois

ELSEVIER
MOSBY

4
BY DR. MOHAMMED ATIAA KAREEM ALNASHY
-------------------------------------------------------------------
ELSEVIER
MOSBY

The Curtis Center


170 S Independence Mall W 300E
Philadelphia, Pennsylvania 19106

Cummings Otolaryngology—Head & Neck Surgery Fourth


Edition Review Copyright © 2005, Elsevier Inc. All rights
reserved.
No part of this publication may be reproduced or transmitted in any form or by any means,
electronic or mechanical, including photocopying, recording, or any information storage and
retrieval system, without permission in writing from the publisher. Permissions may be sought
directly from Elsevier's Health Sciences Rights Department in Philadelphia, PA, USA: phone:
(+1) 215 238 7869, fax: (+1) 215 238 2239, e-mail: healthpermissions@elsevier.com. You may
also complete your request on-line via the Elsevier homepage (http://www.elsevier.com), by
selecting 'Customer Support' and then 'Obtaining Permissions.'

NOTICE

Otolaryngology is an ever-changing field. Standard safety precautions must be followed, but as new
research and clinical experience broaden our knowledge, changes in treatment and drug therapy may
become necessary or appropriate. Readers are advised to check the most current product information
provided by the manufacturer of each drug to be administered to verify the recommended dose, the
method and duration of administration, and contraindications. It is the responsibility of the licensed
prescriber, relying on experience and knowledge of the patient, to determine dosages and the best
treatment for each individual patient. Neither the publisher nor the author assumes any liability for any
injury and/or damage to persons or property arising from this publication.

Previous edition copyrighted 1998.

International Standard Book Number 0-323-03006-8

Acquisitions Editor: Rebecca Schmidt Gaertner


Developmental Editor: Mary Beth Murphy
Editorial Assistant: Suzanne Flint

Printed in the United States of America

Last digit is the print number: 987654321

5
BY DR. MOHAMMED ATIAA KAREEM ALNASHY
-------------------------------------------------------------------
Table of Contents

-------------------------

Volume One

GENERAL CONSIDERATIONS IN HEAD AND NECK

Cummings/Robbins

1. History, Physical Examination, and the Preoperative Evaluation 3


Marion Everett Couch, James E. Blaugrund, Dario Kunar
2. Overview of Diagnostic Imaging of the Head and Neck 4
Nafi Ay gun, Patrick J. Oliverio, S. James Zinreich Questions prepared by:
Matthew Whitley
3. Biophysiology and Clinical Considerations in Radiotherapy 5
Jason K. Rockhill, George E. Laramore
4. Chemotherapy for Head and Neck Cancer 6
Arlene A. Forastiere, Merrill S. Kies Questions prepared by: Matthew
Whitley
5. Skin Flap Physiology and Wound Healing 7
George S. Goding, Jr., Patrick J. Byrne
6. Free Tissue Transfer 8
Douglas A. Girod, Terrance T. Tsue
7. Laser Surgery: Basic Principles and Safety Considerations 9
Robert H. Ossqff, C. Gaelyn Garrett, Lou Reinisch
8. General Considerations of Anesthesia and Management of the
Difficult Airway 10
Lynette J. Mark, Seth Akst, James Michelson
9. Allergy and Immunology of the Upper Airway 12
Fuad M. Baroody, Robert M. Naclerio

6
BY DR. MOHAMMED ATIAA KAREEM ALNASHY
-------------------------------------------------------------------
10. Head and Neck Manifestations of Human Immunodeficiency Virus
Infection 13
Steven D. Fletcher, Andrew N. Goldberg
11. Special Considerations in Managing Geriatric Patients 14
Matthew L. Kashima, W. Jarrard Goodwill, Jr., Thomas J. Balkany, Roy R.
Casiano
12. Genetics and Otolaryngology 15
William J. Kimberling Questions prepared by: lee Ching Anderson
13. Fundamentals of Molecular Biology and Gene Therapy 16
Bert W. O'Malley, Jr., Daqing Li, Hinrich Staecker
14. Molecular Biology of Head and Neck Cancer 17
Patrick K. Ha, David Goldenberg, Matthew Wolpoe, Joseph A. Califano
III
15. Outcomes Research 18
Bevan Yueh
16. Interpreting Medical Data 19
Richard M. Rosen/eld
17. Pain Management in the Head and Neck Patient 20
Peter S. Staats Questions prepared by: Sara Pai
18. Integrating Palliative and Curative Care Strategies in the Practice of
Otolaryngology 21
Michael A. Williams, Cynda Hylton Rushton

19. Graphics and Digital Imaging for Otolaryngologists 22


Lawrence R. Lustig, Nikolas H. Blevins REVIEW QUESTIONS NOT
REQUIRED FOR THIS TOPIC
20. Medical Informatics and Telemedicine 23
David Goldenberg, Marion Everett Couch

7
BY DR. MOHAMMED ATIAA KAREEM ALNASHY
-------------------------------------------------------------------
FACE

Schuller/Thomas

21. Aesthetic Facial Analysis 27


Marc S. Zimbler, Jongwook Ham
22. Recognition and Treatment of Skin Lesions 28
Ken K. Lee, Khosrow (Mark) Mehrany, Neil A. Swanson Questions
prepared by: Seth Cohen
23. Management of Head and Neck Melanoma 29
Cecelia E. Schmalbach, Timothy M. Johnson, Carol R. Bradford
24. Scar Revision and Camouflage 30
J. Regan Thomas, Steven Ross Mobley
25. Facial Trauma: Soft-Tissue Lacerations and Burns 31
Kevin A. Shumrick, Jon B. Chadwell
26. Maxillofacial Trauma 32
Robert M. Kellman Questions prepared by: Seth Cohen
27. Reconstruction of Facial Defects 33
Shan R. Baker Questions prepared by: Matthew Whitley

28. Hair Restoration: Medical and Surgical Techniques 34


Benjamin M. Loos, Sheldon S. Kabaker
29. Management of Aging Skin 36
Stephen W. Perkins, Timothy G. Gillum
30. Rhytidectomy 37
Shan R. Baker
31. Management of the Aging Brow and Forehead 38
Paul S. Nassif, J. Regan Thomas

8
BY DR. MOHAMMED ATIAA KAREEM ALNASHY
-------------------------------------------------------------------
32. Management of the Aging Periorbital Area 39
Oren Friedman, Tom D. Wang, Ted A. Cook Questions prepared by: Lisa
Earnest
33. Suction-Assisted Lipocontouring 40
Edward H. Farrior, Stephen S. Park
34. Mentoplasty and Facial Implants 41
Jonathan M. Sykes, Travis T. Tollefson, John L. Frodel, Jr. Questions
prepared by: Seth Cohen
35. Rehabilitation of Facial Paralysis 42
Roger L. Crumley, William B. Armstrong, Patrick J. Byrne
36. Otoplasty 43
Peter A. Adamson, Suzanne K. Doud Galli

Volume Two

NOSE

Schuller/Thomas

37. Physiology of Olfaction 47


Donald A. Leopold, Eric H. Holbrook
38. Evaluation of Nasal Breathing Function with Objective Airway
Testing 48
Jacob W. Zeiders, John F. Pallanch, Thomas V. McCaffrey
39. Manifestations of Systemic Diseases of the Nose 49
Thomas J. McDonald
40. Epistaxis 50
Evan J. Tobin, Douglas D. Massick Questions prepared by: Seth Cohen

9
BY DR. MOHAMMED ATIAA KAREEM ALNASHY
-------------------------------------------------------------------
41. Nasal Fractures 51
Burke E. Chegar, Sherard A. Tatum HI
42. Allergic Rhinitis 52
Richard L. Mabry, Bradley F. Marple
43. Nonailergic Rhinitis 53
Stephanie Joe, Aaron Benson Questions prepared by: Seth Cohen
44. The Nasal Septum 54
Russell W. H. Kridel, Paul E. Kelly, Allison R. MacGregor
45. Rhinoplasty 55
M. Eugene Tardy, Jr., J. Regan Thomas
46. Special Rhinoplasty Techniques 56
Richard T. Farrior, Edward H. Farrior, Raymond D. Cook

47. Revision Rliinoplasty 57


David W. Kim, Manuel A. Lopez, Dean M. Toriumi
48. Reconstructive Rhinoplasty 58
C. Rose Rabinov, Roger L. Crumley

PARANASAL SINUSES

Schuller/Thomas

49. Radiology of the Nasal Cavity and Paranasal Sinuses 61


Patrick J. Oliverio, S. James Zinreich Questions prepared by: Joshua S.
Schindler
50. Infectious Causes of Rhinosinusitis 62
Jonas T. Johnson, Berrylin J. Ferguson
51. Neoplasms 63
Ernest A. Weymuller, Jr., Thomas J. Gal

10
BY DR. MOHAMMED ATIAA KAREEM ALNASHY
-------------------------------------------------------------------
52. Medical Management of Nasosinus Infectious and Inflammatory
Disease 64
Scott C. Manning Questions prepared by: Seth Cohen
53. Primary Sinus Surgery 65
Kevin J. Hulett, James A. Stankiewicz
54. Revision Endoscopic Sinus Surgery 66
David W. Kennedy, James N. Palmer
55. Cerebrospinal Fluid (CSF) Rhinorrhea 67
Martin J. Citardi

SALIVARY GLANDS

Haughey

56. Physiology of the Salivary Glands 71


Ravindhra G. Elluru, Manoj Kumar
57. Diagnostic Imaging and Fine-Needle Aspiration of the Salivary Glands
72
Dale H. Rice
58. Inflammatory Disorders of the Salivary Glands 73
Agustin J. Arrieta, Thomas V. McCaffrey
59. Trauma of the Salivary Glands 74 Jeffrey R. Holler
Questions prepared by: Seth Cohen
60. Benign Neoplasms of the Salivary Glands 75
Ehab Y. Hanna, Stephen Lee, Chun Y. Fan, James Y. Suen Questions
prepared by: Joshua Schindler
61. Malignant Neoplasms of the Salivary Glands 76
Alfred Simental, Ricardo L. Carrau

11
BY DR. MOHAMMED ATIAA KAREEM ALNASHY
-------------------------------------------------------------------
ORAL CAVITY/PHARYNX/ESOPHAGUS

Haughey

62. Physiology of the Oral Cavity 79


Joseph B. Travers, Susan P. Travers
63. Mechanisms of Normal and Abnormal Swallowing 80
JerilynA. Logemann
64. Oral Mucosal Lesions 81
James J. Sciubba
65. Oral Manifestations of Systemic Disease 82
Jonathan A. Ship, Elisa M. Ghezzi
66. Odontogenesis and Odontogenic Cysts and Tumors 83
Peter E. Larsen
67. Odontogenic Infections 84
Eric R. Carlson, J. W. Hudson
68. Temporomandibular Joint Disorders 85
Daniel M. Laskin
69. Benign Tumors and Tumor-Like Lesions of the Oral Cavity 86
Timothy S. Lian Questions prepared by: Joshua S. Schindler
70. Malignant Neoplasms of the Oral Cavity 87
Richard O. Wein, Randal S. Weber
71. Reconstruction of the Mandible and Maxilla 88
Mark L. Urken, Daniel Buchbinder, Eric M. Genden Questions prepared
by: Joshua S. Schindler
72. Maxillofacial Prosthetics for Head and Neck Defects 89
Jeffery C. Markt, Thomas J. Salinas, William Donald Gay
73. Benign and Malignant Tumors of the Nasopharynx 90
Ranjiv Sivanandan, Willard E. Fee, Jr.

12
BY DR. MOHAMMED ATIAA KAREEM ALNASHY
-------------------------------------------------------------------
74. Pharyngitis in Adults 91
Brian Nussenbaum, Carol R. Bradford
75. Sleep Apnea and Sleep-Disordered Breathing 92
Robert J. Troell, David J. Terris
76. Oropharyngeal Malignancy 93
Holger G. Gassner, Alain N. Sabri, Kerry D. Olsen Questions prepared
by: lee Ching Anderson
77. Reconstruction of the Oropharynx 94
Bruce H. Haughey, S. Mark Taylor
78. Diagnostic Imaging of the Pharynx and Esophagus 95
Barton F. Branstetter IV
79. Endoscopy of the Pharynx and Esophagus 96
Ravindhra G. Elluru, J. Paul Willging
80. The Esophagus: Anatomy, Physiology, and Diseases 97
Jason F. Vollweiler, Michael F. Vaezi Questions prepared by: Lisa
Earnest
81. Zenker's Diverticulum 98
Christopher Y. Chang, Richard L. Scher
82. Neoplasms of the Hypopharynx and Cervical Esophagus 99
Ravindra Uppaluri, John B. Sunwoo Questions prepared by: Joshua S.
Schindler
83. Radiotherapy and Chemotherapy of Squamous Cell Carcinomas of
the Hypopharynx and Esophagus 100
Jean-Louis Lefebvre, Antoine Adenis
84. Reconstruction of Hypopharynx and Esophagus 102
Kristi E. Chang, Eric M. Genden, Gerry F. Funk Questions prepared by:
Joshua S. Schindler

13
BY DR. MOHAMMED ATIAA KAREEM ALNASHY
-------------------------------------------------------------------
Volume Three

LARYNX/TRAGHEA/BRONGHUS

Flint

85. A. Laryngeal and Pharyngeal Function 105


Gayle Ellen Woodson
B. Evaluation and Management of Hyperfunctional Disorders 106
Andrew Blitzer Questions prepared by: Joshua S. Schindler
86. Visual Documentation of the Larynx 107
Randall L. Plant, Robin A. Samlan
87. Voice Analysis 108
Robin A. Samlan
88. Diagnostic Imaging of the Larynx 109
Franz J. Wippold I
89. INeurologic Evaluation of the Larynx and the Pharynx 110
Gayle Ellen Woodson, Andrew Blitzer Questions prepared by: Joshua S.
Schindler
90. Laryngeal and Tracheal Manifestations of Systemic Disease 111
Kim Richard Jones
91. Chronic Aspiration 112
Steven D. Fletcher, David W. Eisele
92. Laryngeal and Esophageal Trauma 113
Steven D. Schaefer

93. Surgical Management of Upper Airway Stenosis 114


David Goldenberg, Ramon M. Esclamado, Paul W. Flint, Charles W.
Cummings Questions prepared by: Joshua S. Schindler

14
BY DR. MOHAMMED ATIAA KAREEM ALNASHY
-------------------------------------------------------------------
94. The Professional Voice 115
Gregory N. Postma, Mark S. Courey, Robert H. Ossoff
95. Benign Vocal Fold Mucosal Disorders 116
Robert W. Bastian Questions prepared by: Joshua S. Schindler
96. Medialization Thyroplasty 117
Paul W. Flint, Charles W. Cummings Questions prepared by: Joshua S.
Schindler
97. Arytenoid Adduction 118
Gayle Ellen Woodson Questions prepared by: Joshua S. Schindler
98. Laryngeal Reinnervation 119
George S. Goding, Jr.
99. Malignant Tumors of the Larynx and Hypopharynx 120
George L. Adams, Robert H. Maisel
100. Management of Early Glottic Cancer 121
Henry T. Hoffman, Lucy H. Karnell, Timothy M. McCulloch, John Buatti,
Gerry F. Funk Questions prepared by: Anton Chen
101. Transoral Laser Micro Resection of Advanced Laryngeal Tumors 122
Bruce W. Pearson, John R. Salassa, Michael L. Hinni
102. Conservation Laryngeal Surgery 123
Ralph P. Tufano, Gregory S. Weinstein, Ollivier Laccourreye, Christopher
H. Rassekh
103. Total Laryngectomy and Laryngopharyngectomy 124
Christopher H. Rassekh, Bruce H. Haughey
104. Radiation Therapy for the Larynx and Hypopharynx 125
Parvesh Kumar Questions prepared by: Anton Chen
105. Vocal Rehabilitation Following Total Laryngectomy 126
Joshua S. Schindler Questions prepared by: Joshua S. Schindler
106. Management of the Impaired Airway in the Adult 127
David Goldenberg, Nasir I. Bhatti Questions prepared by: Anton Chen

15
BY DR. MOHAMMED ATIAA KAREEM ALNASHY
-------------------------------------------------------------------
107. Endoscopy of the Tracheobronchial Tree 128
Rex C. Yung
108. Diagnosis and Management of Tracheal Neoplasms 130
Christine L. Lau, G. Alexander Patterson
109. Upper Airway Manifestations of Gastroesophageal Reflux Disease 131
Savita Collins Questions prepared by: Anton Chen
110. NECK Robbins Deep Neck Infection 135
Harrison G. Weed, L. Arick Forrest Questions prepared by: Matthew
Whitley
111. Blunt and Penetrating Trauma to the Neck 136
Robert H. Maisel, David B. Horn
112. Differential Diagnosis of Neck Masses 137
W. Frederick McGuirt, Sr. Questions prepared by: Matthew Whitley
113. Primary Neoplasms of the Neck 138
Terry A. Day, John K. Joe
114. Lymphomas Presenting in the Head and Neck 139
Nancy Price Mendenhall, Ilona M. Schmalfuss, Matthew C. Hull Questions
prepared by: Matthew Whitley
115. Radiation Therapy and Management of the Cervical Lymph Nodes
Bernard J. Cummings, John Kim, Brian OfSullivan Questions prepared by:
Matthew Whitley
116. Neck Dissection 141
K. Thomas Robbins, Sandeep Samant Questions prepared by: Matthew
Whitley
117. Surgical Complications of the Neck 142
Carol M. Bier-Laning

16
BY DR. MOHAMMED ATIAA KAREEM ALNASHY
-------------------------------------------------------------------
THYROID/PARATHYROID

Robbins

118. Disorders of the Thyroid Gland 145


Phillip K. Pellitteri, Steven W. Ing Questions prepared by: Justin Wittkopf
119. Management of Thyroid Neoplasms 146
Stephen Y Lai, Susan J. Mandel, Randal S. Weber
120. Surgical Management of Parathyroid Disorders 147
Phillip K. Pellitteri, Robert A. Sofferman, Gregory W. Randolph
121. Paranasal Sinuses: Management of Thyroid Eye Disease (Graves'
Ophthalmology) 148
Douglas A. Girod Questions prepared by: Justin Wittkopf

Volume Four

GENERAL

Harker

122. Anatomy of the Skull Base, Temporal Bone, External Ear, and Middle
Ear 151
Oswaldo Laercio M. Cruz Questions prepared by: Justin Wittkopf
123. Neural Plasticity in Otology 152
Robert V. Harrison
124. Tinnitus and Hyperacusis 154
Samuel G. Shiley, Robert L. Folmer, Sean O. McMenomey
125. Management of Temporal Bone Trauma 155
Hilary A. Brodie Questions prepared by: Justin Wittkopf
126. Otologic Symptoms and Syndromes 156
Carol A. Bauer, Herman A. Jenkins

17
BY DR. MOHAMMED ATIAA KAREEM ALNASHY
-------------------------------------------------------------------
127. Otologic Manifestations of Systemic Disease 157
Joseph B. Nadol, Jr., Saumil N. Merchant
128. Noise-Induced Hearing Loss 158
Brenda L. Lonsbury-Martin, Glen K. Martin

129. Autoimmune Inner Ear Disease 159


Steven D. Rauch, Michael J. Ruckenstein
130. Vestibular and Auditory Toxicity 160
Leonard P. Rybak
131. Pharmacologic Treatment of the Cochlea and Labyrinth 161
Anil K. Lalwani, John F. McGuire

INFECTIOUS PROCESSES

Harker

132. Infections of the External Ear 165


Michael J. Ruckenstein Questions prepared by: Justin Wittkopf
133. Chronic Otitis Media, Mastoiditis, and Petrositis 166
Richard A. Chole, Holger H. Sudhojf
134. Complications of Temporal Bone Infections 167
Lee A. Harker, Clough Shelton Questions prepared by: Justin Wittkopf 135
135. Infections of the Labyrinth 168
Larry E. Davis
136. Tympanoplasty and Ossiculoplasty 169
Hussam K. El-Kashlan, Lee A. Harker Questions prepared by: Justin
Wittkopf
137. Mastoidectomy 170
Paul R. Lambert, Questions prepared by: Matthew OMalley

18
BY DR. MOHAMMED ATIAA KAREEM ALNASHY
-------------------------------------------------------------------
VESTIBULAR SYSTEM

Harker

138. Anatomy of Vestibular End-Organs and Neural Pathways 173


Anna Lysakowski
139. Principles of Applied Vestibular Physiology 174
John P. Carey, Charles C. Delia Santina Questions prepared by: Matthew
O'Malley
140. Evaluation of the Patient with Dizziness 175
Timothy E. Hullar, Lloyd B. Minor, David S. Zee
141. Imbalance and Falls in the Elderly 176
Marian Girardi, Horst R. Konrad
142. Meniere's Disease and Other Peripheral Vestibular Disorders 177
David A. Schessel, Lloyd B. Minor, Julian M. Nedzelski, Questions
prepared by: Matthew O'Malley
143. Central Vestibular Disorders 178
Scott D. Z. Eggers, David S. Zee
144. Surgery for Vestibular Disorders 179
Steven A. Telian
145. Vestibular and Balance Rehabilitation Therapy: Program Essentials
180
Neil T Shepard, Steven A. Telian

19
BY DR. MOHAMMED ATIAA KAREEM ALNASHY
-------------------------------------------------------------------
FACIAL NERVE

Harker

146. Tests of the Facial Nerve Function 183


Robert A. Dobie
147. Clinical Disorders of the Facial Nerve 184
Douglas E. Mattox
148. Intratemporal Facial Nerve Surgery 185
Bruce J. Gantz, Jay T. Rubinstein, Ravi N. Samy

AUDITORY SYSTEM

Harker

149. Cochlear Anatomy and Central Auditory Pathways 189


Peter A. Santi, Patrizia Mancini
150. Molecular Basis of Auditory Pathology 190
JoAnn McGee, Edward J. Walsh Questions prepared by: Matthew
OMalley
151. Electrophysiologic Assessment of Hearing 191
Carolyn J. Brown
152. Diagnostic and Rehabilitative Audiology 192
Paul R. Kileny, Teresa A. Zwolan Questions prepared by: Matthew
O'Malley
153. Auditory Neuropathy 193
Robert J. Tibesar, Jon K. Shallop
154. Evaluation and Surgical Management of Conductive Hearing Loss 194
Douglas D. Backous, John K. Niparko

20
BY DR. MOHAMMED ATIAA KAREEM ALNASHY
-------------------------------------------------------------------
155. Sensorineural Hearing Loss: Evaluation and Management in Adults
195
H. Alexander Arts
156. Otosclerosis 196
John W. House, Calhoun D. Cunningham III
157. Surgically-Implantable Hearing Aids 197
Lawrence R. Lustig, Charles C. Delia Santina

COCHLEAR IMPLANTS

Harker

158. Patient Evaluation and Device Selection for Cochlear Implantation


201
P. Ashley Wackym, Jill B. Firszt, Christina L. Runge-Samuelson,
Questions prepared by: Sara Pai
159. Medical and Surgical Considerations in Cochlear Implantation 202
Thomas J. Balkany, Bruce J. Gantz
160. Cochlear Implants: Results, Outcomes, and Rehabilitation 203
John K. Niparko, Jennifer L. Mertes, Charles J. Limb

SKULL BASE

Harker

161. Diagnostic and Interventional Neuroradiology 207


Richard E. Latchaw, Patricia Silva
162. Temporal Bone Neoplasms and Lateral Cranial Base Surgery 208
Michael A. Marsh, Herman A. Jenkins Questions prepared by: Joshua S.
Schindler

21
BY DR. MOHAMMED ATIAA KAREEM ALNASHY
-------------------------------------------------------------------
163. Extra-Axial Neoplasms Involving the Anterior and Middle Cranial
Fossa 209
Timothy M. McCulloch, Russell Smith Questions prepared by: Jeffrey
Cutler
164. Surgery of the Anterior and Middle Cranial Base 210
Daniel W Nuss, Bert W. O'Malley, Jr. Questions prepared by: Jeffrey
Cutler
165. Extra-Axial Neoplasm of the Posterior Fossa 211
Derald E. Brackmann, Moises A. Arriaga
166. Auditory Brainstem Implants 212
Bruce J. Gantz, Ted A. Meyer
167. Transnasal Endoscopic-Assisted Surgery of the Skull Base 213
Aldo Cassol Stamm, Shirley S. N Pignatari Questions prepared by: Lisa
Earnest
168. Intraoperative Monitoring of Cranial Nerves in Neurotologic Surgery
214
Charles D. Yingling, Yasmine A. Ashram Questions prepared by: Gabriela
Sanchez
169. Radiation Therapy of the Cranial (Skull) Base 215
Nancy Y Lee, Edward J. Shin Questions prepared by: Gabriela Sanchez

PEDIATRIC OTOLARYNGOLOGY

Richardson

170. General Considerations 219


J. Scott McMurray, Mark A. Richardson
171. Developmental Anatomy 220
Daniel O. Graney, Kathleen C. Y Sie

22
BY DR. MOHAMMED ATIAA KAREEM ALNASHY
-------------------------------------------------------------------
172. Anesthesia 222
Jeffrey L. Koh, Veronica C. Swanson, Jeffrey Morray Questions prepared
by: Gabriela Sanchez
173. Characteristics of Normal and Abnormal Postnatal Craniofacial
Growth and Development 223
Frederick K Kozak, Juan Camilo Ospina Questions prepared by: Gabriela
Sanchez
174. Hemangiomas and Vascular Malformations of the Head and Neck 224
Reza Rahbar, Trevor J. I. McGill, John B. Mulliken
175. Craniofacial Surgery for Congenital and Acquired Deformities 225
Jonathan Z. Baskin, Sherard A. Tatum III, Questions prepared by:
Gabriela Sanchez
176. Cleft Lip and Palate 226
Oren Friedman, Tom D. Wang, Henry A. Milczuk, Questions prepared by:
Gabriela Sanchez
177. Velopharyngeal Dysfunction 227
Harlan R. Muntz, Helene M. Taylor, Marshall E. Smith, Questions
prepared by: Joshua S. Schindler
178. Congenital Malformations of the Nose 228
Karla Brown, Orval E. Brown Questions prepared by: Joshua S. Schindler
179. Pediatric Chronic Sinusitis 229
Rodney P Lusk
180. Salivary Gland Diseases 230
David L. Walner, Charles M. Myer HI
181. Pharyngitis and Adenotonsillar Disease 231
Brian J. Wiatrak, Audie L. Woolley Questions prepared by: Joshua S.
Schindler
182. Obstructive Sleep Apnea in Children 232
Laura M. Sterni, David E. Tunkel

23
BY DR. MOHAMMED ATIAA KAREEM ALNASHY
-------------------------------------------------------------------
183. Pediatric Head and Neck Malignancies 233
Carol J. MacArthur, Richard J. H. Smith
184. Differential Diagnosis of Neck Masses 234
Ralph F. Wetmore, William P. Potsic
185. Congenital Disorders of the Larynx 235
Anna H. Messner
186. Managing the Stridulous Child 236
David M. Albert, Susanna Leighton
187. Glottic and Subglottic Stenosis 237
George H. Zalzal, Robin T Cotton
188. Gastroesophageal Reflux and Laryngeal Disease 238
Philippe Narcy, Philippe Contencin, Thierry Van Den Abbeele
189. Aspiration and Swallowing Disorders 239
Philippe Narcy, Philippe Contencin, Thierry Van Den Abbeele
190. Voice Disorders 240
Sukgi S. Choi, George H. Zalzal
191. Congenital Disorders of the Trachea 241
Reza Rahbar, Gerald B. Healy
192. Tracheal Stenosis 242
Mark A. Richardson, Greg R. Licameli Questions prepared by: Gabriela
Sanchez
193. Caustic Ingestion 243
Dale Browne, James N. Thompson
194. Foreign Bodies of the Airway and Esophagus 245
Roberto L. Barretto, Lauren D. Holinger
195. Infections of the Airway 246
Newton O. Duncan III, Questions prepared by: Gabriela Sanchez
196. Recurrent Respiratory Papillomatosis 247
Craig S. Derkay, Russell A. Faust

24
BY DR. MOHAMMED ATIAA KAREEM ALNASHY
-------------------------------------------------------------------
197. Early Detection and Diagnosis of Infant Hearing Impairment 248
Susan J. Norton, Jonathan A. Perkins Questions prepared by: Gabriela
Sanchez
198. Congenital Malformations of the Inner Ear 249
Robert K. Jackler, Questions prepared by: Gabriela Sanchez
199. A Reconstruction Surgery of the Ear: Microtia Reconstruction 250
Craig S. Murakami, Vito C Quatela Questions prepared by: Craig S.
Murakami and Gabriela Sanchez
B Reconstruction Surgery of the Ear: Auditory Canal and Tympanum
251
Antonio De la Cruz, Marian R. Hansen
200. Acute Otitis Media and Otitis Media with Effusion 252
Andrew F. Inglis, Jr., George A. Gates
201. Genetic Sensorineural Hearing Loss 253
Murad Husein, Richard J. H. Smith
202. Pediatric Facial Fractures 254
Peter J. Koltai, Paul R. Krakovitz

25
BY DR. MOHAMMED ATIAA KAREEM ALNASHY
-------------------------------------------------------------------

PART ONE
--------------------------------------------------------------------------------------------
--------------------------------------------------------------------------------------------

GENERAL CONSIDERATIONS IN HEAD AND NECK

26
BY DR. MOHAMMED ATIAA KAREEM ALNASHY
-------------------------------------------------------------------
CHAPTER ONE: HISTORY, PHYSICAL EXAMINATION, AND THE
PREOPERATIVE EVALUATION

--------------------------------------------------------------------------------------------

1. When a patient is found to have a Virchow node, it will be located

A. In the supraclavicular nodes of level 5a


B. In the supraclavicular nodes of level 4
C. In the level 6 nodes
D. In the level 5b supraclavicular nodes
E. In the submental sublevel la nodes

2. Which of the following statements regarding Addison's disease is false?

A. It results in both glucocorticoid and mineralocorticoid deficiencies.


B. Hyperpigmentation is caused by overproduction of adrenocorticotropic
hormone and P-lipotropin.
C. Glucocorticoid replacement is required twice a day.
D. Mineralocorticoid therapy is given once a day.
E. Stress-dose steroids are not necessary perioperatively.

3. Latex allergies

A. Are seen in only a small percentage of healthcare workers


B. May lead to anaphylaxis in the operating department
C. Require only premedication to adequately prepare for an operation
D. Do not result in a positive skin test to latex proteins
E. Are not true allergies, but rather a side effect of wearing gloves

27
BY DR. MOHAMMED ATIAA KAREEM ALNASHY
-------------------------------------------------------------------
4. Which of the following statements regarding hyperthyroidism is false?

A. Hyperthyroidism may result in hypercalcemia, thrombocytopenia, and


anemia.
B. Hyperthyroidism may be treated with mithramycin.
C. Hyperthyroidism may be treated with propylthiouracil.
D. Hyperthyroidism may be treated with Lugol's solution to inhibit iodide
organification.
E. Propanolol treats sympathetic hyperactivity and decreases T4- to T3-
conversion.

5. Which of the following is not a risk factor for perioperative


cardiovascular complications?
A. Recent myocardial infarction
B. Older than 50 years of age
C. Third heart sound
D. Nonsinus rhythm
E. Valvular aortic stenosis

28
BY DR. MOHAMMED ATIAA KAREEM ALNASHY
-------------------------------------------------------------------
CHAPTER 1 ANSWERS KEY

History, Physical Examination, and the Preoperative Evaluation

1. B
By definition, in the latest neck classification system developed by the
American Head and Neck Society and the AAO-HNS, the Virchow node is
part of level IV, not sublevel VB, which includes the transverse cervical and
supraclavicular nodes.

2. E
Patients treated for more than 3 weeks with exogenous glucocorticoids should
be assumed to have suppression of their adrenal-pituitary axis and should
receive stress-dose steroids.

3. B
Latex allergies may result in serious, life-threatening anaphylactic reactions.
Proper planning requires avoidance of latex products in the operating room.

4. B
Mithramycin treats hypercalcemia by inhibiting parathyroid hormone-
induced osteocytoclastic activity.

5. B
Age older than 70 years is a risk factor.

29
BY DR. MOHAMMED ATIAA KAREEM ALNASHY
-------------------------------------------------------------------
CHAPTER TWO: OVERVIEW OF DIAGNOSTIC IMAGING OF THE
HEAD AND NECK

-------------------------------------------------------------------------------------------

1. Which of the following is true of the Caldwell view of conventional


radiography?
A. It provides an excellent view of anterior ethmoid cells.
B. It is obtained in the posteroanterior projection with 35 degrees of
caudal angulation of the x-ray beam.
C. It is not suited for evaluation of the frontal sinuses.
D. It provides an excellent view of maxillary sinuses.
E. It provides an excellent view of the posterior ethmoid cells.

2. Which of the following is not true of chemical shift artifact of magnetic


resonance imaging (MRI)?
A. It occurs in areas where fat abuts structures containing predominantly
water.
B. It may produce the appearance of a pseudo-capsule around a lesion.
C. It is seen as a bright band on one side of a structure and a black band
on the opposite side.
D. It is most apparent on T2-weighted images.
E. It may cause obscuration of a small-diameter structure.

3. Which of the following is true of spin (proton) density images?


A. They use a short repetition time and a long echo time.
B. They show air and bone as high signal intensity.
C. They show fluid-containing structures and muscles as high signal
intensity.
D. A solid mass or fluid-filled lesion with a high protein content
demonstrates low signal intensity.
E. Paranasal sinus inflammation appears very bright.

30
BY DR. MOHAMMED ATIAA KAREEM ALNASHY
-------------------------------------------------------------------
4. Which of the following are true regarding imaging of the parotid gland?
A. Lesions in the parotid are better defined on computed tomography (CT)
than on MRI.
B. Ductal anatomy is best delineated by sialography.
C. Pleomorphic adenoma is hyperintense on Tl-weighted images and
hypointense on T2-weighted images.
D. Chronic sialadenitis is brighter on Tl-weighted images than on
T2-weighted images.
E. Facial nerve anatomy is best assessed with ultrasonography.

5. Which of the following radiographic properties of a lymph node on


contrast-enhanced CT does not support identification of cervical
adenopathy as inflammatory (reactive)?
A. Less than 10 mm.
B. Well-defined margins.
C. Central hilar or mild homogeneous enhancement.
D. Central low intensity.
E. Calcification.

CHAPTER 2 ANSWERS KEY

Overview of Diagnostic Imaging of the Head and Neck

1. E

2. D

3. E

4.B

5. D

31
BY DR. MOHAMMED ATIAA KAREEM ALNASHY
-------------------------------------------------------------------
CHAPTER THREE: BIOPHYSIOLOGY AND CLINICAL
CONSIDERATIONS IN RADIOTHERAPY
-----------------------------------------------------------------------------------------
1. Which of the following statements is true regarding the high-energy
x-rays used in radiation therapy?
A. The x-rays, being highly energetic, have a long wavelength
compared with cellular dimensions.
B. The initial interaction of the x-rays with matter typically
produces a high-energy electron, which in turn causes multiple
ionizations.
C. The biologic effect of the x-rays is due to heating caused by
inducing molecular rotations.
D. X-rays have a shorter penetration distance than high-energy
electrons, which are also used in therapy.
E. Bony structures show up better on verification therapy films than
on standard diagnostic x-ray films.

2. Compared with standard photon and electron radiation, high linear


energy transfer (LET) radiation has the following property:
A. The oxygen enhancement ratio is higher, making it more useful in
treating hypoxic tumors.
B. The relative biologic effectiveness factor is lower.
C. LET causes double-stranded breaks in the DNA of the cells, which
are less readily repaired.
D. LET is more effective in treating squamous cell tumors of the head
and neck.
E. Cell survival curves typically have a larger shoulder for high LET
irradiation than for low LET irradiation.

32
BY DR. MOHAMMED ATIAA KAREEM ALNASHY
-------------------------------------------------------------------
3. There are several different ways of fractionating radiotherapy for
head and neck cancers. Which of the following statements is true?
A. Radiotherapy is fractionated to allow tumor cells time to repair
their DNA and thus move into a radiosensitive portion of the cell
cycle.
B. Patients treated with hyperfractionated radiotherapy finish their
treatment quicker than those treated with standard fractionation.
C. Rapidly proliferating tumors can potentially replace a significant
portion of cells killed with each dose of radiation.
D. The acute and late effects seen for both accelerated and continuous
hyperaccelerated radiotherapy are more intense than those seen
for standard fractionation.

4. Intensity-modulated radiation therapy (IMRT) is rapidly becoming


more common in treating head and neck cancers. Which of the
following statements is false?
A. Trials to date with IMRT have a significantly lower number of
treatments than with standard radiotherapy.
B. IMRT treatment plans are designed to have very steep gradients
between isodoses.
C. IMRT can be more conformal than traditional three-dimensional
radiotherapy.
D. Initial results suggest that IMRT for nasopharyngeal carcinoma
actually may improve overall survival and not just local control.

5. Combined-modality therapy with both chemotherapy and


radiotherapy is becoming common for head and neck cancers. Which
statement best summarizes the rationale for the use of combined
modality therapy?
A. Chemotherapy, when given concurrently with radiotherapy, can
sensitize both normal and tumor cells to the effects of
radiotherapy.
B. Certain agents, such as amifostine, may be able to increase the
therapeutic ratio by protecting normal tissues.
C. Chemotherapy may be effective at addressing micrometastases
outside the radiation field.
D. In general, acute affects are more intense with combined modality
therapy than in single modality therapy.
E. A, B, and C
F. All of the above

33
BY DR. MOHAMMED ATIAA KAREEM ALNASHY
-------------------------------------------------------------------
CHAPTER 3 ANSWERS KEY
Biophysiology and Clinical Considerations in Radiotherapy
1. B
The primary interaction between a megavoltage x-ray and matter typically
produces a high-energy x-ray that causes a chain of ionization events as it goes
along its path. These ionization events break molecular bonds, and this causes the
biologic damage and not simple thermal heating. Electromagnetic radiation has
a dual nature and can be described as either quanta of energy or waves. With the
wave picture, the energy goes inversely as the wavelength, with shorter
wavelengths corresponding to higher energies. A 1-MeV photon, which is the
order of magnitude of the x-rays used in treating head and neck cancer, has a
wavelength of approximately 10~2 angstrom, which is considerably smaller than a
cell. The nature lof the primary interaction between x-rays and matter depends
on the energy of the x-ray. X-rays used in therapy have much higher energy than
the x-rays used for diagnostic purposes and primarily interact by way of the
Compton effect. This means that bone and soft tissues show up about the same,
and one does not see the distinction between the two as readily as in diagnostic
films. Megavoltage electrons can also be used in therapy and are produced by the
same linear accelerators used to produce x-rays. They have the same radiobiologic
properties as the x-rays but are advantageous in the treatment of lymph nodes in
the neck that overlie the spinal cord because of their shorter penetration distance.

2. C
LET refers to the energy distributed by the particle along its path. High LET
radiation produces a dense ionization chain as it goes through tissue, which in
turn causes a high percentage of double-stranded breaks in the DNA that are not
readily repairable. This gives rise to small shoulders on radiation cell survival
curves and a steeper slope to the curves. Hence, the RBE of high LET radiation
is higher than standard forms of radiation. Direct interaction with the cellular
DNA means that there is less dependence on a free radical-mediated mechanism,
which in turn means that high LET radiation is more effective in killing hypoxic
cells (e.g., the OER is low). Most of the clinical work on high LET radiation is
with fast neutrons. After a large number of clinical trials, only a few selected
types of tumors have been shown to respond better to fast neutron radiation. In
the head and neck region, it is salivary gland tumors and not the more common
squamous cell tumors that are better treated with high LET radiation.

34
BY DR. MOHAMMED ATIAA KAREEM ALNASHY
-------------------------------------------------------------------
3. C
The whole rationale behind fractionation is to allow normal tissue time to repair
and thus cause less long-term damage. Some tumor cells may repair, but most
tumor cells have defective repair mechanisms and are thus sensitive to the
radiation. Hyperfractionated schemes are the same treatment length as standard
fractionation. Accelerated hyperfractionation escalates the dose and decreases the
treatment time. Repopulation is a significant problem in tumors that seem less
sensitive to radiotherapy. Altered fractionation schemes are designed to deliver
an increased effective dose to the tumor so that the tumor cannot repopulate, while
still allowing normal tissues time to repair. This is the reason that the acute
effects of hyperfractionated therapy are more intense, but the late effects are
similar to standard radiotherapy.

4. A
Trials to date with IMRT have, in general, the same number of treatment days as
standard therapy. However, because the isodose gradients can be very steep and
the dose to normal tissue can be reduced, future studies may have IMRT
treatments taking longer to allow for dose escalation. The steeper isodose gradients
with IMRT also mean that the treatments are more conformal than traditional
three-dimensional con-formal therapy. The dose in three-dimensional therapy is
more homogeneous throughout the field than it is for IMRT.

5.F
Chemotherapy with cytotoxic agents is thought to make the tumor cells more
sensitive to radiotherapy. The exact mechanisms are unknown, but there is
some thought that certain chemotherapy agents might temporarily arrest
cells in a portion of the cell cycle that is more radiation sensitive.
Traditional chemotherapy agents have always been cytotoxic. However, it
may be possible to also increase the therapeutic ratio by protecting normal
tissues. Because radiation will only kill tumor cells that are targeted,
concurrent systemic therapy may be able to clear micrometastatic disease
that is not in the radiation field. Because many chemotherapy agents also
affect rapidly proliferating tissue (much like radiotherapy does), the acute
effects of combined modality therapy can be very intense.

35
BY DR. MOHAMMED ATIAA KAREEM ALNASHY
-------------------------------------------------------------------
CHAPTER FOUR: CHEMOTHERAPY FOR HEAD AND NECK
CANCER
------------------------------------------------------------------------------------------
1. The dose-limiting toxicity of carboplatin is which of the following?
A. Hepatotoxicity
B. Myelosuppression
C. Ototoxicity
D. Peripheral neuropathy
E. Gastritis

2. Which of the following is true of induction chemotherapy?


A. Patients who are resistant to cisplatin-based induction
chemotherapy have a high likelihood of not responding to
radiotherapy.
B. Treatment increases morbidity of surgery.
C. Survival is not prolonged in responders vs nonresponders.
D. No affect on organ preservation or quality of life has been proven.
E. Distant metastases are slightly more likely.

3. Which of the following is true of concurrent chemotherapy and


radiation therapy?
A. It is used primarily before surgery in patients with larger tumor
burden.
B. It increases incidence in acute radiation-induced toxicity
(primarily mucosal toxicity).
C. Concurrent bleomycin and radiotherapy is used for advanced
nasopharyngeal carcinoma.
D. Concurrent chemotherapy and radiation therapy is useful
primarily for pain relief with palliation as a goal.

4. Which of the following is a desirable property of an intra-arterial


chemotherapy agent?
A. The drug should be activated in the systemic circulation.
B. The drug should have a high tissue extraction.
C. The drug should require activation in the liver.
D. The drug should not be cleared by the kidney.
E. The drug should not include platinum compounds.

36
BY DR. MOHAMMED ATIAA KAREEM ALNASHY
-------------------------------------------------------------------
5. Which of the following is not a common toxicity associated with
cisplatin therapy?
A. Nausea
B. Vomiting
C. Renal dysfunction
D. Ototoxicity
E. Severe neutropenia

CHAPTER 4 ANSWERS KEY


Chemotherapy for Head and Neck Cancer
1. B
2. A
3. B
4. B
5. E

37
BY DR. MOHAMMED ATIAA KAREEM ALNASHY
-------------------------------------------------------------------
CHAPTER FIVE: SKIN FLAP PHYSIOLOGY AND WOUND
HEALING
-----------------------------------------------------------------------------------------
1. The failure of a microvascular free flap to survive after a prolonged
ischemia time despite patent anastomoses may be considered a failure
of
A. Zone I
B. Zone II
C. Zone III
D. Zone IV
E. Zone II or III

2. True/False. The paramedian forehead flap is an example of a random


pattern flap.

3. The survival of random pattern flaps is most directly affected by


A. The period of ischemia after the flap is raised.
B. The length/width ratio of the skin flap.
C. The perfusion pressure gradient.
D. The thickness of the raised flap.
E. The smoking status of the patient.

4. True/False. After skin flap elevation, lymphatic obstruction occurs


and affects the skin perfusion and survival.

5. Neovascularization of transferred flaps occurs in


A. 7 days.
B. 2 days.
C. 3 weeks.
D. 3 months

38
BY DR. MOHAMMED ATIAA KAREEM ALNASHY
-------------------------------------------------------------------
CHAPTER 5
Skin Flap Physiology and Wound Healing
1. E
The zones of perfusion may be classified according to the anatomic and
physiologic components of vascular supply to the soft tissue. Zone I refers to
the macrocirculation, the arterial conduits, and venous drainage. In free
tissue transfer, the microvascular anastomoses create an alternative zone I
perfusion to the transferred tissue. Thromboses at the anastomosis represents
zone I failure. Zone II is the capillary circulation, and zone III is the
interstitium. The "no-reflow" phenomenon is the failure of flap survival
despite adequate zone I circulation. It is believed to be the result of the
accumulation of free radicals with extended periods of ischemia. This leads
to swelling of the endothelial and parenchymal cells coupled with
intravascular stasis. Eventually, thrombosis leads to loss of nutritive flow.

2. False
The paramedian forehead flap is based on the distribution of the
supratrochlear artery. This makes the flap an arterial or axial pattern flap.
Random pattern flaps are perfused by no identifiable artery but rather by
the subdermal plexus. Axial pattern flaps are more robust and generally can
be tolerated to much greater lengths without ischemia.

3. C
The surviving length of the random portion of the flap depends on the
physical properties of the supplying vessels (intravascular resistance) and
the perfusion pressure. When the perfusion pressure decreases below the
pressure in the interstitial space, capillary blood flow ceases. The pressure
at which there is no longer enough intravascular blood pressure to maintain
capillary blood flow is called the critical closing pressure.

39
BY DR. MOHAMMED ATIAA KAREEM ALNASHY
-------------------------------------------------------------------
4. True
Impairment of lymphatic drainage with flap elevation also occurs.
Reduction of the cutaneous lymphatic drainage results in an increase in
interstitial fluid pressure that is compounded by increased leakage of
intravascular protein associated with inflammation. The resulting edema
leads to increased interstitial pressure, which decreases capillary perfusion
by increasing the critical closing pressure. Alterations in Starling's forces
result in further ischemic swelling of cells and the interstitial space, setting
a positive feedback cycle in motion. This can threaten flap survival by
limiting the perfusion pressure.

5. A
In surviving flaps, blood flow gradually increases. If the flap is in a
favorable recipient site, a fibrin layer forms within the first 2 days.
Neovascularization of the flap begins 3 or 4 days after flap transposition.
Revascularization adequate for division of the flap pedicle has been shown
as early as 7 days in animal models and humans. During revascularization,
vascular endothelial cells play a major role in the formation of new vessels.
Normally, endothelial cells are in a quiescent state, although when
stimulated by angiogenic growth factors, these cells can dramatically
proliferate.

40
BY DR. MOHAMMED ATIAA KAREEM ALNASHY
-------------------------------------------------------------------
CHAPTER SIX: FREE TISSUE TRANSFER
-----------------------------------------------------------------------------------------
1. The first free tissue transfer for oral cavity reconstruction was
reported in what decade?
A. The early 1960s.
B. The early 1970s.
C. The early 1980s.
D. The early 1990s.

2. The advantage of free tissue transfer over other techniques for head
and neck reconstruction include
A. Versatility of available tissues.
B. Multiple donor site options.
C. Donor site outside the field of radiation therapy.
D. Single-stage reconstruction even for very large defects.
E. All of the above.

3. Which soft-tissue donor site has proven to be the most versatile,


reliable, and commonly used?
A. The ulnar forearm flap.
B. The radial forearm flap.
C. The lateral arm flap.
D. The rectus flap.

4. Donor site selection can be influenced by


A. Prior surgery or trauma.
B. Handedness or footedness.
C. Occupation.
D. Hobbies.
E. All of the above.

5. What is the most reliable method for postoperative free tissue transfer
monitoring?
A. Visible flap inspection and pinprick.
B. Doppler monitoring of the flap pedicle.
C. Color flow Doppler monitoring.
D. Laser Doppler velocimetry.
E. Oxygen tension measurements

41
BY DR. MOHAMMED ATIAA KAREEM ALNASHY
-------------------------------------------------------------------
CHAPTER 6 ANSWERS KEY
Free Tissue Transfer
1. B
The first free tissue transfer was reported in 1959; however, the first report for
oral cavity reconstruction was not until 1973 by Kaplan and others using a free
groin flap.

2. E
The multiple advantages of free tissue transfer for head and neck reconstruction
over all other techniques are outlined in Table 2. The numerous available donor
sites provide tremendous versatility of tissues (bone, skin, and muscle) from
nonirradiated regions of the body. This allows the surgeon to address all
components of the required reconstruction in a single procedure, replacing "like
tissues with like tissues."

3B
Forearm skin is available in large quantities, is thin and pliable, and has
excellent sensory capability, which is ideal for oral cavity reconstruction. The
vascular pedicle is long, vessel-caliber favorable, concurrent harvest easily
performed, and donor site functional morbidity acceptable. These attributes have
made the radial forearm free flap the "workhorse flap" for head and neck
reconstructions.

4. E
Donor site selection is influenced by many factors, including all those listed in
Question 4. Additional factors include surgeon preference and experience and
patient anatomy and vascular status, which may require additional preoperative
testing to verify.

5.A
The direct observation of a portion of the flap with pin prick to assess bleeding
remains the most reliable method of flap monitoring. This approach, however, is
very labor intensive. As a result, there are ongoing efforts to develop less
manpower-intensive approaches for monitoring. To date, none have proven
reliable enough to replace close direct tissue monitoring.

42
BY DR. MOHAMMED ATIAA KAREEM ALNASHY
-------------------------------------------------------------------
CHAPTER SEVEN: LASER SURGERY: BASIC PRINCIPLES AND
SAFETY CONSIDERATIONS
-----------------------------------------------------------------------------------------
1. Which of the following lasers has the highest risk of injury to the
pulmonary vessels during laser bronchoscopy?
A. Carbon dioxide (C02) laser
B. Potassium-titanyl-phosphate (KTP) laser.
C. Neodymium: yttrium-aluminum-garnet (Nd:YAG) laser.
D. Holmium: yttrium-aluminum-garnet (Ho:YAG) laser.
E. Diode laser.

2. The most commonly used lasers in otolaryngology (C02, KTP,


Nd:YAG) exert their biologic effect through which of the following
laser-tissue interactions?
A. Transmission.
B. Scatter.
C. Reflection.
D. Absorption.
E. Photochemical effect.

3. Eye protection for both the patient and operating department


personnel is essential for proper laser safety. The most important laser
parameter in determining ocular structures at risk during surgery is
A. Laser wavelength.
B. Pulse structure.
C. Pulse duration.
D. Power setting.
E. Delivery system.

4. The most effective method of reducing collateral thermal damage to


tissue surrounding the ablation crater of a C02 laser on laryngeal
mucosa is to
A. Cool the tissue before laser application.
B. Reduce the spot size of the laser beam.
C. Use a short-pulse laser setting.
D. Reduce the power setting.
E. Use continuous mode.

43
BY DR. MOHAMMED ATIAA KAREEM ALNASHY
-------------------------------------------------------------------
5. Which of the following vocal fold lesions is most appropriate for C0 2
laser excision?
A. Polyp.
B. Nodule.
C. Intracordal cyst.
D. Sulcus vocalis.
E. Papilloma

44
BY DR. MOHAMMED ATIAA KAREEM ALNASHY
-------------------------------------------------------------------
CHAPTER 7 ANSWERS KEY
Laser Surgery: Basic Principles and Safety Considerations
1. C
The Nd:YAG laser has the deepest thermal penetration of the listed lasers.
Thermal injury can occur 4 mm deep to the ablation crater.

2.D
The above lasers cause tissue effects by absorption of the light energy and
conversion to heat.

3. A
The retina is most at risk with wavelengths in the visible and near-infrared
range of the electromagnetic spectrum. Corneal injury can occur with lasers
in the ultraviolet or infrared range of the spectrum.

4. C
Knowledge of laser-tissue interactions is essential for the surgeon to safely
apply laser technology to tissue. Several parameters are important in laser
use, including power, density, and fluence (see pp. 9, 10).

5. E
For benign lesions of the vocal folds involving the lamina propria such as
cysts, nodules, polyps, and sulcus vocalis, the best surgical technique would
be mucosal sparing excision. The C02 laser would be most appropriate for
RRP, because it is an epithelial disease, and the risk of thermal damage to
otherwise normal tissue is lessened.

45
BY DR. MOHAMMED ATIAA KAREEM ALNASHY
-------------------------------------------------------------------
CHAPTER EIGHT: DIFFICULT AIRWAY/INTUBATION:
IMPLICATIONS FOR ANESTHESIA
-------------------------------------------------------------------------------------------
1. The post anesthesia care unit nurse calls you to evaluate your patient
for disorientation after a uvulopalatopharyngoplasty. On your arrival,
the patient is snoring heavily, with 91% oxygen saturation (Sa02), on
40% oxygen by face mask (FM). Appropriate immediate management
is
A. Administer naloxone.
B. Administer 100% Sa02 via FM Ambu Bag; perform chin lift and jaw
thrust to alleviate the airway obstruction.
C. Perform flexible nasopharyngoscopy to evaluate for edema in the
posterior pharynx.
D. Perform immediate cricothyrotomy to secure an airway in the
presence of mental status changes.

2. You are asked to assist with the awake fiberoptic intubation of a


patient with angioedema and respiratory distress. The patient is
agitated during the process. To note, the patient has a heart rate of 85
beats/min, blood pressure of 150/90 mm Hg, and Sa02 of 98%. Which of
the following medications would you consider giving as part of this
awake intubation?
A. Labetalol.
B. Midazolam.
C. Propofol.
D. Succinylcholine.

3. When a local anesthetic is used for a glossopharyngeal nerve block,


the potential complication of primary concern is
A. Edema from subcutaneous infiltration.
B. Injection into a vein leading to cardiac arrhythmias.
C. Toxic absorption of local anesthetic.
D. Seizure caused by arterial injection.

46
BY DR. MOHAMMED ATIAA KAREEM ALNASHY
-------------------------------------------------------------------
4. Your patient is an elderly man with hypertension who has a 60
pack/year history of cigarette smoking. He is being seen for evaluation
of a vocal cord polyp. Which medication has the potential to cause
tachycardia and hypertension in this patient?
A. Cocaine.
B. Etomidate.
C. Lidocaine.
D. Metoprolol.

5. In which of the following patients is a rapid-sequence


induction/intubation not advisable?
A. Diabetic patient with gastroparesis.
B. Patient with obstructive sleep apnea and Mallampati class IV airway.
C. Trauma patient in whom the time of last oral intake is unknown.
D. Vomiting patient with a small bowel obstruction.

6. The American Society of Anesthesiologist's Difficult Airway


Algorithm reminds the practitioner to consider all of the following
choices except
A. Awake vs asleep intubation.
B. Macintosh vs Miller blade for laryngoscopy.
C. Paralyzed vs spontaneously breathing techniques.
D. Surgical vs nonsurgical airway management.

7. The laryngeal mask airway (LMA) differs from an endotracheal tube


in which important respect?
A. The LMA cannot be used for positive-pressure ventilation.
B. The LMA does not protect the patient's airway from aspiration of
gastric contents.
C. The LMA is difficult to place in patients who are difficult to
intubate.
D. The LMA requires more time to place than an endotracheal tube.

8. All of the following reduce the risk of airway fire except


A. Avoiding the use of nitrous oxide.
B. Minimizing the inspired oxygen concentration.
C. Using bipolar rather than unipolar cautery within the airway.
D. Using special endotracheal tubes during laser surgery

47
BY DR. MOHAMMED ATIAA KAREEM ALNASHY
-------------------------------------------------------------------
9. Which of the following is not part of the treatment of malignant
hyperthermia?
A. Dantrolene.
B. Discontinuation of the volatile anesthetic.
C. Succinylcholine.
D. Symptomatic cooling.

48
BY DR. MOHAMMED ATIAA KAREEM ALNASHY
-------------------------------------------------------------------
CHAPTER 8 ANSWERS KEY
Difficult Airway/Intubation: Implications for Anesthesia
1.B
Appropriate immediate management for this patient with partial upper
airway obstruction is basic noninvasive airway adjuncts such as chin lift
and jaw thrust. Placement of a nasal trumpet or oral 100% oxygen with FM-
Ambu bag and airway, if tolerated, could be an appropriate next step. If
these simple maneuvers failed to relieve the obstruction in this patient with
known sleep apnea, then diagnostic procedures such as naloxone and/or
nasopharyn-goscopy could be performed. If indicated by the clinical
scenario, re-intubation through an orotracheal route would be preferable to
immediate cricothyrotomy.

2. B
Midazolam is a benzodiazepine with anxiolytic properties that is a useful
adjunct for awake intubation in agitated patients. Unlike diazepam, the
anxiolytic properties are not dose-dependent, and midazolam can be titrated
safely (in small doses) to facilitate awake intubations. Labetalol is a
combined a- and P-blocker that is an excellent antihypertensive but has
little role in the acute setting in which any hypertension is most likely
secondary to agitation. Succinylcholine is a fast-acting depolarizing
paralytic that is contraindicated in this patient who is spontaneously
ventilating and awake. Propofol is an induction agent that could be used for
sedation in smaller doses, but a full induction dose causes apnea, and,
therefore, propofol would not typically be a first-line agent for sedation
during an awake intubation.

3.D
The lateral approach to glossopharyngeal nerve block brings the path of the
needle dangerously close to the carotid artery. Injection of even small
amounts of local anesthetic into the artery delivers a bolus of local
anesthetic to the brain and may result in seizure. Local tissue swelling and
total overdose are also relative concerns but not as clinically important with
this nerve block. Toxic absorption of local anesthetic is manifested by
arrhythmia before GNS changes. Therefore, dysrhythmias are a concern, but

49
BY DR. MOHAMMED ATIAA KAREEM ALNASHY
-------------------------------------------------------------------
compared with intracarotid injections, a larger dose of anesthetic would
need to be injected intravenously before seeing these side effects.

4. A
This is a patient who is known to be hypertensive and is also at high risk of
coronary artery disease. Therefore, exacerbations of hypertension and
tachycardia should be avoided. Cocaine can cause these sympathetic effects.
Lidocaine mixed with phenylephrine can be used as a combination
anesthetic and vasoconstrictor with less risk of systemic adverse effects.
Etomidate would be an appropriate induction agent in this patient with
potentially impaired cardiac function, and p-blockade with metoprolol
would be a good technique to reduce risk of myocardial ischemia.

5.B
The cardinal principle is that rapid-sequence induction/intubation has both
potential risks and benefits. The main risk is the "cannot intubate, cannot
ventilate" scenario. In a patient with known obstructive sleep apnea,
difficulty with mask ventilation is to be expected. In addition, this patient
has a Mallampati class IV oral view. Paralysis is best avoided in this patient
until ability to mask ventilate is verified, in case this patient is also difficult
to intubate. Unless this patient has a documented history of recent successful
airway management with conventional laryngoscopes (Mac/Miller),
considerations for awake techniques and/or the immediate availability of
the OLHN rigid. The patient with gastroparesis, the trauma patient
(considered to have a full stomach), and the actively vomiting patient all
present significantly increased risk of aspiration, and, therefore, weighing
risks and benefits would most likely favor rapid sequence
induction/intubation unless there was reason to anticipate difficulty with
their airways.

50
BY DR. MOHAMMED ATIAA KAREEM ALNASHY
-------------------------------------------------------------------
6. B
The ASA Difficult Airway Algorithm does ask the practitioner to consider
broad points, such as awake vs asleep techniques, surgical vs nonsurgical
techniques, and paralyzed vs spontaneously ventilating patients. It does not
offer a checklist of specific intubating tools such as Miller vs Macintosh
blades for laryngoscopy.

7. B
The LMA does not isolate the patient's esophagus and trachea, as does an
endotracheal tube. The LMA is usually fast and easy to place and can be
used for positive pressure ventilation if necessary. When used for positive
pressure ventilation, the LMA can also force air into the stomach and
increase the risk for aspiration. Maintaining airway pressures <20 cm H20
is advised, so that the amount of air pushed past the esophageal sphincter is
lessened.

8. C
The use of any cautery within the airway poses a risk for airway fire. Both
bipolar and unipolar cautery produce sparks. Minimizing the oxygen
concentration by reducing both the inspired oxygen and nitrous oxide
fractions can reduce the risk of fire. Special endotracheal tubes are
manufactured or can be jury-rigged that reduce the risk of heat from laser
surgery igniting the oxygen-rich atmosphere within the tube.

9. C
Malignant hyperthermia can be triggered by succinylcholine or volatile
anesthetics. Therefore, these agents should be avoided in patients at risk for
MH. Once MH is suspected, discontinuing the volatile anesthetic, immediate
administration of dantrolene, and symptomatic cooling are all part of the
treatment for this potential catastrophe.

51
BY DR. MOHAMMED ATIAA KAREEM ALNASHY
-------------------------------------------------------------------
CHAPTER NINE: ALLERGY AND IMMUNOLOGY OF THE UPPER
AIRWAY
-------------------------------------------------------------------------------------------
1. The major histocompatibility complex, which codes for molecules that
allow the immune system to distinguish between self and nonself, is
located on
A. Chromosome 5.
B. Chromosome 13.
C. Chromosome 6.
D. Chromosome 10.
E. Chromosome 21.

2. Which of the following cells are important in antigen presentation


(antigen-presenting cells)?
A. Monocytes.
B. Macrophages.
C. Dendritic cells.
D. B cells.
E. Langerhans' cells.
F. All of the above.

3. Which of the following cytokine(s) is secreted by TH2 CD4+ cells?


A. Interleukin (IL)-2.
B. IL-4.
C. IL-5.
D. Interferon (IFN)-α.
E. IL-13.

4. Which of the following immunoglobulins (Ig) is most important in


memory immune responses?
A. IgG.
B. IgM.
C. IgA.
D. IgE.
E. IgD.

52
BY DR. MOHAMMED ATIAA KAREEM ALNASHY
-------------------------------------------------------------------
5. Eosinophils produce all of the following except
A. Peroxidases.
B. Neurotoxins.
C. Proteins.
D. Cytokines.
E. Histamine

53
BY DR. MOHAMMED ATIAA KAREEM ALNASHY
-------------------------------------------------------------------
CHAPTER 9 ANSWERS KEY
Allergy and Immunology of the Upper Airway
1. C
The essence of specific immunity is the ability to discriminate at the
molecular level between self and nonself. This ability allows the immune
system to attack and destroy potentially harmful microorganisms without
simultaneously destroying the individual infected by these agents. The
molecules determined by the human leukocyte antigen (HLA) complex
mediate this crucial function. The generic term "major histocompatibility
complex" (MHC) has been coined for the HLA complex and its homologues
in other species. In man, the MHC occupies -4000 kb of DNA on the short
arm of chromosome 6 and contains a large number of genes encoding
molecules that serve a variety of functions. Among these molecules, a group
of glycoproteins belonging to the immunoglobulin supergene family are
present on the cell surface and play a major role in allowing the immune
system to distinguish between self and non-self. These are MHC class I
molecules (HLA-A, HLA-B, and HLA-G) and class II molecules (HLA-
DR, HLA-DQ, and HLA-DP).

2. F
Antigen presentation is carried out by specialized cells referred to as
antigen-presenting cells, and these include a diverse group of leukocytes
such as monocytes, macrophages, dendritic cells, and B cells. These cells are
found primarily in the solid lymphoid organs and the skin. Follicular
dendritic cells are specialized antigen-presenting cells in the B-cell areas of
lymph nodes and the spleen. Peripheral-tissue dendritic cells engulf and
process antigen and then leave the tissues and home to T-cell areas in
draining lymph nodes or the spleen. The predominant antigen-presenting
cells of the skin are Langerhans cells, which are found in the epidermis and
deliver antigens entering the skin to the effector cells of the lymph nodes. In
the lymph nodes, these antigen-presenting cells can directly present
processed antigens to resting T cells to induce their proliferation and
differentiation. Monocytes-macrophages exist as monocytes in blood and as
macrophages (a more differentiated form) in various tissues such as the
lungs, liver, and brain. In addition to phagocytic and cytotoxic functions,
these cells have receptors for various cytokines (IL-4, IFN a) that can serve

54
BY DR. MOHAMMED ATIAA KAREEM ALNASHY
-------------------------------------------------------------------
to regulate their function. All antigen-presenting cells have MHC class II
surface molecules.

3. B, C, E
The activities of GD4+ cells are largely mediated by way of the secretion of
cytokines, which are small protein hormones that function in controlling
the growth and differentiation of cells in the microenvironment. The
pattern of cytokine secretion of Th cells allows their further subdivision into
Thl and Th2 cells. Thl cells elaborate inflammatory cytokines involved in
effector functions of cell-mediated immunity, such as IL-2 and IFN-a,
whereas Th2 cells elaborate cytokines such as IL-4, IL-5, and IL-13 that
control and regulate antibody responses. Some GD4+ cells, capable of
secreting both Thl- and Th2-type cytokines, are sometimes designated ThO
and may be the precursors of fully differentiated Thl and Th2 cells.
Differentiation into Thl vs Th2 cells is regulated by positive feedback loops
promoted primarily by IL-12 in the case of Thl cells and IL-4 in the case of
Th2 cells. In addition to their central role in initiating and regulating
immune responses, GD4+ T lymphocytes are important effectors of cell-
mediated immunity by virtue of the cytokines that they elaborate. These
cytokines, particularly IFN-a, are essential contributors to the generation of
chronic inflammatory responses characterized by mononuclear cellular
infiltration and activated macrophages. The cytokine profile observed after
allergen provocation of allergic individuals supports the involvement of
Th2-type lymphocytes in the allergic reaction. Because IL-5 promotes the
differentiation, vascular adhesion, and in vitro survival of eosinophils, as
well as enhances histamine release from basophils, and because IL-4 is a
mast cell growth factor and also promotes the switching of B cells to the
production of IgE, Th2-like T cells are thought to be particularly important
in allergic disease.

4. A
Each of the antibodies contributes differently to the human defense system.
IgM is the predominant class formed on initial contact with antigen
(primary immune response). It is confined mostly to the intravascular
compartment and can efficiently bind antigen and activate complement. The
synthesis of IgM is much less dependent than that of other isotypes on the
activity of T lymphocytes. Certain antigens are capable of stimulating IgM

55
BY DR. MOHAMMED ATIAA KAREEM ALNASHY
-------------------------------------------------------------------
production by B cells in a T-cell-independent fashion, and the resultant
immune response is usually restricted to the IgM iso-type and does not
exhibit immunologic memory. IgG is the most abundant immunoglobulin in
the serum and the principal antibody generated during the secondary
immune response. Because of its capacity to activate complement and the
expression on phagocytes of FC receptors, IgG is regarded as the most
important antibody of memory immune responses. Furthermore, IgG is the
only iso-type that is actively transported across the placenta, providing
newborns with a full repertoire of maternal IgG antibodies. These maternal
antibodies provide the neonate with antibody protection during the early
months of life. IgA is present as a dimer in tears, saliva, and the secretions
of the respiratory, gastrointestinal, and genitourinary systems and is
relatively resistant to enzymatic digestion. It is also abundant in colostrum
and provides passive immunity to the gastrointestinal system of nursing
newborns. It does not fix complement by the antibody-dependent pathway
and does not promote phagocytosis. IgA contributes to the defensive
functions of the immune system by preventing a breach of the mucous
membrane surface by microbes and their toxic products. Finally, IgE is
important in immediate-type hypersensitivity reactions and in host defenses
against parasitic infestation. The latter role is accomplished both by the
direct toxic effects of mast cell and basophil mediators and by the potent
stimulatory effects of T cells and mast cell products such as IL-5 in
promoting eosinophilia and attracting eosinophils to the local environment.
These, in turn, contribute to the eradication of parasitic infestation by
releasing mediators with parasite-toxic properties.

5. E
Eosinophils secrete cationic granule proteins that include major basic
protein (MBP), eosinophil peroxidase (EPO), eosinophil cationic protein
(EGP), and eosinophil-derived neurotoxin (EDN). Another prominent
constituent protein of the eosinophil is the Charcot-Leyden crystal (GLG)
protein, which constitutes an estimated 7% to 10% of total cellular protein,
possesses lysophospholi-pase activity, and forms the distinctive hexagonal
bipyramidal crystals that are the hallmark of eosinophil-associated
inflammation. MBP is a potent cy to toxin and helminthotoxin in vitro. It
is capable of killing bacteria and many types of normal and neoplastic
mammalian cells, stimulating histamine release from basophils and mast
cells, activating neutrophils and platelets, and augmenting superoxide

56
BY DR. MOHAMMED ATIAA KAREEM ALNASHY
-------------------------------------------------------------------
generation by alveolar macrophages. It has also been shown to induce
bronchoconstric-tion and transient airway hyperreactivity when instilled
into the monkey trachea. As for MBP and EGP, EPO is highly cationic and
exerts some cytotoxic effects on parasites and mammalian cells in the
absence of hydrogen peroxide. However, it is highly effective in combination
with hydrogen peroxide and a halide cofactor (iodide, bromide, or chloride)
from which EPO catalyzes the production of the toxic hypohalous acid. In
the presence of these compounds, EPO is highly toxic to a variety of
unicellular, multicellular, and other targets, including viruses,
mycoplasma, bacteria, fungi, and parasites. EGP, like MBP, has marked
toxicity for helminth parasites, blood hemoflagellates, bacteria, and
mammalian cells and tissues. Purified EGP has been used in a number of
studies in which respiratory epithelial damage (epithelial stripping, mucus
plugging) similar to that seen in severe asthma has been reproduced. EDN
has been shown to induce a syndrome of muscle rigidity, ataxia, eventual
paralysis, widespread loss of Purkinje cells, and spongiform degeneration of
the white matter of the cerebellum, brainstem, and spinal cord when
injected intrathe-cally or intracerebrally into experimental rabbits or
guinea pigs. Histamine, a prominent mediator in allergic diseases, is
secreted by mast cells and basophils but not eosinophils.

57
BY DR. MOHAMMED ATIAA KAREEM ALNASHY
-------------------------------------------------------------------
CHAPTER TEN HEAD AND NECK MANIFESTATIONS OF HUMAN
IMMUNODEFICIENCY VIRUS INFECTION
-------------------------------------------------------------------------------------------
1. Which of the following statements regarding HIV replication is false?
A. The reverse transcriptase enzyme is a critical enzyme for viral
replication that is targeted by antiretroviral medications.
B. HIV typically infects and replicates in every cell in the body.
C. Viral proteases are critical for viral replication and are targeted by
antiretroviral medications.
D. A combination of error-prone transcription and prolific
replication results in vast genetic diversity.

2. Which of the following statements regarding cervical adenopathy in


HIV-positive patients is true?
A. Open biopsy should be performed in all HIV-positive patients with
cervical adenopathy.
B. Fine-needle aspiration is unreliable in the setting of HIV.
C. Idiopathic follicular hyperplasia is the most common cause of
cervical adenopathy in HIV-positive patients.
D. In HIV-positive patients, cervical adenopathy most commonly
occurs in the anterior triangle.

3. Which of the following malignancies are most highly associated with


HIV-positive patients?
A. Non-Hodgkin's lymphoma.
B. Kaposi's sarcoma.
C. Hodgkin's lymphoma.
D. Squamous cell carcinoma of the head and neck.

58
BY DR. MOHAMMED ATIAA KAREEM ALNASHY
-------------------------------------------------------------------
4. Which of the following statements regarding HIV infection and
sinusitis is false?
A. HIV-positive patients and the general population report similar
rates of sinonasal symptoms.
B. Pseudomonas and fungal infections may rapidly progress to life-
threatening infections and should be treated aggressively.
C. In HIV-positive patients with sinusitis, sphenoid involvement is
seen at nearly double the rate of that in the general population.
D. Surgical intervention is reserved for complications of sinusitis or
life-threatening infection.

5. Which of the following statements regarding occupational exposure


and HIV infection is true?
A. Postexposure prophylaxis is universally effective.
B. Most surgeons follow universal precautions.
C. Inexperienced surgeons are most likely to have injuries from sharp
instruments.
D. The rate of seroconversion after a needles tick from an HIV-
positive patient is greater than 1%.

59
BY DR. MOHAMMED ATIAA KAREEM ALNASHY
-------------------------------------------------------------------
CHAPTER 10 ANSWERS KEY
Head and Neck Manifestations of Human Immunodeficiency Virus Infection
1. B
HIV infection requires the virus to bind to a GD4 receptor and thus most frequently
infects GD4+ T-lymphocytes and macrophages. The viral proteases and the reverse
transcriptase enzyme are unique from human enzymes and critical for viral replication
and thus have been targeted for antiretroviral therapy. Transcription errors and a
prolific rate of replication create a vast pool of genetic diversity that allows the virus to
evade the immune system and develop resistance to antiretroviral medications.

2. C
Idiopathic follicular hyperplasia is the most common cause of cervical adenopathy in
HIV-positive patients. This is typically seen in the setting of peripheral generalized
lymphadenopathy (PGL). FNA should be the first line of tissue sampling, and open
biopsy should be considered in cases in which an FNA is nondiagnostic. The posterior
triangle is the most common location of HIV-associated cervical adenopathy.

3. A
Increased incidence of Kaposi's sarcoma and both types of lymphoma have been shown
in multivariate analysis to be correlated with HIV infection. Although squamous cell
carcinoma seems to have a more aggressive course in HIV-positive patients, an increased
incidence was not seen in multivariate analysis.

4D
Surgical intervention should be considered in HIV-positive patients who have symptoms
refractory to medical management, not just in those with complications or life-
threatening illness. Although HIV-positive patients seem to have a similar rate of
sinonasal complaints compared with the general population, HIV-positive patients with
sinusitis have an increased rate of sphenoid involvement. Pseudomonas and fungi are
particularly aggressive pathogens in HIV-related sinusitis.

5. C
Inexperienced surgeons such as medical students and junior residents are more likely to
have sharp injuries than more experienced surgeons. Gases of seroconversion have been
documented despite the use of postexposure prophylaxis. Some studies suggest that only
16% of surgeons follow universal precautions. The rate of seroconversion after a needle
stick is estimated at 0.3%. The risk is increased with hollow-bore needles or devices that
are visibly bloody.

60
BY DR. MOHAMMED ATIAA KAREEM ALNASHY
-------------------------------------------------------------------
CHAPTER ELEVEN: SPECIAL CONSIDERATIONS IN MANAGING
GERIATRIC PATIENTS
-------------------------------------------------------------------------------------------
1. The most common manifestation of vestibular dysfunction in the
elderly is
A. Loss of balance
B. Oscillopsia.
C. Benign paroxysmal positional vertigo.
D. Meniere's disease.
E. Labyrinthine fistula.

2. The most significant complication of presbyastasis is.


A. Automobile accidents.
B. Hip fractures.
C. Depression.
D. Impaired activities of daily living.
E. Benign paroxysmal positional vertigo.

3. Changes in nasal physiology associated with aging include


A. Reduction in nasal cilia.
B. Increase in mucociliary clearance.
C. Decrease in olfactory epithelium.
D. Decrease in nasal airflow resistance.
E. General decrease in autonomic function.

4. Decrease in fundamental frequency of voice in aging women most


often is associated with
A. Edema caused by general endocrine changes.
B. History of previous intubation.
C. Benign essential tremor.
D. Cricoarytenoid joint fixation.
E. Increase in tissue hyaluronic acid content.

5. Factors associated with presbydysphonia include all of the following except.


A. Diminished lung vital capacity.
B. Use of antidepressants.
C. Prior intubation.
D. Decreased salivary secretory rate.
E. Laryngeal elevation caused by calcification

61
BY DR. MOHAMMED ATIAA KAREEM ALNASHY
-------------------------------------------------------------------
CHAPTER 11
Special Considerations in Managing Geriatric Patients
1. A
2. B
3. C
4. A
5. E

62
BY DR. MOHAMMED ATIAA KAREEM ALNASHY
-------------------------------------------------------------------
CHAPTER TWELVE: GENETICS AND OTOLARYNGOLOGY
-------------------------------------------------------------------------------------------
1. Which of the following statements is true?
A. Introns are noncoding parts of the gene that are excised before
transcription.
B. Genes are transcribed from the 3' to the 5' end of DNA.
C. Regulatory elements within the gene act primarily to signal when
translation into protein begins and ends.
D. The wobble nucleotide refers to the third nucleotide in a codon,
which can vary for most amino acids.
E. The genetic distance between two genes is always directly
proportional to its physical distance.

2. Which of the following genetic diseases is not correctly classified?


A. Turner's syndrome: aneuploidy
B. Treacher-Collins syndrome: autosomal recessive
C. Branchiootorenal syndrome: autosomal dominant
D. Mohr-Tranebjaerg's syndrome: X-linked recessive
E. Kearns-Sayre's syndrome: mitochondrial disorder

3. In autosomal-recessive disorders
A. An affected man cannot transmit the gene to his son.
B. When normal parents have an affected child, the chance of the
disorder affecting any other children they have is 50%.
C. The abnormal gene is found in higher frequency than would be
expected considering the relative rarity of the disorder.
D. The mechanism of haploinsufficiency is important in influencing
phenotype.
E. Twice as many females as males are affected.

63
BY DR. MOHAMMED ATIAA KAREEM ALNASHY
-------------------------------------------------------------------
4. Which of the following statements regarding genetic expressivity is true?
A. X-linked genes tend to exhibit more variable expressivity in
females than in males.
B. In males, variability in gene expressivity is partly due to a
phenomenon known as Lyon's hypothesis.
C. The expressivity of recessive disorders is usually more variable
than that of dominant disorders.
D. A gene that is not penetrant can still have variable degrees of
expressivity.
E. Variability in gene expression cannot be affected by other genes.

5. Which of the following regarding DNA testing is false?


A. DNA chips can be used to detect single-base differences in DNA.
B. Polymerase chain reaction amplifies target DNA sequences for
further analysis.
C. All modern molecular genetic testing begins with Southern
hybridization of genomic DNA.
D. Heteroduplex testing is one method of screening DNA for
mutations.
E. Novel mutations cannot be detected by DNA chips.

64
BY DR. MOHAMMED ATIAA KAREEM ALNASHY
-------------------------------------------------------------------
CHAPTER 12 Genetics and Otolaryngology

1. D
Introns are noncoding DNA within a gene that are excised from the genetic
message after transcription has occurred. Genes are transcribed from the 5'
to the 3' end. Regulatory elements within the gene influence the rate of
transcription and cellular specificity of gene action. Most amino acids are
associated with more than one codon, and because it can vary for most amino
acids, the third nucleotide is referred to as the wobble nucleotide. The
genetic distance between two genes reflects the frequency of observed
combinations between them and is only imperfectly correlated with its
physical distance or the number of bases between two genes.

2. B
Treacher-Collins syndrome is a monogenic disorder inherited in an
autosomal-dominant fashion. The other disorders listed are correctly
classified with its mode of inheritance.

3. C
In autosomal-recessive disorders, two abnormal copies of the same gene are
required for an individual to be affected. An affected man can transmit the
gene to his son in autosomal-recessive disorders, but not in X-linked
recessive disorders. The chance that two heterozygous parents will have an
affected child is 25%. Although autosomal-recessive disorders are relatively
rare, the abnormal gene is seen in higher frequency in the population,
residing mostly with asymptomatic carriers. Haploinsufficiency, in which
the inactivation of one gene results in an insufficient level of gene product
to maintain normal cellular function, influences pheno-type in autosomal-
dominant disorders. In many autosomal-recessive disorders, heterozygous
carriers are asymptomatic, and the mechanism of haploinsufficiency does
not influence phenotype. Females and males are equally affected in
autosomal-recessive disorders.

65
BY DR. MOHAMMED ATIAA KAREEM ALNASHY
-------------------------------------------------------------------
4. A
Expressivity refers to the severity of the phenotype seen in genetic diseases.
X-linked disorders tend to have more variable expression in females than in
males. One reason for this increased variability of expressivity in females is
due to the random inactivation of one X chromosome early during
development, a phenomenon known as Lyon's hypothesis. The expressivity
of recessive disorders is more consistent than the expressivity of dominant
ones. A gene that has no penetrance has, by definition, zero expression.
Variability in gene expression implies the existence of mechanisms by which
the severity of the disorder can be influenced; such mechanisms may include
expression of background genes or environmental effects.

5. C
DNA chips have oligonucleotides of known sequences arrayed on a chip,
such that homologous RNA or DNA can be detected. DNA chips can be used
both to analyze patterns of gene expression and to detect single-base changes
in DNA. DNA chip technology is limited in that it cannot be used to detect
a novel mutation. Polymerase chain reaction (PCR) amplifies a targeted
sequence of DNA by use of oligonucleotide primers complementary to the 5'
and 3' ends of the desired DNA fragment. Southern hybridization, in which
radiolabeled DNA probes are hybridized with DNA on a stable membrane
support (such as filter paper), has limited use in modern molecular genetics
testing, but it is still useful for analysis of large DNA fragments.
Heterozygous mutations in DNA can be detected by the formation of a
heteroduplex-two strands of DNA with mismatched bases-when the DNA
fragment in question is amplified, heated, and allowed to anneal with itself.

66
BY DR. MOHAMMED ATIAA KAREEM ALNASHY
-------------------------------------------------------------------
CHAPTER THIRTEEN: FUNDAMENTALS OF MOLECULAR
BIOLOGY AND GENE THERAPY
-------------------------------------------------------------------------------------------
1. Which of the following vectors are not useful for gene therapy?
A. Adenovirus
B. Plasmids
C. Herpesvirus vectors
D. Coronavirus vectors

2. Gene therapy may be useful for the following applications except


A. Treatment of head and neck cancer
B. Prevention of spiral ganglion degeneration
C. Treatment of obstructive sleep apnea
D. Treatment of anosmia

3. The three principal genetic components of retroviruses include all except


A. env
B. gag
C. lat
D. pol

4. Gene therapy vectors can be used to


A. Deliver therapeutic genes
B. Decrease expression of targeted genes
C. Change a cell's phenotype
D. All of the above
E. None of the above

5. Which cell type should be excluded from standard gene delivery


protocols?
A. Neurons
B. Muscle
C. Germ cells
D. Lymphoid tissue

67
BY DR. MOHAMMED ATIAA KAREEM ALNASHY
-------------------------------------------------------------------
CHAPTER 13
Fundamentals of Molecular Biology and Gene Therapy
1. D
2. C
3. C
4D
5. C

68
BY DR. MOHAMMED ATIAA KAREEM ALNASHY
-------------------------------------------------------------------
CHAPTER FOURTEEN: MOLECULAR BIOLOGY OF HEAD AND
NECK CANCER
-------------------------------------------------------------------------------------------
1. Head and neck malignancy arises as an alteration in
A. RNA
B. DNA
C. Protein
D. mRNA
E. tRNA

2. Malignancy can develop as a single genetic event or as an


accumulation of genetic alterations. What evidence supports the
notion that head and neck cancer occurs as a result of multiple genetic
alterations?
A. A 20- to 25-year latency period
B. History of toxin exposure
C. Genetic alterations in premalignant lesions
D. Statistical modeling
E. All of the above

3. The first study demonstrating the efficacy of chemopreventive agents


used which compound?
A. Vitamin D
B. Vitamin C
C. Vitamin B12
D. Vitamin A
E. Cyclooxygenase-2 inhibitors

4. Which of the following techniques is not used to detect molecular


genetic sequence alterations in head and neck cancer?
A. Comparative genomic hybridization
B. Microsatellite analysis.
C. Promoter hypermethylation
D. Fluorescence in situ hybridization
E. DNA sequencing

69
BY DR. MOHAMMED ATIAA KAREEM ALNASHY
-------------------------------------------------------------------
5. Theoretical advantages of immunotherapy and molecular-directed
therapy include
A. Targeted therapy for tumor cells.
B. Decreased toxicity profile
C. The ability to combine with traditional surgical and medical
therapies
D. Systemic effects of therapy
E. All of the above

70
BY DR. MOHAMMED ATIAA KAREEM ALNASHY
-------------------------------------------------------------------
CHAPTER 14
Molecular Biology of Head and Neck Cancer

1. B
The basis for the known mechanisms behind the development of head and
neck cancer has been shown to be genetic in origin. DNA is the code from
which mRNA protein products arise and, therefore, is the foundation for
genetic alterations that can lead to carcinogenesis. Ultimately, these genetic
alterations lead to a malignant phenotype that can include altered cell
proliferation, invasion, metastasis, altered immunogenicity, resistance to
therapy, genetic instability, as well as other phenotypic characteristics
common to malignancy.

2. E
For all of these reasons, head and neck cancer is thought to arise as a result
of a series of genetic alterations, the sum of which leads to malignancy.
Rennan and others (1993) suggested that between 6 and 10 genetic
alterations were required for the development of head and neck carcinoma.
The detection of microsatellite alterations in premalignant lesions in a
progression model by Califano and others (1996) also indicates that there are
early, predictable changes that occur in the pathway to malignant
transformation.

3D
Chemoprevention was first studied and demonstrated efficacy in patients
with oral leukoplakias in a study by Hong and others (1986) using vitamin
A derivatives. Since then, many studies have examined the possibility of
using other chemopreventive agents to reduce the incidence of tumor
development, tumor recurrence, and tumor progression, including answers
a, b, and e.

71
BY DR. MOHAMMED ATIAA KAREEM ALNASHY
-------------------------------------------------------------------
4. C
These are just some of the techniques researchers use to detect genetic
sequence alterations in head and neck cancer cells. Promoter
hypermethylation is an epigenetic event that does not involve DNA sequence
change, or loss or gain of DNA.

5. E
Translating the known molecular alterations of tumors into practical
therapy modalities has long been the goal of many researchers. With these
very specific alterations, one could theoretically devise a treatment plan that
would target only the cells with abnormalities and preserve the normal cells.
This has proven to be very difficult, but strides are underway to create
therapies that can be used alone or in conjunction with conventional
treatment.

72
BY DR. MOHAMMED ATIAA KAREEM ALNASHY
-------------------------------------------------------------------
CHAPTER FIFTEEN: OUTCOMES RESEARCH
------------------------------------------------------------------------------------------
1. After an extensive review of the literature, an otolaryngologist finds
that the best supporting evidence for a new procedure is a case series
of 13 patients. This is an example of
A. Grade A, level 1 evidence
B. Grade B, level 2 or 3 evidence
C. Grade C, level 4 evidence
D. Grade D, level 5 evidence

2. In an observational cohort study of treatment of sinusitis, patients are


treated with either antibiotics or surgery. Failure to recognize that the
patients who receive surgical intervention generally have more severe
symptoms represents a problem with.
A. Selection bias.
B. Intervention bias.
C. Comorbidity.
D. Detection bias

3. A double-blind, randomized clinical trial for allergic rhinitis


demonstrates that rhinorrhea is reduced more often with nasal
steroids than with antihistamines. Nasal steroids are
A. More efficacious.
B. More efficient.
C. More effervescent
D. More effective

4. With respect to appropriate measurement of outcomes in a double-


blind, randomized clinical trial of hearing aids, which of the
following statement is most accurate?
A. Quality-of-life (QOL) scales are too soft for controlled studies.
B. Physiologic measures such as pure tone thresholds are the most
reliable.
C. A generic outcome measure (e.g., the Medical Outcomes Study
Short Form-36 [SF-36]) is the most sensitive.
D. A disease-specific QOL scale facilitates comparisons with other
types of treatment.
E. None of the above.

73
BY DR. MOHAMMED ATIAA KAREEM ALNASHY
-------------------------------------------------------------------
5. An otolaryngologist wishes to determine whether a history of
childhood tonsillectomy affects rates of adult atopic disease. He
assembles a group of adults with and without atopic disease and then
compares the rates of tonsillectomy by reviewing their records. This is
an example of a
A. Prospective observational study
B. Retrospective observational study
C. Case-control study
D. Case series study
E. Poorly designed study

CHAPTER 15
Outcomes Research
1. C
2. A
3. A
4. E
5. C

74
BY DR. MOHAMMED ATIAA KAREEM ALNASHY
-------------------------------------------------------------------
CHAPTER SIXTEEN: INTERPRETING MEDICAL DATA
------------------------------------------------------------------------------------------
1. A clinician reviews the medical records of all patients who had
tonsillectomy over the past 10 years and records the frequency of
primary hemorrhage. Follow-up is available for all subjects. The
hemorrhage rate is reported using
A. Prevalence, because the method of data collection is retrospective
B. Incidence, because the method of data collection is prospective
C. Prevalence, because the direction of inquiry is retrospective
D. Incidence, because the direction of inquiry is prospective
E. Survival analysis, because some observations are censored

2. When the change in hearing levels is assessed after ossicular


reconstruction, a 95% confidence interval aids in data interpretation
because it
A. Gives the range of results consistent with the data
B. Estimates the accuracy of observed results
C. Adjusts for systematic error that may have occurred when results were
measured
D. Defines the variability of observed data relative to the mean
E. Ensures adequate statistical power

3. A researcher reports that a new antibiotic is more effective than an


established standard for bacterial sinusitis, P = .015. Which of the
following statements is true?
A. The new antibiotic increased relative efficacy by 15%.
B. The new antibiotic increased absolute efficacy by 15%.
C. The likelihood of a type I statistical error (false positive) is 1.5%.
D. The likelihood of a type II statistical error (false negative) is 1.5%.
E. The likelihood of adequate statistical power is 98.5%.

75
BY DR. MOHAMMED ATIAA KAREEM ALNASHY
-------------------------------------------------------------------
4. Evidence-based medicine is defined as the judicious, explicit, and
systematic use of current best evidence in caring for individual
patients. Which statement is true concerning levels of evidence and
corresponding grades of recommendation?
A. Expert consensus can support only a grade C or D recommendation.
B. Expert consensus is unacceptable as the sole criterion for a
recommendation.
C. Grade A recommendations are required to justify surgical therapy.
D. Grade A or B recommendations are required to justify medical
therapy.
E. Levels of evidence differ for studies of therapy, diagnosis, or
prognosis.

5. The most important aspect of analyzing and interpreting medical data


is
A. Choosing the right statistical test for the right dataset
B. Recognizing uncertainty and quantifying error rates
C. Avoiding the multiple P value problem by using multivariate
techniques
D. Reporting statistical power whenever a significant P value is obtained
E. Reporting statistical power whenever a non-significant P value is
obtained

76
BY DR. MOHAMMED ATIAA KAREEM ALNASHY
-------------------------------------------------------------------
CHAPTER 16
Interpreting Medical Data

1. D
Retrospective and prospective refer to directions of study inquiry, not to
methods of data collection. A prospective study records new events
(incidence) and may be conducted by record review or by observing future
events. Survival analysis, which adjusts for censored observations, is
unnecessary because follow-up is available for all subjects.

2. A
A 95% confidence interval aids in data interpretation, because it estimates
the range of results consistent with the observed data. This permits
extrapolation of results beyond the study (inference) based on the single set
of measurements made on a limited number of study subjects. Conversely,
accuracy reflects nearness to the truth (bias) and has nothing to do with
precision or confidence intervals. Similarly, statistical power is an unrelated
concept. Variability of observed data relative to the mean is described by
standard deviation, not by confidence intervals.

3. C
When study results are statistically significant, the P value is the
probability of making a type I error: concluding the drugs have differing
efficacy when in fact they are really comparable. Alternatively, we could
state that there is only a 1.5% chance that the differences observed by the
investigators are strictly fortuitous (e.g., explainable by random error).
Relative and absolute efficacy is assessed by the relative risk and the rate
difference, respectively, and measures effect size not statistical significance.
Power is related to type II error, not to P values.

77
BY DR. MOHAMMED ATIAA KAREEM ALNASHY
-------------------------------------------------------------------
4. E
Evidence-based medicine is based on a hierarchy of research evidence, with
different classification schemes for studies of therapy, diagnosis, or
prognosis. The level of evidence is then used to derive a grade of
recommendation, which is the same for medical and surgical interventions.
Randomized trials and prospective controlled studies yield the highest level
of evidence (and recommendation grades), but expert consensus yields only
the lowest level of evidence (grade D recommendation). Expert consensus,
however, is an acceptable basis for recommendations if higher quality
studies are unavailable, unfeasible, or unethical.

5D
Effective interpretation of medical data is a systematic process of moving
from observations to generalizations with predicable degrees of certainty
(and uncertainty). The most important part of the process is recognizing the
inherent variability in all biologic systems and the inevitable uncertainties
in related measurements and observations. Because error can never be
avoided, it is estimated with confidence intervals, P values (rates of type I
error), and power calculations (rates of type II error). Choosing the right
statistical test and avoiding multiple P values are of secondary importance.
Statistical power is irrelevant when a significant P value is obtained.

78
BY DR. MOHAMMED ATIAA KAREEM ALNASHY
-------------------------------------------------------------------
CHAPTER SEVENTEEN: PAIN MANAGEMENT IN THE HEAD
AND NECK PATIENT
-------------------------------------------------------------------------------------------
1. Which of these medical therapies is not effective in treating
neuropathic pain?
A. Oxycodone
B. Ibuprofen
C. Carbamazepine.
D. Lidocaine
E. Amitriptyline

2. Which specific nerve block can lead to the greatest morbidity after a
misplaced injection?
A. Sphenopalatine ganglion
B. Maxillary nerve
C. Mandibular nerve
D. Glossopharyngeal nerve
E. Stellate ganglion

3. Which of the following statements regarding migraines is not true?


A. Migraine is an undertreated or inadequately treated syndrome,
because patients generally rely on over-the-counter medications.
B. Migraines may proceed through four phases: prodromal phase, aural
phase, headache phase, and resolution phase.
C. Migraines are the result of a cervical-trigeminal-vascular disorder.
D. Migraine treatment can be prophylactic, abortive, or acute.
E. Migraines are associated with lower levels of serotonin centrally.

4. Which treatment is effective in treating pain associated with


whiplash?
A. Corticosteroid injections into the cervical zygapophyseal joint
B. Percutaneous radiofrequency neurotomy
C. Single sessions of extension-retraction exercises
D. Botulinum toxin A injections into certain trigger points

79
BY DR. MOHAMMED ATIAA KAREEM ALNASHY
-------------------------------------------------------------------
5. Which of the following statements is not true?
A. Intractable chronic cluster headaches can resolve with nerve blockade
of the sphenopalatine ganglion.
B. Overactive pericranial muscles may result in a constant bandlike pain
in the forehead.
C. Regular analgesic use by patients with a history of migraine will
likely decrease the development of chronic daily headaches.
D. Ocular and frontotemporal pain provoked by certain neck movements
or pressure in the upper back are characteristics of paroxysmal
hemicrania.
E. Facet joint syndrome can be diagnosed by the patient's response to a
nerve block into the zygapophyseal joints.

80
BY DR. MOHAMMED ATIAA KAREEM ALNASHY
-------------------------------------------------------------------
CHAPTER 17
Pain Management in the Head and Neck Patient

1. B
Neuropathic pain can be treated with anticonvulsant agents such as
carbamazepine, local anesthetics such as lidocaine, tricyclic antidepressants
that decrease the emotional depression that amplifies pain, and opioids.
Nonsteroidal antiinflammatory drugs such as ibuprofen are used to treat
mild pain associated with inflammation. NSAIDs alter the inflammatory
process by blocking expression of the cyclooxygenase (COX) enzymes that
mediate production of the prostaglandins that sensitize pain afferents.

2D
The proximity of the glossopharyngeal nerve to the carotid artery dictates
extreme care when performing a nerve block to avoid profound toxicity from
a misplaced injection.

3. E
Several migraine triggers include alcohol, certain foods, changes in
hormonal levels, stress, and sleep patterns. Serotonin levels have been found
to be higher centrally and lower peripherally during migraines.

4B
Multiple radiofrequency lesioning of target nerves has been found to reduce
pain in patients with cervical zygapophyseal joint pain or whiplash. No
evidence supports the effectiveness of single sessions of extension-retraction
exercises or corticosteroid injections in relieving pain. Botulinum toxin A
injections have been shown to lead to a trend toward improved function, but
not in treating pain.

81
BY DR. MOHAMMED ATIAA KAREEM ALNASHY
-------------------------------------------------------------------
5. C
Regular analgesic use has been implicated as a cause of chronic headache.
Regular analgesic use will likely lead to chronic daily headaches in patients
with a history of migraine. Intractable chronic cluster headaches can resolve
with blockade of the trigeminal ganglion or the sphenopalatine ganglion.
Overactive pericranial muscles may play a role in the pathophysiology of
chronic tension headaches that consist of a constant bandlike pain that is
bilateral and contained in the forehead. Paroxysmal hemi-crania is a
unilateral headache characterized by excruciating pain in the ocular and
frontotemporal area that is provoked by certain neck movements and
pressure in the upper back. Facet joint syndrome can be differentiated by
the response to radiographically guided injections of local anesthetics into
the zygapophyseal joints or around the dorsal medial branches of the
posterior primary rami.

82
BY DR. MOHAMMED ATIAA KAREEM ALNASHY
-------------------------------------------------------------------
CHAPTER EIGHTEEN: INTEGRATING PALLIATIVE AND
CURATIVE CARE STRATEGIES IN THE PRACTICE OF
OTOLARYNGOLOGY
-------------------------------------------------------------------------------------------
1. Palliative care is a comprehensive approach to treating serious illness
that focuses on patients'
A. Physical needs
B. Psychological needs
C. Social needs
D. Spiritual needs
E. All of the above

2. True/False. Palliative care and end-of-life care are synonymous


.
3. True/False. Palliative care and curative therapies are incompatible,
and palliative care should be pursued only when it is clear that
curative strategies are futile.

4. A patient with advanced but treatable cancer wants to refuse


treatment—a choice that seems irrational based on the current
literature. Which of the following is a potential reason for the
patient's refusal?
A. Fear of disfigurement
B. Fear of pain
C. Fear of death
D. Fear of isolation
E. All of the above

5. Which of the following actions is least consistent with effective


negotiation strategies to persuade the patient to reconsider?
A. Promising the patient that the treatment will cure the cancer
B. Offering to respect the patient's choice
C. Allowing the patient time to voice concerns
D. Setting up another meeting at another time to talk again
E. Assessing whether the patient has understood enough about the
condition and treatment options to make an informed refusal

83
BY DR. MOHAMMED ATIAA KAREEM ALNASHY
-------------------------------------------------------------------
CHAPTER 18
Integrating Palliative and Curative Care Strategies in the Practice of
Otolaryngology
1. E
2. B
3. B
4. E
5. A

84
BY DR. MOHAMMED ATIAA KAREEM ALNASHY
-------------------------------------------------------------------
CHAPTER NINETEEN: GPAPHICS AND DIGITAL IMAGING FOR
OTOLARYNGOLOGISTS
-------------------------------------------------------------------------------------------
NO REVIEW QUESTIONS NECESSARY FOR THIS TOPIC

85
BY DR. MOHAMMED ATIAA KAREEM ALNASHY
-------------------------------------------------------------------
CHAPTER TWENTY: MEDICAL INFORMATICS AND
TELEMEDICINE
-------------------------------------------------------------------------------------------
1. Which of the following statements about literature searches is false?
A. The quality of the literature search can be assessed by search precision
and recall.
B. You should take full advantage of the provided training materials to
obtain the best possible search results.
C. Thorough literature searches should be performed only by a trained
medical librarian.
D. Your topic should be systematically defined and a specific aspect
chosen, especially if the topic is broad.
E. You should take the time to become familiar with the Medical Subject
Headings terms (MeSH).

2. All of the following statements about information technology systems


are true except
A. Information technology systems are readily used at the institutional
level by local area networks but have less value for individuals and
society.
B. They are facilitated by physician-oriented web portals.
C. They are facilitated by development of common technology standards.
D. They may lead to a reduction in errors, potentially saving patients'
lives.

3. In describing patient use of the Internet, all of the following are true
except
A. The Internet allows patients to get a virtual second opinion in some
cases.
B. The Internet usurps the physician position as the most authoritative
source of medical information.
C. The Internet allows patients to gather information before visiting
their physician.
D. The age of information has empowered the patient, as well as the
doctor.

86
BY DR. MOHAMMED ATIAA KAREEM ALNASHY
-------------------------------------------------------------------
4. Which of the following statements is true regarding telemedicine?
A. Telemedicine is already integrated into many facets of the practice of
medicine.
B. Telemedicine is known to be detrimental to doctor-patient interaction.
C. In a store-and-forward method, the referring physician collects all
relevant information and forwards it to the remote specialist, which
causes scheduling problems for programs that cover different time
zones.
D. Telemedicine will be integrated into the practice of medicine only in
5 to 10 years because of immature technology.
E. Telemedicine depends on live video teleconferencing.

5. Which statement about the electronic patient record (EPR) is false?


A. EPR is in a structured format that allows greater accessibility and
understanding of health conditions and medications of one's patients.
B. An EPR is simply an electronic version of the paper medical record
that stores patient information in a more efficient way.
C. An EPR adds information management tools to provide reminders
and alerts, knowledge source linkages, specialized medical decision
support, and analysis of aggregated data.
D. Integrated access to all patients' data by legitimate users is one of the
primary purposes of the EPR.

CHAPTER 20
Medical Informatics and Telemedicine
1. C
2A
3B
4. A
5B

87
BY DR. MOHAMMED ATIAA KAREEM ALNASHY
-------------------------------------------------------------------

PART TWO
FACE

88
BY DR. MOHAMMED ATIAA KAREEM ALNASHY
-------------------------------------------------------------------
CHAPTER TWENTY-ONE: AESTHETIC FACIAL ANALYSIS
---------------------------------------------------------------------------------

1. What are the anatomic landmarks that define the Frankfort


horizontal line?
A. Inferior wall of the external auditory canal to infraorbital rim
B. Root of the superior helical rim to the infraorbital rim
C. Superior tragal root to nasojugal fold
D. Superior wall of the external auditory canal to infraorbital rim

2. When the face is analyzed, the width of one eye may be used as a basic
unit of measurement that is equal to the following except
A. Intercanthal distance
B. Alar base width
C. One-fifth of the facial width
D. One-half the nasal length

3. Which of the following is a facial landmark used as a reference point


when dividing the face into thirds?
A. Rhinion
B. Nasion
C. Menton
D. Pogonion
E. Stomion

4. The following descriptions of the nasolabial angles are correct except


A. 95 to 110 degrees in women
B. 90 to 95 degrees in men
C. A measure of the nasal projection in relation to the upper lip
D. A measure of nasal tip rotation

5. All of the following regarding the analysis of the ears are correct except
A. The ear protrudes from the skull at an angle approximately 20 to 30 degrees.
B. The helix of the ear lies 15 to 25 mm lateral to the mastoid skin.
C. The long axis of the ear is parallel to the long axis of the nasal dorsum
and is noted to have a posterior rotation of approximately 15 degrees.
D. The width of the ear is approximately two-thirds its length.

89
BY DR. MOHAMMED ATIAA KAREEM ALNASHY
-------------------------------------------------------------------
CHAPTER 21 ANSWERS: Aesthetic Facial Analysis

1. D
The Frankfort horizontal is the standard reference point in which to position
the patients' head and gaze. The patient is positioned standing or sitting
upright with the legs uncrossed, hair tucked behind the ears, and jewelry
removed. The head is positioned adjusting the chin to achieve a Frankfort
line parallel to the ground and eyes in forward gaze. This reference point
helps to standardize patient photographs to achieve consistency and eliminate
variability that may exist from day-to-day with the same patient, from patient
to patient, and between different photographers.

2D
After evaluating facial symmetry, the face may be divided into fifths. The basic
unit for dividing the face vertically is the width of the eye. Each eye is one-fifth of
the total facial width. The intercanthal distance approximates the width of one eye.
Moreover, a line dropped from each medial canthus approximates the side of the
ala of the nose, making the nasal base one-fifth of the facial width.

3. C
When assessing facial height, the face is divided into thirds. The landmarks are
the trichion to glabella, from glabella to the subnasale, and from the subnasale to
the menton. A second method of assessing the facial height disregards the upper
third of the face because of the variability of the hairline. Measurements are made
from the nasion to subnasale and from the subnasale to menton representing the
midface or nasal height and the lower facial height, respectively.

4. C
The nasolabial angle defines the angular inclination of the columella with the
upper lip. In the female, the ideal angle ranges from 95 to 110 degrees, and in the
male from 90 to 95 degrees. A nasolabial angle less than that of the ideal is
described as under-rotated, and an angle greater than that of the ideal is described
as over-rotated.

5D
The width of the ear is approximately one-half its length. The superior and
inferior aspect of the ear should approximate the level of the brow and the
ala, respectively

90
BY DR. MOHAMMED ATIAA KAREEM ALNASHY
-------------------------------------------------------------------
CHAPTER TWENTY-TWO: RECOGNITION AND TREATMENT OF
SKIN LESIONS
--------------------------------------------------------------------------------------------

1. Studies indicate that up to what percent of actinic keratoses can become


squamous cell carcinoma?
A. 10%
B. 20%
C. 30%
D. 40%
E. 50%

2. What is the treatment of choice for keratoacanthomas?


A. Retinoic acid
B. Chemical peel
C. Mohs' surgical excision
D. Laser resurfacing
E. Observation

3. What is the preferred biopsy technique for melanoma?


A. Excisional biopsy
B. Shave biopsy
C. Cryotherapy
D. Curettage
E. Electrodessication

4. Treatment of hemangiomas may be indicated for all circumstances except


A. Ocular involvement
B. Airway involvement
C. Rapidly growing
D. Ulceration
E. Beginning involution

91
BY DR. MOHAMMED ATIAA KAREEM ALNASHY
-------------------------------------------------------------------
5. A patient with bifid ribs, skin lesions, frontal bossing, jaw cysts, and
calcified cerebra has
A. Xeroderma pigmentosa
B. Gardner's syndrome
C. Nevoid basal cell syndrome
D. Sturge-Weber syndrome
E. Kasabach-Merritt syndrome

CHAPTER 22

Recognition and Treatment of Skin Lesions

1B

2. C

3. A

4. E

5. C

92
BY DR. MOHAMMED ATIAA KAREEM ALNASHY
-------------------------------------------------------------------
CHAPTER TWENTY-THREE: MANAGEMENT OF HEAD AND NECK
MELANOMA
--------------------------------------------------------------------------------------------

1. Desmoplastic melanoma
A. Is most often black with asymmetry and irregular borders and is larger
than 6 mm in diameter
B. Rarely arises on the head and neck region
C. Is associated with an increased incidence of cervical metastasis
compared with other melanoma variants
D. Often arises in the setting of a lentigo malignant melanoma histologic
subtype and has a propensity for neurotropic spread
E. Has a worse prognosis even when corrected for other risk factors

2. Adjuvant radiation therapy is considered in all of the following


settings except
A. Neurotropism
B. Extracapsular spread
C. Multiple lymph node involvement
D. Tumor ulceration
E. Recurrent melanoma

3. Which of the following criteria is not implemented in the staging of


cutaneous melanoma under the 2001 American Joint Committee on
Cancer guidelines?
A. Primary tumor thickness
B. Primary tumor ulceration
C. Number of metastatic lymph nodes
D. Size of metastatic lymph nodes
E. Microscopic vs macroscopic nodal disease

93
BY DR. MOHAMMED ATIAA KAREEM ALNASHY
-------------------------------------------------------------------
4. Which of the following has higher sensitivity and specificity for
detecting micrometastasis?
A. Sentinel lymph node mapping with biopsy
B. Computed tomography scan
C. Magnetic resonance imaging
D. Positron emission tomography scan
E. Elective neck dissection

5. Which of the following criterion excludes a patient from consideration


for sentinel lymph node mapping with biopsy?
A. Previous punch biopsy of primary lesion
B. Tumor ulceration
C. Depth of invasion <1.0 mm
D. Depth of invasion >4.0 mm
E. 1-cm cervical lymph node positive for melanoma on fine-needle
aspiration

94
BY DR. MOHAMMED ATIAA KAREEM ALNASHY
-------------------------------------------------------------------
CHAPTER 23

Management of Head and Neck Melanoma

1. C

Desmoplastic melanoma (DM) accounts for 1% of all cutaneous melanoma


cases. Up to 75% of lesions arise in the head and neck region, often in the
setting of lentigo maligna and lentigo maligna melanoma. DM is a diagnostic
challenge, because many lesions lack the typical ABGD warning signs
associated with melanoma, and up to 75% of cases are amelanotic. DM has a
propensity for neurotropic spread. The 12.5% rate of regional metastasis in the
setting of DM is lower than other melanoma subtypes.

2D

Most authors advocate the use of radiation as adjuvant therapy for patients
with adverse prognostic markers such as neurotropism, extracapsular spread,
multiple lymph node involvement (>4 nodes), or recurrence. Primary tumor
ulceration alone is not a common reason for adjuvant radiation.

3D

The two most important factors in T classification of localized disease are


primary tumor thickness and ulceration. The number of metastatic lymph
nodes, as opposed to the gross size of metastatic lymph nodes, is used to define
the N category. Delineation of occult (microscopic) nodal metastasis vs
clinically/radiographically apparent (macroscopic) nodal metastasis is also
made within the N category. In doing so, the AJCC has now incorporated the
technique of sentinel lymph node mapping with biopsy (SLNB) into the new
staging system.

95
BY DR. MOHAMMED ATIAA KAREEM ALNASHY
-------------------------------------------------------------------
4. A

Sentinel lymph node mapping with biopsy (SLNB) is considered the method
of choice for staging of regional nodal basins. Both the sensitivity and
specificity for detecting micrometastasis is higher with SLNB than
radiographic studies such as GT scan, MRI, and PET scan. The SLNB
technique provides the pathologist with a limited number of nodes to
thoroughly evaluate with serial sectioning, hematoxylin and eosin staining,
and melanoma-specific immunohistochemistry if warranted. Therefore, the
histologic analysis of sentinel lymph nodes is more thorough and complete
than traditional evaluation of the entire lymphadenectomy specimen from an
elective neck dissection.

5. E

Patients with primary melanoma tumors measuring >1 mm in thickness


should be considered for SLNB. Patients with primary melanoma tumors
measuring <1 mm in thickness should also be considered if they have poor
prognostic indicators such as tumor extension to the deep margin, ulceration,
extensive regression to 1.0 mm, young age, high mitotic rate, or Glark level
IV invasion. Patients with regional metastasis warrant a therapeutic lymph
node dissection and, therefore, are not deemed candidates for the SLNB
staging procedure. Other exclusion criteria include distant metastasis,
previous surgical disruption of the lymphatics and previous resection of the
primary tumor with wide margins. Punch biopsy alone will not limit the
ability to perform SLNB.

96
BY DR. MOHAMMED ATIAA KAREEM ALNASHY
-------------------------------------------------------------------
CHAPTER TWENTY-FOUR: SCAR REVISION AND CAMOUFLAGE
--------------------------------------------------------------------------------------------

1. Which of the following statements regarding scar re-excision is true?


A. Re-excision with primary closure is an irregularization procedure.
B. Sharp excision of the scar should proceed with the scalpel exactly
parallel to the skin surface.
C. Direct closure of the area should be performed without undermining.
D. Vertical mattress sutures are an effective way to properly evert the
wound edges.

2. Which of the following patients is most amenable to surgical scar


revision surgery?
A. A 2-month-old with a slightly depressed scar of the forehead 2 weeks
after a fall that resulted in closure of a laceration in the emergency
department
B. A patient 6 weeks after a bilobed flap repair of the nasal sidewall with
pincushioning and edema
C. A patient 6 months after a motor vehicle accident with depressed scar
along the cheek and malar prominence area
D. A patient with a red scar in the preauricular crease 2 months after
harvest of a preauricular full-thickness skin graft

3. Which of the following statements regarding tissue expansion is false?


A. Rectangular expanders provide the greatest expansion.
B. In general, the base of an expander should be approximately 2.5 to 3.0
times as large as the area to be reconstructed.
C. The effects of tissue expansion on skin include epidermal thinning,
decreasing melanin production, and derma thickening.
D. Expansion can proceed until the skin blanches or the patient complains
of discomfort.
E. Tissue expanders can cause impressive disfigurement of the head and
neck, which naturally results in significant emotional stress to the
patient.

97
BY DR. MOHAMMED ATIAA KAREEM ALNASHY
-------------------------------------------------------------------
4. Regarding dermabrasion, which is true?
A. Wire fraises are ideal for the neophyte surgeon.
B. Prophylaxis for herpetic outbreak is unnecessary in dermabraded
patients.
C. Dermabrasion is performed best 6 to 8 weeks after surgical scar
revision.
D. The dermabrasion bit should be moved parallel to the direction of
rotation of the bit.
E. Dermabrasion should be performed through the layer of the reticular
dermis.

5. Which of the following statements regarding scar irregularization is true?


A. Irregularization is not an effective way to camouflage a raised scar that
falls between two facial aesthetic units.
B. A classic 30-degree Z-plasty will increase the length of the final scar
by 25%.
C. The limbs of the triangles in W-plasty should be between 2 and 4 mm
in length.
D. Geometric broken line closure is a poor choice of technique for long
linear scars that do not fall within relaxed skin tension lines.
E. Dermabrasion is rarely used after scar irregularization.

98
BY DR. MOHAMMED ATIAA KAREEM ALNASHY
-------------------------------------------------------------------
CHAPTER 24: Scar Revision and Camouflage

1. D

When re-excision with direct primary closure is undertaken for an unsightly


scar, the following principles should be followed. This is not an
irregularization procedure such as geometric broken line closure or running
W-plasty. The purpose is to close a wound using soft tissue principles that
will help form an optimal scar. This includes beveling of the scalpel slightly
outward to prepare for wound edge eversion, undermining of 1 to 2 cm of
surrounding skin, and closure with interrupted vertical mattress sutures to
evert the wound edges.

2. C

A depressed scar along the cheek/malar prominence area is a scar favorable


for improvement through surgical scar revision. If the scar's main
unfavorable characteristic is that it is depressed, then re-excision with
primary closure is indicated. If the scar is depressed and also not in alignment
with relaxed skin tension lines, then re-excision combined with a scar
irregularization technique (geometric broken line closure, W-plasty) would
be indicated likely followed by dermabrasion 6 to 8 weeks later. A 2-month-
old with a scar 2 weeks after closure should be followed to see how the wound
appears after 6 to 12 months of maturation. An edematous bilobed flap 6
weeks out is normal and does not need surgical scar revision but would likely
benefit from intradermal steroid injection. A red scar in the preauricular
area 2 months out is likely undergoing normal healing, and the erythema
usually fades with increased scar maturation and does not need surgical scar
revision.

99
BY DR. MOHAMMED ATIAA KAREEM ALNASHY
-------------------------------------------------------------------
3. C

All of the statements are true regarding tissue expansion except that during
expansion the currently accepted understandings on the fate of tissue layers
during tissue expansion are: Epidermis is thickened, melanin production
increases, mitotic activity is increased, dermis is thinned (30%-50%), collagen
synthesis is enhanced, hair follicle number remains unchanged, hair density
decreases, muscle thins and can atrophy, and blood vessels proliferate.

4. C

Dermabrasion is best performed at 6 to 8 weeks. Wire fraises are best reserved


for experienced dermabraders, and diamond fraises are best suited for
neophyte dermabraders. Prophylaxis against herpetic outbreak should be
provided to any patient with a history of previous herpetic outbreak. When
performing dermabrasion, one should move the bit perpendicular to the axis
of rotation of the bit. Dermabrasion should proceed into the papillary dermis.

5B

Irregularization would not be the surgical scar revision technique of choice


for an unsightly scar that falls between two facial aesthetic units. In this
situation, re-excision with primary closure or intrale-sional steroids would
be a reasonable alternative. The limbs of a W-plasty should be 5 to 7 mm in
length. Shorter limbs undergo contraction and are too small to create
sufficient irregularization. Geometric broken line closure is an excellent
choice for a long linear scar that does not fall within relaxed skin tension
lines. Dermabrasion should be routinely offered to patients 6 to 8 weeks after
scar irregularization to obtain maximum camouflage.

100
BY DR. MOHAMMED ATIAA KAREEM ALNASHY
-------------------------------------------------------------------
CHAPTER TWENTY-FIVE: FACIAL TRAUMA: SOFT-TISSUE
LACERATIONS AND BURNS
--------------------------------------------------------------------------------------------
1. Which of the following statements regarding facial laceration repair is true?
A. Extensive soft-tissue debridement should be undertaken before repair
of a simple laceration.
B. Sutures should be left in place for 7 to 10 days to achieve optimal
cosmetic results.
C. Obtaining an accurate clinical history regarding the facial injury is of
little value in making treatment decisions.
D. Thorough removal of debris from a wound before primary closure will
help prevent debris tattooing.
E. Repair of facial lacerations is best accomplished with 3-0 or 4-0 sutures.

2. Injury to Stensen's duct as a result of a deep laceration to the cheek


A. Is rarely associated with an injury to the buccal branch of the facial nerve
B. Can often be confirmed by cannulating the duct intraorally with a
lacrimal probe
C. Is easier to repair in a delayed fashion after the wound has matured
after primary closure
D. Never results in salivary fistula or sialocele, even if the laceration is
not repaired
E. Is best repaired with 4-0 or 5-0 absorbable sutures

3. Which of the following statements regarding facial laceration repair is false?


A. A soft-tissue avulsion should immediately be covered with a full-thickness skin
graft.
B. In patients who will not likely return for suture removal or in children who
may not be cooperative with suture removal, the use of fast-absorbing plain gut
is acceptable.
C. TAG (tetracaine [0.5%], adrenaline [1:2000], cocaine [11.8%]) applied topically
to a laceration will minimize the pain of injected anesthetics and may obviate
the need for any injection of anesthetics.
D. Special consideration should be given to lacerations that extend through an
anatomic margin (i.e., nostril margin or vermillion border of the lip) for
appropriate realignment.
E. Neutralization of acidic injectable anesthetics with sodium bicarbonate can
decrease the discomfort associated with the injection.

101
BY DR. MOHAMMED ATIAA KAREEM ALNASHY
-------------------------------------------------------------------
4. Which of the following statements regarding facial burns is true?
A. Early, aggressive soft-tissue debridement is recommended in an oral
commissure electrical burn.
B. Tarsorrhaphy is required in all thermal injuries to the periorbital
region.
C. The sooner facial burns are sealed by spontaneous re-epithelization or
skin grafting, the better the ultimate cosmetic and functional outcome
will be.
D. In general, topical antimicrobial agents are contraindicated in facial
burns.
E. If a burn spontaneously heals at 6 to 8 weeks, there will be little
scarring, and skin quality will be excellent.

5. A painful facial burn with significant erythema and blistering is best


classified as a
A. Full-thickness burn
B. Third-degree burn
C. First-degree burn
D. Second-degree burn

102
BY DR. MOHAMMED ATIAA KAREEM ALNASHY
-------------------------------------------------------------------
CHAPTER 25: Facial Trauma: Soft-Tissue Lacerations and Burns

1. D

The phenomenon of debris tattooing results when particulate matter gets


embedded in the dermis and the epidermis heals over it. The best treatment
for debris tattooing is prevention. Therefore, at the time of primary repair,
foreign material should be aggressively removed from the wound by scrubbing
and irrigation. After debris tattooing has resulted, full-thickness excision is
the most effective treatment. With simple facial lacerations, no soft tissue
debridement is usually necessary. Fine (nothing larger than 6-0),
nonabsorbable, monofilament suture should be used, and the sutures should
be removed in 3 to 5 days to obtain the optimal cosmetic result.

2B

Although injury to Stensen's duct is uncommon, an unrecognized and


unrepaired duct can result in a sialocele or salivary fistula that can easily be
avoided with repair. The most common sign of injury is saliva draining from
the wound, but sometimes the injury may be subtle. In these cases, the best
way to confirm injury is by cannulating the duct intraorally with a lacrimal
probe. Repair is best performed with 8-0 or 9-0 nylon suture under
magnification and possibly using a silastic stent left in place for 7 to 10 days
after repair.

3. A

Placement of a skin graft in the acute setting when facial soft tissue has been
lost stops secondary healing, which results in a poor tissue match and a
noticeable contour defect. Therefore, skin grafts should be avoided in these
situations, unless there is massive tissue loss or third-degree burns. Instead,
the wound should be left to stabilize, and, if needed, a local or regional flap
may be planned in a delayed fashion. This will likely result in a better
outcome.

103
BY DR. MOHAMMED ATIAA KAREEM ALNASHY
-------------------------------------------------------------------
4. C

If a burn spontaneously heals within 10 to 14 days, then scarring will be


minimized and skin quality will be good. After 14 days, the chances of
unsatisfactory hypertrophic scarring increase dramatically. Therefore, at
approximately 10 days, the burn should be closely evaluated for its viability,
and if at this time healing seems to be delayed, then eschar excision and skin
grafting should be performed. Oral commissure burns often result from a
child biting an electrical cord. Injury occurs from the current passing
through the tissues rather than from heat on the skin surface; therefore, the
initial injury may seem limited. Tissue necrosis, however, may progress over
the next 36 to 48 hours. Because of the difficulty distinguishing viable from
nonviable tissue, early debridement is generally not recommended. With
periorbital burns, a major concern is corneal protection. Tarsorrhaphy was
once routinely recommended, but now is reserved difficult cases. With
meticulous corneal lubrication and appropriate ophthalmologic consultation,
often tarsorrhaphy can be avoided. Colonization and superinfection of the
eschar is a major source of morbidity with burns and in the past has been a
major source of sepsis in burn patients. Topical antimicrobials have served to
decrease the risk of eschar superinfection.

5. D

Second-degree burns extend into the dermis for a variable distance.


Clinically, they are characterized by severe pain, erythema, and blistering.
Third-degree burns extend the full thickness of the dermis into subcutaneous
and sometimes even down to bone. Clinically, there is no bleeding, blistering,
or pain. First-degree burns are superficial injuries only involving the
epidermis. Clinically, they cause only pain and erythema.

104
BY DR. MOHAMMED ATIAA KAREEM ALNASHY
-------------------------------------------------------------------
CHAPTER TWENTY-SIX: MAXILLOFACIAL TRAUMA
--------------------------------------------------------------------------------------------

1. The mesiobuccal cusp of the maxillary first molar sitting within the
mesiobuccal groove of the mandibular first molar designates which
type of occlusion?
A. Glass I
B. Glass II
C. Glass III
D. Glass IV
E. Glass V

2. Lag screw fixation is recommended for which type of mandible


fracture?
A. Nondisplaced
B. Comminuted
C. Oblique
D. Contaminated
E. Unfavorable

3. In panfacial fractures, what should be the first and primary focus?


A. Orbital reconstruction
B. Establishing facial height
C. Occlusion
D. Repositioning the zygoma
E. Repairing the central face

4. Preservation of what structure is important in repairing the frontal


sinus/anterior cranial fossa?
A. Supraorbital rim
B. Temporalis fascia
C. Galea
D. Pericranium
E. Periosteum

105
BY DR. MOHAMMED ATIAA KAREEM ALNASHY
-------------------------------------------------------------------
5. To prevent postoperative enophthalmos, the surgeon must remember
that the medial orbital wall
A. Should not be repaired
B. Is concave
C. Is convex
D. Cannot be accessed through the approach used to the orbital floor
E. Lies in a sagittal plane

CHAPTER 26

Maxillofacial Trauma

1. A

2. C

3B

4D

5. C

106
BY DR. MOHAMMED ATIAA KAREEM ALNASHY
-------------------------------------------------------------------
CHAPTER TWENTY-SEVEN: RECONSTRUCTION OF FACIAL
DEFECTS
--------------------------------------------------------------------------------------------
1. Which of the following is not true of rotational flaps?
A. Rotational flaps are best used to close triangular defects.
B. Rotational flaps usually have a random vascular supply.
C. Rotational flaps are ideally superiorly based.
D. Rotational flaps are useful for posterior cheeks.
E. Rotational flaps feature curvilinear shapes.

2. Which of the following is not true of the mid-forehead flap?


A. The midforehead flap has an excellent color/texture match for midface
defect repair.
B. The axial flap is based on the supratrochlear artery.
C. Frontalis muscle and fascia can be included when more depth is needed.
D. The midforehead flap represents one type of rotational flap.
E. Cartilage grafts can be included for nasal reconstruction.

3. Which of the following is not true of advancement flaps?


A. Advancement flaps have a linear configuration.
B. Advancement flaps are moved into a defect by being stretched forward.
C. Closure of advancement flaps depends on both primary and secondary
movement.
D. Advancement flaps are most commonly used for very large facial defects.
E. Advancement flaps are categorized as pedicle, bipedicle, and V-Y types.

4. A 57-year-old woman has a squamous cell cancer removed from her


upper lip, leaving a defect that encompasses approximately two-thirds
of her upper lip. Which of the following represents the most
appropriate choice for reconstruction?
A. Microvascular tissue transfer
B. Rotational flap
C. Advancement or transpositional flap from the cheek
D. Mucosal hinge flap
E. Primary closure

107
BY DR. MOHAMMED ATIAA KAREEM ALNASHY
-------------------------------------------------------------------
5. Which of the following is not a goal in repairing forehead defects?
A. Preservation of frontalis muscle function
B. Preservation of sensation of the forehead skin
C. Maintenance of eyebrow symmetry
D. Maintenance of natural-appearing temporal and frontal hairlines
E. Creation of vertical instead of horizontal scars whenever possible
(except in the midline forehead)

CHAPTER 27

Reconstruction of Facial Defects

1. C

2D

3D

4. C

5. E

108
BY DR. MOHAMMED ATIAA KAREEM ALNASHY
-------------------------------------------------------------------
CHAPTER TWENTY-EIGHT: HAIR RESTORATION: MEDICAL AND
SURGICAL TECHNIQUES
--------------------------------------------------------------------------------------------
1. In androgenetic alopecia, hair growth of specific hair follicles on the
scalp is sensitive to which of the following androgens?
A. Testosterone
B. Estradiol
C. Estrone
D. Dihydrotestosterone
E. Dehydroepiandrosterone

2. Telogen effluvium is a physiologic response to a variety of stressors,


including hormonal or systemic conditions manifesting as a form of
diffuse, nonscarring alopecia. Which of the following statements
correctly characterizes this physiologic response?
A. The hair follicle gradually shifts from the anagen phase to the catagen
phase.
B. The hair follicle precipitously shifts from the anagen phase to the catagen
phase.
C. The hair follicle gradually shifts from the anagen phase to the telogen phase.
D. The hair follicle precipitously shifts from the anagen phase to the telogen
phase.
E. The hair follicle precipitously shifts from the catagen phase to the telogen
phase.

3. In terms of medical management of androgenetic alopecia, which of the


following statements about finasteride is false?
A. Finasteride is a 5-oc-reductase inhibitor that does not interfere with the
actions or effects of testosterone.
B. In 5-year studies, 65% of men taking finasteride maintained or improved
their hair counts, whereas men on placebo continued to lose hair.
C. In 5-year studies based on reviewing standardized clinical photographs, 90%
of men taking finasteride were rated as having no further baseline hair loss.
D. Patients taking finasteride need to be on continued therapy to preserve the
renewed hair growth.
E. Patients taking finasteride for an extended period of time report an increase
in the incidence of adverse effects as high as 8%, including decreased sexual
libido and erectile dysfunction.

109
BY DR. MOHAMMED ATIAA KAREEM ALNASHY
-------------------------------------------------------------------
4. In terms of follicular-unit hair transplantation, which of the following
statements is false?
A. Follicular-unit hair grafts consist of naturally occurring groups of one
to four hairs in addition to the supporting structures, including
sebaceous glands and a circumferential band of collagen.
B. Ideal candidates for hair transplantation have a significant contrast
between hair color and skin color and are old enough that future hair
loss is more likely to be predictable.
C. In planning the anterior hairline, consideration should be given to
recreating a "feathering" transition zone of approximately 0.5 to 1.0 cm,
as well as augmenting a dense frontal forelock.
D. The donor strip is harvested within the predicted hair fringe margin,
which extends from each temporoparietal region to the midoccipital
scalp.
E. In creating recipient incisions, specific attention should be given to the
surrounding natural hair growth in terms of proper hair direction and
angulation from the scalp.

5. In the past, scalp reductions were used extensively to minimize or


eliminate areas of alopecia. Which of the following statements about
scalp reductions is false?
A. Excising a bald portion of scalp requires wide subgaleal undermining.
B. In patients with a high anterior hairline and a prominent forehead, a
hairline advancement can be performed through an irregular,
trichophytic incision and excision of pretrichial skin.
C. In terms of complications associated with scalp reductions, the most
significant complication is intraoperative bleeding associated with
extensive scalp dissection.
D. The vertical "slot" defect is created in the posterior scalp and is related
to misdirected hair growth from repetitive scalp reductions.
E. Modified scalp reductions still have a role in hair restoration surgery,
often in conjunction with hair grafting.

110
BY DR. MOHAMMED ATIAA KAREEM ALNASHY
-------------------------------------------------------------------
CHAPTER 28

Hair Restoration: Medical and Surgical Techniques

1D

Different androgens have various influences on hair growth based on the


region. Testosterone affects primarily axillary and pubic hair growth.
Dihydrotestosterone (DHT) affects beard growth and scalp hair, primarily in
frontal and vertex regions of the scalp, resulting in androgenetic alopecia.

2D

During telogen effluvium, the hair shaft is shed as the follicle abruptly
transitions from the growth or anagen phase to the resting of telogen phase.
Telogen effluvium often occurs in response to certain stressors, including
hormonal and systemic conditions, as well as exposure to a broad range of
medications.

3. E

Finasteride is a type 2 5-oc-reduc-tase inhibitor that results in the


suppression of circulating dihydrotestosterone (DHT) while having no
affinity or effect on testosterone and other hormones. In 5-year studies, 65%
of men taking finasteride maintained or improved their hair counts, and 90%
of men were noted in clinical photography as having no further visible hair
loss compared with baseline. These studies also found that the incidence of
sexual side effects decreased with duration of therapy from 0.8% to 1.8% to 0.3%
after the fifth year. As with minoxidil, continued therapy with finasteride is
required to preserve renewed hair growth.

111
BY DR. MOHAMMED ATIAA KAREEM ALNASHY
-------------------------------------------------------------------
4B

Follicular-unit hair transplantation has become the established method of


choice in hair restoration surgery. Individual follicular units are dissected
based on the natural grouping of one to four terminal hairs and include all
of their support structures. In terms of ideal candidates, minimal contrast
should exist between hair color and skin color to improve the illusion of a
natural anterior hairline. This illusion also relies on recreating a
"feathering" transition zone with fine, single hair grafts placed in recipient
incisions made to emulate the surrounding natural hair in terms of proper
hair direction and angulation.

5. C
Although modified scalp reductions still have a role in select patients with
alopecia, both the surgeon and the patient need to be aware of the potential
complications, most noticeably misdirected hair, decreased hair density, and
potentially detectable scars. The vertical "slot" defect is created in the
occipital scalpel by closing an elliptical resected area into an elongated scar.
This unfavorable scar can be prevented or corrected by use of a series of three
rotational flaps as described by Frechet.

112
BY DR. MOHAMMED ATIAA KAREEM ALNASHY
-------------------------------------------------------------------
CHAPTER TWENTY-NINE: MANAGEMENT OF AGING SKIN
--------------------------------------------------------------------------------------------
1. Which Fitzpatrick sun-reactive skin type does the following
characterize: a fair-skinned individual with blond, red, or brown hair,
usually burns and tans less than the average person?

2. Which of the following is not and absolute contraindication to


chemical peeling?
A. Collagen vascular disease
B. Emotional instability
C. Isotretinoin treatment 8 months ago
D. Immunosuppressed patient
E. Fitzpatrick type IV skin type

3. What is the most important ingredient in creating the depth of a


Baker's solution peel?
A. Phenol 88%
B. Septisol
C. Croton oil
D. Distilled water
E. Hydroxy-acid

4. Describe the physiologic sequence seen with phenol toxicity from a


chemical peel, and explain how such toxicity can be avoided.

5. Describe the mechanism of action of retinoids (i.e., tretinoin) and


common side effects in facial rejuvenation.

113
BY DR. MOHAMMED ATIAA KAREEM ALNASHY
-------------------------------------------------------------------
CHAPTER 29

Management of Aging Skin

1. Type II

2. E

3. C

4. Systemic toxicity if first suggested by CNS stimulation (tremors,


hyperreflexia, and hypertension) followed by CNS depression with
respiratory failure, hypotension, and cardiac arrhythmia, which can be fatal.
Preventing such toxicity is possible with IV fluid volume loading before,
during, and after a peel; forced diuresis with furosemide 10 minutes before
the peel, waiting 20 to 30 minutes between each facial region to be peeled (25%-
30% surface area), and careful patient selection, avoiding patients with
hepatorenal disease.

5. Tretinoin induces type I procollagen formation, thickens the epidermis,


reduces melanin content, increases glycosamino-glycans, and stimulates
angio-genesis, giving a "rosy" appearance to the skin. Histologically,
tretinoin-treated skin is more stable because of increased anchoring fibrils
and decreased activity of collage-nase enzyme. Tretinoin can also reduce UV-
induced collagen destruction and therefore helps prevent photodamage
(Fisher and others 1999). Side effects include mild to moderate dryness, skin
irritation, peeling, and photosensitivity; therefore, sun protection is
important.

114
BY DR. MOHAMMED ATIAA KAREEM ALNASHY
-------------------------------------------------------------------
CHAPTER THIRTY: RHYTIDECTOMY
--------------------------------------------------------------------------------------------

1. Which is a disadvantage of a subperiosteal mid-face lift?


A. It requires preauricular incision.
B. It impairs vascular supply of facial skin.
C. It weakens orbicularis occuli.
D. It risks injury to buccal branch of facial nerve.

2. Which surgical approach is not used to perform a subperiosteal


midface lift?
A. Transtemporal
B. Transfacial
C. Transorbital
D. Transoral
E. Combined

3. Which mimetic muscle is part of the superficial muscular aponeurotic


system (SMAS)?
A. Zygomatic major
B. Zygomatic minor
C. Orbicularis occuli
D. Platysma
E. Orbicularis oris

4. An SMAS rhytidectomy is a facelift in which


A. An extended supra-SMAS dissection is performed.
B. The SMAS is tightened solely with suture suspension.
C. A flap consisting of a portion of the SMAS is dissected.
D. A sub-SMAS dissection is performed to lift the midface.
E. An SMASectomy is performed over the parotid gland.

115
BY DR. MOHAMMED ATIAA KAREEM ALNASHY
-------------------------------------------------------------------
5. The most effective method of managing anterior neck bands is
A. Suture suspension of the cervical platysma to mastoid fascia
B. Dissection of a platysmal flap below the angle of the mandible and
advancing the flap pos-terosuperiorly
C. Horizontal division of the cervical platysma and posterior advancement
of the muscle flap
D. Submental fat excision and horizontal division of the anterior border
of the platysma
E. Excision of the anterior platysmal muscles and midline suture
apposition

116
BY DR. MOHAMMED ATIAA KAREEM ALNASHY
-------------------------------------------------------------------
CHAPTER 30: Rhytidectomy
1D
The buccal branch of the facial nerve is at risk during subperiosteal dissection
of the midface at the time of release of the periosteum from the inferior aspect
of the zygoma and zygomatic arch.

2B
The transfacial approach is used to perform an extended supra-SMAS
rhytidectomy in which the midface is lifted by dissecting to the upper lip
while remaining superficial to the SMAS but deep to the cheek fat. It is not
used as an approach for subperiosteal midface lifts.

3D
The platysma is a rhomboidal subcutaneous sheet of muscle. The muscle
crosses the entire length of the mandible and accounts for the mobility of the
skin along the jawline. It continues above the lower cheek as a superficial
aponeurotic fascia that invests the muscles of facial expression located in the
midface.

4. C
A SMAS rhytidectomy is performed by approximately 20% of facelift
surgeons. This may take the form of dissecting a SMAS flap limited to the
area over the parotid gland or extending the SMAS flap anterior to the
parotid gland. The SMAS flap is then suspended postero-superiorly.

5. E
By resecting redundant platysmal muscle and advancing the platysma
medially toward the midline, the surgeon is addressing the deformity at its
origin and is advancing tissue in the same direction as the gravitational forces
on the neck.

117
BY DR. MOHAMMED ATIAA KAREEM ALNASHY
-------------------------------------------------------------------
CHAPTER THIRTY-ONE: MANAGEMENT OF THE AGING BROW
AND FOREHEAD
--------------------------------------------------------------------------------------------

1. Which of the following statements regarding pretrichal forehead lift


is true?
A. Pretrichal forehead lift preserves the hairline.
B. Pretrichal forehead lift is indicated in patients with a low hairline.
C. Pretrichal forehead lift has no risk of an exposed scar.
D. Pretrichal forehead lift does not treat all aspects of the aging forehead
and brow.
E. The temporal incision in the pretrichal forehead lift is not similar to
that in the coronal lift.

2. Upper blepharoplasty performed before a selected brow elevation


procedure can be associated with
A. Minimal elevation of the brow-lid complex
B. Lagophthalmos
C. Elevation of the lateral canthus
D. Upper eyelid ptosis
E. Injury to the supratrochlear nerve

3. All of the following are brow depressor muscles except


A. Gorrugator
B. Frontalis
C. Procerus
D. Supraorbital orbicularis oculi
E. Depressor supercilii

4. To achieve brow elevation, all of the following structures are "released"


in the endoscopic browlift except
A. Supraorbital periosteum
B. Temporal conjoint fascia
C. Temporal conjoint tendon
D. Temporalis muscle
E. Brow depressor musculature

118
BY DR. MOHAMMED ATIAA KAREEM ALNASHY
-------------------------------------------------------------------
5. Which nerve may be injured if cautery is performed on the
undersurface of the temporoparietal fascia in the region of the
"sentinel" vein?
A. Zygomaticotemporal nerve
B. Supratrochlear nerve
C. Supraorbital nerve
D. Frontal branch of the facial nerve
E. Auriculotemporal nerve

119
BY DR. MOHAMMED ATIAA KAREEM ALNASHY
-------------------------------------------------------------------
CHAPTER 31

Management of the Aging Brow and Forehead

1. A
In patients with an elongated forehead and high hairline, the pretrichial
forehead lift may be used. The pretrichial incision is located at the junction
of the cephalic aspect of the forehead and hairline or is placed just within
the hairline to further camouflage the scar. A beveled incision to allow hair
follicle growth through the scar adds additional camouflage, and an
irregularized scar pattern is created. The temporal incision is connected to
the pretrichial incision and is posterior to the temporal hairline similar to
the coronal lift. The advantage of the pretrichial incision is that the forehead
is not elevated, and the frontal hairline is preserved. The pretrichial forehead
lift treats all aspects of the aging forehead and brow.

2B
In general, the selected brow elevation procedure should be performed before
upper blepharoplasty so that the facial plastic surgeon can judge the precise
amount of upper eyelid skin to be removed. This helps prevent excessive
elevation of the brow-lid complex with the potential for causing
lagophthalmos. In some cases, the need for upper blepharoplasty maybe
eliminated after brow lifting procedures.

3B
The brow depressor musculature include the corrugator, procerus, depressor
supercilii, and supraorbital orbicularis oculi muscles. The frontalis muscle is
the only brow elevator.

120
BY DR. MOHAMMED ATIAA KAREEM ALNASHY
-------------------------------------------------------------------
4D
The term "release" means to elevate, incise, and spread. The technique that
achieves excellent brow elevation is release of the periosteum from one
inferolateral orbit to the other and release of the brow depressor musculature
(corrugator, procerus, depressor supercilii, and supraorbital orbicularis
oculi). The temporal conjoint fascia (fusion of the galea and the
temporoparietal fascia) is released with a periosteal elevator in an inferior
direction to the level of the supraorbital rim. Near the supraorbital rim, a
thickening of periosteum termed the conjoint tendon is incised sharply or
bluntly. Adequate release of the conjoint tendon at the lateral supraorbital
rim is an essential factor of the periosteal release. The dissection occurs over
the deep temporal fascia and temporalis muscle without release.

5D
The "sentinel" vein is a reliable marker for the frontal branch of the facial
nerve, which lies superficial to the dissection on the undersur-face of the
temporoparietal fascia. If the vein is cauterized, the bipolar forceps are placed
at the base of the sentinel vein to help prevent a thermal neuropraxic injury
to the frontal branch of the facial nerve. Lateral and slightly inferior to the
sentinel vein, the zygomaticotemporal sensory nerve is encountered and is
usually considered the lateral border of the dissection.

121
BY DR. MOHAMMED ATIAA KAREEM ALNASHY
-------------------------------------------------------------------
CHAPTER THIRTY-TWO: MANAGEMENT OF THE AGING
PERIORBITAL AREA
--------------------------------------------------------------------------------------------
1. All the following statements about the frontal branch of the facial
nerve are true except
A. It lies deep to the superficial musculoaponeurotic system fascia.
B. It enters the orbicularis oculi muscle and frontalis muscle along the
deep surface of the muscles.
C. As it crosses the zygomatic arch, it lies deep to the periosteum.
D. It supplies the muscles of the forehead and the orbicularis oculi muscle.
E. It courses anterior to the superficial temporal artery.

2. All of the following statements about ideal brow anatomy in females


are true except
A. The medial end should lie in a horizontal plane 2 mm inferior to the
lateral end.
B. The medial end should have a club head appearance.
C. The lateral end should gradually taper to a point.
D. The medial origin should be at the level of a vertical line drawn from
the nasal alar facial junction.
E. The brow should arch superiorly with the highest point at the lateral
limbus.

3. The advantages of subcutaneous dissection for browlift and


blepharoplasty compared with deeper subgaleal and subperiosteal
dissection include all of the following except
A. Sensory nerve branches are spared with subcutaneous dissection.
B. Subcutaneous dissection results in a lower incidence of skin slough and
hair loss from vascular compromise.
C. Subcutaneous dissection provides an effective method for abolishing
forehead rhytids and moderate to severe brow ptosis.
D. Subcutaneous dissection protects the facial nerve branches from injury.
E. Subcutaneous dissection may enable a greater degree of accuracy for
brow placement and postoperative brow symmetry.

122
BY DR. MOHAMMED ATIAA KAREEM ALNASHY
-------------------------------------------------------------------
4. All of the following statements are true regarding upper eyelid
blepharoplasty except
A. The initial lid marking is made at the natural skin crease or 1 mm
above the natural crease.
B. The medial end of the incision is carried to but not beyond the punctum
of the medial can thus.
C. If blepharoplasty is performed in conjunction with a browlift, the
markings for the blepharoplasty incisions are made first.
D. Orbital fat may be addressed by cauterizing through the orbital septum
or by opening the orbital septum to remove fat.
E. A variable amount of orbicularis oculi muscle should be resected to
create a distinct upper eyelid crease.

5. Compared with the transconjunctival approach for lower eyelid


blepharoplasty, the subciliary approach is useful for which of the
following group of patients?
A. Patients desiring associated midface lifting
B. Patients primarily desiring treatment for fat pseudoherniation
C. Patients with thick skin
D. Patients who smoke
E. Patients with large amounts of excess skin

123
BY DR. MOHAMMED ATIAA KAREEM ALNASHY
-------------------------------------------------------------------
CHAPTER 32
Management of the Aging Periorbital Area

1. C
As the nerve crosses the zygomatic arch, it lies between the periosteum of the
zygoma and the SMAS. This is an important relationship to keep in mind,
because dissection in the region of the arch should be carried out
subcutaneously or subperiosteally. The nerve courses from the parotid gland
toward its final destination, where it pierces the undersurface of the frontalis
muscle 1.5 cm above the lateral canthus.

2A
The ideal female brow has medial and lateral ends that lie on the same
horizontal plane. In addition to the other descriptions, the lateral extent of
the brow should reach a point on a line drawn from the nasal alar facial
junction through the lateral Canthus of the eye. In men, there should be less
of an arch to the brow position and more of a horizontal contour along the
supraorbital ridge.

3B
The major vascular supply lies within the subcutaneous fat and the
superficial fascia of the frontalis muscle. Dissecting in this plane thus results
in a higher incidence of vascular compromise with skin slough and hair loss.
This plane is most useful in brow lifting techniques involving incisions
anterior to the hairline, where hair loss is not an issue. Subgaleal and
subperiosteal dissection result in flaps that are thicker and well vascularized
but also more limited as a result of inelasticity.

124
BY DR. MOHAMMED ATIAA KAREEM ALNASHY
-------------------------------------------------------------------
4. C
When performed in conjunction with a browlift, the blepharoplasty incisions
must be marked only after the browlift has been performed and all incisions
closed. Upward repositioning of the brow reduces the amount of upper eyelid
skin excess, resulting in a decrease in the amount of skin to be excised with
blepharoplasty. Failure to adhere to this may result in the development of
postoperative lagophthalmos.

5. E
The subciliary lower eyelid blepharoplasty approach is most useful for
patients with large amounts of excess skin. In this technique, an incision is
made 2 mm below the lash line extending from 1 mm lateral to the inferior
punctum to 10 mm lateral to the lateral can-thus. It extends through the skin
and orbicularis oculi muscles. After the skin-muscle flap is elevated and
redraped and bulging orbital fat addressed, a variable amount of skin and
muscle are excised as needed. The transconjunctival approach is more
appropriate for patients with pseudoherniation of fat and little need for skin
excision. It is also good for patients prone to hypertrophic scar formation and
patients unwilling to accept an external incision.

125
BY DR. MOHAMMED ATIAA KAREEM ALNASHY
-------------------------------------------------------------------
CHAPTER THIRTY-THREE: SUCTION-ASSISTED
LIPOCONTOURING
--------------------------------------------------------------------------------------------

1. When considering whether a patient is a candidate for lipocontouring,


the surgeon need not be concerned with which of the following?
A. The age of the patient
B. The adjacent anatomic structures and skeletal framework
C. Whether the fat responds to weight loss
D. The gender of the patient

2. The area of liposuction will


A. Have a greater tendency to gain or lose weight with fluctuation in
overall body mass
B. Have a response to changes in weight that is similar to other regions of
the body
C. Replace removed adipocytes with new ones during periods of weight
gain
D. Never return to the preexisting contour

3. During liposuction
A. Adipocytes are selectively removed because of their structural integrity.
B. Vessels, nerves, and muscles are protected because of their loose
intercellular connections.
C. Maintaining bridges of uninterrupted tissue between the deep and
superficial tissue is important.
D. Only hypertrophic adipocytes are removed.

4. When ultrasonic liposuction is performed


A. The risk of hematoma is greater than with standard liposuction.
B. Heat may be generated at the distal subdermis and the cutaneous
incision site, thereby leading to potential complications.
C. The wound must be bathed in ultrasound jelly.
D. Fluctuations in the weight gain and loss at the treated area are less
significant than those that occur at areas treated with traditional
liposuction.

126
BY DR. MOHAMMED ATIAA KAREEM ALNASHY
-------------------------------------------------------------------
5. The ideal patient for liposuction
A. Has loose skin and a weak jawline
B. Has young elastic skin, good bone structure, and a low anterior hyoid
C.Has young elastic skin, a strong jawline, a high posterior hyoid, and
fat that is not responsive to weight loss
D. Would like to have surgery that would help him or her attract a mate,
get a raise, and look like a model

127
BY DR. MOHAMMED ATIAA KAREEM ALNASHY
-------------------------------------------------------------------
CHAPTER 33: Suction-Assisted Lipocontouring

1. B
The gender of the patient does have an impact on the effectiveness of
liposuction. The thicker skin of the male beard may help to camouflage
irregularities that can occasionally occur but does not determine the ability
to contour.

2B

The remaining adipocytes in areas that have undergone lipocontouring will


respond to weight gain and weight loss in relation to overall body fat
deposition. The evidence indicates that weight gain occurs through the
enlargement of individual fat cells and not by the addition of new cells. If
excessive weight gain occurs, then the original contour could be obtained
through hypertrophy of the remaining fat cells.

3. C
Maintaining bridges of uninterrupted tissue between the deep and superficial
tissue is important to ensure viability of the elevated skin flap. The fat cells
are selectively aspirated as a result of their lack of structural integrity,
whereas the vessels, nerves, and muscles are protected.

4B
With the undulations of the ultrasonic liposuction cannula, heat can be
generated, which could lead to a thermal injury. No advantage is obtained by
the liberal use of lubricating jelly. Other risks associated with liposuction are
not significantly influenced by the use of ultrasonic liposuction. There is also
no impact on the response of remaining fat cells to weight loss or gain.

5. C
When selecting a patient for liposuction, young elastic skin, a strong skeletal
structure with a favorable position of the hyoid larynx complex, and fat that is not
responsive to weight loss are the ideal characteristics. Older patients with loose
inelastic skin will not achieve significant improvement with liposuction alone and
will require some form of skin reduction, either through direct cervical excision or
facelift. Any patient that considers cosmetic surgery to create a major improvement
in his or her socioeconomic status should be counseled and avoided; only patients
with realistic expectations should be treated.

128
BY DR. MOHAMMED ATIAA KAREEM ALNASHY
-------------------------------------------------------------------
CHAPTER THIRTY-FOUR: MENTOPLASTY AND
FACIAL IMPLANTS
--------------------------------------------------------------------------------------------

1. In contrast to alloplastic augmentation mentoplasty, the osseous


genioplasty allows
A. Correction in the horizontal plane only
B. Correction in the vertical plane only
C. Correction in the transverse, horizontal, and vertical plane
D. Correction in the transverse plane only
E. A reversible technique

2. If bony osteotomies are made, where should they be in relation to the


mental foramen?
A. 2 mm above
B. 2 mm below
C. 5 mm above
D. 5 mm below
E. At the level of the foramen

3. When a chin implant is placed, all of the following are true except
A. Supraperiosteal placement may cause bone erosion.
B. Subperiosteal placement may cause bone erosion.
C. The mentalis should be reapproximated.
D. Subperiosteal placement increases fixation of the implant.
E. An intraoral or extraoral approach may be used.

4. A disadvantage of the use of expanded polytetrafluoroethylene


(ePTFE) as a facial implant is
A. Difficulty with removal
B. Capsule formation
C. Extrusion
D. Instability
E. Difficulty contouring the implant

129
BY DR. MOHAMMED ATIAA KAREEM ALNASHY
-------------------------------------------------------------------
5. Malar and/or submalar augmentation is indicated for all of the
following except
A. Congenital defects
B. Traumatic deficiencies
C. Accentuating the nasolabial fold
D. Cheek soft-tissue ptosis
E. Submalar wasting

CHAPTER 34
Mentoplasty and Facial Implants
1. C
2D
3. A
4. C
5. C

130
BY DR. MOHAMMED ATIAA KAREEM ALNASHY
-------------------------------------------------------------------
CHAPTER THIRTY-FIVE: REHABILITATION OF
FACIAL PARALYSIS
--------------------------------------------------------------------------------------------

1. Which branch of the facial nerve has the highest priority when
reinnervation procedures are being considered?
A. Frontal
B. Buccal
C. Mandibular
D. Cervical
E. None. They are equally important.

2. After transection of the facial nerve, the distal branches retain their
stimulability with a portable electrical stimulator for how long?
A. 1 day
B. 3 days
C. 1 week
D. 3 weeks
E. 12 months

3. A healthy 68-year-old patient develops House-Brackmann grade VI


facial paralysis after resection of an acoustic neuroma. One year later,
he has no discernible return of function. The patient desires the best
possible outcome. Which treatment would provide optimal
rehabilitation for this patient?
A. Reinnervation with sural nerve graft
B. Static sling, lateral canthoplasty, and gold weight
C. XII-VII crossover graft
D. Temporalis transfer
E. Masseter transfer

131
BY DR. MOHAMMED ATIAA KAREEM ALNASHY
-------------------------------------------------------------------
4. Advantages of polytetrafluoroethylene (PTFE; Gore-Tex) for static
suspension of the midface include all of the following except
A. No donor site morbidity
B. Shorter operative time
C. Lower infection rate than with autologous fascia grafts
D. Less overcorrection necessary than autologous fascia grafts
E. Immediate improvement of facial symmetry compared with
reinnervation procedures

5. A patient with complete facial paralysis for 1&1/2 years after


parotidectomy is referred to you for management. While reviewing the
operative report, you learn that the patient had transection of the facial
nerve during parotidectomy proximal to the pes anserinus. The patient
has had no discernible return of function but desires a reinnervation
procedure. Which diagnostic test is most important to obtain before
proceeding with reinnervation?
A. Magnetic resonance imaging
B. Audiogram
C. Muscle biopsy
D. Schirmer test
E. An electromyogram

132
BY DR. MOHAMMED ATIAA KAREEM ALNASHY
-------------------------------------------------------------------
CHAPTER 35
Rehabilitation of Facial Paralysis

1B
The zygomatic and buccal branches of the facial nerve have equal importance
in reinnervation procedures. These branches innervate the midface structures
responsible for the smile and some orbicularis oculi function. Restoration of
innervation to these muscles provides gross facial symmetry. The cervical
branch is least important for facial expression, although selective disruption
of this nerve can result in subtle lip asymmetry noted on mouth opening. The
frontal branch is relatively less important in comparison as well. The
mandibular nerve branch innervates the depressor muscles of the mouth and,
although important, does not take the same precedence as reinnervation of
the midface muscles.

2B
Electrical stimulability in the distal nerve branches remains intact for up to
72 hours. After acute facial nerve transection, exploration should be
performed within this time frame to repair injured nerves if possible. If
reinnervation is not performed in this time frame, then marking the position
of the distal nerve branches after identification is suggested to aid subsequent
reinnervation efforts.

3. C
This patient has a proximal nerve injury, and delayed nerve grafting to the
brainstem will not provide satisfactory results. The individual has a viable
hypoglossal nerve that can be used. Muscle transfers could be performed, but
the function with nerve transposition is superior. Static techniques and upper
eyelid adjunctive procedures could be used in selected cases to provide
immediate restoration of symmetry and eyelid closure, but by themselves
would provide suboptimal results.

133
BY DR. MOHAMMED ATIAA KAREEM ALNASHY
-------------------------------------------------------------------
4. C
Like autologous material, PTFE and acellular dermis (Alloderm) provide
immediate restoration of symmetry when used for static midface suspension.
Autologous materials generally require gross overcorrection to compensate for
laxity developing in the postoperative period, but this is not necessary for
PTFE. Advantages of biocompatible alloplastic materials such as PTFE
include no need to harvest tissue, which prevents donor site morbidity and
can shorten the operative time. The infection rate with alloplastic graft
materials is not less than with autologous materials. Although PTFE is very
biocompatible, infection rates of up to 9% have been reported with its use.

5. E
EMG is indicated in any facial paralysis lasting longer than 1 year. It can
indicate whether reinnervation is occurring and can also provide information
about the viability of facial musculature. If there is evidence of
reinnervation, procedures could be delayed to observe for return of function.
If there is total electrical silence, indicating severe facial muscle atrophy,
reinnervation procedures would not be indicated. Muscle biopsy is useful in
selected cases to confirm lack of viable facial musculature.

134
BY DR. MOHAMMED ATIAA KAREEM ALNASHY
-------------------------------------------------------------------
CHAPTER THIRTY-SIX: OTOPLASTY
---------------------------------------------------------------------------------
1. What is the embryologic basis for protruding ears?
A. Autosomal-dominant inheritance
B. Overgrowth of ectoderm from the first branchial arch
C. Overgrowth of mesoderm from the third branchial arch
D. Hypertrophy of the otic placode
E. Maldevelopment of the forth hillock of His

2. What is the most appropriate indication for pursuing otoplasty?


A. Age of patient
B. Auriculocephalic angle
C. Cartilage proportions
D. Distance of helical rim from scalp
E. Cartilage stiffness

3. A 29-year-old man is initially seen with a unilateral prominent ear.


There is minimal furling of the antihelix, yet the cartilage seems
thick and stiff. What is a practical treatment modality?
A. Octyl-2-cyanoacrylate
B. Postauricular fusiform skin excision
C. Horizontal mattress sutures from the conchal wall to the mastoid
periosteum
D. Scoring of the anterior cartilage surface followed by horizontal
mattress sutures from the scapha to the concha
E. Scoring of the anterior cartilage surface alone

4. A 4-year-old child has bilateral protruding ears. During the initial


consultation, it is apparent that the mother has a similar affliction.
The mother requests correction before the child enters kindergarten.
What is the best plan of action?
A. The parents should be advised to wait until the child is 5 or 6 years
old and ear growth is nearly complete.
B. The parents should be advised to use octyl-2-cyanoacrylate to hold the
ears back until surgical correction is achieved.
C. Postauricular fusiform skin excision should be performed.
D. Scoring of the anterior cartilage surface should be performed followed
by horizontal mattress sutures from the scapha to the concha.
E. Scoring of the anterior cartilage surface alone should be performed.

135
BY DR. MOHAMMED ATIAA KAREEM ALNASHY
-------------------------------------------------------------------
5. A 32-year-old woman with Ehlers-Danlos syndrome undergoes bilateral
otoplasty. At 3:00 AM, she contacts you through the answering service to
complain of pain. What is the appropriate action?
A. Instruct the patient to remove any dressings or bolsters.
B. Instruct the patient to increase pain medications.
C. See the patient and remove the dressing.
D. See the patient and reassure him or her that fullness under the skin
will resolve.
E. See the patient and evaluate the ear under dressing and drain any
possible collection.

136
BY DR. MOHAMMED ATIAA KAREEM ALNASHY
-------------------------------------------------------------------
CHAPTER 36 :-Otoplasty

1. E
Although autosomal dominance may be the mode of inheritance, it is
descriptive of the embryology underlying protruding ears. The otic placode
is the first precursor to the ear; however, the antihelix derives from the
fourth hillock of His and the conchal cartilage from the ectoderm of the
first branchial groove. The hillocks derive from the mesoderm of the first
and second arch.

2. C
Although each of these is considered in the preoperative assessment, it is
really the cartilage size that determines whether to proceed with otoplasty.

3D
Octyl-2-cyanoacrylate is reserved for neonatal otoplasty to avert future
surgical otoplasty. A postauricular skin incision may set the ear back but
does not address the antihelix. In the case of stiff cartilage, scoring may
impede the cartilage spring and facilitate reestablishment of the antihelical
fold by suture technique.
4. A

5. E

137
BY DR. MOHAMMED ATIAA KAREEM ALNASHY
-------------------------------------------------------------------

PART THREE
--------------------------------------------------------------------------------------------
--------------------------------------------------------------------------------------------
NOSE

138
BY DR. MOHAMMED ATIAA KAREEM ALNASHY
-------------------------------------------------------------------
CHAPTER THIRTY-SEVEN: PHYSIOLOGY OF OLFACTION
----------------------------------------------------------------------------------

1. The primary neuron cell body for cranial nerve I is located in the
A. Olfactory bulb
B. Nasal mucosa
C. Entorhinal cortex
D. Cribriform plate
E. Prefrontal cortex

2. A 38-year-old man with history of a sudden "bad cold" 4 months ago


complains of bland taste of foods. On further questioning, he notes the
inability to detect smells that are strong to his wife. In addition, he
recalls burning the eggs last week and had not noticed the fire until he
saw the smoke. However, he does remember smelling the ammonia 2
days ago that he was using to clean the bathroom. He can most likely
detect the ammonia because of a functioning
A. Cranial nerve I
B. Cranial nerve V
C. Cranial nerve VII
D. Cranial nerve IX
E. Cranial nerve X

3. While undergoing olfactory identification testing, it was noted that the


patient was consistently unable to identify the same specific odorant
over many trials; however, all other odorants were identified correctly.
The best possible explanation is a mutation in a gene encoding for a/an
A. Olfactory G-protein
B. Cyclic adenosine monophosphate
C. Inositol phosphate
D. Olfactory receptor protein
E. Calcium/sodium channel

139
BY DR. MOHAMMED ATIAA KAREEM ALNASHY
-------------------------------------------------------------------
4. Of a random population of people, which person would do best on an
olfactory identification test?
A. A 38-year-old man
B. A 40-year-old woman
C. A 68-year-old woman
D. A 67-year-old man
E. A 5-year-old girl

5. A 43-year-old woman comes to your office with a history of the


inability to smell for the past 6 years. She does not remember any
previous head trauma but remembers an upper respiratory tract
infection around the time of the loss of smell. She denies current nasal
obstruction, recurrent sinus infections, headaches, and epistaxis. A
neurologic examination is negative. Nasal endoscopy is normal, and
smell identification testing is consistent with anosmia. The most
important next step is to
A. Obtain magnetic resonance imaging of the head
B. Obtain a computed tomography scan of the sinuses
C. Refer her for an electroolfactogram
D. Counsel her on hazards of anosmia
E. Admit her for high-dose intravenous steroids

140
BY DR. MOHAMMED ATIAA KAREEM ALNASHY
-------------------------------------------------------------------
CHAPTER 37
Physiology of Olfaction

1B
The cell body of the olfactory receptor neuron resides in the olfactory epithelium
within the nasal cavity. It is a bipolar cell with a dendrite extending to the mucosal
surface, where the olfactory knob gives rise to several cilia containing the olfactory
receptors. The axon extends into the lamina propria and travels through the
cribriform plate to synapse within glomeruli of the olfactory bulb.

2B
This patient most likely has anosmia related to an upper respiratory tract infection,
which decreases the flavor of food. The inability to detect smoke at close distances
reflects the severity of the smell loss. Ammonia is a strong stimulator of the trigeminal
system, which usually remains intact. It is rare for both cranial nerve I and cranial
nerve V to be damaged at the same time.

3D
There are approximately 1000 genes encoding olfactory receptor proteins. A mutation
in one of the "functional" genes may result in an odorant specific anosmia. A mutation
in genes encoding G-protein, cyclic AMP, or the calcium/sodium channel would result
in the inability to detect all odorants. Inositol phosphate is not thought to play a role
in the olfactory receptor signal transduction cascade.

4B
Over all testable ages, females do better than males in olfactory identification. A rapid
drop in odorant identification testing occurs in the seventh decade. Odorant
identification testing is thought to be unreliable in young children.

5. D
This patient most likely had anosmia develop in relation to an upper respiratory tract
infection. Her history has been unchanged for the past 6 years, and she is too young
for age-related loss. Her nasal examination is without any evidence of obstruction,
neoplasia, or inflammation. Because her symptoms are not atypical and her
neurologic history and examination are otherwise negative, an intracranial lesion is
highly unlikely. An MRI or GT scan would, therefore, be unnecessary.
Electroolfactograms are limited to research settings and would only confirm her
anosmia but may be helpful in patients suspected of confabulation. High-dose steroids
have not been shown to offer dramatic improvements in anosmia of this type. The most
important step in all patients with complete smell loss is to educate the patient on
health risks associated with anosmia.

141
BY DR. MOHAMMED ATIAA KAREEM ALNASHY
-------------------------------------------------------------------
CHAPTER THIRTY-EIGHT: EVALUATION OF NASAL BREATHING
FUNCTION WITH OBJECTIVE AIRWAY TESTING
--------------------------------------------------------------------------------------------

1. The nasal valve area consists of


A. The septum and distal end of the upper lateral cartilage
B. The septum, piriform aperture, and head of the inferior turbinate
C. The septum, lower lateral cartilage, and floor of nose
D. The distal end of the upper lateral cartilage, head of the inferior
turbinate, caudal end of the septum, and tissues surrounding the
piriform aperture
E. The union of the lateral nasal wall with the upper lateral cartilage
forming an anatomic ridge

2. Acoustic rhinometry can be used to measure which of the following?


A. Size of the minimal cross-sectional area (MCA)
B. Distance to various cross-sectional areas in the nose
C. Total volume of the nose
D. Location of the MCA
E. All of the above

3. Which of the following is true regarding rhinomanometry?


A. Measurement of total nasal resistance is possible only by calculation
from unilateral measurements.
B. Inability to calculate a cross-sectional area from rhinomanometric
measurements is a limiting factor in its use.
C. In a more obstructed nose, the pressure-flow curve is rotated toward the
pressure axis.
D. The typical pressure-flow curve obtained in a normal nose is a linear
relationship represented by a straight line.
E. Passive rhinomanometry requires the patient's own respiratory effort
as a source of pressure and flow.

142
BY DR. MOHAMMED ATIAA KAREEM ALNASHY
-------------------------------------------------------------------
4. Which of the following can be a source of variability in objective
airway testing?
A. The nasal cycle
B. Posture
C. Time of day
D. Smoking
E. All of the above

5. In a patient with nasal obstruction, examination reveals pathology on


the same side as the patient's symptoms. Objective testing confirms the
restriction, and the patient reports improvement after decongestion.
Which of the following is the best approach to treatment as outlined in
the algorithm?
A. Surgery
B. Medical management followed by surgical intervention if no
improvement
C. Medical management
D. Repeat examination and objective testing with imaging
E. Surgery with option for medical treatment if symptoms are still present

143
BY DR. MOHAMMED ATIAA KAREEM ALNASHY
-------------------------------------------------------------------
CHAPTER 38
Evaluation of Nasal Breathing Function with Objective Airway Testing

1. D
According to Kasperbauer and Kern, the nasal valve area is the functional
unit that "includes the distal end of the upper lateral cartilage, the head of
the inferior turbinate, the caudal septum, and the remainder of the tissues
surrounding the piriform aperture." The nasal valve alone was the slitlike
opening between the caudal end of the upper lateral cartilage and the nasal
septum.

2. E
Acoustic rhinometry results are displayed as an area-distance curve
that allows the investigator to determine the cross-sectional area of the nose
at various distances into the nose. This allows the calculation of the MGA
(minimal cross-sectional area of the nose) by noting the distance into the nose
where the greatest restriction to airflow occurs. Because AR gives an
anatomic picture of the nasal cross-sectional area, it makes sense that the
curve generated can also be used to calculate the total volume of one or both
sides of the nose.

3. C
Rhinomanometry measures the pressure differential across the nose from
front to back. This can be done on one side at a time as in anterior rhinometry
or on both sides at the same time as in posterior rhinometry. Active
rhinometry refers to data collected using the patients own respiratory effort,
whereas passive refers to data collected while the patient holds his or her
breath and a known rate of air is introduced into the nose. The
rhinomanometric data obtained are typically displayed as a pressure-flow
curve. This curve is sigmoidal in shape. The data from this curve can be then
be used to calculate resistance in the nose. Both posterior and anterior
methods can be used to calculate total resistance. By tradition, the data are
displayed with pressure on the x-axis and flow on the y-axis. Therefore, in a
more obstructed nose, greater pressures are required to generate the same flow
and the axis rotates toward the pressure or x-axis.

144
BY DR. MOHAMMED ATIAA KAREEM ALNASHY
-------------------------------------------------------------------
4. E
Objective airway testing has many factors that lead to variability in results.
The difficulty encountered in controlling for all of the variables severely
limits the clinical usefulness of these tools. Other factors influencing
objective testing include secretions, exercise, medications, temperature, and
race.

5B
The question refers to the situation of a combined structural problem as
outlined in the algorithm. In this instance, a patient gets some relief from
decongestion, but it is not complete. Furthermore, there is some observed
anatomic pathology on the same side as the subjective sensation of
obstruction, which is confirmed by objective testing. According to the
algorithm, these patients benefit from a trial of medical management
followed by surgical intervention if the response is unsatisfactory.

145
BY DR. MOHAMMED ATIAA KAREEM ALNASHY
-------------------------------------------------------------------
CHAPTER THIRTY-NINE: MANIFESTATIONS OF SYSTEMIC
DISEASES OF THE NOSE
--------------------------------------------------------------------------------------------

1. With regard to anti-neutrophil cytoplasmic antibody (ANGA) testing


and Wegener's granulomatosis (WG), which of the following
statements is incorrect?
A. A negative c-ANGA test does not exclude the diagnosis of WG.
B. c-ANGA specificity is >90% during the systemic vasculitic phase of the
disease.
C. The characteristic c-ANGA pattern is caused by antibodies against
proteinase 3 (PR3).
D. Perinuclear-ANCA testing is superior to c-ANGA testing in the
diagnosis of WG.

2. In patients with large, dry septal perforations without evidence of


ulceration and in whom histopathologic findings are nonspecific and
c-ANGA testing is negative, the most likely cause of the nasal changes is
A. Wegener's granulomatosis
B. T-cell lymphoma
C. Sarcoidosis
D. Tuberculosis
E. Substance abuse

3. Which of the following laboratory results is incorrect in the diagnosis


of patients suspected of having sarcoidosis?
A. Hypocalcemia
B. Hypercalciuria
C. High number of T lymphocytes in bronchoalveolar lavage fluid
D. Elevated angiotensin-converting enzyme
E. Increased levels of sIL-2R serum

4. All of the following are true about atypical mycobacterial infections except
A. Purified protein derivative (PPD) skin testing is often negative.
B. Patients classically are initially seen with cervical adenopathy.
C. Causative organisms can inevitably be cultured from biopsy specimens.
D. Auramine-rhodamine staining is a useful initial step in the diagnosis.
E. Nasal involvement is typified by anterior septal perforations.

146
BY DR. MOHAMMED ATIAA KAREEM ALNASHY
-------------------------------------------------------------------
5. A 20-year-old patient is seen with progressive unilateral nasal
destruction with involvement of the adjacent maxillary sinus and early
involvement of the oral cavity. ANCA testing is negative. Biopsy shows
cells with angiocentric and angioinvasive features. The most likely
diagnosis is
A. Wegener's granulomatosis
B. Sarcoidosis
C. T-cell lymphoma
D. Atypical mycobacterial infection
E. Histiocytosis X

CHAPTER 39

Manifestations of Systemic Diseases of the Nose

1. A

2B

3. A

4. A

5. E

147
BY DR. MOHAMMED ATIAA KAREEM ALNASHY
-------------------------------------------------------------------
CHAPTER FORTY: EPISTAXIS
-----------------------------------------------------------------------------------------

1. Epistaxis presenting in a delayed fashion after maxillofacial trauma


should raise suspicion for
A. Undiagnosed nasal fracture
B. Arterial aneurysm
C. Continued traumatic insult to nasal mucosa
D. Altered nasal airflow
E. Bacterial infection

2. The most common familial bleeding disorder to consider in frequent,


difficult-to-manage epistaxis is
A. Hemophilia A
B. Hemophilia B
C. von Willebrand's disease
D. Thrombocytopenia
E. Osler-Weber-Rendu disease

3. In managing epistaxis, posterior nasal packing


A. Should be the first-line therapy
B. Should be avoided
C. Should not be performed with anterior nasal packing
D. Requires the patient to be monitored for hypoxia, apnea, and arrhythmias
E. May predispose to hypocarbia

4. Signs of fever, nausea, vomiting, and diarrhea in the presence of nasal


packing may indicate
A. Toxic shock syndrome
B. Streptococcal infection
C. Viral infection
D. Contamination of the packing material
E. Anaphylactic reaction to packing material

148
BY DR. MOHAMMED ATIAA KAREEM ALNASHY
-------------------------------------------------------------------
5. The most effective laser for treatment of telangiectasias in Osler-
Weber-Rendu disease is
A. Nd:YAG
B. C02
C. KTP
D. Argon
E. Pulse dye laser

CHAPTER 40

Epistaxis

1B

2. C

3D

4. A

5. E

149
BY DR. MOHAMMED ATIAA KAREEM ALNASHY
-------------------------------------------------------------------
CHAPTER FORTY-ONE: NASAL FRACTURES
-------------------------------------------------------------------------------------------

1. A 25-year-old man is seen in the emergency department 3 hours after


being struck in the nose with a basketball during a game. Examination
reveals significant midfacial edema, ecchymosis, and crepitance of the
nasal pyramid. The remainder of the head and neck examination is
normal. What is the next best step in management?
A. Plain films of the nasal bones
B. Computed tomography scan of the maxillofacial skeleton
C. Administration of local sedation followed by closed nasal reduction
D. Outpatient follow-up in 48 to 72 hours to reassess nasal structure
E. Open septorhinoplasty in 6 months

2. What is the most common reason for failure after a closed nasal
reduction (CNR) performed within 1 week of injury?
A. Nasal bone comminution with poor underlying support
B. Fibrous tissue formation between bony fragments
C. Nasal septal fracture
D. Additional mid-facial fractures
E. Greenstick fracture of the nasal bones

3. Management of deviated nasal fractures in children should include all


of the following except
A. Early closed nasal reduction (<4 days)
B. General anesthesia during closed nasal reduction
C. Immediate incision and drainage of septal hematoma
D. Nasal stenting after closed nasal reduction for infants less than 6
months of age
E. Open reduction for injuries with deviation greater than one-half the
width of the nasal bridge

150
BY DR. MOHAMMED ATIAA KAREEM ALNASHY
-------------------------------------------------------------------
4. (Case) A 34-year-old woman is seen with a nasal deformity after a high-
speed car accident. Examination reveals a flattened dorsum and
widening between the inner canthi. What approach is best for repair of
the injury?
A. Bicoronal scalp flap
B. Lateral rhinotomy
C. "Open sky" incision
D. Intranasal, intercartilaginous incisions
E. Open rhinoplasty approach (bilateral marginal and transcolumellar
incisions)

5. Which of the following statements regarding nasal septal hematomas


is not true?
A. Children are more likely than adults to have septal hematomas develop
after nasal injury.
B. Hematoma collection results in cartilage necrosis in 3 days.
C. Nasal septal hematomas often produce long-term complications such as
saddle nose deformity, perforation, and columellar retraction.
D. Nasal septal hematomas often appear as blue, noncompressible
intranasal masses.
E. An untreated nasal septal hematoma can lead to an intracranial
infection.

151
BY DR. MOHAMMED ATIAA KAREEM ALNASHY
-------------------------------------------------------------------
CHAPTER 41
Nasal Fractures

1D
Radiographs are not indicated for evaluation of nasal fractures. GT scans
may be used when an injury associated with a high velocity impact has
occurred or when physical findings are indicative of additional maxillofacial
fractures. Immediate closed nasal reduction is appropriate when the patient
is seen soon after the trauma and when nasal edema does not obscure nasal
bone position. For the remaining cases, outpatient reassessment is performed
48 to 72 hours once swelling has subsided and the results of a delayed closed
reduction can be more easily determined.

2. C
Nasal bone comminution, surrounding facial fractures, and greenstick
fractures all can be potential causes for persistent nasal deviation after CNR.
They should be considered after a nasal septal fracture has been ruled out.
Fibrous tissue formation associated with healing is generally not significant
in adults less than a week after injury.

3. E
Because of faster rates of healing, closed nasal reduction should be performed
early in children when edema no longer obscures nasal bone position. Unless
there is a medical contraindication, general anesthesia should be used for
most pediatric cases to ensure patient comfort and ease of reduction for the
surgeon. Nasal septal hematomas are considered emergent conditions that
should be drained immediately on diagnosis. Pediatric nasal fractures should
be treated conservatively to avoid further disruption of important growth
centers. Open techniques should only be considered for extensive fractures
(i.e., nasal-orbital-ethmoid fractures) or when closed reduction cannot
reasonably reduce the deformity.

152
BY DR. MOHAMMED ATIAA KAREEM ALNASHY
-------------------------------------------------------------------
4. A
Bicoronal scalp flaps are a good choice for exposure of nasal-orbital-ethmoid
fractures, especially in cases when a split calvarial bone graft may be
necessary to reconstruct severely comminuted nasal bones. Lateral rhinotomy
incisions are only useful in unilateral nasal injuries. Both lateral rhinotomy
and open-sky incisions leave conspicuous facial scarring that may not be
desirable. An intranasal incision or open rhinoplasty approach
does not provide adequate exposure in this case and would not allow for proper
reduction and fixation.

5d
Children are more likely than adults to acquire nasal septal hematomas
because of softer nasal tissues that are more susceptible to shear forces. The
hematoma collection under the mucoperichondrium separates the tenuous
blood supply to the quadrangular cartilage, resulting in necrosis within 3
days. This destruction then often leads to significant internal and external
deformities. Nasal septal hematomas tend to be compressible masses and
usually are not discolored. Bacterial seeding of a hematoma can result in
abscess formation that then has the ability to spread to contiguous areas
including the intracranial vault.

153
BY DR. MOHAMMED ATIAA KAREEM ALNASHY
-------------------------------------------------------------------
CHAPTER FORTY-TWO: ALLERGIC RHINITIS
-----------------------------------------------------------------------

1. Which statement most accurately describes the importance of allergy


in the practice of otolaryngology—head and neck surgery?
A. It will be of no importance to the average otolaryngologist.
B. It will rarely be encountered in a subspecialty practice such as otology
or pediatric otolaryngology.
C. It is responsible for a small fraction of healthcare costs in the United
States.
D. It may represent a primary or contributory diagnosis in up to half the
patients seen by the general otolaryngologist.
E. It will frequently require referral to an allergist, because it is outside
the capabilities of a surgical subspecialty.

2. The most important type of allergy encountered by the otolaryngologist,


as classified by Gell and Coombs, is
A. Perennial
B. Delayed
C. Immediate
D. Seasonal
E. Cytotoxic

3. The most important tool in making the diagnosis of allergy is


A. A positive skin prick test
B. A positive intradermal skin test
C. A positive history for symptoms associated with exposure
D. A total IgE >100 IU/mL
E. An mRAST value of class II or higher

154
BY DR. MOHAMMED ATIAA KAREEM ALNASHY
-------------------------------------------------------------------
4. Which of the following statements is most accurate regarding
treatment of allergy?
A. Most patients can be controlled with the use of antihistamines, which
are now available without a prescription.
B. Any patient with positive skin and/or in vitro tests for allergen-specific
IgE should receive allergen immunotherapy.
C. Topical nasal corticosteroids should be used daily by patients with
allergic rhinitis and are safe for long-term use.
D. The best method of managing inhalant allergy is environmental
control, and the most avoidable allergens are the perennial group: dust
mite, mold, and animal danders.
E. Leukotriene modifiers attack the allergic reaction at its source and
should be first-line therapy for patients with allergic rhinitis.

5. Which statement regarding immunotherapy is most accurate?


A. Candidates for immunotherapy are cooperative patients with proven
IgE-mediated allergy, in whom pharmacotherapy and/or avoidance are
ineffective or impractical, producing symptoms that are severe.
B. Monoclonal antibody (anti-IgE) therapy given as a 3- to 5-year therapy
may produce the same effects as conventional immunotherapy.
C. Inhalant immunotherapy is indicated in patients with seasonal rather
than perennial allergy.
D. The risk of anaphylaxis associated with immunotherapy on the basis of
quantitative testing is so negligible that it may be administered by a
minimally trained assistant in any office setting.
E. The technique of immunotherapy involves administering progressively
smaller doses of antigen to modify the patient's immune response to
antigen challenge.

155
BY DR. MOHAMMED ATIAA KAREEM ALNASHY
-------------------------------------------------------------------
CHAPTER 42 Allergic Rhinitis

1. D
Allergy may affect all aspects of the ear, nose, and throat, including otitis media,
Meniere's disease, rhinosinusitis, laryngitis, chronic sore throats, as well as asthma
and chronic cough. All otolaryngologists should be able to suspect allergy based on
history, prescribe appropriate pharmacotherapy, and give advice regarding empiric
avoidance measures. Depending on the training they have received, otolaryngologists
should be able to either refer patients for appropriate immunotherapy or administer
it themselves.

2. C
The Gell and Coombs type I (immediate or anaphylactic) reaction is the mechanism
of allergic rhinitis (hay fever) commonly encountered by the otolaryngologist. In
addition, it is the mechanism of hypersensitivity reactions to drugs, contrast materials,
and insect stings. Thus, it is important to understand it, be able to recognize it, and
treat it appropriately.

3. C
A high total IgE does not necessarily indicate the presence of allergy. Skin tests and
RAST (and other in vitro) tests demonstrate the presence of allergen-specific IgE.
However, the sine qua non of allergy is the production of specific and typical
symptoms on exposure to one or more allergens.

4. D
It is generally accepted that prevention of the allergic reaction is much preferred to
treating its consequences. Although immunotherapy may also provide protection,
avoidance remains the best and safest treatment when it is feasible. Although allergy
patients will generally require "rescue medications," no specific type of
pharmacotherapy is universally effective or applicable, and all methods have
drawbacks.

5A
Because immunotherapy carries a risk of severe reactions, even though the likelihood
of anaphylaxis is small when quantitative testing is used, it should not substitute for
simpler and safer measures such as environmental control and pharmacotherapy. It
offers benefits to patients who have perennial allergy and those with seasonal
symptoms (typically it is best used in patients with symptoms covering several seasons
or severe single season symptoms). The use of anti-IgE shows promise, because
provides nonspecific results (regardless of allergens involved) and must only be given
over a short period of time.

156
BY DR. MOHAMMED ATIAA KAREEM ALNASHY
-------------------------------------------------------------------
CHAPTER FORTY-THREE: NONALLERGIC RHINITIS
--------------------------------------------------------------------------------------------

1. Nonallergic rhinitis usually presents with all of the following


symptoms except
A. Rhinorrhea
B. Nasal congestion
C. Nasal osbstruction
D. Itching
E. Negative skin testing

2. The pathophysiology of nonallergic rhinitis may involve


A. Underactivity of the afferent reflex pathway
B. Underactivity of the efferent reflex pathway
C. Excess secretion of mucous and congestion by the sympathetic system
D. Excess secretion of mucous and congestion by the parasympathetic
system
E. Inhibition of G-fibers

3. Rhinitis of pregnancy may be related to increased vascular volume and


vascular relaxation due to what hormone?
A. Estrogen
B. Progesterone
C. Placental growth factor
D. Human chorionic gonadotropin
E. Luteinizing hormone

4. Which of the following statements about atrophic rhinitis is false?


A. May be related to granulomatous diseases
B. Aggressive nasal surgery
C. Associated with aging
D. Presents with crusting and foul odor
E. Demonstrates preservation of nasal airflow

157
BY DR. MOHAMMED ATIAA KAREEM ALNASHY
-------------------------------------------------------------------
5. Gustatory rhinitis appears related to
A. Food allergy
B. Excess sympathetic activity
C. Stimulation of afferent sensory nerves
D. Positive skin testing
E. Altered mucociliary clearance

CHAPTER 43

Nonallergic Rhinitis

1. D

2. D

3. B

4. E

5. C

158
BY DR. MOHAMMED ATIAA KAREEM ALNASHY
-------------------------------------------------------------------
CHAPTER FORTY-FOUR: THE NASAL SEPTUM
-------------------------------------------------------------------------------------------

1. During which week of embryonic development does the nasobuccal


membrane rupture and by doing so allow communication of the nasal
passage and the nasopharynx?

2. Describe the most accurate method(s) for assessment of the nasal valve
angle.

3. Which septoplasty incision is the best for addressing the deviated


caudal septum and least likely to disrupt tip support mechanisms?

4. Which two operative maneuvers during septoplasty decrease the risk of


permanent perforation?

5. To straighten the significantly bowed septum, which operative


maneuver is paramount in this effort?

159
BY DR. MOHAMMED ATIAA KAREEM ALNASHY
-------------------------------------------------------------------
CHAPTER 44
The Nasal Septum

1. During the 10th week of embryonic development, the nasobuccal


membrane ruptures and creates a communication between the nasal
passages and the nasopharynx. Failure of this membrane to rupture
may result in the membranous variant of posterior choanal atresia.

2. Nasal valve compromise can be appreciated by visual inspection, use of


the Cottle maneuver, or by lateralization of the ULC. Lateralization of
the ULC by insertion of a cotton-tipped applicator or a thin metal
curette provides perhaps the best assessment tool.

3. The hemi-transfixion incision allows superb access to the caudal


septum and minimally disrupts supporting tip structures. Conversely,
the full-transfixion incision and the open approach to the septum
significantly disrupt tip support and are used primarily when septal
perforation repair is being performed, or in conjunction with an
external rhinoplasty procedure. The Killian incision created within the
respiratory epithelium does not allow access to the caudal septal edge.

4. Perforations are avoided by careful membrane elevation and closure of


all membrane rents that may occur. Interposition of crushed cartilage
bolsters the repair and ensures against permanent perforation.

5. The significantly bowed septum is approached by first freeing the septal


floor from the maxillary crest and creating a superiorly based swinging
door. The septum can then be replaced into the midline and secured
with suture to the periosteum of the nasal spine.

160
BY DR. MOHAMMED ATIAA KAREEM ALNASHY
-------------------------------------------------------------------
CHAPTER FORTY-FIVE: RHINOPLASTY
--------------------------------------------------------------------------------------------
1. The proper favorable tissue plane in which dissection should be carried
out when uncovering the nose is located
A. Immediately subcutaneous
B. Within the superficial musculoaponeurotic tissue layer
C. Beneath the periosteum
D. Between the SMAS layer and the cartilaginous structure of the nose
E. In the fatty tissue plane beneath the dermis

2. Wide dome angles and a broad interdomal distance should be narrowed


and refined during tip rhinoplasty by
A. Transdomal sutures and/or single dome sutures
B. Division of the dome angles with suture-repair
C. Interdomal sutures only
D. Resection of the interdomal intermediate crura
E. Removal of the upper half of the lateral crura

3. Preoperative evaluation of the surgical needs of the nasal tip should


include all but which of the following
A. Need for cephalic rotation
B. Need for nasal spine reduction
C. Need for narrowing of the dome angle
D. Need for projection change
E. Need for interdomal distance change

4. The complete transfixion incision


A. Promotes cephalic rotation
B. Corrects a caudal septal deflection
C. Helps to narrow a wide tip
D. Is always combined with an intercartilaginous incision
E. Results in tip retroprojection

161
BY DR. MOHAMMED ATIAA KAREEM ALNASHY
-------------------------------------------------------------------
5. Which of the following is not true about microosteotomes?
A. Less trauma results from the use of 2- or 3-mm osteotomies.
B. Microosteotomes should be used only for percutaneous osteotomies.
C. Some periosteum is left intact after microosteotomes.
D. Microosteotomes do not require guards.
E. Lateral osteotomies with microosteotomes should begin at or just above
the junction of the inferior concha with the lateral nasal wall.

162
BY DR. MOHAMMED ATIAA KAREEM ALNASHY
-------------------------------------------------------------------
CHAPTER 45
Rhinoplasty

1D
Dissection of the soft tissues covering the nasal dorsum is best carried out in
the favorable tissue dissection plane between the overly SMAS fascia covering
and the underlying cartilage and bone of the supporting structures of the nose.
If dissection proceeds within the SMAS layer, various arteries, veins, nerves,
and lymphatics are damaged, leading to increased intraoperative bleeding,
swelling, and prolonged healing.

2. A
If overwide domal angles combined with a wide interdomal distance (bifidity)
is found in the nasal tip, suture reorientation of the domal angles and wide
tip defining points is recommended as the most effective and safe tip
technique. Vertical division of the domes runs the very real risk of
asymmetric healing and loss of tip support.

3B
Although noses are encountered in which the nasal spine is overlarge or even
deviated, and thus requires correction, surgery of the nasal tip size and shape
per se is not principally affected by the nasal spine. The spine may, however,
play a role in nasal tip projection or deviation and require alteration.

4. E
A complete transfixion separates the medial crural footplates from the caudal
septum, which in most patients is a major tip support
mechanism. Tip retroprojection ordinarily results from this incision, and
thus it is often the initial step in retroprojecting an overprojecting tip.

5B
Although microosteotomes are preferred for percutaneous osteotomies, the
latter are not the only circumstance in which microosteotomes are useful.
These small osteotomes create less damage to the nasal side walls, produce less
bleeding and swelling, and thus aid in rapid healing.

163
BY DR. MOHAMMED ATIAA KAREEM ALNASHY
-------------------------------------------------------------------
CHAPTER FORTY-SIX: SPECIAL RHINOPLASTY TECHNIQUES
--------------------------------------------------------------------------------------------

1. If a patient requires osteotomies, the order in which they should be


performed is
A. Lateral, intermediate, medial
B. Medial, intermediate, lateral
C. Lateral, medial, intermediate
D. Medial, lateral, intermediate
E. Intermediate, medial, lateral

2. In the twisted nose, a C-shaped deformity can be corrected by the use of


A. A spreader graft between the upper lateral cartilage and the septum on
the concave side
B. A spreader graft between the upper lateral cartilage and the septum on
the convex side
C. Placement of an onlay graft on the concave side
D. A and B
E. A and C

3. In rhinoplasty, the DART graft is used to


A. Camouflage columellar asymmetries
B. Increase tip rotation and projection
C. Correct an acute nasolabial angle
D. Correct a shallow nasofrontal angle
E. Derotate the nose and increase tip projection

4. Wier incisions are used to


A. Narrow a wide alar base
B. Increase tip projection
C. Lengthen the columellar skin
D. Decrease the convexity of the lateral ala
E. Augment the nasal dorsum

164
BY DR. MOHAMMED ATIAA KAREEM ALNASHY
-------------------------------------------------------------------
5. Anatomic characteristics of the unilateral cleft lip-nasal deformity
include
A. Cleft ala displaced laterally, inferiorly, and posteriorly
B. Dislocation of the caudal end of septum toward the noncleft side
C. Tip deflected toward the noncleft side
D. Bony deficiency of the maxilla on the cleft side
E. All of the above

CHAPTER 46

SPECIAL RHINOPLASTY TECHNIQUES

1. B
2. E
3.
4.
5.

165
BY DR. MOHAMMED ATIAA KAREEM ALNASHY
-------------------------------------------------------------------
CHAPTER FORTY-SEVEN: REVISION RHINOPLASTY
------------------------------------------------------------------------------------------

1. A patient is seen with supra-alar pinching and alar retraction. A common


cause of this deformity is
A. Inadequate osteotomies
B. Over-resected lower lateral crura
C. Improper graft placement
D. Excessive hump removal
E. Avulsion of the upper lateral cartilages

2. The base view provides information about all of the following except
A. Shape of columella
B. Size of columella
C. Alar base
D. Radix
E. Lobule

3. The nasolabial angle in men should be


A. 65 to 75 degrees
B. 75 to 85 degrees
C. 90 to 95 degrees
D. 100 to 110 degrees
E. 110 to 120 degrees

4. When harvesting costal cartilage, the ribs most commonly used are the
A. First and second
B. Third and fourth
C. Fifth and sixth
D. Seventh and eighth
E. Eleventh and twelfth

5. An open roof deformity can occur after


A. Bony hump removal with inadequate osteotomies
B. Excessive resection of lower lateral crura
C. Avulsion of the upper lateral cartilages
D. Excessive soft tissue in the supratip
E. Over-resection of septal cartilage

166
BY DR. MOHAMMED ATIAA KAREEM ALNASHY
-------------------------------------------------------------------
CHAPTER 47
Revision Rhinoplasty

1. B
Overresection of the lower lateral crura causes lack of support of the supratip
area. A common deformity seen with this is supra-alar pinching and alar
retraction. On lateral view one should see 2 to 4 mm of columellar show.

2D
Preoperative photography is essential in rhinoplasty. It is useful during the
initial consultation with the patient, as well as postoperative follow-up. The
base view provides information about the size and shape of the columella, alar
base, nostrils, and the lobule. In a true base view, the tip should obscure the
radix.

3. C
The nasolabial angle in men is should be between 90 and 95 degrees, and in
women it is between 95 and 105 degrees. Depending on the amount of tissue
excess or deficiency at the premaxilla, this angle may not reflect the amount
of rotation at the tip and infratip lobule.

4D
Costal cartilage is most commonly taken from the seventh, eighth, or ninth
ribs. The medial portion of the rib is taken, leaving the inner perichondrium
intact to prevent entry into the pleural space.

5. A
An open roof deformity occurs when a bony hump is removed and the
osteotomies do not adequately medialize the nasal bones. If standard lateral
osteotomies do not sufficiently mobilize the bones, then one can try
percutaneous osteotomies.

167
BY DR. MOHAMMED ATIAA KAREEM ALNASHY
-------------------------------------------------------------------
CHAPTER FORTY-EIGHT: RECONSTRUCTIVE RHINOPLASTY
-------------------------------------------------------------------------------------------

1. In selecting the donor site for a full-thickness skin graft of a nasal tip
defect, which of the following areas of skin matches most closely the
thickness, color, and texture of nasal skin?
A. Nasolabial
B. Supraclavicular
C. Postauricular
D. Upper eyelid
E. Thigh

2. Which of the following statements regarding the forehead flap is false?


A. The forehead flap is primarily based on the supraorbital artery.
B. The forehead flap is an axial flap.
C. The donor site can usually be closed primarily when the defect is less
than 3.5 cm wide.
D. The forehead flap is elevated in the supragaleal plane.
E. The distal end of the flap can usually be thinned without injuring the
pedicles.

3. Which of the following statements regarding composite grafting is false?


A. Graft size is critical to survival.
B. Survival depends on transfer of fluids and nutrients from graft edges
immediately after placement.
C. The auricular donor site is associated with minimal morbidity.
D. Composite grafting should not be considered when a graft larger than 1
cm is required.

4. Which of the following statements regarding calvarial bone grafting is


false?
A. Grafts are usually harvested from the parietal region.
B. Grafts can be harvested with lower donor site morbidity.
C. Grafts are usually harvested from the outer cortex of the cranium.
D. Grafts resist resorption because of their endochondral origin.

168
BY DR. MOHAMMED ATIAA KAREEM ALNASHY
-------------------------------------------------------------------
5. Which of the following is the material of choice in reconstructing the
nasal dorsum?
A. Calvarial bone
B. Iliac bone
C. Rib
D. Irradiated cartilage
E. Alloplastic implants

CHAPTER 48

Reconstructive Rhinoplasty

1. A

2. A

3D

4D

5A

169
BY DR. MOHAMMED ATIAA KAREEM ALNASHY
-------------------------------------------------------------------

PART FOUR
--------------------------------------------------------------------------------------------
--------------------------------------------------------------------------------------------
PARANASAL SINUSES

170
BY DR. MOHAMMED ATIAA KAREEM ALNASHY
-------------------------------------------------------------------
CHAPTER FORTY-NINE: RADIOLOGY OF THE NASAL CAVITY
AND PARANASAL SINUSES

--------------------------------------------------------------------------------------------

1. Which of the following is false with regard to the agger nasi cell?
A. It is an ethmoturbinal remnant.
B. It is present in about half of patients.
C. It is usually aerated.
D. Its roof usually borders the ostium or floor of the frontal sinus.
E. Its size directly influences the size of the frontal sinus drainage tract.

2. Which of the following is false with regard to the uncinate process?


A. It is part of the ethmoid bone.
B. It is part of the lateral nasal wall.
C. It contacts the agger nasi cell.
D. It defines the infundibulum.
E. It is lateral to the hiatus semilunaris.

3. The sinus lateralis is the


A. Space between the uncinate and the ethmoid bulla
B. Space between the posterior ethmoid and the basal lamella
C. Space between the ethmoid bulla and the basal lamella
D. Space between the agger nasi cell and the middle turbinate when the
turbinate inserts on the cribriform plate

4. Radiographic signs of chronic sinusitis include all of the following except


A. Drainage of intermediate attenuation on CT
B. Thickening of the bony walls
C. Opacification of the middle meatus
D. Mucoperiosteal thickening of the maxillary sinus
E. Hyperintense drainage on Tl-weighted MRI images

171
BY DR. MOHAMMED ATIAA KAREEM ALNASHY
-------------------------------------------------------------------
5. Which of the following radiographic findings is believed to be
associated with inflammatory sinusitis?
A. Haller cells
B. Uncinate pneumatization
C. Horizontal orientation of the uncinate process
D. Paradoxic turbinates
E. Concha bullosa

CHAPTER 49

Radiology of the Nasal Cavity and Paranasal Sinuses

1. B

2. E

3. C

4. A

5. C

172
BY DR. MOHAMMED ATIAA KAREEM ALNASHY
-------------------------------------------------------------------
CHAPTER FIFTY: INFECTIOUS CAUSES OF RHINOSINUSITIS
--------------------------------------------------------------------------------------------

1. What fungus is most commonly responsible for invasive fungal


sinusitis in uncontrolled diabetics?
A. Rhizopus oryzae (mucormycosis)
B. Aspergillus fumigatus
C. Aspergillus flavus
D. Candida albicans
E. Alternaria

2. The patient with a sinus computed tomography showing mucosal


thickening occluding the ostiomeatal complex
A. Requires endoscopic sinus surgery
B. May have a cold, a bacterial sinus infection, nasal polyps, or an
irreversible obstruction of the maxillary infundibulum
C. Should be treated with a broad-spectrum antibiotic
D. Requires a culture-directed antibiotic and nasal steroid sprays

3. The histologic findings in allergic fungal sinusitis are


A. Allergic (eosinophilic) mucin with hyphae present
B. Fungal invasion of the epithelium but not of deeper structures
C. A tangled mass of hyphae
D. Eosinophilic mucin with Charcot-Leyden crystals but no hyphae present

4. The most common bacterial causes of acute rhinosinusitis are


A. Staphylococcus aureus and anaerobes
B. Streptococcus viridans, S. aureus, and anaerobes
C. Haemophilus influenza and Streptococcus pneumoniae
D. Pseudomonas and coagulase-negative staphylococci

5. Complications of bacterial sinusitis include all of the following except


A. Pseudotumor cerebri
B. Subperiosteal orbital abscess
C. Cavernous sinus thrombosis
D. Epidural abscess

173
BY DR. MOHAMMED ATIAA KAREEM ALNASHY
-------------------------------------------------------------------
CHAPTER 50

Infectious Causes of Rhinosinusitis

lA

2. B

3. A

4. C

5. A

174
BY DR. MOHAMMED ATIAA KAREEM ALNASHY
-------------------------------------------------------------------
CHAPTER FIFTY-ONE: NEOPLASMS
------------------------------------------------------------------------------------------

1. Ohngren's line is an important delineator of prognosis in the management


of carcinoma of the maxillary sinus. This imaginary line is described as
a plane created by the intersection of a line drawn between
A. The tip of the nose and the tragus
B. The medial canthus and the angle of the jaw
C. The nasal tip and the angle of the jaw

2. A maxillary sinus carcinoma involving the inferior lateral superior and


medial walls of the maxillary sinus and the anterior ethmoid would be
classified
A. T1
B. T2
C. T3
D. T4

3. The first-echelon nodal drainage for tumors of the nasal space and
maxillary sinus is
A. Lateral retropharyngeal node
B. Facial node
C. Parotid node
D. Jugulodigastric node

4. The imaging study most likely to provide accurate information


regarding tumor extension intracranially or intraorbitally is
A. Ultrasonography
B. Computed tomography scan
C. Magnetic resonance imaging
D. Positron emission tomography scan

5. Which of the following is the most helpful in controlling postoperative


cerebrospinal fluid leakage?
A. Antibiotics
B. Postoperative radiation
C. Tissue glue
D. Skull base reconstruction with soft tissue or bone

175
BY DR. MOHAMMED ATIAA KAREEM ALNASHY
-------------------------------------------------------------------
CHAPTER 51
Neoplasms

1. B

2. C

3. A

4. C

5. no answer

176
BY DR. MOHAMMED ATIAA KAREEM ALNASHY
-------------------------------------------------------------------
CHAPTER FIFTY-TWO: MEDICAL MANAGEMENT OF NASOSINUS
INFECTIOUS AND INFLAMMATORY DISEASE
--------------------------------------------------------------------------------------------

1. In the treatment of allergic rhinitis, cromolyn preparations


A. Should be used after an attack occurs
B. Are effective for nasal congestion
C. Effectively treat sneezing and rhinorrhea
D. Should not be used with antihistamines
E. Have long duration of action

2. All of the following are true about nasal steroids except


A. Are contraindicated in infectious sinusitis.
B. More potent nasal steroids like fluticasone may have some pituitary
suppression.
C. Should be used cautiously in elderly patients taking inhaled steroids.
D. Septal perforation is a rare side effect.
E. Are effective against acute and late-phase effects.

3. Which of the following about macrolide antibiotics is true?


A. Chronic use often leads to resistance.
B. Are recommended primarily for acute sinusitis.
C. Should not be used in conjunction with nasal steroids.
D. Have important antiinflammatory effects
E. Have no role in treating sinusitis.

4. Children may have a physiologic immunodeficiency predisposing them


to chronic rhinosinusitis until what age?
A. 6
B. 8
C. 10
D. 12
E. 14

177
BY DR. MOHAMMED ATIAA KAREEM ALNASHY
-------------------------------------------------------------------
5. Patients with allergic fungal sinusitis have all of the following except
A. Allergic mucin
B. Nasal polyps
C. Atopy
D. Fungal allergies
E. Immunodeficiency

CHAPTER 52

Medical Management of Nasosinus Infectious and Inflammatory Disease

1. C

2. A

3D

4D

5. E

178
BY DR. MOHAMMED ATIAA KAREEM ALNASHY
-------------------------------------------------------------------
CHAPTER FIFTY-THREE: PRIMARY SINUS SURGERY
--------------------------------------------------------------------------------------------

1. Anatomic variations of the paranasal sinuses that may predispose a


surgeon to inadvertent penetration of the orbit or the anterior cranial
cavity include
A. Lamina papyracea lying medial to the maxillary ostium
B. Maxillary sinus hypoplasia
C. Fovea ethmoidalis abnormalities such as low or sloping fovea
D. Lamina papyracea dehiscence
E. All of the above

2. Absolute indications for endoscopic sinus surgery include all of the


following except
A. Mucoceles
B. Headaches
C. CSF rhinorrhea
D. Complications of rhinosinusitis
E. Tumors

3. Dividing the infundibulum into thirds, the natural ostium to the


maxillary sinus will most commonly be found where?
A. Superior third
B. Middle third
C. Inferior third
D. Outside infundibulum
E. None of the above

4. The most common minor complication after endoscopic sinus surgery is


A. Headache
B. Hyposmia
C. Synechia
D. Periorbital ecchymosis
E. Periorbital emphysema

179
BY DR. MOHAMMED ATIAA KAREEM ALNASHY
-------------------------------------------------------------------
5. The uncinate process can have all of the following superior attachments
EXCEPT
A. Superior turbinate
B. Lamina papyracea
C. Skull base
D. Middle turbinate
E. None of the above

180
BY DR. MOHAMMED ATIAA KAREEM ALNASHY
-------------------------------------------------------------------
CHAPTER 53
Primary Sinus Surgery

1. E
In a study by Meyers and Valvassori, 400 preoperative CT scans were reviewed with
attention to anatomic variations. They found six specific variations that may
predispose a surgeon to inadvertent penetration of the orbit or the anterior cranial
cavity.
These variations include
(1) lamina papyracea lying medial to the maxillary ostium;
(2) maxillary sinus hypoplasia;
(3) fovea ethmoidalis abnormalities, such as low or sloping fovea;
(4) lamina papyracea dehiscence;
(5) sphenoid sinus wall variations, such as septa attached to the carotid, or dehiscence
of the carotid or optic nerve;
(6) sphenoethmoid cells.
(Meyers RM, Valvassori G. Interpretation of anatomic variations of computed
tomography scans of the sinuses: a surgeon's perspective. Laryngoscope 108[3]:422-425,
1998.)

3. C
Dividing the infundibulum into thirds, Van Alyea found the ostium to be in the
superior third in 10% of cases, middle third in 25% of cases, and inferior third in 65%
of cases.
(Van Alyea OE . The ostium maxillare. Anatomic study of its surgical accessibility.
Arch Otolaryngol 24:553-569, 1936.)

4. C
Minor complications such as hyposmia, headache, periorbital ecchymosis, periorbital
emphysema, and facial pain can all occur. The most common minor complication is
the formation of synechia, which usually does not require revision surgery.
(Stammberger H, Posawetz W. Functional endoscopic sinus surgery. Concept,
indications and results of the Messerklinger technique. Eur Arch Oto-Rhino-
Laryngol 247[2]:63-76, 1990.)

5. A
Superiorly, the uncinate process has three possible attachments: the lamina papyracea,
skull base, or middle turbinate. These variants are important to identify preopera-
tively because of variations in frontal sinus drainage. (Stammberger HR, Kennedy
DW. Paranasal sinuses: anatomic terminology and nomenclature. The Anatomic
Terminology Group. Ann Otol Rhinol Laryngol Suppl 167:7, 1995.)

181
BY DR. MOHAMMED ATIAA KAREEM ALNASHY
-------------------------------------------------------------------
CHAPTER FIFTY-FOUR: REVISION ENDOSCOPIC SINUS SURGERY
--------------------------------------------------------------------------------------------

1. Which of the following is not necessary to make the diagnosis of


chronic rhinosinusitis?
A. CT scans with mucosal thickening
B. Culture result for resistant bacteria
C. History with length of symptoms greater than 3 months
D. Characteristic findings on nasal endoscopy

2. Which of the following is not a common reason for failure of maxillary


antrostomy?
A. Missed ostium sequence/recirculation phenomenon
B. Infected secretions dropping into maxillary sinus "catch basin"
C. Scarring of nasolacrimal duct from antrostomy that is too anterior
D. Scarred over maxillary sinus ostium
E. Retained foreign body in the sinus

3. Important landmarks in revision sinus surgery include all the


following except
A. Lateral nasal wall/lamina papyracea
B. Posterior wall maxillary sinus
C. Superior turbinate
D. Anterior wall maxillary sinus

4. Important considerations in preoperative management of the chronic


rhinosinusitis patient include
A. Complete delineation of host and environmental factors
B. Judicious use of antiinflammatories, especially corticosteroid nasal
sprays and oral corticosteroids before surgery
C. Empiric antimicrobials, especially broad-spectrum antibiotics to cover
gram-negative organisms and anaerobes
D. Comprehensive history, including the symptoms that brought the
patient to a primary surgery

182
BY DR. MOHAMMED ATIAA KAREEM ALNASHY
-------------------------------------------------------------------
5. MRI of the sinuses should be obtained in all these situations except
A. Tumor
B. Opacification against skull base
C. Opacified sphenoid sinus
D. Dehiscent bone along skull base
E. To evaluate mucosal disease

183
BY DR. MOHAMMED ATIAA KAREEM ALNASHY
-------------------------------------------------------------------
CHAPTER 54
Revision Endoscopic Sinus and Surgery

1.B
Culture results are not required to make the diagnosis of chronic
rhinosinusitis but may provide a useful adjunct for therapy. History alone is
now not make the diagnosis of chronic rhinosinusitis. Objective findings on
CT scans and nasal endoscopy are required adjuncts to appropriate history to
make the diagnosis of chronic rhinosinusitis.

2. C
Although involvement/scarring of the nasolacrimal duct can commonly occur
when the maxillary ostium is widened too far anterior with a backbiter, it
rarely causes recurrent chronic rhinosinusitis.

3D
The lateral wall and lamina papyracea are important first landmarks to
identify in revision endoscopic sinus surgery, because identifying these
landmarks will help ensure protection against orbital injury. The posterior
wall of the maxillary sinus is often at the same depth as the sphenoid face,
which helps gauge the depth of ethmoid dissection. The superior turbinate
can be a critical landmark to help identify the natural ostium of the sphenoid
sinus. The anterior wall of the maxillary sinus is not a commonly used
landmark in revision sinus surgery.

4. C
In the patient with recalcitrant chronic rhinosinusitis who may require
further surgery, all antibiotics administered should be in culture-directed
format.

5. E
Magnetic resonance image of the sinuses with and without gadolinium is
important to evaluate any areas of dehiscent bone, opacification at the skull
base to rule out encephalocele, and in completely opacified sphenoids to help
evaluate for fungus and aneurysm or pseudoaneurysm. MRI is notorious for
overestimating the extent of mucosal disease and has no role for evaluation of
mucosal disease only.

184
BY DR. MOHAMMED ATIAA KAREEM ALNASHY
-------------------------------------------------------------------
CHAPTER FIFTY-FIVE: CEREBROSPINAL FLUID (CSF)
RHINORRHEA
--------------------------------------------------------------------------------------------

1. Which of the following statements are false?


A. The best classification system for CSF categorizes CSF rhinorrhea as
traumatic or nontraumatic.
B. Causes of traumatic CSF rhinorrhea include head injury and sinus
surgery.
C. Nontraumatic CSF rhinorrhea may also be more appropriately termed
"spontaneous" or "idiopathic" CSF rhinorrhea.
D. Elevated intracranial pressure may occur in nontraumatic CSF
rhinorrhea.
E. Elevated intracranial pressure may result from intracranial tumors or
abnormalities in CSF resorption and circulation.

2. Which of the following statements are true?


A. All patients with nontraumatic CSF rhinorrhea are likely to have
benign intracranial pressure.
B. Nontraumatic CSF rhinorrhea has been associated with the empty sella
syndrome and benign intracranial pressure.
C. An empty sella on MRI represents low intracranial pressure.
D. Most nontraumatic CSF leaks occur in healthy, thin young men.
E. None of the above

3. The β2-transferrin assay


A. Serves as a specific marker for CSF
B. Requires large quantities of sample for reliable result
C. Requires the administration of intrathecal contrast
D. Provides information about the location of the skull base defect
E. None of the above

185
BY DR. MOHAMMED ATIAA KAREEM ALNASHY
-------------------------------------------------------------------
4. Which of the following statements are true?
A. Most instances of CSF rhinorrhea caused by closed-head trauma resolve
with conservative management.
B. Most instances of nontraumatic CSF rhinorrhea require operative repair.
C. Endoscopic repair of CSF rhinorrhea has emerged as the preferred
method for surgical closure of skull base defects when operative closure
is indicated.
D. Only pedicled mucosal flaps can be reliably used to reconstruct the site
of a CSF leak.
E. All of the above

5. In the management of CSF rhinorrhea,


A. Prophylactic antibiotics should be routinely used.
B. (β2-Transferrin testing and high-resolution computed tomography may
eliminate the need for CSF tracer studies.
C. Radionuclide tracer studies provide a sensitive and specific method for
confirming and localizing a CSF leak.
D. Magnetic resonance cisternography requires the administration of
intrathecal contrast.
E. CT cisternography can reliably identify more than 95% of CSF leaks.

186
BY DR. MOHAMMED ATIAA KAREEM ALNASHY
-------------------------------------------------------------------
CHAPTER 55
Cerebrospinal Fluid (CSF) Rhinorrhea

1. C
Use of the term "spontaneous CSF rhinorrhea" should be discouraged, because
it implies that the underlying etiology cannot be determined. In reality, the
underlying etiologic factors in CSF rhinorrhea can be determined. Only those
cases that are idiopathic are truly "spontaneous." CSF rhinorrhea is best
categorized as traumatic and nontraumatic. The traumatic group includes
head injury and surgery. The nontraumatic group may be further broken
down in to those cases with normal intracranial pressure and those cases with
abnormal intracranial pressures. Altered CSF physiology and tumors are both
causes increased intracranial pressure.

2B
Several studies have noted an association between nontraumatic CSF
rhinorrhea and unrecognized elevated intracranial pressure and suggest a
relationship between CSF rhinorrhea, the empty sella syndrome (ESS) and
benign intracranial hypertension (BIH). Despite this relationship, not all
patients with nontraumatic CSF rhinorrhea have ESS or BIH; other causes
of nontraumatic CSF rhinorrhea include intracranial and skull base
neoplasms. The MRI finding of an empty sella suggests possible elevated
intracranial pressure, not lowered intracranial pressure. An association
between nontraumatic CSF rhinorrhea and middle-aged obese women has
been described.

3. A
β2 transferrin assay provides a reliable method for confirming the presence
of a CSF leak. (β2 transferrin is specific marker of CSF, and this assay
requires relatively small amounts of sample for adequate results. The test is
noninvasive; the sample is collected as the nasal fluid passively drains from
the nose. Obviously, it cannot provide specific information about the location
of the CSF leak; it only indicates the presence of a leak.

187
BY DR. MOHAMMED ATIAA KAREEM ALNASHY
-------------------------------------------------------------------
4. E
CSF rhinorrhea associated with closed head trauma typically resolves with
conservative management, but nontraumatic CSF rhinorrhea is much less
likely to resolve with conservative management and hence is more likely to
require operative repair. Numerous reports confirm that endoscopic repair is
the preferred surgical technique if operative repair is indicated. Although a
wide variety of grafts have been used with great success in CSF leak repair,
only pedicled mucosal grafts have been associated with a relatively high rate
of failure.

5B
β2 transferrin testing accurately confirms the presence of a CSF leak. High-
resolution CT can identify the presence of skull base dehiscences that may
represent the site of leakage. Together a positive p-2 transferrin study plus a
CT showing a suspected skull base defect warrant operative exploration. As a
result, traditional CSF tracer studies may not be routinely necessary. Today,
prophylactic antibiotics are not routinely recommended, because they do not
seem to change outcomes, and they may lead to the development of resistant
bacteria. Radionuclide studies and CT cisternography at best can identify 80%
of CSF leaks. MR cisternography does not require intrathecal contrast; this
imaging study relies on the intrinsic signal characteristics of CSF.

188
BY DR. MOHAMMED ATIAA KAREEM ALNASHY
-------------------------------------------------------------------

PART FIVE
--------------------------------------------------------------------------------------------
--------------------------------------------------------------------------------------------

SALIVARY GLANDS

189
BY DR. MOHAMMED ATIAA KAREEM ALNASHY
-------------------------------------------------------------------
CHAPTER FIFTY-SIX: PHYSIOLOGY OF THE SALIVARY GLANDS
------------------------------------------------------------------------------------------

1. Which of the following statements regarding salivary secretion is true?


A. The average daily volume of saliva produced is between 500 and 750 mL.
B. Sixty to seventy percent of the total daily saliva is produced by the
parotid glands.
C. Minor salivary glands are responsible for 20% of the daily saliva
produced.
D. Most of the unstimulated saliva is secreted by the submandibular gland.
E. Hyposalivation is defined as an unstimulated salivary flow of less than
0.5 mL/min.

2. Which of the following statements regarding the structure of a normal


salivary gland secretory unit is not true?
A. Salivary acini are classified as serous, mucinous, and mixed.
B. Serous cells are filled with basophilic secretary granules.
C. Acini lead to intercalated ducts lined by a single layer of cuboidal cells.
D. The intralobular ducts are commonly known as striated ducts.
E. Myoepithelial cells are not seen in normal salivary gland acini.

3. Which of the following statements is not true regarding salivary


secretion?
A. It is a two-stage process composed of primary acinar secretions and
ductal secretions.
B. It is increased by the stimulation of the parasympathetic nervous system.
C. It is believed to occur by way of osmotic coupling of transepithelial
fluxes of sodium chloride and water.
D. Ii is not dependent on the activity of Na+-K+-ATPase.
E. Sympathetic stimulation leads to high fluid output.

4. Which of the following statements regarding salivary flow rates is not true?
A. They are reduced during sleep.
B. They are increased during exercise.
C. They are increased by mastication.
D. They are increased before an episode of vomiting
E. They steadily increase as the child grows and reach a maximum value
by the age of 3 to 4 years.

190
BY DR. MOHAMMED ATIAA KAREEM ALNASHY
-------------------------------------------------------------------
5. All of the following statements are true except
A. IgG is the predominant immunoglobulin.
B. The relationship between IgA and the formation of dental plaque is
unknown.
C. IgA in saliva is in the form of a dimer.
D. Lactoferrin scavenges free iron in fluids and inflamed areas so as to
suppress free radical-mediated damage and decrease the availability of
the metal to invading microbial and neoplastic cells.

191
BY DR. MOHAMMED ATIAA KAREEM ALNASHY
-------------------------------------------------------------------
CHAPTER 56
Physiology of the Salivary Glands

1D
The average daily volume of saliva produced is between 1000 mL and 1500 mL. Sixty
to seventy percent of the total volume of saliva formed in a day is secreted by the
submandibular gland, 20% to 30% is secreted by the parotid glands, and 5% to 10% is
secreted by the sublingual glands and minor salivary glands. Hyposalivation is defined
as an unstimulated salivary flow <0.25 mL/min.

2. E
Myoepithelial cells are elongated or star-shaped nonsecreting cells with long
branching processes that surround the acinus and proximal ducts. The observation
that myoepithelial cells possess adenosine triphosphate activity, have intercellular gap
junctions, and contain myofilaments has led to the hypothesis that these cells have
contractile properties and play a role in expelling preformed secretions.

3D
Operation of the Na+-K+-ATPase is essential for secretion of saliva. Hydrolysis of one
ATP results in the active transport of 3 Na+ out of the cell and 2 K+ into the cell. The
Na+-K+-ATPase is localized on the basolateral domain of the acinar cells. The Na+-K+-
ATPase thus maintains a high intracellular concentration of K+ and a low intracellular
concentration of Na+, thus contributing to the resting membrane potential.

4B
A role for the sympathetic nervous system in decreased salivary flow after exercise has
been proposed. In addition, an anticipatory decrease in saliva flow rate has been shown
immediately beforeNexercise compared with a no-exercise control day. It is thought
that exercise-related increases in sympathetic nervous system activity cause
constriction of blood vessels to the salivary glands, leading to a reduction in saliva
secretion. Dehydration during exercise is believed to be a contributing factor for the
reduced flow.

5. A
Immunoglobulin A (IgA) is the predominant immunoglobulin in saliva and plays an
important role in the local immune defense system. The parotid saliva contains 30 to
160 jig/mL of IgA. Although IgG and IgM are also present in the parotid saliva, their
concentration is significantly lower. IgA exerts its antiinflammatory protective
functions and down-regulates inflammation by inhibiting IgG- and IgM-modulated
functions. The role of IgA as a protective agent against dental plaque formation in
humans is controversial.

192
BY DR. MOHAMMED ATIAA KAREEM ALNASHY
-------------------------------------------------------------------
CHAPTER FIFTY-SEVEN: DIAGNOSTIC IMAGING AND FINE-
NEEDLE ASPIRATION OF THE SALIVARY GLANDS
--------------------------------------------------------------------------------------------
1. The percentage of calculi in the submandibular gland is
A. 20%
B. 40%
C. 50%
D. 60%
E. 80%

2. The most common tumor to scan positively with technetium is


A. Pleomorphic adenoma
B. Oncocytoma
C. Warthin's tumor
D. Mucoepidermoid carcinoma
E. Malignant mixed tumor

3. Ultrasonography can be used to


A. Separate intrinsic from extrinsic parotid masses
B. Separate super facial lobe from deep lobe parotid masses
C. Distinguish facial nerve from tumor
D. Separate deep lobe from parapharyngeal masses
E. Show where the marginal branch of the facial nerve crosses the
retromandibular vein

4. Computed tomography scans are the best imaging study for detection of
A. Neoplasms
B. Calculi
C. Abscesses
D. Chronic inflammatory disease
E. Parapharyngeal masses

5. Magnetic resonance imaging is most useful in evaluating


A. Parotid abscesses
B. Calculi
C. Intrinsic versus extrinsic parotid masses
D. Malignant neoplasms at the skull base
E. Chronic inflammatory disease

193
BY DR. MOHAMMED ATIAA KAREEM ALNASHY
-------------------------------------------------------------------
CHAPTER 57
Diagnostic Imaging and Fine-Needle Aspiration of the Salivary Glands

1. E

2. C

3. A

4B

5D

194
BY DR. MOHAMMED ATIAA KAREEM ALNASHY
-------------------------------------------------------------------
CHAPTER FIFTY-EIGHT: INFLAMMATORY DISORDERS OF THE
SALIVARY GLANDS
--------------------------------------------------------------------------------------------

1. Which salivary gland is most commonly affected in acute suppurative


sialadenitis?
A. Submandibular gland
B. Parotid gland
C. Sublingual gland
D. Minor salivary glands

2. Which would be the best antimicrobial for the treatment of acute


suppurative sialadenitis?
A. Erythromycin
B. Tetracycline
C. β-Lactamase-resistant penicillin
D. Fluoroquinolone

3. Which factors predispose the submandibular gland to sialolithiasis?


A. Smaller-sized duct
B. Viscosity of saliva from the submandibular gland
C. Larger-sized duct
D. Weight of the duct
E. B and C
F. Band D

4. In most cases involving cat-scratch disease, the treatment includes


A. Clindamycin
B. Augmentin
C. No treatment
D. Doxycycline

5. Which autoantibodies are tested for the diagnosis of Sjogren's syndrome?


A. Antimitochondrial antibodies
B. Rheumatoid factor
C. Antimicrosomal antibodies
D. SS-A and SS-b autoantibodies

195
BY DR. MOHAMMED ATIAA KAREEM ALNASHY
-------------------------------------------------------------------
CHAPTER 58
Inflammatory Disorders of the Salivary Glands

1B
The parotid gland is most susceptible to such infections. The parotid gland
produces saliva that is mainly serous as opposed to saliva from the
submandibular and sublingual glands that is primarily mucoid. Serous
saliva, unlike mucinous saliva, is deficient in lysosomes, IgA antibodies, and
sialic acid, which have antimicrobial properties. In addition, the saliva from
the submandibular and sublingual glands contains high molecular weight
glycoproteins that competitively inhibit bacterial attachment to the epithelial
cells of the salivary ducts.

2. C
Initial treatment of acute suppurative sialadenitis begins with aggressive medical
management. This includes prompt fluid and electrolyte replacement, oral hygiene,
reversal of salivary stasis, and antimicrobial therapy. Stimulation of salivary flow
is accomplished by the use of sialogogues such as lemon drops and orange juice. In
addition, capable patients should be instructed on regular external and bimanual
massage, starting from the distal bed of the gland and working in the direction of
duct drainage. Analgesics and local heat application ease the discomfort.
Antimicrobial therapy is an essential part of the management of acute salivary
gland infections. Antimicrobial therapy is initiated empirically toward gram-
positive and anaerobic bacteria. However, the recovery of p-lactamase-producing
bacteria in 75% of patients requires the use of augmented penicillin and
antistaphylococcal penicillin or a first-generation cephalosporin. Culture results
should be used to further direct antimicrobial treatment. Methicillin-resistant S.
aureus infection may require the use of vancomycin or linezolid. The use of
clindamycin or the addition of metronidazole to the first-line agents to broaden
anaerobic coverage has been advocated by some authors. Response to antimicrobial
therapy is seen within 48 to 72 hours of initiating treatment and should continue
for 1 week after resolution of symptoms. Rarely, conservative measures fail to
eradicate the infection, and surgical drainage of a loculated abscess is necessary.
The surgical approach involves elevation of an anterior-based facial flap with
abscess drainage by way of radial incisions in the parotid fascia parallel to the
facial nerve branches. A drain should be placed, and the wound edges should be
loosely approximated with the central aspect left to heal by secondary intention.

196
BY DR. MOHAMMED ATIAA KAREEM ALNASHY
-------------------------------------------------------------------
3. E
Several factors may account for the propensity of salivary stones to form in
the submandibular gland. Wharton's duct is longer, has a larger caliber, and
is angulated against gravity as it courses around the mylohyoid muscle, all of
which results in slower salivary flow rates. Also, the saliva produced by the
gland itself is more viscous and has a higher calcium and phosphorous
concentration.

4. C
In most cases, no active therapy is required. The patient should be reassured
that the lymphadenopathy is self-limited and usually will resolve
spontaneously in 2 to 4 months. However, in patients who are systemically ill,
highly symptomatic antibiotic therapy is recommended. The (3-lactam
antibiotics are ineffective in the treatment of GSD. The antibiotics reported
to be most effective are rifampin, erythromycin, gentamycin, azithromycin,
and ciprofloxacin.

5D
In general, the diagnosis consists of establishing the presence of
keratoconjunctivitis sicca and xerostomia by clinical examination and
objective testing. This testing should include objective measurements of
decreased salivary and tear flow along with a minor salivary gland biopsy. In
addition, laboratory evidence suggesting a systemic autoimmune disease,
specifically against SS-A and SS-B ribonuclear proteins, is necessary for the
diagnosis of Sjogren's syndrome. The presence of another autoimmune
disorder, such as rheumatoid arthritis or systemic lupus erythematosus,
would mandate a diagnosis of secondary Sjogren's syndrome. Patients who
have objective signs of sicca complex but no evidence of an autoimmune
process should be evaluated for other causes.

197
BY DR. MOHAMMED ATIAA KAREEM ALNASHY
-------------------------------------------------------------------
CHAPTER FIFTY-NINE: TRAUMA OF THE SALIVARY GLANDS
--------------------------------------------------------------------------------------------
1. In extensive proximal parotid duct injury, appropriate management
includes
A. Duct ligation
B. Superficial parotidectomy
C. Pressure dressings
D. Primary anastomosis
E. Observation

2. Which statement about the use of electromyography in managing facial


nerve injuries is true?
A. It is most helpful within the first 2 weeks of injury.
B. It has no role.
C. It provides prognostic information 3 weeks after injury.
D. It has supplanted electroneuronography as a prognostic tool.
E. It should not be used before 6 weeks after injury.

3. Which statement about facial nerve injury is true?


A. Repair should be delayed 7 days.
B. Best results are obtained when repair occurs after 14 days.
C. It should be performed on injuries medial to the lateral canthus.
D. Best results are obtained the sooner repair is performed.
E. Branches may be stimulated up to 96 hours after injury.

4. Which of the following is an advantage of using the great auricular nerve


for facial nerve grafting?
A. Minimal donor site morbidity
B. Location within the operating field
C. Width
D. Length
E. Branching pattern

5. Treating parotid fistulas or sialoceles may include all of the following except
A. Repeat aspiration
B. Compression
C. Tympanic neurectomy
D. Parotidectomy
E. Ligating the chorda tympani

198
BY DR. MOHAMMED ATIAA KAREEM ALNASHY
-------------------------------------------------------------------
CHAPTER 59
Trauma of the Salivary Glands

1. A

2. C

3D

4B

5. E

199
BY DR. MOHAMMED ATIAA KAREEM ALNASHY
-------------------------------------------------------------------
CHAPTER SIXTY: BENIGN NEOPLASMS OF THE SALIVARY
GLANDS
------------------------------------------------------------------------------------------

1. Warthin's tumors are thought to arise from which of the following cell
types?
A. Acinar cells
B. Intercalated duct cells
C. Striated duct cells
D. Excretory duct cells
E. Myoepithelial cells

2. Which of the following is not associated with the development of


salivary gland malignancy?
A. Smoking
B. Ionizing radiation
C. Silica dust
D. Nitrosamines
E. Nulliparity

3. Which of the following statements is false?


A. Pleomorphic adenomas are the most common salivary gland neoplasms.
B. Pleomorphic adenomas are the most common parotid deep lobe
neoplasm.
C. Pleomorphic adenomas are the most common submandibular gland
neoplasms.
D. Pleomorphic adenomas are the most common minor salivary gland
neoplasms.
E. Papillary cystadenoma lymphomatosum is the second most common
benign salivary lesion.

200
BY DR. MOHAMMED ATIAA KAREEM ALNASHY
-------------------------------------------------------------------
4. Which of the following is true of fine-needle aspiration (FNA) in
salivary gland neoplasms?
A. The sensitivity of FNA is <85%.
B. The specificity of FNA is <95%.
C. FNA rarely results in a change in management.
D. It can be difficult to differentiate oncocytic and adenoid cystic
neoplasms by FNA.
E. It can be difficult to distinguish mucoepidermoid carcinoma and
sialolithiasis by FNA.

5. Which of the following is false with regard to salivary neoplasms?


A. The presence of lymphoid tissue differentiates Warthin's tumors from
oncocytomas.
B. Pleomorphic adenomas may metastasize to lymph nodes.
C. Warthin's tumors may metastasize to lymph nodes.
D. Pleomorphic adenomas are always well encapsulated.
E. Arteriovenous fistulas typically result from trauma

CHAPTER 60
Benign Neoplasms of the Salivary Glands

1. C

2. A

3D

4. E

5D

201
BY DR. MOHAMMED ATIAA KAREEM ALNASHY
-------------------------------------------------------------------
CHAPTER SIXTY-ONE: MALIGNANT NEOPLASMS OF THE
SALIVARY GLANDS
--------------------------------------------------------------------------------------------
1. Acinic cell carcinoma occurs most commonly in which gland?
A. Parotid gland
B. Submandibular gland
C. Sublingual gland
D. Minor salivary glands
E. None of the above

2. The second most common parotid malignancy is


A. Mucoepidermoid carcinoma
B. Adenoid cystic carcinoma
C. Acinic cell carcinoma
D. Squamous cell carcinoma
E. Polymorphous low-grade adenocarcinoma

3. Which tumor should routinely receive elective neck dissection?


A. Polymorphous low-grade adenocarcinoma
B. Adenoid cystic carcinoma
C. Acinic cell carcinoma
D. Adenocarcinoma
E. Lymphoma

4. The most common site of distant failure in patients with parotid malignancy is
A. Brain
B. Bone
C. Lungs
D. Liver
E. Neck

5. Which statement regarding postoperative radiation is true?


A. Postoperative radiation is given for any parotid malignancy.
B. Postoperative radiation improves overall survival and regional control.
C. Postoperative radiation is only given for positive margins and unresectable
disease.
D. Postoperative radiation improves regional control in advanced stage tumors.
E. Because neutron therapy has an improved radiobiologic effect on salivary
cancers, it has become the standard technique for postoperative radiation.

202
BY DR. MOHAMMED ATIAA KAREEM ALNASHY
-------------------------------------------------------------------
CHAPTER 61
Malignant Neoplasms of the Salivary Glands

1. A
Acinic cell carcinoma occurs most frequently in the parotid gland. In fact, it
is rare that another gland would contain acinic cell carcinoma.

2. B
The most common parotid malignancy is mucoepidermoid carcinoma, with the
second most common histologic pattern being adenoid cystic carcinoma. Adenoid
cystic carcinoma is the most common primary malignancy of the submandibular
glands. Acinic cell carcinoma occurs most frequently in the parotid gland, whereas
polymorphous low-grade adenocarcinoma is most commonly found in the minor
salivary glands, specifically the palate. The presence of squamous cell carcinoma
in a salivary gland should immediately raise the question of a second primary
tumor, because this is clearly more common than primary disease.

3D
Elective neck dissection is not routinely advocated for salivary malignancy.
However, the indications for elective neck dissection are stage III/IV tumors,
high-grade mucoepidermoid carcinoma, squamous cell carcinoma, and
adenocarcinoma. The presence of cervical metastasis is an indication for neck
dissection but would be classified as therapeutic rather than elective.

4. C
The most common site of distant failure in patients with parotid malignancy
is the lungs. Liver, bone, and brain metastases can occur but are clearly less
common. Although cervical and lung metastasis occur with similar
frequency, cervical metastases are classified as regional failure, not distant.

5. D
Postoperative radiation is indicated in high-grade malignancies, nodal
involvement, stage III/IV tumors, and in those with positive margins.
Neutron therapy is advocated for adenoid cystic carcinoma, although it is
clearly not the standard technique, because very few centers provide this
modality. Although postoperative radiation does not improve survival, it is
believed to improve regional control, especially in advanced and high-grade
malignancy.

203
BY DR. MOHAMMED ATIAA KAREEM ALNASHY
-------------------------------------------------------------------

PART SIX
--------------------------------------------------------------------------------------------
--------------------------------------------------------------------------------------------
ORAL CAVITY/PHARYNX/ ESOPHAGUS

204
BY DR. MOHAMMED ATIAA KAREEM ALNASHY
-------------------------------------------------------------------
CHAPTER SIXTY-TWO: PHYSIOLOGY OF THE ORAL CAVITY
--------------------------------------------------------------------------------------------

1. Which of the following statements is false?


A. The lingual nerve is sensitive to chemical stimulation of the tongue.
B. Loss of periodontal mechanoreceptors does not eliminate intradental
discrimination.
C. Subnucleus caudalis is the only brain stem trigeminal nucleus that
mediates pain.
D. Stimulation of the hypoglossal nerve can result in reflex action in the
trigeminal system.
E. The chorda tympani innervates fungiform papillae on the front of the
tongue.

2. Which of the following is true?


A. A jaw-opening reflex is mediated by muscle spindle afferents in the
anterior digastric muscle.
B. The jaw-closing reflex is a disynaptic reflex through the spinal
trigeminal complex.
C. Cephalic-phase insulin release can be initiated by gustatory stimuli.
D. The masticatory rhythm is generated by alternating jaw-opening and
jaw-closing reflexes.
E. Stimulation of the lingual and glossopharyngeal nerves results
primarily in a protrusive movement of the tongue.

3. The initiating cause of the dentinal hypersensitivity caused by exposure


of dentinal tubules, which occurs, for example, with a cracked tooth or
a cavity, can best be explained by
A. The "hydrostatic" theory
B. The "hydrodynamic" theory
C. Central sensitization
D. Release of neuropeptides into the tooth pulp
E. Chemesthesia

205
BY DR. MOHAMMED ATIAA KAREEM ALNASHY
-------------------------------------------------------------------
4. Gustatory transduction may involve
A. Direct entry of a stimulus into the receptor cell
B. Activation of G-protein-coupled receptors
C. Changes in the intracellular pH
D. All of the above

5. The loss of the chorda tympani nerve after middle ear surgery in
humans
A. Results in loss of taste sensation from the back of the mouth
B. Requires precise psychophysical procedures to demonstrate any loss of
function
C. Influences the sensation of thirst
D. Results in a profound disruption in salt intake
E. None of the above

206
BY DR. MOHAMMED ATIAA KAREEM ALNASHY
-------------------------------------------------------------------
CHAPTER 62
Physiology of the Oral Cavity

1. C
Oral sensation is mediated by nerves with multiple sensitivities and functions.
Thus, the lingual nerve responds to mechanical, thermal, and chemical
stimuli. Likewise, interdental discrimination and three-dimensional
recognition (stereognosis) is mediated by multiple nerves, not just those
innervating the periodontal ligament. Although oral pain is frequently
associated with neural processing in the subnucleus caudalis, other brainstem
structures are likely involved as well.

2. C
There are numerous oral reflexes mediating both digestive and protective
functions. The central substrates for many of these reflexes are only partially
understood. Several reflexes that result in mandible elevation (jaw-closure)
are monosynaptic reflexes from muscle spindle afferents directly exciting jaw
closer motoneurons in the motor trigeminal nucleus. Cephalic phase reflexes
include insulin release in response to gustatory stimulation that influences
parasympathetic preganglionic neurons in the dorsal vagal complex.

3B
The dentine is permeated with fluid-filled tubules that allow osmotic,
thermal, and mechanical stimuli to activate A-delta fibers located in the
proximal end of the tubule. The fluid-filled dentinal tubule allows
"hydrodynamic" forces set up by a distal stimulus to activate a nociceptor.
Once this process is underway, other processes, such as the release of
neuropeptides into the pulp, or central sensitization can exasperate the
hypersensitivity.

4. D
Because there are many different types of molecules that the taste system must
transduce, at a minimum, both direct stimulus entry into cells (e.g., Na+ and
acids) and stimulus binding to G-protein coupled receptors (bitter, sweet, and
amino-acid stimuli) make up the first step of transduction. Interestingly,
entry of acid stimuli into cells is accompanied by small, reliable changes in
intracellular pH, which are correlated with the intensity of sourness.

207
BY DR. MOHAMMED ATIAA KAREEM ALNASHY
-------------------------------------------------------------------
5B
Each chorda tympani nerve innervates the ipsilateral taste buds in the
fungiform papillae on the anterior two thirds of the tongue. If a precise
testing procedure is used, so that taste stimuli are applied only to this part of
the tongue, destruction of the chorda tympani nerve would be obvious, because
the individual would not be able to detect any type of taste sensation.
However, if the person was allowed to take the stimulus into the mouth, so
that it contacted receptors on the back of the tongue or palate, taste loss would
be subtle, because the innervated taste buds on the back of the tongue and
palate seem to be able to compensate for chorda tympani loss. A similar type
of testing procedure could detect glossopharyngeal damage if stimuli were
restricted to the foliate or circumvallate papillae. However, this nerve is not
as vulnerable to iatrogenic damage Although animals with lesions of their
chorda tympani nerve do demonstrate profound, specific losses in the ability
to discriminate sodium, such a deficit is not as apparent in humans, implying
that the different regions of the mouth may not be as specialized as in
animals.

208
BY DR. MOHAMMED ATIAA KAREEM ALNASHY
-------------------------------------------------------------------
CHAPTER SIXTY-THREE: MECHANISMS OF NORMAL AND
ABNORMAL SWALLOWING
--------------------------------------------------------------------------------------------

1. Two of the stages of swallow are under voluntary control. They are
A. Oral and pharyngeal
B. Pharyngeal and esophageal
C. Oral and oral preparation
D. Oral preparatory and pharyngeal
E. Oral and esophageal

2. Your patient has a suspected pharyngeal stage swallowing disorder


after chemoradiation. What is your suggestion for an assessment to
determine the management plan?
A. Videofluoroscopy
B. Manometry
C. Endoscopy
D. Scintigraphy
E. Ultrasonography

3. You have a patient who aspirates the minute liquid enters his mouth. You
suspect two possible physiologic reasons for the aspiration. What are they?
A. Delayed pharyngeal swallow and reduced contraction of the pharyngeal
constrictors
B. Delayed pharyngeal swallow and reduced airway closure
C. Delayed pharyngeal swallow and reduced control of the tongue
D. Reduced airway closure and reduced cricopharyngeal opening
E. Reduced laryngeal closure and delayed pharyngeal swallow

4. Your patient has a suspected oral and tongue base disorder. Which of
the following assessments do you recommend?
A. Scintigraphy
B. Scintigraphy and endoscopy
C. Manometry and endoscopy
D. Videofluoroscopy and ultrasonography
E. Videofluoroscopy and endoscopy

209
BY DR. MOHAMMED ATIAA KAREEM ALNASHY
-------------------------------------------------------------------
5. Patients who have undergone supraglottic laryngectomy may exhibit
A. Reduced cricopharyngeal opening
B. Reduced laryngeal elevation, laryngeal closure, and pharyngeal
contraction
C. Reduced tongue control
D. Reduced cricopharyngeal opening and reduced laryngeal closure
E. Reduced laryngeal elevation, laryngeal closure, and reduced tongue
base movement

210
BY DR. MOHAMMED ATIAA KAREEM ALNASHY
-------------------------------------------------------------------
CHAPTER 63
Mechanisms of Normal and Abnormal Swallowing

1. C
The oral and oral preparatory stages of swallow are under cortical voluntary
control, whereas the pharyngeal stage of swallow is
under brainstem or involuntary control.

2. A
Videofluoroscopy shows all stages of swallow in detail. A patient with
chemoradiation may have oral problems related to xerostomia that could
affect the pharyngeal function. Endoscopy does not view the oral stage of
swallow.

3. C
Aspirating as soon as food, particularly liquid, enters the mouth is usually
caused by an abnormality in tongue control to hold the bolus cohesively or a
delay in triggering the pharyngeal swallow. If there is a pharyngeal delay,
liquid can quickly enter the airway before the pharyngeal swallow triggers.

4. E
With both an oral and a tongue base disorder, you will want to examine the
oral and pharyngeal stages of swallow simultaneously. This requires
videofluoroscopy. With a tongue base disorder, endoscopy will allow you to
visualize the degree to which the tongue base and pharyngeal wall make
contact.

5. E
Because supraglottic laryngectomy involves removal of a part of the tongue
base, the top two sphincters of the larynx, and disconnection of strap muscles
from the hyoid to the larynx, these swallowing disorders are predictable based
on the structures resected.

211
BY DR. MOHAMMED ATIAA KAREEM ALNASHY
-------------------------------------------------------------------
CHAPTER SIXTY-FOUR: ORAL MUCOSAL LESIONS
--------------------------------------------------------------------------------------------

1. Aphthous ulceration of the oral mucosa is often mistaken for recurrent


intraoral herpes simplex infection but is distinguishable on the basis
of all of the following except
A. Location
B. Vesicular phase
C. Viral cytopathic effect
D. Duration

2. Oral mucosal melanoma is not generally thought to parallel its


cutaneous counterpart in terms of discrete preinvasive categories but
may be best considered to parallel which type of melanoma from a
precursor lesion standpoint?
A. Thin melanoma
B. Acral lentiginous melanoma
C. Plantar melanoma
D. Cellular blue nevus

3. A diagnosis of "leukoplakia" effectively rules out all of the following except


A. Lichen planus
B. White sponge nevus
C. Leukoedema
D. Epithelial dysplasia

4. The entity, proliferative verrucous leukoplakia, may be separable from


the common form of leukoplakia by virtue of its
A. Location
B. Relationship to smokeless tobacco
C. High rate of cancer development
D. Relationship to use of certain mouthwashes

212
BY DR. MOHAMMED ATIAA KAREEM ALNASHY
-------------------------------------------------------------------
5. Oral lichen planus may present as a desquamative process involving the
attached gingiva in a manner similar to which of the following
diseases/conditions?
A. Contact mucositis
B. Mucosal pemphigoid
C. Nutritional deficiencies (vitamin C)
D. Leukemic infiltrate

213
BY DR. MOHAMMED ATIAA KAREEM ALNASHY
-------------------------------------------------------------------
CHAPTER 64
Oral Mucosal Lesions

1D
See Table 1. Aphthous ulcers are distributed over the nonkeratinized mucosa but do
not have a virally associated vesicular phase or cytopathic effect as does the infection
produced by herpes simplex.

RECURRENT (2-DEGREE) HSV VS RAS (MINOR)


HSV RAS Herpes simplex I/II Varied/immune dysfunction
Etiology
Location Keratinized tissue mucosa Moveable/Nonkeratinized
Vesicle phase Yes No
Duration 7-14 days Varies (usually 7-10 days)
Management Topical (docosanol,penciclovir) Topically based steroids
Oral antivirals

Prodrome Often Uncommonly


Triggers Stress, trauma Stress, ultraviolet light, foods
Biopsy findings Viral cytopathic effect Nonspecific

2B
Oral melanoma precursor phase or developmental biology may best be compared with
nodular or acral lentigenes melanoma in the absence of the usual corresponding
phase(s) associated with cutaneous melanoma.

3D
Epithelial dysplasia is a microscopically defined term. Choices a, b, and c are
clinically distinguishable from each other and, by virtue of their appearance, can be
diagnosed with relative confidence.

4. C
Proliferative verrucous leukoplakia, unlike the more common form of leukoplakia,
carries a significant risk of carcinoma development and higher rate of recurrence.
Location does not enter into this separation, given the widespread nature of
proliferative verrucous leukoplakia.

5B
Both mucosal pemphigoid and erosive lichen planus may involve the attached gingiva
and might, because of basement membrane zone alterations, present as a desquamative
process.

214
BY DR. MOHAMMED ATIAA KAREEM ALNASHY
-------------------------------------------------------------------
CHAPTER SIXTY-FIVE: ORAL MANIFESTATIONS OF SYSTEMIC
DISEASE
--------------------------------------------------------------------------------------------

1. Which of the following classifications of medications does not cause


salivary hypofunction?
A. Tricyclic antidepressants
B. Antihistamines
C. Cox-2 nonsteroidal antiinflammatories
D. Diuretics

2. Which of the following diseases is most likely to have oral


mucocutaneous ulcers?
A. Pemphigus vulgaris
B. Parkinson's disease
C. Renal osteodystrophy
D. Down syndrome

3. Oral manifestations of bleeding disorders include all of the following except


A. Sublingual ecchymotic lesions
B. Hard palate petechiae
C. Fungiform papillae
D. Gingival hemorrhage

4. Antibiotic prophylaxis before dentoalveolar surgery is absolutely


required for which of the following conditions?
A. Pin placement after femur fracture
B. Mitral valve prolapse with regurgitation
C. Three months after a CVA
D. Indwelling cardiac pacemaker

5. Squamous cell carcinoma of the tongue may manifest the following oral
sequelae except
A. Nonhealing oral ulcer
B. Erythroplakic lesion
C. Exophytic erythroleukoplakic pustule
D. Mucocele of the lower lip

215
BY DR. MOHAMMED ATIAA KAREEM ALNASHY
-------------------------------------------------------------------
CHAPTER 65

Oral Manifestations of Systemic Disease

1. C

2. A

3. C

4B

5D

216
BY DR. MOHAMMED ATIAA KAREEM ALNASHY
-------------------------------------------------------------------
CHAPTER SIXTY-SIX: ODONTOGENESIS AND ODONTOGENIC
CYSTS AND TUMORS
-------------------------------------------------------------------------------------------

1. Which is a radiographic feature often found in benign odontogenic


cysts and tumors?
A. Radiopaque border
B. Blunted tooth roots
C. Cortical expansion
D. All of the above

2. A patient with multiple odontogenic keratocysts should be evaluated


for Gorlin's syndrome. This diagnosis is important because
A. These patients must have genetic counseling.
B. These cysts become malignant.
C. These patients often have basal cell carcinoma develop in
noncharacteristic areas.
D. Keratocysts in these patients are more likely to recur.

3. The acceptable surgical excision of a solid ameloblastoma without


cortical bone perforation is
A. Simple enucleation
B. Enucleation and curettage
C. Excision with bony margins of 1 cm and preservation of adjacent soft
tissue
D. Composite resection of bone and soft tissue with 1-cm margins

4. On a panoramic radiograph taken to evaluate a patient with a suspected


mandible fracture, a molar with a large amalgam restoration and a 0.5-
cm diameter radiolucent lesion associated with the apex of the root is
seen. What is the best course of action?
A. Dental evaluation and root canal
B. Extraction of the tooth
C. Incisional biopsy
D. Enucleation of the lesion

217
BY DR. MOHAMMED ATIAA KAREEM ALNASHY
-------------------------------------------------------------------
5. After removal of a 2 x 2-cm radiolucent lesion of the mandible, the
hospital pathology report indicates a diagnosis of "benign odontogenic
cyst." What is the next course of action?
A. Frequent radiographic evaluation for recurrence.
B. No other treatment is needed, because simple removal of this type of
cyst is adequate.
C. Histopathologic evaluation by an oral pathologist.

218
BY DR. MOHAMMED ATIAA KAREEM ALNASHY
-------------------------------------------------------------------
CHAPTER 66
Odontogenesis and Odontogenic Cysts and Tumors

1D
Most odontogenic cysts and tumors are slow growing. They rarely perforate cortical bone
or natural tissue boundaries despite getting quite large. This slow growth tends to produce
a sclerotic border that is radiopaque. It also allows expansion of bone rather than
perforation, and this expansion, if between teeth, will also push the teeth apart and
produce some blunting of the roots because of resorption.

2. C
Multiple odontogenic keratocysts are found in Gorlin's syndrome, also known as basal
cell nevus syndrome. Although genetic counseling may be appropriate, the most
significant problem that these patients face is that they have basal cell carcinoma develop
in non-sun-exposed areas. Frequent and thorough total skin evaluation must be done
frequently by a dermatologist. These lesions do not undergo malignant change, and in
and of themselves are no more likely to recur than OKC in nonsyndromic patients.

3. C
Ameloblastoma has a significant recurrence rate and can become quite large and locally
destructive. It is not a malignant process. Recurrence with only simple enucleation or
enucleation followed with curettage is unacceptably high. One centimeter margins and
extension to the adjacent uninvolved soft tissue plane is adequate.

4. A
Most lesions of this type are inflammatory responses to pulpal involvement secondary to
dental disease/caries. The large amalgam indicates previous caries. It would be
appropriate to first evaluate the tooth for restorability, and, if it is salvageable, root canal
therapy would remove inflammation and generally lead to resolution of the lesion
without surgical intervention. This approach does not produce tissue for diagnosis and so
follow-up is needed to ensure resolution.

5. C
The term "benign odontogenic cyst" is often used by general pathologists to describe any
cyst within the jaws. In this case, determination of only that the lesion is benign is not
adequate. There are a number of different cysts, some of which require more aggressive
treatment than others and with higher recurrence rates. A vague diagnosis should always
be investigated and tissue evaluated by someone with specific knowledge of odontogenic
lesions.

219
BY DR. MOHAMMED ATIAA KAREEM ALNASHY
-------------------------------------------------------------------
CHAPTER SIXTY-SEVEN: ODONTOGENIC INFECTIONS
--------------------------------------------------------------------------------------------

1. Which of the following best describes a typical odontogenic infection?


A. Exclusively aerobic bacteria
B. Exclusively anaerobic bacteria
C. Mixed aerobic and anaerobic bacteria
D. Nosocomial bacteria
E. Fungi and viruses

2. Which of the following signs and symptoms is most commonly


associated with odontogenic infections?
A. Constipation
B. Diarrhea
C. Productive cough
D. Mental confusion
E. Facial swelling

3. Which of the following fascial spaces is not primarily involved in


Ludwig's angina?
A. Submandibular space
B. Submental space
C. Sublingual space
D. Lateral pharyngeal space

4. If required in the surgical management of osteomyelitis of the jaws,


skeletal stabilization is best accomplished by
A. External bandaging
B. External skeletal fixation
C. Internal skeletal fixation
D. Wire fixation

5. Diffuse sclerosing osteomyelitis of the facial skeleton demands


A. Long-term antibiotic maintenance and surgical intervention as necessary
B. Treatment similar to florid osseous dysplasia
C. Consideration for osteoradionecrosis as a comorbidity
D. Comparison with primary chronic osteomyelitis of childhood.

220
BY DR. MOHAMMED ATIAA KAREEM ALNASHY
-------------------------------------------------------------------
CHAPTER 67
Odontogenic Infections

1B
It has been estimated that only 5% of odontogenic infections are caused
exclusively by aerobic bacteria, 60% are caused exclusively by anaerobic
bacteria, and 35% are caused by mixed infections.

2. E
Constipation may be associated with a dehydrated patient taking narcotic
analgesics for an odontogenic infection but is not as common as facial
swelling. Mental confusion is a late finding in patients with odontogenic
infections. Diarrhea and productive cough are best described as possible
comorbid conditions in patients with odontogenic infections.

3D
By definition, Ludwig's angina involves the bilateral submandibular and
sublingual spaces and the submental space. The lateral pharyngeal space may
become secondarily involved in the patient with Ludwig's angina, but its primary
involvement is not required to make the diagnosis of Ludwig's angina.

4. C
External bandaging is only used to support soft tissue wounds. External skeletal
fixation, although acceptable, is technically difficult and creates patient
compliance problems. Internal skeletal fixation is technically efficient and
physiologically acceptable if applied correctly. Wire stabilization is not much
better than soft tissue bandaging and offers no real immobility except with regional
fixation.

5A
Diffuse sclerosing osteomyelitis can widely vacillate in its clinical presence
and symptoms over the course of a lifetime, necessitating both palliative and
definitive support. Florid osseous dysplasia resembles an odontogenic tissue
pathosis and should be treated very conservatively. Osteoradionecrosis is a
complication of radiation therapy in which the soft tissue, as well as the bone,
must be definitively treated on an aggressive basis, such as hyperbaric oxygen
therapy, debridement, and reconstruction. Chronic osteomyelitis of childhood
shows marked resolution with skeletal aging.

221
BY DR. MOHAMMED ATIAA KAREEM ALNASHY
-------------------------------------------------------------------
CHAPTER SIXTY-EIGHT: TEMPOROMANDIBULAR JOINT
DISORDERS
--------------------------------------------------------------------------------------------

1. Which of the following statements regarding the changes produced in


the mandible by unilateral condylar hypoplasia is incorrect?
A. The mandibular body on the affected side is shorter than the
contralateral side.
B. The chin is deviated to the affected side.
C. There is decreased antegonial notching.
D. The unaffected side is long and flat.
E. The face appears fuller on the affected side.

2. Which of the following is not an indication for open reduction and


fixation of fractures of the mandibular condyloid process?
A. The presence of bilateral fractures in a dentate patient
B. The presence of bilateral fractures in an edentulous patient
C. The presence of interference of the fracture with jaw movement
D. The presence of an intracapsular fracture
E. The presence of associated fractures of the mandible

3. Which of the following is the least important consideration in the


surgical management of ankylosis of the temporomandibular joint?
A. The age of the patient
B. The amount of limitation of jaw movement
C. The use of postoperative physical therapy
D. The prevention of fusion of the new joint
E. The etiology of the problem

4. The preferred treatment of a patient with an anteriorly displaced,


nonreducing disk is
A. The use of nonsteroidal anti-inflammatory drugs and a bite appliance
B. Surgical repositioning of the disk (discoplasty)
C. Doing an arthrocentesis
D. Arthroscopic repositioning of the disk
E. Surgical removal of the disk (diskectomy)

222
BY DR. MOHAMMED ATIAA KAREEM ALNASHY
-------------------------------------------------------------------
5. Which of the following are the most important factors involved in the
cause of myofascial pain dysfunction syndrome?
A. Muscular overextension
B. Chronic clenching and grinding of the teeth
C. Psychological stress
D. Malocclusion of the teeth
E. Muscular overcontraction

223
BY DR. MOHAMMED ATIAA KAREEM ALNASHY
-------------------------------------------------------------------
CHAPTER 68
Temporomandibular Joint Disorders

1. C
Condylar hypoplasia is a condition caused by trauma to the mandibular
condyle occurring during the growth period. Because the condyle is an
important growth site for the mandible, injury in this area results in
mandibular deformity. Therefore, early diagnosis and treatment are
important. Tile features described, except for the absence of antegonial
notching, are characteristic of the changes produced by condylar hypoplasia.
Increased antegonial notching is pathognomonic of retarded mandibular
growth and is an important diagnostic feature.

2. A, B
Unless there is gross displacement of the condyloid processes, patients with
bilateral fractures can be treated by closed reduction and maxillomandibular
fixation as long as the occlusion can be reestablished. This is not a problem
in a dentate patient and is also possible in edentulous patients who have
dentures.

3. B
The age of the patient is an important consideration in the surgical
management of mandibular ankylosis, because the associated lack of
mandibular growth caused by damage of the condyle, as well as the ankylosis,
need to be addressed. To maintain the new joint space, it is important to use
an interpositional material. In the child, a costochondral graft provides both
growth potential and an excellent interpositional tissue. In treating
ankylosis, an understanding of the cause of the problem is also essential,
because an autogenous graft or flap should not be used in patients with active
rheumatoid arthritis, in whom such interpositional materials can be
destroyed by the inflammatory process. Such patients require an alloplastic
joint replacement. Active, prolonged physical therapy is necessary in all cases
of ankylosis to maintain movement and to strengthen the masticatory
muscles. Although the amount of jaw limitation can be a problem in
performing general anesthesia, and the surgery can be more difficult in those
patients with severe limitation, this factor is less important than the others
noted.

224
BY DR. MOHAMMED ATIAA KAREEM ALNASHY
-------------------------------------------------------------------
4. C
Patients with locking caused by an anteriorly displaced, nonreducing disk
will not respond to medical management and require immediate surgical
intervention. The least-invasive, highly effective procedure is lysis of
adhesions and lavage of the joint by arthrocentesis. In those patients who do
not respond positively to such treatment, surgical disk repositioning, or disk
removal if it is not salvageable, are indicated.

5 B, C
Although masticatory muscle over-contraction or overextension can cause
myofascial pain and dysfunction in some patients, the most common cause is
increased muscle tension caused by psychological stress. Whereas the
associated presence of chronic clenching or grinding of the teeth is not
essential, patients who engage in such parafunctional activities are more
likely to have clinical symptoms. Malocclusion of the teeth is not a
contributing factor.

225
BY DR. MOHAMMED ATIAA KAREEM ALNASHY
-------------------------------------------------------------------
CHAPTER SIXTY-NINE: BENIGN TUMORS AND TUMOR-LIKE
LESIONS OF THE ORAL CAVITY
--------------------------------------------------------------------------------------------

1. Optimal treatment of nasoalveolar cysts requires


A. Marsupialization
B. Marsupialization with curettage
C. Conservative surgical excision
D. Complete surgical excision
E. Complete surgical excision followed by radiation therapy

2. Which of the following lesions grows in response to local trauma?


A. Nasoalveolar cyst
B. Mandibular torus
C. Fibroma
D. Choristoma
E. Parulis

3. Which of the following lesions is most easily mistaken for malignancy?


A. Fibrous histiocytoma
B. Necrotizing sialometaplasia
C. Granular cell tumor
D. Squamous papilloma
E. Pyogenic granuloma

4. Which of the following lesions should be followed closely for


resolution?
A. Pseudoepitheliomatous hyperplasia
B. Pyogenic granuloma
C. Granular cell tumor
D. Lingual thyroid
E. Necrotizing sialometaplasia

226
BY DR. MOHAMMED ATIAA KAREEM ALNASHY
-------------------------------------------------------------------
5. A woman is seen for denture placement with a bony lesion on the
lingual surface of the mandible. Panorex imaging demonstrates this
lesion to be multilobular with expansion of cortical bone. The best
option for treatment of this lesion is
A. Curettage of the lesion with resurfacing of the mandible for denture
placement
B. Simple excision of the lesion
C. Complete excision of the lesion
D. Marginal mandibulectomy with 1-cm margins
E. Segmental mandibulectomy with 2-cm margins and fibula free flap
placement followed by radiation therapy

CHAPTER 69
Benign Tumors and Tumor-Like Lesions of the Oral Cavity

1. D

2. C

3B

4. E

5D

227
BY DR. MOHAMMED ATIAA KAREEM ALNASHY
-------------------------------------------------------------------
CHAPTER SEVENTY: MALIGNANT NEOPLASMS OF THE ORAL
CAVITY
-------------------------------------------------------------------------------------------

1. In patients with an oral tongue squamous cell carcinoma and a depth


of invasion of 4 mm
A. Elective treatment of the neck is not necessary.
B. The risk of perineural invasion within the primary is significantly
increased.
C. Elective treatment of the N0 neck is advocated, because the risk of
occult metastasis exceeds 20%.
D. Resection of the primary tumor with a minimum of 2-cm margins is
required.
E. The likelihood of bilateral neck metastases is greater than 30%.

2. A 75-year-old edentulous patient is seen with an ulcerated 4-cm


retromolar trigone lesion that appears to invade the ascending ramus
of the mandible. Appropriate care includes
A. External beam radiation to the primary site and the ipsilateral neck
B. Transoral resection of the primary with selective neck dissection
C. Midline mandibulotomy with resection of the primary and ipsilateral
neck dissection
D. Composite resection of the primary tumor after lip-splitting incision,
ipsilateral neck dissection, and reconstruction with local or pedicled
soft tissue flap
E. Resection with fibular free flap reconstruction of the lateral mandible

3. The best treatment option for a 3-cm lower lip squamous cell carcinoma
extending to the oral commissure includes
A. Resection with primary closure
B. External beam radiation
C. Resection with a Bernard von Burow flap reconstruction
D. Free flap reconstruction
E. Resection with reconstruction by Estlander flap

228
BY DR. MOHAMMED ATIAA KAREEM ALNASHY
-------------------------------------------------------------------
4. When considering elective treatment of the neck in a patient with a T3
lateral tongue squamous cell carcinoma
A. A level I to IV neck dissection is advocated because of the potential for
skip metastases.
B. Postoperative radiation therapy can include the ipsilateral neck.
C. Radical neck dissection is required because of the advanced T stage.
D. A "watch and wait" philosophy can be used.
E. Sentinel node localization should be performed to determine the need
for neck dissection.

5. In a patient with a T3 oral tongue carcinoma with extension to the floor


of mouth, for whom free flap reconstruction is necessary and
postoperative radiation likely, the best approach to the primary is
A. Midline mandibulotomy
B. Lateral mandibulotomy
C. Transoral resection, if possible
D. Paramedian mandibulotomy
E. Use of a pull-through technique

229
BY DR. MOHAMMED ATIAA KAREEM ALNASHY
-------------------------------------------------------------------
CHAPTER 70
Malignant Neoplasms of the Oral Cavity

1. C
In patients with oral tongue carcinomas and a depth of invasion of greater
than 2 mm, the rate of regional metastasis exceeded 40%. Elective treatment
of the neck has been advocated when the risk of occult metastases exceeds 20%.

2D
The specific situation presented does not represent a situation that requires
bony reconstruction. An elderly edentulous patient with a lateral mandibular
lesion can tolerate composite resection of the mandible with soft tissue
reconstruction and maintain adequate speech and swallowing function.

3. E
With the average lip length of approximately 6 to 7 cm, this lesion involves
about half of the lower lip. Primary closure is not an option, given the size of
the lesion. The Bernard-von Burow flap is intended for lesions greater than
two thirds the length of a lip. An Estlander flap (lip-switch) can use the
upper lip to reconstruct the lower and will recreate the patients commissure
at the same setting.

4. A
Byers and others demonstrated that with lateralized oral tongue carcinomas
skip metastases in the N0 to level III and/or IV occur in approximately 16%
of patients. The use of a supraomohyoid neck dissection could potentially miss
this regional disease, as such the authors advocated a level I to IV neck
dissection in this setting.

5. E
In a patient with an oral lesion that will require soft tissue free flap
reconstruction and the potential need for postoperative radiation therapy,
removal of the lesion without the creation of a mandibulotomy is the best
available option. A pull-through technique allows the flap access to the neck
vasculature and spares the clinician from having to radiate a recent
mandibulotomy site.

230
BY DR. MOHAMMED ATIAA KAREEM ALNASHY
-------------------------------------------------------------------
CHAPTER SEVENTY-ONE: RECONSTRUCTION OF THE MANDIBLE
AND MAXILLA
------------------------------------------------------------------------------------------

1. Autogenous bone graft sources used in oromandibular reconstruction


include all of the following except
A. Calvarium
B. Rib
C. Radium
D. Scapula
E. Femur

2. The key substance that alone has been found to induce differentiation
of fibroblasts and mesenchymal bone cells into osteoblasts is
A. Interferon
B. Substance P
C. Bone morphogenic protein
D. Cartilage-inducing factors
E. Osteoinductive factor

3. The open anteromedial approach to the ilium minimizes postoperative


gait disturbance because
A. The incision is smaller.
B. Bone is harvested in a less traumatic fashion.
C. Attachment of the gluteal muscles is maintained.
D. The incidence of postoperative hematoma is lower.
E. The incidence of postoperative hypoesthesias is lower.

4. The main reason for hardware removal after a bone grafting procedure is to
A. Prevent extrusion
B. Prevent the long-term effects of stress shielding
C. Prevent infection
D. Avoid effects on dosimetry in postoperative radiation therapy
E. Improve the lower facial contour

231
BY DR. MOHAMMED ATIAA KAREEM ALNASHY
-------------------------------------------------------------------
5. Which of the following statements is not a desirable qualifier for bone
used in reconstruction of the mandible?
A. It has a natural shape or easy contourability to conform to the missing
mandible.
B. It is of sufficient length for reliable placement of endosteal dental
implants.
C. It is well vascularized.
D. Its vascular anatomy is easily preserved while contouring the graft.
E. There are no significant functional or aesthetic deficits at the donor
site following harvest.

CHAPTER 71
Reconstruction of the Mandible and Maxilla

1. E

2. C

3. C

4B

5B

232
BY DR. MOHAMMED ATIAA KAREEM ALNASHY
-------------------------------------------------------------------
CHAPTER SEVENTY-TWO: MAXILLOFACIAL PROSTHETICS FOR
HEAD AND NECK DEFECTS
--------------------------------------------------------------------------------------------
1. A maxillary surgical prosthesis for a dentate patient places emphasis on
preservation of
A. The alveolar ridge
B. The nasopalatine papilla
C. The hard palate
D. The molar teeth
E. The soft palate

2. Mandibular discontinuity is particularly problematic for functional


disabilities of
A. Speech
B. Swallowing
C. Mastication
D. Respiration
E. Lip sealing with absence of the inferior alveolar nerve

3. Resection of the tongue may indicate the use of a palatal augmentation


prosthesis, which has a primary purpose of
A. Improving speech production
B. Lowering the vault of the palate for undulation during deglutition
C. Creating an anterior barrier for food boluses
D. Compensating for lip incompetence
E. Decreasing resonance during speech

4. Resection of tumors of the soft palate may create


A. Paralysis of levator palatini muscle
B. Velopharyngeal incompetence
C. Velopharyngeal insufficiency
D. Hyponasal speech
E. Chronic otitis media

5. Placement of a palatal lift prosthesis


A. Can be successful without pharyngeal muscle mobility
B. Aids with masticatory ability
C. Decreases the oral transit time
D. Aids the soft palate with stimulation and closing off the nasopharynx
E. Should be directed to the tubercle of the second cervical vertebrae

233
BY DR. MOHAMMED ATIAA KAREEM ALNASHY
-------------------------------------------------------------------
CHAPTER 72
Maxillofacial Prosthetics for Head and Neck Defects

1. c
The hard palate is the primary stabilizing structure necessary for prosthetic
support. Without the hard palate, the prosthesis will be further seated into
the defect, resulting in impingement of the structures within the nasal cavity.
Removal of the inferior conchae, grafting of the cheek flap and sinus with
split-thickness skin are also important. Disease eradication is
yet even more a primary goal before preservation of any tissues.

2. C
Whether anterior or posterior, mandibular discontinuity usually creates
problems associated with swallowing from either interference with lip
sealing, pull of unopposed pterygomasseteric slings, suprahyoid muscles, or
tethered oral tongue used for wound closure. Mastication and speech are
secondary effects of mandibular discontinuity depending on their location.

3B
The primary role of the tongue is for swallowing. Respiration, speech
production, and assistance during mastication are important functions as
well. However, swallowing is best achieved through both the oral actions of
the tongue and pharyngeal as well.

4. C
Velopharyngeal insufficiency is a common result of resection of the soft
palate. Frequently, the soft palate is "insufficient in form" to close off the
nasopharynx for swallowing and speech to occur. Incompetency and paralysis
are possible sequelae with surgery of this type but are uncommon.

5D
A palatal lift prosthesis serves to close off the nasopharynx with the incompetent
soft palate. A secondary effect may be achieved by stimulating the soft palate into
increased functional level. Pharyngeal muscle activity should be present, and the
extension should be aimed at the first cervical vertebrae. Although a palatal lift
does not directly decrease oral transit times, addition of material to the palate
portion may serve the purpose of a combination palatal augmentation as well for
patients who also have paralytic tongue effects.

234
BY DR. MOHAMMED ATIAA KAREEM ALNASHY
-------------------------------------------------------------------
CHAPTER SEVENTY-THREE: BENIGN AND MALIGNANT TUMORS
OF THE NASOPHARYNX
--------------------------------------------------------------------------------------------

1. Which of the following statements regarding the management of


juvenile nasopharyngeal angiofibroma are true?
A. Best results are obtained by the combination of surgery and adjuvant
radiotherapy.
B. The main consideration to the surgical approach is the age of the patient.
C. Endoscopic resection is ideal for all tumors as it avoids facial incisions
and soft tissue and bony disruption.
D. Locally advanced tumors with intracranial extension can be effectively
managed with radiotherapy alone.
E. Recurrence seldom occurs if the tumor is completely removed.

2. With regard to histopathology in nasopharyngeal carcinoma (NPC),


A. Keratinizing squamous cell carcinoma of the nasopharynx (type I) is
the most common variety globally.
B. Types II and III have distinct ultrastructural features that allows for
a clear distinction between the two.
C. Type I is the most sensitive to radiotherapy and provides the best local
control rates.
D. Epstein-Barr virus titer seems to correlate better with types II and III
than with type I.
E. Open biopsy of a neck node is the simplest and most expeditious means
of obtaining pathological diagnosis.

3. With regard to staging in NPG


A. Ho's classification closely resembles the TNM staging system.
B. Ho's classification is the most widely used in Asia and has withstood
the test of time, despite not being validated.
C. The 1992 UICC/AJCC staging classification accurately distinguished T1
from T2 disease and shows good correlation with survival.
D. Both Ho's and the 1992 UICC/AJCC staging classification take into
account lateral parapharyngeal spread.
E. The 1997 UICC/AJCC staging classification has been prognostically
validated both in Asia and the West.

235
BY DR. MOHAMMED ATIAA KAREEM ALNASHY
-------------------------------------------------------------------
4. Which of the following statements regarding the management of NPC
is true?
A. Radiotherapy alone is seldom used in the treatment of NPC.
B. The use of chemotherapy in advanced disease is supported by level I data.
C. Neck irradiation need not be given if there is no clinical evidence of
neck disease.
D. Both three-dimensional conformal radiation therapy and intensity
modulated radiation therapy have been conclusively shown to improve
tumor coverage, locoregional control, and long-term complications.
E. Hyperfractionation and accelerated fractionation have been shown to
improve local control rates without increase in toxicity.

5. Surgery in NPC
A. Is sometimes used initially in the treatment of neck disease
B. Is used mainly in the treatment of residual or recurrent disease in the
primary site
C. Must take into account tumor extent, exposure, and control of the
internal carotid artery
D. Is not usually associated with significant morbidity
E. Does not provide good local control and survival in patients with
recurrent disease

236
BY DR. MOHAMMED ATIAA KAREEM ALNASHY
-------------------------------------------------------------------
CHAPTER 73
Benign and Malignant Tumors of the Nasopharynx

1D
Surgery is the treatment of choice for JNA. Combination therapy or radiation
alone is reserved for more advanced tumors. The main considerations for the
approach are tumor location and extent, as well as the available surgical
expertise. Endoscopic surgery, although ideal for avoiding soft tissue and
bony disruption, is at present indicated for low-volume disease with minimal
extension. Radiation therapy alone has been able to achieve local control rates
of 80% for locally advanced disease. Recurrence is often difficult to predict
even after complete extirpation and ranges between 30% and 46%.

2D
Keratinizing SCC accounts for 20% to 30% of cases seen in North America, but
less than 3% of all NPC in Asia. WHO histopathologic types II and III have
similar ultra-structural features that make it difficult sometimes to
distinguish between the two. Type I is the least radiosensitive and exhibits
the poorest local control. EBV titers have an 82% to 100% correlation with
types II and III compared with between 20% and 40% for type I. Fine-needle
aspiration biopsy is the most expeditious; open biopsy should be avoided if at
all possible.

3. E
Ho's classification comprises three T and N stages, and five overall stages,
which differs from the TNM classification. Ho's classification is the most
widely used in Asia and has been prognostically validated. No survival
difference has been shown between Tt and T2 disease based on the 1992
UICC/AJCC classification. The 1992 UICC/AJCC classification does not
take into account lateral parapharyngeal extension. The revised 1997
UICC/AJCC classification has been prognostically validated both in Asia and
the West.

237
BY DR. MOHAMMED ATIAA KAREEM ALNASHY
-------------------------------------------------------------------
4B
Radiotherapy alone is used in the treatment of stage I and II disease. Meta-
analyses of six randomized controlled trials found a progression-free and
overall survival improvement of 34% and 21% with the use of chemotherapy.
Neck irradiation is routinely given even in the absence of cervical
lymphadenopathy because of the high incidence of subclinical disease.
Although three-dimensional CRT has improved tumor coverage, there is no
conclusive evidence that locoregional and long-term complications are
improved. Accelerated fractionation may improve local control, but a
randomized controlled trial using accelerated hyperfractionation had to be
prematurely terminated because of excessive complications without
improvement in local control.

5. C
Surgery is never used in the initial management of even bulky neck disease,
because high response rates can be achieved with chemotherapy and
radiotherapy. The main role of surgery is for treatment of recurrent or
residual disease in the neck. The surgical approach must take into account
the tumor extent, the required exposure, and often control of the internal
carotid from the neck for tumors extending into the parapharyngeal space.
Surgical morbidity, especially if the patient has received more that one course
of radiation, is significant and includes skull base osteomyelitis, necrosis, or
bleeding. Surgery for recurrent disease can achieve local control rates of
between 43% and 67%.

238
BY DR. MOHAMMED ATIAA KAREEM ALNASHY
-------------------------------------------------------------------
CHAPTER SEVENTY-FOUR: PHARYNGITIS IN ADULTS
--------------------------------------------------------------------------------------------

1. A 24-year old, otherwise healthy man is seen with a 3-day history of


sore throat, low-grade fever, nasal stuffiness, and a nonproductive
cough. The oropharynx has mild erythema on examination. Which of
the following is the most likely diagnosis?
A. Laryngopharyngeal reflux
B. Group A-α-hemolytic Streptococcus pyogenes pharyngitis
C. Fungal pharyngitis
D. Allergy exacerbation
E. Viral pharyngitis

2. Which of the following is not part of the Centor scoring system for
predicting the diagnosis of group A-α-hemolytic streptococcal
pharyngitis?
A. History of fever
B. Anterior cervical adenopathy
C. Odynophagia
D. Tonsillar exudates
E. Absence of cough

3. Which antibiotic is currently considered the first choice by the


Infectious Disease Society of America, American College of Physicians,
and American Academy of Family Physicians for treatment of acute
pharyngitis caused by group A-α-hemolytic Streptococcus?
A. Erythromycin
B. Clindamycin
C. Tetracycline
D. Penicillin
E. Azithromycin

239
BY DR. MOHAMMED ATIAA KAREEM ALNASHY
-------------------------------------------------------------------
4. A 25-year-old man with a history of intravenous drug use is seen with
a 3-day history of sore throat, lethargy, high fevers, and headaches.
Which diagnostic test should be performed to evaluate for acute
retroviral syndrome?
A. Enzyme-linked immunosorbent assay (ELISA) for HIV
B. CD4 count
C. Western blot for HIV
D. Throat culture
E. Quantitative plasma HIV-1 RNA level

5. A 19-year-old man with a history of orogenital contact presents with a


5-day history of a painless ulcer in the left tonsil. What is the most
likely diagnosis?
A. Gonococcal pharyngitis
B. Primary syphilis
C. Secondary syphilis
D. Chlamydial pharyngitis
E. Infectious mononucleosis

240
BY DR. MOHAMMED ATIAA KAREEM ALNASHY
-------------------------------------------------------------------
CHAPTER 74
Pharyngitis in Adults

1. E
A viral etiology, most commonly from rhinovirus, is the most common cause of
acute sore throat in adults. The patient described in this question has symptoms
characteristic of the common cold. Nasal stuffiness and nonproductive cough are
not characteristic for a streptococcal pharyngitis.

2. C
The Centor scoring system is helpful for determining the likelihood for a
streptococcal etiology in adults that are seen with an acute sore throat. This scoring
system does not include odynophagia but includes the remainder of the choices in
the question.

3D
Penicillin is considered the first choice antibiotic for patients not allergic to it
because of its narrow spectrum of action that includes group A-p hemolytic
Streptococcus pyogenes (GABHS), low cost, and safety. No evidence exists for
tolerance or resistance of GABHS to this antibiotic. Patients with multiply
recurrent infections may have copathogens in the oropharyngeal tissues that
produce p-lactamases, and an alternative antimicrobial would be indicated for
these cases.

4. E
Acute retroviral syndrome caused by human immunodeficiency virus type 1 (HIV-
1) is characterized by a high viral load caused by the initial burst of viremia.
ELISA and Western blot are not positive until approximately 4 weeks after
acquiring the infection. The CD4 count is normal during acute infection.

5B
A painless, unilateral oral ulcer in a young patient with risk factors for sexually
transmitted diseases should be highly suspicious for primary syphilis. Proper
recognition and treatment are essential for preventing the infection from going into
a latent phase and then re-presenting as secondary syphilis. Secondary syphilis
manifestations in the oropharynx reveal enlarged, reddened tonsils, and a rash
commonly involves the palms and soles. Exudative pharyngitis is more
characteristic of gonococcal pharyngitis and infectious mononucleosis. The role of
chlamydia in causing pharyngitis in the absence of bronchitis or pneumonia is now
being questioned.

241
BY DR. MOHAMMED ATIAA KAREEM ALNASHY
-------------------------------------------------------------------
CHAPTER SEVENTY-FIVE: SLEEP APNEA AND SLEEP-
DISORDERED BREATHING
----------------------------------------------------------------------------------------

1. What is the international classification of obstructive sleep apnea


syndrome as a sleep disorder?
A. Intrinsic dyssomnia
B. Extrinsic dyssomnia
C. Intrinsic parasomnia
D. Extrinsic parasomnia

2. What are the characteristics of upper airway resistance syndrome?


A. Apnea-hypopnea index <5.
B. Minimal number of arousals during sleep.
C. The chief complaint is daytime sleepiness.
D. One-third of patients snore.

3. What is the association of the pathophysiology of obstructive sleep


apnea (OSA) in children and adults?
A. The cause of OSA in children and adults is tissue obstruction.
B. The cause of OSA in children and adults is primarily hypotonia.
C. Children have tissue obstruction and adults have hypotonia.

4. What medical conditions are associated with obstructive sleep apnea


syndrome?
A. Depression
B. Congestive heart failure
C. Gastroesophageal reflux
D. Diabetes mellitus
E. All of the above

5. Perioperative airway obstruction may be minimized by the following


actions?
A. Do not give premedications.
B. Ensure the patient can be masked ventilated before paralysis.
C. Use nasal continuous positive airway pressure postoperatively.
D. Use a patient-controlled anesthesia device (PCA) for pain control.

242
BY DR. MOHAMMED ATIAA KAREEM ALNASHY
-------------------------------------------------------------------
CHAPTER 75
Sleep Apnea and Sleep-Disordered Breathing
1. A
Dyssomnia is a sleep disorder that produces either difficulty initiating or
maintaining sleep (insomnia) or excessive daytime sleepiness. An intrinsic
disorder originates or develops within the body or arises from causes within
the body.

2. A, C
Upper airway resistance syndrome by definition has an apnea-hypopnea
index <5 or it would be classified as mild obstructive sleep apnea syndrome. It
is associated with frequent respiratory-related arousals during sleep, two-
thirds of patients snore, and the main complaint is daytime sleepiness or
fatigue. It is diagnosed during a sleep study by an elevated esophageal
pressure, elevated diaphragmatic eleetromyogram, or other respiratory
monitor illustrating a respiratory effort associated with an arousal. This is
one of the reasons that ambulatory or home sleep studies have limitations,
because they rarely suggest this diagnosis.

3. C
In general, the pediatric population has frank anatomic obstruction
as the cause of the obstructive sleep apnea. This is the rationale for a 75%
surgical success rate with adenoidectomy and tonsillectomy alone in the
pediatric patient. Adults have primarily upper airway hypotonia as the main
factor and frank tissue obstruction as the second factor. This is one of the
reasons why a uvulopalatopharyngoplasty has only a 39% to 40% surgical
success rate as defined as an apnea-hypopnea index <20 or an apnea index <10
with at least a 50% improvement.

243
BY DR. MOHAMMED ATIAA KAREEM ALNASHY
-------------------------------------------------------------------
4. E
Twenty percent of patients with obstructive sleep apnea have depression, and
many others may have personality changes including irritability.
Cardiovascular associations are well known to be associated with OSAS, with
35% of patients with hypertension, 85% with three medications to control
hypertension, 50% of congestive heart failure patients, 35% of angina and
stroke patients having an underlying diagnosis of obstructive sleep apnea.
Because of the increased intrathoracic pressure during the upper airway
obstructions during sleep, up to 45% of patients experience indigestion. OSAS
causes insulin resistance, and 70% of patients with OSAS are obese; these
factors cause this association.

5. A, B, C
Cautious use of intraoperative and postoperative use of narcotics is
imperative. A PCA device is not recommended, because patients can suppress
their respiratory drive to a dangerous level. These patients should receive the
degree of monitoring that the surgeon believes ensures a safe recovery, taking
into account many variables including the body habitus of the patient, the
patient's medical condition, the severity of the obstructive sleep apnea, and
the number and types of upper airway reconstruction.

244
BY DR. MOHAMMED ATIAA KAREEM ALNASHY
-------------------------------------------------------------------
CHAPTER SEVENTY-SIX: OROPHARYNGEAL MALIGNANCY
--------------------------------------------------------------------------------------------

1. Which of the following is not correctly associated with its embryologic


structure of origin?
A. Salivary glands: first pharyngeal arch
B. Tonsillar crypts: second pharyngeal pouch
C. Anterior tongue epithelium: second pharyngeal arch
D. Posterior tongue epithelium: third pharyngeal arch
E. Epiglottis: third and fourth pharyngeal arches

2. Which of the following statements regarding squamous cell carcinoma


(SCC) of the oropharynx is true?
A. Nonkeratinizing SCC is more common than keratinizing SCC.
B. Verrucous carcinoma almost always presents with metastasis.
C. 80% of all malignant oropharyngeal neoplasms are SCC.
D. Spindle cell carcinoma exhibits clinical behavior similar to that of
conventional SCC.
E. Basaloid SCC occurs most commonly in the tonsil.

3. Regarding neck metastases from oropharyngeal carcinoma,


A. Computed tomography is significantly more accurate at detecting neck
metastases than clinical evaluation alone.
B. Supraclavicular and posterior triangle nodal metastasis have the worst
prognosis for survival.
C. Contralateral metastasis occurs in 30% of tongue base tumors.
D. Given the moderate morbidity of a staging neck dissection for clinically
N0 nodes, a selective neck dissection is only performed in a minority of
patients initially seen with oropharyngeal SCC.
E. A T2 primary tumor resected with negative margins and no neck disease
necessitates postoperative radiation therapy.

245
BY DR. MOHAMMED ATIAA KAREEM ALNASHY
-------------------------------------------------------------------
4. Which of the following statements regarding treatment of
oropharyngeal carcinoma is true?
A. For early tonsil cancer, there is no significant difference in survival
between surgery and primary radiotherapy.
B. Primary closure of a tongue base defect can be performed only if less
than 25% of the tongue base is removed.
C. Tumors of the upper pharyngeal wall are usually accessible through a
transoral route.
D. For soft palate cancer, radiotherapy should be considered for lesions
less than 2 cm in diameter.
E. Wide resection of the tonsil and surrounding soft tissues can result in
significant adverse effects on function.

5. Which of the following regarding microvascular free flaps is true?


A. Advantages of the rectus abdominis free flap include ease of harvest,
length of the vascular pedicle, and ease of providing a sensate flap.
B. The forearm free flap uses the posterior cutaneous nerve of the arm to
provide sensation.
C. One advantage of the lateral arm free flap is the large size of its feeding
vessel.
D. In a fibula free flap, its muscle and soft tissue components cannot be
epithelialized.
E. The scapular free flap provides two skin paddles for use in
reconstruction.

246
BY DR. MOHAMMED ATIAA KAREEM ALNASHY
-------------------------------------------------------------------
CHAPTER 76
Oropharyngeal Malignancy

1. C
The epithelium of anterior tongue up to the terminal sulcus is derived from the
first pharyngeal arch. The other tissues are correctly associated with their
embryologic structures of origin.

2D
Nonkeratinizing SCC is less common than keratinizing SCC. Verrucous carcinoma
essentially does not metastasize. More than 90% of all malignant oropharyngeal
neoplasms are SCC. Basaloid SCC occurs most commonly in the tongue base,
followed by larynx, hypopharynx, and tonsil.

3B
CT, MRI, and clinical evaluation are equivalently accurate at detecting neck
metastases, at a rate of 70% to 80%. Contralateral metastasis occurs in 20% of tongue
base tumors. A staging neck dissection has low morbidity, so it is indicated for most
patients with oropharyngeal SCC. After a margin-negative resection of a T2
primary tumor with minimal or no neck disease, postoperative radiotherapy can be
avoided.

4. A
For resections of up to 50% of the tongue base, there is no adverse effect on function;
the defect can be closed primarily, through secondary intention, or with a small
thin flap. Tumors of the upper pharyngeal wall are considered challenging, because
access and reconstruction are difficult. The transoral route is used for resection of
most tonsillar cancers. In soft palate cancer, radiotherapy is favored when surgical
resection would result in considerable functional impairment, which is usually the
case when the lesion is >2 cm in diameter. Resection of the tonsil and surrounding
soft tissue does not usually result in impairment of function.

5. E
It is difficult to provide sensation with the rectus abdominis flap. Advantages of
the rectus abdominis flap include its ease of harvest, versatility, length of its
vascular pedicle, and reliability. The forearm free flap uses the lateral
antebrachial cutaneous nerve to provide sensation. The lateral arm free flap has a
small-caliber feeding vessel. The muscle and soft tissue components of a fibular
free flap can be epithelialized with a split-thickness skin graft with acceptable
results.

247
BY DR. MOHAMMED ATIAA KAREEM ALNASHY
-------------------------------------------------------------------
CHAPTER SEVENTY-SEVEN: RECONSTRUCTION OF THE
OROPHARYNX
--------------------------------------------------------------------------------------------

1. Which of the following statements regarding microvascular


reconstruction of advanced oropharyngeal tumors is true?
A. This form of reconstruction has improved prognosis and locoregional
control.
B. Other forms of reconstruction are no longer warranted.
C. Quality of life and functional status are restored at 6 months, and most
are improved at 1 year.
D. Fasciocutaneous flaps are preferred for all defects.
E. No reconstruction is needed because of the poor prognosis of these
patients.

2. The Gehanno technique of soft palate reconstruction is indicated in


cases in which
A. Complete soft palatectomy has been performed
B. A 25% defect exists
C. A 75% defect exists
D. A 50% or greater defect is present
E. Any soft palate defect is present

3. The goals of tongue base reconstruction include the following except


A. Articulation
B. Taste perception
C. Swallowing
D. Maintenance of the airway
E. To create a neotongue with the necessary bulk to create a shelf above
the laryngeal inlet

248
BY DR. MOHAMMED ATIAA KAREEM ALNASHY
-------------------------------------------------------------------
4. The platysma myocutaneous flap
A. Has not been used successfully in the tongue base
B. Cannot be performed in cases in which the facial artery has been divided
C. Is not a reliable form of reconstructing defects of the oropharynx
D. Flap survival rate in the tongue base is in the order of 65%
E. Provides a thin pliable skin paddle that is ideal for oropharyngeal
reconstruction

5. With regard to the lateral arm flap,


A. Elevation of the flap risks ischemia to the arm.
B. Neurotization of the flap is not possible.
C.It is not regarded as a good choice for oropharyngeal defects.
D. It is regarded as a good choice for combined defects of the pharyngeal
wall and tongue base.
E. The vascular pedicle is more robust than the radial forearm flap.

249
BY DR. MOHAMMED ATIAA KAREEM ALNASHY
-------------------------------------------------------------------
CHAPTER 77
Reconstruction of the Oropharynx

1. C
It has been shown that quality of life and functional status can be restored at 6 months
and usually is improved 1 year after microvascular reconstruction of advanced
oropharyngeal tumors. Despite the fact that the overall prognosis is poor,
reconstruction is justified to achieve the highest level of function possible. There is no
one free flap that is superior for all oropharyngeal defects, and despite our advances
in reconstruction, the overall survival rate has not changed. Several factors come into
play when deciding on which form of reconstruction is best for each individual
patient.

2D
The Gehanno technique has been used for defects that encompass 50% or more of the
soft palate. Kimata and others reported that the incidence of flap dehiscence is higher
in the above case when this technique was not used. This leads to contracture and
subsequent velopharyngeal insufficiency. This technique is not universally accepted,
and the other option is to perform a folding technique to reestablish the velopharynx.

3B
The goals of tongue base reconstruction in order of importance are maintenance of the
airway, swallowing, and articulation. These objectives are achieved with a form of
reconstruction that provides the necessary bulk to position the neotongue above the
laryngeal inlet. The perception of taste is not a goal in reconstructing defects of the
tongue base.

4. E
The platysma flap has been underused for oropharyngeal reconstruction. It provides
a reliable amount of pliable tissue for reconstruction. Overall flap survival in the
tongue base has been reported to be greater than 90%. Radiotherapy and ligation of the
facial artery are not contraindications for the use of this flap.

5D
The lateral arm flap is thought to be ideal for reconstructing combined defects of the
pharyngeal wall and tongue base. The distal aspect of the flap can be harvested over
the upper forearm, providing thin pliable tissue for the pharyngeal wall, while the
bulky upper arm component may be used in the tongue base. The flap is supplied by
the posterior radial collateral artery, which tends to be smaller than the radial artery.
However, it is a terminal artery, thus not putting the arm at risk of ischemia.
Neurotization may be performed with the posterior cutaneous nerve of the arm.

250
BY DR. MOHAMMED ATIAA KAREEM ALNASHY
-------------------------------------------------------------------
CHAPTER SEVENTY-EIGHT: DIAGNOSTIC IMAGING OF THE
PHARYNX AND ESOPHAGUS
--------------------------------------------------------------------------------------------
1. A patient receives surgery, radiation, and chemotherapy for an
oropharyngeal squamous cell carcinoma. One month after the completion of
therapy, the most accurate way to assess for residual tumor is
A. Computed tomography (CT)
B. Magnetic resonance imaging
C. Endoscopy
D. Positron emission tomography (PET)
E. Combined PET/CT

2. In a patient with a suspected leak after supra-glottic laryngectomy, which is


the most appropriate initial oral contrast agent?
A. Thin barium suspension
B. Thick barium suspension
C. Gastrografin
D. Nonionic intravenous CT contrast agents
E. An esophagogram should not be performed on this patient

3. Esophagogram is superior to endoscopy in the assessment of which disease?


A. Epidermolysis bullosa
B. Nutcracker esophagus
C. Mallory-Weiss tear
D. Barrett's esophagus
E. Laryngeal carcinoma

4. Esophageal ulcers are seen in all the following except


A. Herpes esophagitis
B. Acute radiation esophagitis
C. Intramural pseudodiverticulosis
D. Barrett's esophagus
E. Crohn's disease

5. CT is superior to an esophagogram in the assessment of


A. Prevertebral spread of a hypopharyngeal tumor
B. Staging of an esophageal carcinoma
C. Postsurgical leak
D. Fourth branchial cleft fistula
E. Recurrence in a jejunal interposition graft

251
BY DR. MOHAMMED ATIAA KAREEM ALNASHY
-------------------------------------------------------------------
CHAPTER 78
Diagnostic Imaging of the Pharynx and Esophagus

1. E
Gross-sectional imaging, such as CT and MR, rely on morphologic changes
over time. An examination performed 1 month after therapy would serve
primarily as a baseline for further tests. Endoscopy is limited to mucosal
recurrences. PET is sensitive for residual tumor but not as accurate as
combined PET/CT, particularly in the head and neck.

2D
If a perforation or leak is suspected, barium should not be used initially.
Barium may inspissate in the soft tissues and cause a granulomatous reaction.
If Gastrografin is aspirated, it may induce respiratory distress; this is of
particular concern after a supraglottic laryngectomy. Non-ionic CT contrast
agents are the most appropriate first choice; barium may then be used if no
leak is detected.

3. A
Endoscopy may cause strictures in patients with epidermolysis bullosa, so
fluoroscopy is preferred. Manometry is most useful in nutcracker esophagus.
The other diseases are best assessed endoscopically.

4. C
Intramural pseudodiverticulosis is dilation of mucous glands and is not
confused with ulceration on an esophagogram. The other disease may all
present with esophageal ulcers.

5B
Staging of esophageal carcinoma relies on cross-sectional techniques for
extent of tumor and lymph node involvement. Prevertebral spread of
hypopharyngeal tumors is best assessed fluoroscopically by examining the
motion of the larynx against the vertebral column. Sinus tracts and fistulas,
in general, are not well seen on CT. The CT appearance of a jejunal graft may
be confused with recurrent tumor, whereas esophagogram shows a
characteristic mucosal pattern.

252
BY DR. MOHAMMED ATIAA KAREEM ALNASHY
-------------------------------------------------------------------
CHAPTER SEVENTY-NINE: ENDOSCOPY OF THE PHARYNX AND
ESOPHAGUS
--------------------------------------------------------------------------------------------

1. The following are not well tolerated by the awake or nonsedated patient.
A. Rigid esophagoscopy
B. Flexible esophagoscopy
C. Rigid pharyngoscopy
D. Muller maneuver

2. The role of functional endoscopic evaluation of swallowing does not include


A. Assessment of delay in swallowing
B. Assessment of laryngeal penetration
C. Assessment of esophageal motility
D. Assessment of vocal cord mobility

3. Complications of esophagoscopy include


A. Esophageal perforation
B. Dental trauma
C. Bleeding
D. Cardiac arrhythmia
E. All of the above

4. Indications for the use of pharyngoscopy include


A. Assessment of velopharyngeal insufficiency
B. Evaluation of a patient with obstructive sleep apnea
C. Tumor surveillance
D. Evaluation of a patient with dysphagia
E. All of the above

5. Preoperative assessment of the patient undergoing an esophagoscopy


does not routinely include
A. History and physical examination
B. Radiologic evaluation
C. Dental X-rays
D. Evaluation for cervical spine instability

253
BY DR. MOHAMMED ATIAA KAREEM ALNASHY
-------------------------------------------------------------------
CHAPTER 79
Endoscopy of the Pharynx and Esophagus

1. A
Rigid esophagoscopy, although it can be performed on the awake patient, is
not well tolerated and carries an increased risk of complications.

2. C
Functional endoscopic evaluation of swallowing (FEES) is typically used to
assess delay in swallowing, laryngeal penetration, aspiration, and pharyngeal
residue. As part of the FEES vocal cord, mobility is examined as well.

3. E
Complications of rigid and flexible esophagoscopy include dental trauma,
bleeding, perforation, mediastinitis, cardiac arrhythmia, and carotid artery
dissection.

4. E
Indications for pharyngoscopy are varied and are listed in Table 1.

5. C
Although dental trauma is a complication of any peroral endoscopy,
preoperative dental x-rays are not routinely obtained. Instead, the dentition
is carefully examined before and after the procedure. Preoperative evaluation
includes a thorough physical examination, review of medical and surgical
history, review of current medicine regimen and drug allergies, and possibly
radiographic evaluation. Cervical spine instability should be suspected in any
patient with congenital anomalies and evaluated preoperatively, especially if
rigid esophagoscopy is planned.

254
BY DR. MOHAMMED ATIAA KAREEM ALNASHY
-------------------------------------------------------------------
CHAPTER EIGHTY: THE ESOPHAGUS: ANATOMY, PHYSIOLOGY,
AND DISEASES
--------------------------------------------------------------------------------------------

1. All of the following are true regarding ambulatory 24-hour esophageal


pH monitoring except
A. The single most important parameter to measure is the amount of time
that the pH is less than 6.0.
B. The primary indications for this procedure are to document excessive
acid reflux in patients with expected gastroesophageal reflux disease
(GERD) and to evaluate the efficacy of medical or surgical therapy.
C. Standard pH monitoring measures distal esophageal acid exposure by
use of a single pH electrode catheter positioned 5 cm about the superior
margin of the LES.
D. One advantage of this method is the ability to correlate symptoms with
reflux episodes.
E. Less than 20% of reflux episodes in patients with well-documented
GERD are associated with symptoms.

2. A patient with oropharyngeal dysphagia is likely to demonstrate all of


the following except
A. An improvement in symptoms when swallowing liquids
B. Difficulty with initiating a swallow
C. Localization of symptoms to the cervical or throat region
D. A disruption in the finely coordinated act of swallowing secondary to
neuromuscular dysfunction
E. Diseases that affect nerves or muscles, as well as structural abnormalities.

3. All of the following statements regarding extraesophageal GERD are


true except
A. The primary esophageal cause of noncardiac chest pain is GERD in 40%
to 60% of patients.
B. Laryngeal signs related to GERD include hoarseness, throat clearing,
dysphagia, increased phlegm, and globus sensation.
C. Approximately 10% to 20% of patients with asthma have GERD.
D. GERD is the third most common cause of chronic cough.
E. Patients with extraesophageal GERD do not typically demonstrate
esophagitis

255
BY DR. MOHAMMED ATIAA KAREEM ALNASHY
-------------------------------------------------------------------
4. A white man with a history of GERD and Barrett's esophagus is
initially seen with rapidly progressive solid food dysphagia. He is
found to have a neoplastic lesion in the distal esophagus at the
gastroesophageal junction. Biopsy of this lesion is most likely to reveal
which of the following
A. Leiomyoma
B. Adenocarcinoma
C. Squamous cell carcinoma
D. Lymphoma

5. All of the following are true regarding a patient who is seen in the
emergency department after ingestion of a strong alkali chemical except
A. Upper endoscopy should be performed during the first 24 to 48 hours
after ingestion.
B. The patient will have an estimated thousandfold increase in the risk of
squamous cell carcinoma of the esophagus.
C. The esophageal injury is the result of a coagulative necrosis.
D. The patient may complain of oropharyngeal, retrosternal, or epigastric pain.
E. The patient should be examined for evidence of oropharyngeal injury.

256
BY DR. MOHAMMED ATIAA KAREEM ALNASHY
-------------------------------------------------------------------
CHAPTER 80
The Esophagus: Anatomy, Physiology, and Diseases
1. A
The percent time that the pH is less than 4 is the single most
important parameter to measure during ambulatory 24-hour esophageal pH monitoring.
A reflux episode is defined when the esophageal pH drops below 4.0. This value is chosen
based on the proteolytic activity of pepsin, which is most active at and below this pH. In
addition, pH less than 4.0 best distinguishes between symptomatic patients and
asymptomatic controls.

2. A
The distinction between oropharyngeal and esophageal dysphagia is crucial, because they
have different causes. Oropharyngeal dysphagia is most commonly caused by disruptions
in the finely coordinated act of swallowing secondary to neuromuscular dysfunction. In
this setting, the symptoms may be more severe when swallowing liquids. Any disease that
affects the nerves or muscles can produce oropharyngeal dysphagia, with the more
common associations including ALS, myasthenia gravis, and Parkinson's disease.

3. C
There is a clear association between asthma and GERD, and 70% to 80% of patients with
asthma have GERD. There are two main proposed mechanisms of acid-induced asthma.
The first is that proximal esophageal reflux leads to microaspiration/bronchospasm. The
second is that a vagally mediated esophageal-bronchial reflex results in bronchospasm.
GERD is the third most common cause of chronic cough after postnasal drip and asthma.
It accounts for 21% of cases of chronic cough.

4B
The two main types of esophageal carcinoma are squamous cell carcinoma (SCCA) and
adenocarcinoma. More than half of the cases in the United States are now
adenocarcinoma. The epidemiology of the two is quite different. SCCA is typically a
disease of African-American men and is associated with alcohol and tobacco abuse.
Patients with a history of caustic esophageal injury are also at increased risk.
Adenocarcinoma is predominately a disease of white men and has a well-documented
association with GERD and Barrett's esophagus. Adenocarcinoma occurs predominately
in the distal esophagus and at the GE junction.

5. C
Strong alkali and acids are the most likely to produce esophageal injury when ingested.
Strong alkalis are contained in drain cleaners and other household cleaning products.
Lye is a generic term for a strong alkali, usually sodium or potassium hydroxide, used in
these cleansing agents. Injury to the esophagus is more severe with alkali substances than
acid substances, because they produce a liquefactive necrosis. This results in rapid and
deep tissue injury compared with acidic agents, which produce a coagulative necrosis that
limits penetration and injury.

257
BY DR. MOHAMMED ATIAA KAREEM ALNASHY
-------------------------------------------------------------------
CHAPTER EIGHTY-ONE: ZENKER'S DIVERTICULUM
--------------------------------------------------------------------------------------------

1. Where is Killian-Jamieson's region located?


A. Between the cricopharyngeal and inferior constrictor muscle
B. Between the oblique and transverse fibers of the cricopharyngeal
muscle
C. Between the cricopharyngeal and most superior esophageal circular
muscle
D. Between the fibers of the inferior constrictor muscles
E. None of the above

2. What diagnostic test(s) should be obtained to evaluate a patient


suspected of having an esophageal diverticulum?
A. Chest x-ray
B. Barium swallow
C. Computed tomography scan
D. Esophagogastroduodenoscopy (EGD)
E. Rigid endoscopy

3. What are the advantages of the endoscopic stapling method over other
endoscopic techniques?
A. Faster operative times
B. No thermal injury to the recurrent laryngeal nerve
C. The incised mucosa is sealed mechanically
D. Lower complication rate
E. No external scar is produced

4. Which statement(s) is/are false?


A. No cricopharyngeal myotomy is performed with endoscopic techniques.
B. ESD can be performed more quickly than external techniques.
C. There is no need for perioperative antibiotics for the electrostatic
discharges (ESD) procedure.
D. Zenker's diverticulum is thought to be due to discoordination of the
cricopharyngeal muscle during glutition.
E. There is no role for external techniques with the development of
endoscopic techniques.

258
BY DR. MOHAMMED ATIAA KAREEM ALNASHY
-------------------------------------------------------------------
5. What are some methods to help completely divide the common wall
between the esophagus and diverticulum during ESD?
A. Retraction sutures
B. Placing the longer stapling blade containing the cartridge into the esophagus
C. Sawing off the distal part of the stapler anvil
D. Use of multiple stapler cartridges
E. Placing the blades of the Weerda laryngoscope directly into the
diverticulum and esophagus as distally as possible

259
BY DR. MOHAMMED ATIAA KAREEM ALNASHY
-------------------------------------------------------------------
CHAPTER 81
Zenker's Diverticulum

1B
Zenker's diverticulum is classically defined as a pulsion diverticulum found
bulging through the hypopharyngeal mucosa and sub-mucosa between the
cricopharyngeal muscle and inferior constrictor muscle in an area of
weakness called Killian's dehiscence or triangle. However, there are other
areas of weaknesses where pulsion diverticula may form, including Killian-
Jamieson's area between the oblique and transverse fibers of the
cricopharyngeus and Laimer's triangle formed between the cricopharyngeal
muscle and the most superior esophageal wall circular muscles. More rarely,
they also may be found in the posterolateral or lateral areas of the pharynx
and hypopharynx (pharyngocele).

2. B
The diagnostic test of choice is barium swallow radiography. The test will
allow the size and position of the sac to be defined. Although the chest x-ray
may reveal a hazy opacity over a lung apex suggesting a diverticulum, it does
not have any surgical value. It may have value in assessing preoperative
pulmonary status of the patient, however. Occasionally, a diverticulum may
be first discovered incidentally during esophagogastroduodenoscopy or rigid
cervical esophagoscopy but is unnecessary in diagnosis. However, if other
causes of dysphagia are suspected, these tests may be worthwhile. GT scans
are unnecessary, unless one suspects a neck mass contributing to the patient's
symptoms.

3. B, C, D
All endoscopic procedures take approximately 30 minutes to perform. ESD
does not induce thermal injury to the mucosa and surrounding tissues as laser
and cautery methods. Such thermal injury could potentially injure the
recurrent laryngeal nerve. ESD simultaneously incises and seals the mucosa
with staples. ESD does have a lower complication rate compared with other
endoscopic, as well as external, techniques. All endoscopic techniques do not
produce an external scar.

260
BY DR. MOHAMMED ATIAA KAREEM ALNASHY
-------------------------------------------------------------------
4. A, E
An internal cricopharyngeal myotomy is performed with endoscopic
techniques when dividing the common wall that contains the cricopharyngeal
muscle. ESD procedures take approximately 30 minutes to perform, whereas
external techniques take several hours to complete. We have not found
perioperative antibiotics necessary for the ESD procedures. Among the many
theories attempting to explain the etiology of Zenker's diverticulum
formation is the dis-coordination of the cricopharyngeal muscle during
glutition, a theory first proposed by Bell in 1816. Even with the development
and numerous advantages of endoscopic techniques, there is still a role for
external approaches to address Zenker's diverticulum. These include difficult
and/or impossible exposure of the diverticulum caused by patient anatomy
such as kyphosis, large cervical osteophytes, or small oropharyngeal opening.
Also, retraction of the common wall may not be possible in patients with
recurrent small ZD from prior external approaches secondary to scarring,
making exposure and divisibility of the cricopharyngeal muscle difficult if
not impossible, even with stitches to help retract the common wall. Last, ESD
should not be performed if diverticular carcinoma is highly suspected or
confirmed on intraoperative biopsy of a diverticular lesion. External
diverticulectomy should be performed in this case.

5. A, B, D
Retraction sutures, placing the longer stapling blade containing the cartridge
into the esophagus, and using multiple stapler cartridges for large diverticula
are all good methods to help completely divide the common wall between the
esophagus and diverticulum during ESD. Sawing off the distal part of the
stapler anvil as suggested by Collard is unnecessary when using retraction
sutures and potentially may even adversely affect the integrity of the stapler
introducing unnecessary risk to the procedure. One should never place the
blades of the Weerda laryngoscope directly into the diverticulum and
esophagus as distally as possible. The reason is because the diverticular walls
are composed of only mucosa and submucosa, which may be easily perforated
with insertion of the laryngoscope blades. One should insert the laryngoscope
just far enough to expose the superior border of the common wall completely.

261
BY DR. MOHAMMED ATIAA KAREEM ALNASHY
-------------------------------------------------------------------
CHAPTER EIGHTY-TWO: NEOPLASMS OF THE HYPOPHARYNX
AND CERVICAL ESOPHAGUS
--------------------------------------------------------------------------------------------
1. Which of the following statements is false?
A. Hypopharyngeal cancer is more common in men.
B. Hypopharyngeal cancer is more common in black men.
C. The most common presenting complaint of hypopharyngeal cancer is otalgia.
D. Most hypopharyngeal cancers present with associated lymphadenopathy.
E. In total, 5-year survival is less than 35% in patients with hypopharyngeal
cancer.

2. A 62-year-old smoker and former alcoholic is seen with a 3.5-cm left-


sided neck mass that is firm, minimally mobile, and nontender.
Clinical examination fails to identify a primary tumor. The best initial
imaging study to evaluate for hypopharyngeal cancer is
A. Barium swallow
B. Noncontrast enhanced high-resolution computed tomography (CT) scan
of the neck
C. Contrast-enhanced high resolution CT scan of the neck
D. Magnetic resonance imaging (MRI) with gadolinium enhancement of
the neck
E. Whole body positron emission tomography (PET) scan

3. Which of the following tumor types is treated primarily with radiation


therapy in the hypopharynx?
A. Squamous cell carcinoma
B. Lymphoepithelioma
C. Adenosquamous carcinoma
D. Basosquamous carcinoma
E. Synovial cell sarcoma

262
BY DR. MOHAMMED ATIAA KAREEM ALNASHY
-------------------------------------------------------------------
4. A 65-year-old man is seen with a neck mass and otalgia on the right.
Laryngoscopy demonstrates a squamous cell carcinoma of
approximately 2.5 cm in the lateral wall of the pyriform sinus
extending to the apex, but without involvement of the cervical
esophagus. The most conservative surgical option for this patient is
likely to be
A. Lateral pharyngectomy and primary closure
B. Lateral pharyngectomy and pectoralis flap reconstruction
C. Lateral pharyngectomy and radial forearm free-flap reconstruction
D. Partial laryngopharyngectomy
E. Total laryngectomy and partial pharyngectomy

5. An MRI scan of the preceding patient demonstrates a 3-cm necrotic


node on the side of the tumor without any additional
lymphadenopathy. Management of the neck in the patient described in
the preceding should include
A. Bilateral I to V neck dissection
B. Bilateral II to V neck dissection
C. Ipsilateral I to V neck dissection and contralateral II to IV neck dissection
D. Ipsilateral II to IV neck dissection
E. Ipsilateral I to III neck dissection

CHAPTER 82

Neoplasms of the Hypopharynx and Cervical Esophagus

1. C

2. D

3. B

4. E

5. C

263
BY DR. MOHAMMED ATIAA KAREEM ALNASHY
-------------------------------------------------------------------
CHAPTER EIGHTY-THREE: RADIOTHERAPY AND
CHEMOTHERAPY OF SQUAMOUS CELL CARCINOMAS OF THE
HYPOPHARYNX AND ESOPHAGUS
--------------------------------------------------------------------------------------------
1. Which of the following statements regarding the treatment of advanced
hypopharynx cancer is true?
A. Conventional external beam irradiation is the treatment of choice for
T4 hypopharyngeal cancer.
B. When treated by radical surgery and postoperative irradiation, these
cancers usually recur in half the cases at the primary site.
C. The most frequent evolution after radical surgery and postoperative
irradiation is the appearance of distant metastases.
D. Large randomized trials have concluded in a similar outcome either
after radical surgery and postoperative irradiation or after definitive
irradiation alone.
E. Adjuvant chemotherapy has improved the outcome after either radical
surgery and postoperative irradiation or definitive irradiation alone.

2. Which of the following statements regarding locoregional control after


total laryngectomy with partial laryngectomy and radical neck
dissection for a preliminary untreated pyriform sinus cancer is true?
A. Preoperative radiation therapy has been proven to be able to improve
locoregional control.
B. Postoperative radiation therapy has been proven to be able to improve
locoregional control.
C. Preoperative chemotherapy has been proven to be able to improve
locoregional control.
D. Perioperative chemotherapy has been proven to be able to improve
locoregional control.
E. Postoperative chemotherapy has been proven to be able to improve
locoregional control.

264
BY DR. MOHAMMED ATIAA KAREEM ALNASHY
-------------------------------------------------------------------
3. Which of the following statements regarding larynx preservation with
induction chemotherapy is true?
A. After chemotherapy, a subsequent irradiation is the treatment of choice
whatever the response to chemotherapy.
B. Apart from its ability to allow preservation of some of the larynx,
induction chemotherapy has significantly improved overall survival.
C. Apart from the ability to allow to preservation of some of the larynx,
induction chemotherapy has definitively suppressed distant failures.
D. Induction chemotherapy has decreased the incidence of metachronous
cancers.
E. This strategy assessed in a randomized trial has allowed preservation
of the larynx in half the survivors at 3 and 5 years.

4. Which of the following statements regarding the treatment of


squamous cell esophageal cancer is true?
A. Postoperative radiation therapy improves survival in localized cancer.
B. Postoperative chemotherapy therapy improves survival in node-positive
cancer.
C. Increasing radiation dose from 50.4 to 64.8 Gy did not translate into
enhanced survival in cancer treated by definitive chemoradiation.
D. Split-course should be preferred as conventional protracted radiation
therapy in esophageal cancer treated by definitive chemoradiation.
E. A Cochrane systematic review supports the need of preoperative
chemotherapy in localized cancer.

5. Which of the following statements regarding the treatment of


squamous cell esophageal cancer is true?
A. In locally advanced operable cancer responding to chemoradiation, the
continuation of chemoradiation is an alternative to surgery.
B. Preoperative chemoradiation improves survival in operable cancer.
C. Preoperative chemoradiation improves the disease-free survival in
operable cancer.
D. In metastatic disease, chemotherapy provides a slight but significant
increase in survival vs best supportive care.
E. Definitive chemoradiation increases survival compared with radiation
therapy alone.

265
BY DR. MOHAMMED ATIAA KAREEM ALNASHY
-------------------------------------------------------------------
CHAPTER 83
Radiotherapy and Chemotherapy of Squamous Cell Carcinomas of the
Hypopharynx and Esophagus

1. C
Advanced hypopharyngeal cancers, when resectable, are better controlled by
radical surgery (i.e., total laryngectomy and partial pharyngectomy and radical
neck dissection) and postoperative irradiation. This treatment may control four of
five patients above the clavicles, but most of the patients subsequently have distant
metastases develop.

2B
All studies have concluded that the only one adjunct that has been able to improve
locoregional control after radical surgery of the hypopharynx. Preoperative
irradiation has demonstrated a deleterious impact, whereas chemotherapy,
whatever the setting, has no improved the locoregional control.

3. E
Induction chemotherapy-based larynx-preserving strategies have demonstrated an
ability to allow preservation of the larynx in good responders to chemotherapy. On
the contrary, there was no impact (favorable or unfavorable) on locoregional
control, distant metastases, second primary tumors, and overall survival.

4. C, E
None of the studies that have examined the role of either postoperative radiation
or postoperative chemotherapy have observed any benefit with the adjuvant
treatment. Increasing radiation dose did not translate into enhanced survival but
provides increased morbidity and mortality rates. Protracted radiation therapy
increases the 2-year disease-free survival in a definitive chemoradiation regimen
compared with split-course radiation therapy. A Cochrane systematic review favors
the preoperative chemotherapy.

5. A, C, E
Continuing chemoradiation is an alternative to surgery in locally advanced
operable cancer responding to chemoradiation. Preoperative chemoradiation does
not improve survival but improves disease-free survival. Up to now, we still do not
know whether chemotherapy provides any benefit in terms of survival vs best
supportive care in metastatic disease. Definitive chemoradiation with 5-
fluorouracil and cisplatin significantly increases survival compared with radiation
therapy alone.

266
BY DR. MOHAMMED ATIAA KAREEM ALNASHY
-------------------------------------------------------------------
CHAPTER EIGHTY-FOUR: RECONSTRUCTION OF HYPOPHARYNX
AND ESOPHAGUS
--------------------------------------------------------------------------------------------

1. Which of the following was not a reason for abandoning the


deltopectoral flap for reconstruction of the pharynx following
laryngopharyngectomy?
A. High fistula rates
B. Delay in postoperative chemoradiation therapy
C. Stenosis of the flap
D. High donor site morbidity
E. Need for multiple procedures

2. Which of the following is not an advantage of pectoralis major


myocutaneous flap over deltopectoral flap reconstruction?
A. Lower flap failure rates
B. Single-stage reconstruction possible
C. Lower mortality rate
D. More versatile for range of pharyngectomy defects
E. Shorter delay for initiation of radiation therapy

3. Which of the following procedures has the highest morbidity and


mortality rates?
A. Tubed pectoralis major myocutaneous flap
B. Gastric pull-up
C. Colon transposition graft
D. Free radial forearm fasciocutaneous graft
E. Free jejunum interposition graft

4. Which of the following reconstruction methods gives the highest rate


of return to oral feeding?
A. Tubed pectoralis major myocutaneous flap
B. Gastric pull-up
C. Colonic transposition flap
D. Free radial forearm fasciocutaneous graft
E. Free jejunum interposition graft.

267
BY DR. MOHAMMED ATIAA KAREEM ALNASHY
-------------------------------------------------------------------
5. Which of the following flaps is the most susceptible to ischemic injury?
A. Deltopectoral flap
B. Pectoralis major myocutaneous flap
C. Radial forearm free flap
D. Lateral thigh free flap
E. Jejunal free flap

CHAPTER 84

Reconstruction of Hypopharynx and Esophagus

1. D

2. D

3. C

4. B

5. C

268
BY DR. MOHAMMED ATIAA KAREEM ALNASHY
-------------------------------------------------------------------

PART SEVEN
-------------------------------------------------
-------------------------------------------------
LARYNX/TRACHEA/BRONCHUS

269
BY DR. MOHAMMED ATIAA KAREEM ALNASHY
-------------------------------------------------------------------
CHAPTER EIGHTY-FIVE A:- LARYNGEAL AND PHARYNGEAL FUNCTION
--------------------------------------------------------------------------------------------

1. Which muscle opens the larynx?


A. Cricothyroid
B. Thyroarytenoid
C. Lateral cricoarytenoid
D. Posterior cricoarytenoid
E. Interarytenoid

2. Laryngospasm is more likely to occur in a patient who is


A. Hypoxic
B. Hypercarbic
C. Acidotic
D. Lightly anesthetized
E. Deeply anesthetized

3. Strong respiratory demand alters activity of the posterior


cricoarytenoid (PCA) muscle so that the PCA
A. Mirrors activity in the diaphragm
B. Mirrors activity in the cricothyroid muscle
C. Begins to contract before onset of inspiration
D. Stops contracting before the inspiration ends
E. Continues to contract during expiration

4. Vocal pitch drops with which pattern of laryngeal muscle activity?


A. Isolated thyroarytenoid (TA) muscle contraction
B. Isolated cricothyroid (CT) muscle contraction
C. Isolated PCA
D. Cocontraction of the TA and CT
E. Cocontraction of the PCA and lateral cricoarytenoid (LCA)

5. The shaping of vocal sound into words is termed


A. Phonation
B. Resonance
C. Articulation
D. Intonation
E. Transduction

270
BY DR. MOHAMMED ATIAA KAREEM ALNASHY
-------------------------------------------------------------------
CHAPTER 85A
Laryngeal and Pharyngeal Function

1D
The posterior cricoarytenoid muscle pulls medially and inferiorly on
the muscular process of the arytenoids, rotating that cartilage so that the
vocal process of the arytenoids moves rostrally and laterally.

2D
Laryngospasm is most like to occur in response to laryngeal stimulation in a
well-oxygenated patient in a light plane of anesthesia.

3. E
During normal breathing, the PCA begins contracting just before onset of
inspiration and is silent during exhalation. With increasing respiratory
demand, the PCA continues contracting after the onset of exhalation to
facilitate the egress of air.

4. A
Thyroarytenoid muscle contraction shortens and thickens the vocal fold,
lowering vocal pitch.

5. C
The phonated voice produced by the larynx is articulated into words by
actions of the upper aerodigestive tract.

271
BY DR. MOHAMMED ATIAA KAREEM ALNASHY
-------------------------------------------------------------------
CHAPTER EIGHTY-FIVE B: - EVALUATION AND MANAGEMENT
OF HYPERFUNCTIONAL DISORDERS
--------------------------------------------------------------------------------------------

1. Rhythmic movements of the tongue, jaw, and lips may be associated


with which of the following drugs?
A. Diazepam
B. Diphenhydramine
C. Cyclobenzaprine
D. Chlorpromazine
E. Clozapine

2. Which of the following statements is true about dystonias?


A. Most patients with idiopathic dystonia show an autosomal-dominant
inheritance pattern with reduced penetrance.
B. Most patients with primary laryngeal dystonia will have abnormal
movements develop in another body part.
C. X-linked torsion dystonia is associated with parkinsonism.
D. Most cases of laryngeal dystonia are associated with other neurologic
conditions.
E. Older age is associated with a higher probability of spread to another
body part.

3. Which of the following is not a feature of Meige's syndrome?


A. Blepharospasm
B. Adductor spasmodic dysphonia
C. Oromandibular dystonia
D. Writer's cramp
E. Lingual dystonia

4. Features that support a diagnosis of spasmodic dysphonia include all


of the following except
A. Voice improvement with singing
B. Abnormal response to stress on personality testing
C. Voice improvement with alcohol ingestion
D. Worsening of voice on the telephone
E. Voice improvement with pinching of the nares

272
BY DR. MOHAMMED ATIAA KAREEM ALNASHY
-------------------------------------------------------------------
5. The mechanism of action of botulinum toxin is
A. Blockade of muscarinic acetylcholine receptors at the neuromuscular
junction
B. Blockade of nicotinic acetylcholine receptors at the neuromuscular
junction
C. Inhibition of acetylcholine reuptake at the neuromuscular junction
D. Inhibition of acetylcholine release into the neuromuscular junction
E. Inhibition of intracellular acetylcholine formation

CHAPTER 85B

Evaluation and Management of Hyperfunctional Disorders

1D

2. C

3D

4B

5. D

273
BY DR. MOHAMMED ATIAA KAREEM ALNASHY
-------------------------------------------------------------------
CHAPTER EIGHTY-SIX: - VISUAL DOCUMENTATION OF THE LARYNX
-------------------------------------------------------------------------------------------
1. What is the best way to minimize a moire pattern during laryngeal
examinations?
A. Record the examination with National Television Standards Committee
(NTSC) format instead of Phase Alternating Line (PAL) format.
B. Focus the camera.
C. Defocus the camera
D. Use a flexible laryngoscope instead of a rigid telescope.
E. Use a digital format for recording the images.

2. The following parameter(s) can be accurately assessed only with


stroboscopic lighting:
A. Vocal fold closure
B. Mucosal wave
C. Vocal fold edges
D. Supraglottic constriction
E. Vibration symmetry

3. Which closure patterns are sometimes found in subjects without voice


disorders?
A. Spindle-shaped gap
B. Small posterior gap
C. Irregular gap
D. Anterior gap
E. Incomplete

4. What conditions would not cause decreased mucosal wave?


A. Scarring
B. Incomplete closure with large glottic gap
C. Cyst
D. Increased pitch
E. Sulcus vocalis

5. Supraglottic constriction during phonation can be


A. A sign of muscle tension dysphonia
B. A sign of dehydration
C. Masking an underlying lesion or bowing
D. A normal variant
E. A sign of an autoimmune disorder

274
BY DR. MOHAMMED ATIAA KAREEM ALNASHY
-------------------------------------------------------------------
CHAPTER 86
Visual Documentation of the Larynx

1. C
A moire pattern is a colored fringe effect produced when there is overlap of
linear features in an image. It can be reduced by slightly defocusing the image
or with a filter placed between the eyepiece and the camera.

2. A, B, E

3. A, B, D, E

4. All would cause decreased mucosal wave.

5. A, C, D

275
BY DR. MOHAMMED ATIAA KAREEM ALNASHY
-------------------------------------------------------------------
CHAPTER EIGHTY-SEVEN: - VOICE ANALYSIS
------------------------------------------------------------------------------------------

1. Patient scales
A. Are less important than the results of the objective evaluation
B. Can measure satisfaction, quality of life, handicap, or a particular
aspect of voice production
C. Add nothing to a good case history
D. Are not reliable
E. All of the above

2. Which feature is not important to voice production?


A. Hypernasality
B. Clenched jaw
C. Neck extension
D. Decreased thyrohyoid space
E. None of the above

3. Which statement is true?


A. Frequency is the same as pitch.
B. Loudness is the perceptual correlate of frequency.
C. Frequency and intensity are independent.
D. Frequency range can be stated in semitones.
E. A and C.

4. Which statement is true?


A. Narrow-band spectrograms can highlight a loss of harmonic structure
in the voice signal.
B. Jitter and shimmer are the best measures of hoarseness.
C. Cepstral peak prominence (CPP) is an excellent measure of breathiness
that is based on frequency analysis.
D. Mean nasalance above 50% for sustained /m/ generally corresponds to
hyponasality.
E. All of the above

276
BY DR. MOHAMMED ATIAA KAREEM ALNASHY
-------------------------------------------------------------------
5. Which of the following statements is false?
A. Maximum phonation time (MPT) can be influenced by respiratory
function, laryngeal valving, velopharyngeal closure, practice,
frequency, intensity, vowel, and instructions.
B. Laryngeal airway resistance (RLar) is a ratio of translaryngeal air
pressure to translaryngeal airflow.
C. Electroglottographic (EGG) traces show degree of vocal fold closure.
D. High mean airflow is common with glottic incompetence, such as
unilateral vocal fold motion impairment.
E. None of the above

277
BY DR. MOHAMMED ATIAA KAREEM ALNASHY
-------------------------------------------------------------------
CHAPTER 87
Voice Analysis

1B
Patient scales are extremely variable and can measure many different things.
Some scales are well constructed and demonstrate both reliability and
validity. Others do not. Using a scale such as the VHI brings consistency and
structure to questions about how the disorder affects physical, functional, and
emotional well-being. They are a valuable addition to a thorough voice
evaluation.

2. E
Hypernasality is often an indication of a structural or neurologic process
affecting voice and speech. A clenched jaw, neck extension, and decreased
thyrohyoid space reflect increased musculoskeletal tension, which often
adversely affects voice.

3D
The semitone scale is often used to state frequency range, because it equalizes
the differences between two frequencies. To illustrate, a 100-Hz difference
between two tones is perceived as a greater difference at low frequencies than
at high frequencies. There are 12 semitones between 98 Hz and 196 Hz,
whereas there are only 2 semitones between 880 Hz and 988 Hz. Frequency is
only one dimension of pitch, so answer "a" is incorrect. Loudness is the
perceptual correlate of intensity, and frequency and intensity are interrelated.

4. A
Narrow-band spectrograms show the fundamental frequency and harmonic
structure. Jitter and shimmer are influenced by many
factors and not necessarily reliable or valid, especially for acoustic signals
that lack a single fundamental frequency. GPP seems to correlate with
breathiness but is not based in frequency analysis. Mean nasalance below 50%
for sustained In! generally corresponds to hyponasality, not above 50%.

5. C
EGG traces show degree of vocal fold contact but in a relative manner. EGG
does not show actual degree of closure. The other answers are true.

278
BY DR. MOHAMMED ATIAA KAREEM ALNASHY
-------------------------------------------------------------------
CHAPTER EIGHTY-EIGHT: - DIAGNOSTIC IMAGING OF THE LARYNX
--------------------------------------------------------------------------------------------

1. Imaging signs of retropharyngeal abscess include


A. Thickening of the retropharyngeal soft tissues on lateral soft-tissue
radiography of the neck
B. Hypodense fluid collection in the retropharyngeal space on computed
tomography (CT)
C. Hyperintense fluid collection in the retropharyngeal space on T2-
weighted magnetic resonance imaging (MRI)
D. Ring-enhanced pattern in the retropharyngeal space on CT or MRI
E. All of the above

2. Regarding the imaging evaluation of vocal cord paralysis, which of the


following statements is true?
A. Most causes can be detected with MRI of the brain.
B. Imaging evidence of perineural spread of the vagus nerve caused by
remote carcinoma metastasis is commonly seen.
C. Imaging from the skull base to the pulmonary hila should be
performed.
D. Imaging signs include widely abducted vocal cords and ipsilateral
collapsed pyriform sinus.
E. CT tends to be superior for posterior fossa evaluation compared with
MRI.

3. Features of MRI include all of the following except


A. Acquisition of images is generally slower than CT.
B. Cortical bone and calcifications do not produce a signal.
C. MRI is contraindicated in patients with cardiac pacemakers, metallic
cochlear implants, and cerebral aneurysm clips.
D. MRI is limited to the transverse plane.
E. Motion artifacts may severely degrade images.

279
BY DR. MOHAMMED ATIAA KAREEM ALNASHY
-------------------------------------------------------------------
4. Which of the following statements regarding imaging of glottic
carcinoma is true?
A. CT reliably distinguishes between benign cord paralysis and direct
involvement with tumor.
B. MRI may demonstrate tumor infiltration within the paraglottic and
preepiglottic spaces.
C. Phases of respiration have little impact on cord appearance.
D. Soft tissue thickening of the anterior commissures up to 5 mm may be
considered normal.
E. Soft tissue plain film radiography is the best means for detecting
cartilage invasion.

5. Regarding imaging of the postoperative neck, which of the following


statements is true?
A. MRI is the preferred modality for imaging the posttherapy neck.
B. Expanding nodular soft tissue masses within an irradiated field
usually herald tumor recurrence.
C. Scar within an operated field tends to remain stable over time.
D. Hemorrhage or edema within a surgical bed may persist for 4 to 6 weeks.
E. All of the above

280
BY DR. MOHAMMED ATIAA KAREEM ALNASHY
-------------------------------------------------------------------
CHAPTER 88
Diagnostic Imaging of the Larynx

1. E
Retropharyngeal abscess consists of a masslike collection of purulent fluid in
the retropharyngeal space. Imaging findings reflect this pathophysiology.
Plain films, which poorly define soft tissues, nevertheless reveal the masslike
properties by demonstrating displacement of the airway, thickening of the
soft tissues, and occasionally soft tissue emphysema. CT and MR imaging
better display the actual fluid within the retropharyngeal space. Fluid tends
to be hypodense compared with soft tissue on CT and hyperintense on T2-
weighted MR images. Administration of contrast on either CT or MRI often
reveals a ring pattern bordering the fluid. Because of its availability, speed
of imaging, and excellent anatomic display, CT has become the preferred
modality for confirming retropharyngeal abscess.

2. C
Evaluation of vocal cord paralysis should include the entire course of the
vagus nerve from the skull base to the pulmonary hila. Most causes of
paralysis are peripheral, and, therefore, brain imaging alone is inadequate
for thorough evaluation. CT tends to be excellent for neck and chest
evaluation, and MRI is superb for skull base evaluation. Perineural
infiltration of the nerves by distant disease is very rare. Signs of paralysis
include paramedian position of the cords, displaced arytenoid cartilage,
ipsilateral dilation of the pyriform sinus, tilting of the thyroid cartilage, and
prominent laryngeal ventricle.

3D
MRI offers significant advantages for evaluation of complex disease of the
neck. Its superior soft tissue differentiation provides excellent information
about primary lesion location and extent of spread. Submucosal disease is
especially well studied. One of the major advantages of MRI is the capability
of multiplanar display. This feature permits evaluation of anatomy and
lesion in three dimensions. The remaining choices represent some of the other
limitations of MRI.

281
BY DR. MOHAMMED ATIAA KAREEM ALNASHY
-------------------------------------------------------------------
4B
Although CT can detect the difference in densities of soft tissues such as
tumor and fat, it is much more limited in differentiating similar tissues such
as tumor and muscle. Therefore, infiltration of the thyroarytenoid muscle
with tumor may appear similar to a flaccid paralyzed thyroarytenoid muscle.
MRI better demonstrates tumor infiltration into fatty spaces, such as the
paraglottic and preepiglottic spaces, because of its superior soft tissue
differentiation. Phase of respiration can dramatically alter cord
configuration and lead to misdiagnosis of tumor extent. The anterior
commissure should be no greater than 1 mm thick. Values >1 mm imply tumor
infiltration. Both CT and MRI are preferred for detecting cartilage invasion.
MRI may have advantages over CT according to some investigators. Plain
film radiography has no role in cartilage invasion.

5. E
Imaging of the posttherapy neck remains challenging, although MRI has
emerged as the most reliable readily available cross-sectional modality. In
general, scar tends to remain stable or even contract with time, whereas
recurrent tumor presents as expanding nodular scars within the posttherapy
field. Despite these guidelines, hemorrhage and edema may persist for 4 to 6
weeks and confound interpretation. Therefore, a baseline is best postponed
for 6 to 8 weeks after therapy.

282
BY DR. MOHAMMED ATIAA KAREEM ALNASHY
-------------------------------------------------------------------
CHAPTER EIGHTY-NINE: - NEUROLOGIC EVALUATION OF THE
LARYNX AND THE PHARYNX
--------------------------------------------------------------------------------------------
1. Isolated superior laryngeal injury results in
A. Rotation of the glottis to the side of the injury
B. Rotation of the glottis to the side opposite the injury
C. Prolapse of the arytenoid
D. Rowing of the vocal fold
E. No appreciable change to the glottis

2. The most common cause of unilateral vocal fold paresis is


A. Thyroid surgery
B. Thoracic surgery
C. Stroke
D. Idiopathic
E. Arnold-Chiari malformation

3. Laryngeal and speech abnormalities found in patients with Parkinson's


disease include all the following except
A. Dysarthria secondary to hypokinetic lingual function
B. Dysphagia secondary to decreased laryngeal sensation
C. Decreased loudness secondary to bowing of the vocal folds
D. Decreased loudness secondary to hypokinetic bellows function
E. Dysphonia secondary to vocal tremor

4. Which of the following statements regarding laryngeal electromyography


(EMG) is false?
A. It is essential before reinnervation procedures.
B. Both superior and recurrent laryngeal nerve function can be tested.
C. Demonstration of polyphasic action potentials means that function will be
restored.
D. Fibrillation potentials are noted about 2 weeks after nerve transaction.
E. It is useful in localizing muscles for botulinum injection.

5. Edrophonium is used to test for


A. Multiple sclerosis
B. Amyotrophic lateral sclerosis (ALS)
C. Syringomyelia
D. Shy-Drager syndrome
E. Myasthenia gravis

283
BY DR. MOHAMMED ATIAA KAREEM ALNASHY
-------------------------------------------------------------------
CHAPTER 89
Neurologic Evaluation of the Larynx and the Pharynx

1. E

2D

3B

4. C

5. E

284
BY DR. MOHAMMED ATIAA KAREEM ALNASHY
-------------------------------------------------------------------
CHAPTER NINETY: LARYNGEAL AND TRACHEAL MANIFESTATIONS
OF SYSTEMIC DISEASE

1. Which of the following has not been shown to be of significant benefit in


the treatment of croup?
A. Humidified air
B. Racemic epinephrine
C. Nebulized budesonide
D. Oral dexamethasone
E. Intravenous dexamethasone

2. Adult patients with epiglottitis are more likely to require intubation if they
are initially seen with
A. Involvement of other supraglottic structures
B. Symptoms for more than 5 days
C. Tachycardia
D. Positive soft-tissue neck radiograph
E. Blood cultures positive for Streptococcus

3. Which of the following statements are true regarding pertussis?


A. The incidence is rising in infants but not adults.
B. The cause is known to be failure of the vaccine in some children.
C.Once a patient has contracted pertussis, they have natural immunity for life.
D. The classic "whooping" cough associated with pertussis is more common in
infants than children.
E. None of the above

4. Which of the following is commonly seen in patients with laryngeal


tuberculosis?
A. Chest radiograph positive for tuberculosis
B. History of tobacco and alcohol use
C. Bloody sputum
D. Negative purified protein derivative skin test
E. History of HIV

5. Pseudoepitheliomatous hyperplasia can be seen in all of the following except


A. Blastomycosis
B. Histoplasmosis
C. Cryptococcosis
D. Actinomycosis
E. All of the above

285
BY DR. MOHAMMED ATIAA KAREEM ALNASHY
-------------------------------------------------------------------
CHAPTER 90
Laryngeal and Tracheal Manifestations of Systemic Disease

1. A
Clinical studies have demonstrated the efficacy of all of the above treatments
except humidified air.

2. C
The factors that have been shown to correlate with likelihood of intubation
are those that present with stridor, tachycardia, rapid progression of
symptoms, or blood cultures positive for H. influenza.

3. E
Incidence is rising in infants, teenagers, and adults. Multiple causes for the
increase have been hypothesized, but none proven. Acquiring pertussis by
contact from another infected individual protects one from future infection
for at least 3 years, but then this protection starts to wane. Infants (or adults)
often do not exhibit the classic "whooping" cough, although this sign is
commonly seen in children.

4B
Chest radiographs are positive only 60% to 80% of the time, and other
manifestations of pulmonary disease (such as bloody sputum) are even less
common. HIV has not been shown to be a risk factor for laryngeal
tuberculosis, but there is a strong association between smoking and alcohol
use and the presence of laryngeal TB. PPD test is usually positive in patients
with laryngeal tuberculosis.

5. D
Pseudoepitheliomatous hyperplasia can be seen in all of the above except
actinomycosis.

286
BY DR. MOHAMMED ATIAA KAREEM ALNASHY
-------------------------------------------------------------------
CHAPTER NINETY-ONE: CHRONIC ASPIRATION

1. Which of the following statements regarding laryngotracheal


separation is true?
A. It is irreversible.
B. It may be performed at the bedside.
C. The stasis of secretions in the laryngeal pouch results in chronic,
symptomatic irritation.
D. It is not recommended for children.
E. Patients who have undergone previous tracheotomy are not candidates
for this procedure.

2. Which of the following statements regarding tracheotomy and


aspiration is true?
A. A properly placed tracheotomy tube with an inflated cuff effectively
prevents aspiration.
B. Tracheotomy has been definitively shown to cause aspiration.
C. Tracheotomy should be the first surgical procedure for all patients with
chronic aspiration.
D. A causal relationship between tracheotomy and aspiration has yet to be
demonstrated.
E. Tracheotomy has no role in patients with chronic aspiration.

3. Which of the following statements regarding videofluoroscopic


swallowing studies and functional endoscopic evaluation of swallowing
(FEES) is true?
A. Videofluoroscopic examination is preferred because it is more sensitive.
B. Videofluoroscopic examination is preferred because it is an easier
bedside test.
C. FEES is preferred because it is more sensitive.
D. FEES or videofluoroscopic examination have similar sensitivity and
specificity and may be used according to hospital availability and
preference.
E. FEES should only be performed after a videofluoroscopic swallowing
examination.

287
BY DR. MOHAMMED ATIAA KAREEM ALNASHY
-------------------------------------------------------------------
4. Which of the following statements regarding aspiration is false?
A. Normal, healthy patients never aspirate.
B. Chronic aspiration may have severe, long-term pulmonary consequences.
C.Most patients with chronic aspiration have severe underlying medical
conditions.
D. Cerebrovascular accidents are the most common underlying medical
condition in adults with chronic aspiration.
E. The volume and character of the aspirated material has a marked
impact on the clinical impact of aspiration.

5. Nonsurgical management of chronic aspiration includes all of the


following except
A. Discontinuation of all oral intake
B. Initiation of parenteral antibiotics to cover impending pulmonary
infections
C. Institution of an alternative route of alimentation
D. Swallowing therapy including chin-tuck and multiple swallow
techniques

288
BY DR. MOHAMMED ATIAA KAREEM ALNASHY
-------------------------------------------------------------------
CHAPTER 91
Chronic Aspiration

1B
Laryngotracheal separation may be performed at the bedside, making this
procedure possible for unstable patients who are unsafe for transportation.
Successful reversals of this procedure have been demonstrated, and several series
have reported efficacy in children. Laryngotracheal separation is frequently
performed in patients with prior tracheotomy, because the diversion procedure
may be technically difficult in such patients. The chronic pooling of secretions
in the laryngeal pouch has not been found to be clinically significant.

2D
A causal role for tracheotomy in the development of aspiration has yet to be
demonstrated. The only prospective trial with preoperative evaluation of
aspiration, although limited by small sample size, failed to demonstrate a causal
relationship with tracheotomy. Tracheotomy does not prevent aspiration but
does decrease dead space and improve pulmonary toilet for patients with chronic
aspiration.

3D
The sensitivity and specificity of FEES and videofluoroscopic swallow studies
are similar, thus they should be performed according hospital and provider
preference. For unstable patients who are unable to be transferred to the
radiology suite, a FEES is a better option in that it may be performed at the
bedside.

4. A
Fifty percent of normal, healthy patients have some degree of aspiration while
sleeping. This is typically clinically insignificant.

5B
Patients with chronic aspiration should not be started on empiric antibiotics.
Antibiotic therapy should be initiated if and when their clinical picture suggests
pneumonia. Discontinuation of oral intake combined with establishment of an
alternative route of alimentation and swallowing therapy are standard
nonsurgical treatments for aspiration.

289
BY DR. MOHAMMED ATIAA KAREEM ALNASHY
-------------------------------------------------------------------
CHAPTER NINETY-TWO: - LARYNGEAL AND ESOPHAGEAL TRAUMA

1. Which of the following statements best describes the role of computed


tomography of the larynx after blunt trauma?
A. All patients with suspected laryngeal injuries should undergo computed
tomography to document the extent of their injury.
B. Computed tomography should be used only when the results of the study
will influence the course of treatment.
C. Suspected fractures of the laryngeal skeleton that are difficult to
document by physical examination should be viewed by computed
tomography.
D. B and C
E. All of the above

2. Understanding the mechanism of trauma in blunt laryngeal injuries is


important because
A. Physical examination of the patient is often unreliable.
B. Patients are frequently unable to give an adequate history.
C. The degree of wounding or energy imparted to the anterior neck may
provide useful information about the severity of injury.
D. Early examination of the larynx may be misleading because the injury
may still be evolving.
E. A and B
F. C and D

3. Endotracheal intubation of patients with laryngeal injuries is


A. Problematic and should be avoided
B. Beneficial in all patients and avoids the need for tracheotomy
C.Permissible and frequently safely performed in the emergency room
setting
D. Permissible when the airway appears intact and skilled personnel are
available.
E. Cricothyrotomy or tracheotomy are the only options

290
BY DR. MOHAMMED ATIAA KAREEM ALNASHY
-------------------------------------------------------------------
4. Which of the following statements best describes the use and
characteristics of laryngeal stenting?
A. All patients undergoing open reduction and internal fixation of laryngeal
fractures should undergo stenting.
B. Laryngeal stents are problematic and can be replaced by stabilization of the
laryngeal skeleton with such fixation devices as mini-plates or microplates.
C. Laryngeal stents are necessary to stabilize complex laryngeal skeletal
fractures and should be used for a minimum of 3 months to permit
significant wound healing.
D. Laryngeal stents are necessary to stabilize complex laryngeal skeletal
fractures and therefore should be fixated with wire to ensure their prolonged
positioning within the larynx.
E. Laryngeal stents are selectively indicated, and prolonged stenting may give
rise to further injury of the larynx.

5. The timing of surgical treatment for laryngeal fractures or significant


mucosal injuries should be
A. Delayed for 3 or more days while the patient is placed on high-dose
corticosteroids to minimize edema and to permit more careful
restoration of the soft and hard tissue of the larynx
B. Performed as soon as possible to permit more careful restoration of the
soft and hard tissue of the larynx
C. Irrelevant because essentially all patients with significant injuries do
poorly, and repair of these injuries seems to be of minimal value
D. Inconsequential given the lack of sufficient studies evaluating the early
management of such injuries

CHAPTER 92
Laryngeal and Esophageal Trauma

1. D

2. F

3D

4. E

5B

291
BY DR. MOHAMMED ATIAA KAREEM ALNASHY
-------------------------------------------------------------------
CHAPTER NINETY-THREE: - SURGICAL MANAGEMENT OF
UPPER AIRWAY STENOSIS

1. The characteristics of an ideal mesodermal graft include all of the


following except
A. Rapid healing with minimal long-term graft resorption
B. Adequate strength, size, and pliability of graft to contour it to the defect
C. Minimal donor site morbidity
D. Absence of an accompanying epithelial lining
E. A donor site within the same operative field

2. Which sentence about laryngeal stenting is correct?


A. A soft stent is required when the cartilaginous framework is disrupted.
B. A hollow stent is preferable to a solid stent for minimizing aspiration.
C. A finger cot type stent will minimize pressure on mucosal surfaces.
D. A Montgomery T-tube is used to prevent phonation.

3. The principles in laryngeal keels usage include which of the following?


A. The material used should be inert.
B. The length should be sufficient to extend from the cricothyroid
membrane to at least 2 to 3 mm above the anterior commissure.
C. The posterior wing of the keel should lie at the vocal processes and
should not touch the posterior commissure.
D. All of the above are correct.

4. Which of the following statements is correct?


A. The use of topical mitomycin-C is a proven time tested treatment for
the prevention of subsequent restenosis.
B. Segmental resection and primary anastomosis provide optimal results
for complete tracheal stenosis.
C. Tracheal mobilization and laryngeal release techniques are not
required when a gap greater than 6 cm in tracheal continuity occurs.
D. Suprahyoid laryngeal release usually requires transecting only the
insertion of the digastric muscles.

292
BY DR. MOHAMMED ATIAA KAREEM ALNASHY
-------------------------------------------------------------------
5. Requirement for successful repair of laryngotracheal stenosis include
which of the following?
A. Establishment of an intact, reasonably shaped skeletal framework to
provide a scaffold for the airway
B. Establishment of a completely epithelialized lumen of reasonably
normal size and shape.
C. Primary closure of mucosal lacerations after minimal debridement of
nonviable tissue is preferable in acute case.
D. All of the above are correct.

CHAPTER 93
Surgical Management of Upper Airway Stenosis

1D

2. C

3. D

4. B

5.D

293
BY DR. MOHAMMED ATIAA KAREEM ALNASHY
-------------------------------------------------------------------
CHAPTER NINETY-FOUR:- THE PROFESSIONAL VOICE

1. Optimal results after vocal fold surgery may be best achieved by


including
A. Postoperative absolute voice rest
B. Smoking cessation
C. Antireflux therapy
D. Preoperative and postoperative voice therapy
E. Perioperative steroids

2. Extraesophageal reflux in a singer may be manifested as


A. Decreased vocal range
B. Vocal fatigue
C. Frequent throat clearing
D. Cough
E. All of the above

3. Which of the following is considered an absolute indication for the


cancellation of a performance?
A. Upper respiratory infection
B. Vocal process granuloma
C. Vocal fold hemorrhage
D. Vocal fold varix
E. Muscle tension dysphonia

4. Laryngeal examination of a professional voice user with the flexible


fiberoptic laryngoscope has what significant advantage compared with
the rigid telescope of mirror?
A. Superior light intensity
B. Ability to evaluate laryngeal biomechanics
C. Superior magnification
D. Higher quality laryngeal photographs
E. Provides more natural color

294
BY DR. MOHAMMED ATIAA KAREEM ALNASHY
-------------------------------------------------------------------
5. Vocal nodules
A. Often require surgical therapy
B. Always result in dysphonia
C. Are congenital
D. Are synonymous with vocal cord cysts
E. Usually respond to medical and behavioral therapy

6. Which of the following statements regarding laryngovideostroboscopy


is false?
A. It is an objective examination.
B. It allows observation of vocal fold vibration.
C. It assists in differentiating vocal fold cysts from polyps.
D. Evaluation criteria include symmetry and enclosure.
E. Stroboscopy was first reported by Oertel.

CHAPTER 94

The Professional Voice

1. D
2. E
3. C
4. B
5. E
6. A

295
BY DR. MOHAMMED ATIAA KAREEM ALNASHY
-------------------------------------------------------------------
CHAPTER NINETY-FIVE: BENIGN VOCAL FOLD MUCOSAL DISORDERS

1. Which of the following is true regarding vocal fold nodules?


A. They are often seen in men.
B. They are rarely unilateral.
C. They result from trauma to capillaries.
D. They are most often treated surgically.
E. They are rarely associated with vocal abuse.

2. A professional vocalist notes a sudden breathy hoarseness and inability to


maintain high notes during a performance. The best initial management is
likely to be
A. Voice rest for 2 weeks with oral corticosteroids
B. Voice rest for 2 weeks with proton-pump inhibitor
C. Careful continuation of performance schedule with aggressive singing voice
therapy, increased hydration, and proton-pump inhibitor
D. Careful continuation of performance schedule with singing voice therapy,
increased hydration, and steroid injection into vocal cords
E. Microlaryngoscopy with removal of hyaline polyp

3. Use of aspirin may predispose one to have which of the following lesions develop
A. Vocal nodules
B. Sulcus vocalis
C. Intracordal cyst
D. Vocal fold polyp
E. Capillary ectasia

4. Which of the following lesions is most commonly associated with smoking?


A. Vocal fold nodules
B. Vocal fold polyps
C. Vocal fold granuloma
D. Vocal fold cysts
E. Reinke's edema

5. A smooth 7-mm lesion at the vocal process of a nonsmoker is best treated with
A. Voice therapy and increased hydration
B. Oral corticosteroids and voice rest
C.Aggressive surgical excision followed by radiation therapy if incompletely
excised
D. Limited surgical excision, steroid injection and aggressive voice therapy
E. Limited surgical excision and intralesional Cidofovir injection

296
BY DR. MOHAMMED ATIAA KAREEM ALNASHY
-------------------------------------------------------------------
CHAPTER 95

Benign Vocal Fold Mucosal Disorders

1B

2. A

3D

4. E

5D

297
BY DR. MOHAMMED ATIAA KAREEM ALNASHY
-------------------------------------------------------------------
CHAPTER NINETY-SIX: MEDIALIZATION THYROPLASTY

1. Which of the following materials predictably provides the longest


duration of function when used for injection laryngoplasty?
A. Gelfoam
B. Autologous fat
C. Hydroxyapatite
D. Teflon
E. Micronized human collagen

2. Which of the following materials would be best suited for injection


laryngoplasty for unilateral vocal fold paresis after thyroidectomy,
knowing the recurrent laryngeal nerve is intact?
A. Bovine collagen
B. Teflon
C. Micronized human collagen
D. Autologous fat
E. Hydroxyapatite

3. During a medialization thyroplasty procedure, as the inner


perichondrium is elevated the patient coughs violently and complains of
pain in the throat and ear. When the patient is asked to phonate air freely
bubbles from the thyroplasty window. The appropriate next step is
A. Fill the window with tissue glue
B. Pack the window with gauze and leave the wound open to heal
secondarily
C. Terminate the procedure and close the wound with a drain in place
D. Place an implant large enough to fill the hole
E. Perform a reinnervation procedure instead

4. The optimal position of a medialization prosthesis is


A. At the anterior commissure
B. At the mid-cord level
C. At the posterior cord level
D. At the anterolateral aspect of the vocal process
E. At a point posterior to the vocal process

298
BY DR. MOHAMMED ATIAA KAREEM ALNASHY
-------------------------------------------------------------------
5. Six months after thyroidectomy, a patient has persistent left vocal fold
motion impairment and hoarseness. Palate and tongue function are
normal. Laryngeal EMG is performed demonstrating fibrillation
potentials only in the left thyroarytenoid muscle. The most appropriate
recommendation would be
A. Teflon injection
B. Gelfoam injection
C. Wait for spontaneous recovery to occur
D. Medialization thyroplasty
E. MRI of the neck and chest to rule out an occult lesion

CHAPTER 96

Medialization Thyroplasty

1D

2. C

3. C

4D

5D

299
BY DR. MOHAMMED ATIAA KAREEM ALNASHY
-------------------------------------------------------------------
CHAPTER NINETY-SEVEN: ARYTENOID ADDUCTION

1. In the paralyzed vocal cord, the purpose of the arytenoid adduction


suture is to mimic the action of which muscle?
A. Thyroarytenoid
B. Posterior cricoarytenoid
C. Lateral cricoarytenoid
D. Interarytenoid
E. Cricothyroid

2. Tension on the muscular process of the arytenoid results in


A. External rotation of the arytenoid with medialization and downward
displacement of the vocal cord
B. External rotation of the arytenoid with medialization and upward
displacement of the vocal cord
C. External rotation of the arytenoid with lateralization and upward
displacement of the vocal cord
D. Internal rotation of the arytenoid with medialization and downward
displacement of the vocal cord
E. Internal rotation of the arytenoid with medialization and upward
displacement of the vocal cord

3. During an awake Type I thyroplasty with arytenoid adduction and


endoscopic monitoring, the patient's voicing is noted to be harsh, tight,
and choppy. With good endoscopic appearance of the paralyzed
hemiglottis, the most likely cause is
A. Oversizing of the implant, resulting in overcorrection of the anterior
glottic gap
B. Excessive tension on the arytenoid adduction suture, resulting in
overclosure of the posterior glottic gap
C. Paresis of the cricothyroid muscle caused by local anesthetic block of
the superior laryngeal nerve
D. Edema of the paraglottic space after implant placement
E. Persistent hyperfunction of the contralateral vocal fold

300
BY DR. MOHAMMED ATIAA KAREEM ALNASHY
-------------------------------------------------------------------
4. Which of the following is the least likely complication to be associated
with arytenoid adduction?
A. Dysphagia
B. Airway obstruction
C. Worsening of vocal quality
D. Salivary fistula
E. Carotid artery injury

5. Relative to the cadaveric position of the denervated vocal cord, the


reinnervated vocal cord is likely to lie
A. Lateral and superior to the cadaveric cord
B. Lateral and inferior to the cadaveric cord
C. Medial and superior to the cadaveric cord
D. Medial and inferior to the cadaveric cord
E. In the same position as the cadaveric cord

CHAPTER 97

Arytenoid Adduction

1. C

2D

3. E

4. A

5. C

301
BY DR. MOHAMMED ATIAA KAREEM ALNASHY
-------------------------------------------------------------------
CHAPTER NINETY-EIGHT: LARYNGEAL REINNERVATION

1. Performing an anastomosis of a divided recurrent laryngeal nerve at


the level of the main trunk is most likely to produce
A. Vocal fold motion symmetric with the normally innervated side
B. Normal abduction of the reinnervated vocal fold but little or no adduction
C. Normal adduction of the reinnervated vocal fold but little or no abduction
D. Minimal purposeful movement of the reinnervated vocal fold

2. The neuromuscular pedicle technique and the ansa cervicalis to


recurrent laryngeal nerve anastomosis are similar in that they both
A. Rely on the transfer of intact neuromuscular units
B. Reinnervate multiple muscles with a single pedicle or anastomosis
C. Use a branch of the ansa cervicalis as a donor nerve
D. Require a midline thyrotomy approach to the muscle to be reinnervated

3. Which of the following muscles has the fastest contraction time?


A. The posterior cricoarytenoid
B. The thyroarytenoid
C. The soleus
D. The thyrohyoid

4. The ansa cervicalis-to-recurrent laryngeal nerve anastomosis is


indicated for
A. Unilateral vocal fold paralysis
B. Bilateral vocal fold paralysis
C. Glottic stenosis
D. All of the above

5. Which of the following is not an advantage of the ansa cervicalis-to-


recurrent laryngeal nerve (RLN)?
A. No permanent implant material is used.
B. Ansa-RLN anastomosis does not interfere with the spontaneous
recovery of vocal fold motion.
C. Ansa-RLN anastomosis does not interfere with the subsequent use of
thyroplasty or vocal fold injection.
D. The sacrifice of the sternothyroid muscle is clinically insignificant.

302
BY DR. MOHAMMED ATIAA KAREEM ALNASHY
-------------------------------------------------------------------
CHAPTER 98
Laryngeal Reinnervation

1D
Most of the time after an RLN anastomosis, vocal fold motion does not return.
Instead, a laryngeal synkinesis occurs, with adductor and abductor nerve
fibers nonselectively innervating the laryngeal muscles. The result of this
neuromuscular mismatching is that counteracting forces are applied to the
arytenoid by all the muscles innervated by the RLN and little or no
functional movement occurs.

2. C
The neuromuscular pedicle (NMP) technique and the ansa cervicalis to
recurrent laryngeal nerve (ansa-RLN) anastomosis both use a branch of the
ansa cervicalis as a donor nerve. The NMP typically uses the branch to the
superior belly of the omohyoid muscle, whereas the ansa-RLN anastomosis
typically uses the branch to the sternothyroid or the sternohyoid muscle.

3B
Of the muscles listed, the thyroarytenoid muscle is the fastest. In the body as
a whole, the extraocular muscles are faster. The soleus muscle is one of the
slowest. The speed of contraction is related to the fiber type of the muscle,
which is related to its myosin heavy chain composition.

4. A
The ansa cervicalis-to-recurrent laryngeal nerve anastomosis is indicated for
unilateral vocal fold paralysis. Bilateral vocal fold paralysis and glottic
stenosis are contraindications, because vocal fold movement is not expected as
a result of the procedure.

5B
The recurrent laryngeal nerve is divided, and the distal portion of the RLN
is attached to the proximal ansa cervicalis nerve during an ansa-RLN
anastomosis. This effectively eliminates the possibility of spontaneous
recovery of the RLN nerve. It is important to wait until the likelihood of
spontaneous recovery of RLN is minimal before performing an ansa-RLN
anastomosis.

303
BY DR. MOHAMMED ATIAA KAREEM ALNASHY
-------------------------------------------------------------------
CHAPTER NINETY-NINE: MALIGNANT TUMORS OF THE LARYNX
AND HYPOPHARYNX

1. Laryngeal organ preservation is based on the premise that the addition


of chemotherapy will
A. Reduce the incidence of distant metastatic disease
B. Provide an increased 5-year survival
C. Improve the quality of life
D. Select individuals that will have a favorable response to radiation therapy
E. Extend the interval until a laryngectomy is required

2. A residual mass in the neck in a patient treated with a larynx organ


preservation protocol who initially had a T2N+ carcinoma of the larynx
should
A. Be observed for 3 months or longer for resolution
B. Undergo a laryngectomy with neck dissection regardless of response at
the primary site
C. Should undergo a neck dissection as soon as feasible
D. Should undergo a neck dissection only if the initial tumor was an N3
E. Does not need a neck dissection if the initial lesion was staged Nx

3. Overall 5-year survival rates for hypopharyngeal carcinoma rates are


poor because
A. Of the high incidence of comorbid conditions
B. Of the high incidence of distant metastases
C. Of the high incidence of second primary malignancies
D. More than 60% of patients are initially seen with positive cervical adenopathy
E. All of the above

304
BY DR. MOHAMMED ATIAA KAREEM ALNASHY
-------------------------------------------------------------------
4. Which one of the following principles apply to the management of the
neck in patients with supraglottic squamous cell carcinoma?
A. Level I should be dissected in patients with positive adenopathy in level II
or III.
B. Radiation therapy is as effective as surgery for control of the contralateral
neck.
C. Recurrence in the previously modified neck dissection and postoperative
irradiated neck can be controlled by a radical neck dissection.
D. In an N1+ neck, a radical neck dissection is required.
E. The submandibular gland should never be resected in a modified neck
dissection.

5. Which of the following findings is the most important in the decision


as to which treatment should be selected for an early vocal cord cancer?
A. Tumor differentiation
B. Limited subglottic extension
C. Involvement of the anterior commissure
D. Arytenoid cartilage involvement
E. Vocal cord mobility

305
BY DR. MOHAMMED ATIAA KAREEM ALNASHY
-------------------------------------------------------------------
CHAPTER 99
Malignant Tumors of the Larynx and Hypopharynx

1. D
Ensley and others, demonstrated that a positive response to chemotherapy, especially a
pathologic GR, had a favorable prognosis. This concept was challenged by Poulsen, but
even that study showed that in a select group of patients, the need for laryngectomy would
be reduced. The NIH Intergroup study showed that there was a positive effect from
chemotherapy/radiation therapy when given concomitantly in extending the length of
time until laryngectomy.

2. C
Wolf and others provide long-term follow-up in the original VA laryngeal preservation
study. Persistent neck disease was best treated by neck dissection as the result of persistent
or recurrent disease in the neck, even when response at the primary tumor was poor. The
primary laryngeal tumor and neck disease should be evaluated separately. There is still
controversy on the management of N2 neck disease as to the necessity of a routine neck
dissection after chemotherapy/radiation therapy if there is a clinical GR. All authors
agree, though, that persistent disease needs to be addressed surgically, especially if not
resolved by 3 months.

3. E
Most patients with hypopharyngeal cancer are initially seen with T3N+ disease, usually
with the primary malignancy in the pyriform sinus. These patients have a high incidence
of comorbid disease (Carpenter's study) and a high incidence of second primary tumors
(Raghavan study). The incidence of distant disease is higher in patients with positive
adenopathy. Therefore, the 5-year overall survival remains poor even with more
aggressive treatment or multimodality treatment.

4. A
Multiple series have reported that a modified neck dissection is all that is necessary for
an N0 or Nx positive neck. However, recent studies confirm that a bilateral modified neck
dissection is required. In some series, the incidence of recurrence is highest in the
contralateral neck, even if it has been treated with postoperative radiation therapy. The
submandibular gland is included when level I is suspected to be positive, and in many
cases because chronic sialadenitis may be confused with recurrent disease.

5. E
Decreased vocal cord mobility implies invasion into the thyroarytenoid muscle or
involvement of the cricothyroid joint. This can lead to paraglottic space involvement,
which limits the capability of a limited resection and likelihood of cure by radiation
therapy.

306
BY DR. MOHAMMED ATIAA KAREEM ALNASHY
-------------------------------------------------------------------
CHAPTER ONE HUNDRED: MANAGEMENT OF EARLY GLOTTIC
CANCER

1. Which of the following risk factors are associated with laryngeal cancer?
A. Gastroesophageal reflux
B. Human papillomavirus
C. Alcohol use
D. Second-hand tobacco smoke
E. All of the above

2. According to the sixth edition of the American Joint Committee on


Cancer (AJCC) staging manual, a transglottic tumor involving the right
true vocal fold with impaired vocal fold motion and invasion limited
to the paraglottic space is classified as
A. T2a
B. T2b
C. T3
D. T4
E. None of the above

3. Excision of the vocal cord and a segment of underlying thyroid


cartilage with or without the ipsilateral arytenoid is classified as a
A. Vertical partial laryngectomy
B. Laryngofissure with cordectomy
C. Vestibulectomy
D. Supracricoid laryngectomy
E. None of the above

4. Which of the following statements is true regarding reconstruction of


the neocord after partial laryngectomy?
A. Supraglottic structures do not contribute to postoperative phonation.
B. Forearm fascial flap with buccal mucosal graft is ideal in the setting
of arytenoid fixation.
C. Injection laryngoplasty is hindered by paraglottic space scarring.
D. Closure of the posterior glottic gap by partial cricoid resection poses no
risk of diminishing the airway.
E. None of the above

307
BY DR. MOHAMMED ATIAA KAREEM ALNASHY
-------------------------------------------------------------------
5. Which of the following is true regarding external beam radiation therapy?
A. Compared with lower dose regimens, accelerated fractionation of
external beam radiation therapy may offer improved control.
B. Compared with lower dose regimens, accelerated fractionation of
external beam radiation therapy causes less dysphagia and mucositis in
the first 2 months of therapy.
C. Duration of treatment does not predict survival.
D. Radiation failures usually manifest as distant metastases.
E. Radiation therapy precludes conservation laryngeal surgery.

CHAPTER 100
Management of Early Glottic Cancer

1. E

2. C

3. A

4. C

5A

308
BY DR. MOHAMMED ATIAA KAREEM ALNASHY
-------------------------------------------------------------------
CHAPTER ONE HUNDRED AND ONE: TRANSORAL LASER MICRO
RESECTION OF ADVANCED LARYNGEAL TUMORS
--------------------------------------------------------------------------------------------

1. Which one of the following statements about transoral laser


microresection (TLM) is true?
A. TLM is a surgical treatment strategy for primary cancers of the larynx
but not the pharynx or mouth.
B. In aggregate, the resected tissue volume parallels that of an open
operation.
C. Radiotherapy is offered after TLM to finesse margins but not to treat
the N0 neck.
D. TLM has never been successfully used for the treatment of a T3
laryngeal cancer.
E. A negative second look guarantees no local recurrence will happen.

2. All of the following are possible reasons a laser tumor transection


might be considered safe except
A. Cancer cells do not adhere to a beam of light so the cutting instrument
does not provide a physical carrier to transplant tumor.
B. Cancer cells revealed by laser energy should be thermocoagulated, hence
not viable.
C. Cancer cells falling unseen into a laser wound light on a thin layer of
coagulum that, left open, is superficially sloughed.
D. Suctions and forceps could never transport viable cancer cells exposed
by tearing of the specimen
E. C02 laser incisions seal lymphatic vessels in the wound margin
immediately, and lymphatic vessels remain sealed for approximately 10
days.

3. All of the following improves access to the anterior commissure except


A. A laryngoscope with a wide body to spread out the tongue
B. Tape across the cricoid to tip the larynx
C. A laryngoscope with less bevel at the tip
D. A laryngoscope with a narrow vertically oval body to sink into the
tongue

309
BY DR. MOHAMMED ATIAA KAREEM ALNASHY
-------------------------------------------------------------------
4. When TLM is performed in the patient with a high-risk N0 neck,
which of the following is not a logical reasons to perform the neck
dissection at a separate time?
A. Micrometastases "in transit" at the time of the TLM will have time to
lodge in the nodes.
B. A patient with serious comorbidities may have recovered from the
primary resection.
C. An elderly patient may have regained swallowing after a laser
supraglottic laryngectomy.
D. The neck is already violated to access the primary tumor by laser
endoscopic surgery.
E. Staging the primary tumor and the neck surgery at separate sittings
reduces the chance of a pharyngocutaneous fistula to zero.

5. Contraindications to TLM will include all of the following except


A. Extensive tumor spread to the great vessels, the esophagus, or the
thyroid gland
B. Inability to expose the larynx or tumor
C. Recurrent cancer in an irradiated "bed"
D. Advanced cancer needing reconstruction
E. Patients with functional disorders after extensive partial resections
(like severe persistent aspiration or secondary stenosis)

CHAPTER 101
Transoral Laser Micro Resection of Advanced Laryngeal Tumors

1B

2D

3. A

4D

5. C

310
BY DR. MOHAMMED ATIAA KAREEM ALNASHY
-------------------------------------------------------------------
CHAPTER ONE HUNDRED AND TWO: CONSERVATION
LARYNGEAL SURGERY

1. The cricoarytenoid unit consists of


A. The recurrent laryngeal nerve, the associated musculature, the cricoid
and arytenoid
B. The superior laryngeal nerve, the associated musculature, the cricoid
and arytenoid
C. The recurrent laryngeal and superior laryngeal nerve, the cricoid and
arytenoid
D. The recurrent laryngeal and superior laryngeal nerve, the associated
musculature, the cricoid and arytenoid
E. The recurrent laryngeal and superior laryngeal nerve, the vocal folds,
the associated musculature, the cricoid and arytenoid

2. Which of the following is not a principle of organ preservation surgery?


A. Local control is of paramount importance
B. Resection of normal tissue is necessary to allow for functional reconstruction.
C. Cricoarytenoid unit is the essential functional unit of the larynx.
D. Accurately assessing the three dimensional extent of the tumor
E. The T-stage indicates the conservation laryngeal procedure to be performed.

3. Which of the following statements regarding supracricoid


laryngectomy is false?
A. The resection includes the entire thyroid cartilage, false cords, true
cords with or without the epiglottis sparing at least one arytenoids.
B. It is contraindicated in tumors that extend to the preepiglottic space.
C. The reconstruction is performed by pexy of the hyoid to cricoid with or
without the epiglottis.
D. The superior laryngeal nerves are preserved whenever possible.
E. Interarytenoid involvement with tumor is a contraindication.

311
BY DR. MOHAMMED ATIAA KAREEM ALNASHY
-------------------------------------------------------------------
4. Which of the following statements regarding supraglottic
laryngectomy is true?
A. Involvement of the preepiglottic space necessitates hyoid bone resection.
B. It is contraindicated in tumors that extend to the vallecula.
C. It is contraindicated in tumors that extend into the ventricle.
D. Vocal fold fixation is not a contraindication.
E. Interarytenoid involvement with tumor is not a contraindication.

5. Which of the following lesions is not suitable for supracricoid


laryngectomy?
A. T3 glottic lesion with arytenoid mobility
B. Anterior commissure lesion with involvement of the thyroid cartilage
not extending through it
C. T2 glottic lesion with 2 cm of subglottic extent
D. A transglottic lesion with a fixed vocal cord and epiglottic involvement
E. T2 glottic lesion with impaired mobility

312
BY DR. MOHAMMED ATIAA KAREEM ALNASHY
-------------------------------------------------------------------
CHAPTER 102
Conservation Laryngeal Surgery

1D
The cricoarytenoid unit is the essential functional unit of the larynx in the
organ preservation paradigm for laryngeal cancer. It includes the recurrent
and superior laryngeal nerves, the associated musculature, along with the
arytenoid and cricoid cartilage.

2. E
The T-staging system for glottic and supraglottic cancer does not indicate the
conservation laryngeal procedure to be performed. The T-staging system does
not include the precise, detailed anatomic information necessary to be able to
perform these surgeries. For example, a T3 glottic lesion will be amenable to
supracricoid laryngectomy, whereas a T2 lesion with interarytenoid
involvement is not a candidate for this procedure.

3. B
The supracricoid laryngectomy can be performed to include the entire
preepiglottic space along with the epiglottis for transglottic tumors affecting
this area. The reconstruction would require a cricohyoidopexy.

4. C
The supraglottic laryngectomy requires a cut that is performed at the level of
the ventricle to preserve the true vocal folds. If tumor involves the ventricle,
this cut cannot be made safely.

5. C
The supracricoid laryngectomy requires at least one functional
cricoarytenoid unit. This includes the cricoid cartilage. A tumor that extends
into the subglottis below the level of the cricoid cartilage will not allow for
preservation of the cricoid.

313
BY DR. MOHAMMED ATIAA KAREEM ALNASHY
-------------------------------------------------------------------
CHAPTER ONE HUNDRED AND THREE: TOTAL LARYNGECTOMY
AND LARYNGOPHARYNGECTOMY
--------------------------------------------------------------------------------------------
1. A patient underwent a laryngectomy for glottic carcinoma that
previously failed radiotherapy 6 months earlier. He complains of
dysphagia to both liquids and solids. A barium swallow is obtained and
reveals a structure in the hypopharynx that seems to be an epiglottis.
The most likely reason for this problems is
A. Vertical closure of the pharynx
B. T-shaped closure of the pharynx
C. Lack of flap reconstruction
D. Pharyngeal stricture caused by closure of muscle
E. A remnant of epiglottis that was not resected

2. Which is true of organ preservation strategies for laryngeal and


hypopharyngeal carcinomas?
A. Concurrent chemotherapy with radiotherapy provides superior local
control and organ preservation compared with sequential
chemotherapy with radiotherapy for laryngeal carcinoma.
B. The rate of organ preservation is consistently higher in hypopharyngeal
carcinomas compared with laryngeal carcinomas.
C. Chemotherapy combined with radiotherapy improves survival over
radiotherapy alone or sequential chemotherapy and radiation.
D. Concurrent chemotherapy with radiotherapy provides better local
control than surgery plus radiotherapy.
E. The rate of pharyngocutaneous fistula is higher after salvage surgery
for patients after sequential chemotherapy and radiation compared
with after concurrent radiation.

3. A patient is treated with concurrent chemotherapy and radiotherapy


for T3N0 glottic cancer. The patient has persistent disease, and a
laryngectomy is performed. What is the risk of pharyngocutaneous
fistula postoperatively?
A. 30%
B. <10%
C. 15%
D. 45%
E. 60%

314
BY DR. MOHAMMED ATIAA KAREEM ALNASHY
-------------------------------------------------------------------
4. A patient who is not a suitable candidate for organ preservation surgery
opts for surgical therapy of a supraglottic cancer that involved the
epiglottis, left false vocal cord, arytenoid, and the anterior commissure.
He has bilateral adenopathy. Which of the following is true about
performing thyroidectomy at the time of total laryngectomy?
A. Total thyroidectomy may be indicated because of paratracheal lymph
node involvement.
B. Total thyroidectomy should be performed regardless of the tumor extent
if tumor invasion is suspected.
C. Left thyroid lobectomy should be performed in continuity with the
larynx, because the tumor is transglottic.
D. Preservation of both thyroid lobes will nearly eliminate the risk of
hypothyroidism after treatment.
E. If there is 5 mm of subglottic extension, thyroid invasion is likely, so
thyroidectomy is indicated.

5. After entry into the vallecula for an endolaryngeal tumor, the most
logical next step in total laryngectomy is to
A. Cut along the aryepiglottic fold from vallecula to pyriform sinus
B. Elevate the pyriform sinus mucosa
C. Divide the constrictor muscles at the lateral border of the thyroid
cartilage
D. Transect the trachea
E. Divide the sternohyoid muscle and the thyroid isthmus

CHAPTER 103
Total Laryngectom and Laryngopharyngectomy

1. A

2A

3. A

4. A

5. A

315
BY DR. MOHAMMED ATIAA KAREEM ALNASHY
-------------------------------------------------------------------
CHAPTER ONE HUNDRED AND FOUR: RADIATION THERAPY FOR
THE LARYNX AND HYPOPHARYNX

1. Which of the following regarding lymphatic drainage is true?


A. The supraglottis has a more extensive lymphatic network than the glottis.
B. The subglottic lymphatics drain exclusively to the paratracheal nodes.
C. The supraglottic lymphatics only drain to the jugulodigastric nodes.
D. The false vocal folds are devoid of lymphatics.
E. None of the above

2. Which of the following is a potential contraindication for supraglottic


laryngectomy?
A. Bilateral arytenoid disease
B. Postcricoid disease
C. Thyroid cartilage involvement
D. True vocal fold fixation
E. All of the above

3. Which of the following is true?


A. Neck lymph nodes should be treated with radiation in T1 disease to
prevent occult metastases.
B. Local control for T1 disease with radiation therapy is 68% to 88%.
C. All patients with T2N0 disease should undergo neck radiation treatment.
D. The radiation field should be limited to the nodal region involved with
gross disease.
E. None of the above

4. In the management of advanced laryngeal carcinoma, the Radiation


Therapy Oncology Group protocol 9111 and Head and Neck Intergroup
3-arm phase III trial concluded that
A. Survival at 2 years for radiation therapy alone was superior to concurrent
chemoradiation.
B. The rate of laryngeal preservation was higher with concurrent
chemoradiation than chemotherapy alone.
C. The rate of laryngeal preservation was higher with radiation alone than
chemoradiation.
D. Radiation therapy alone has higher rates of toxicity than chemoradiation.
E. None of the above

316
BY DR. MOHAMMED ATIAA KAREEM ALNASHY
-------------------------------------------------------------------
5. In external beam radiation given before total laryngectomy, the typical
total radiation dose is
A. 10 to 20 Gy
B. 30 to 40 Gy
C. 50 to 60 Gy
D. 70 to 80 Gy
E. 90 to 100 Gy

CHAPTER 104
Radiation Therapy for the Larynx and Hypopharynx

1. A

2. E

3. E

4. B

5. C

317
BY DR. MOHAMMED ATIAA KAREEM ALNASHY
-------------------------------------------------------------------
CHAPTER ONE HUNDRED AND FIVE: VOCAL REHABILITATION
FOLLOWING TOTAL LARYNGECTOMY
--------------------------------------------------------------------------------------------
1. Which of the following is not a primary reason for the failure of early surgical
shunt procedures for the rehabilitation of voice following laryngectomy?
A. Aspiration
B. Pharyngocutaneous fistula formation
C. Stenosis
D. Need for multiple procedures
E. Failure to attain adequate voicing.

2. The most widely practiced method of surgical voice rehabilitation after


laryngectomy is
A. Esophageal speech
B. Tracheoesophageal (TE) fistula speech without prosthesis
C. TE fistula speech with a prosthesis
D. The Asai procedure
E. Electrolaryngeal speech

3. Which of the following is not a common reason for the failure of TE fistula
speech acquisition?
A. Pharyngeal hypertonia
B. Pharyngeal hypotonia
C. Failure of neoglottic mucosa vibration
D. Fungal colonization of the prosthesis
E. Poor stoma and fistula design

4. What is the most appropriate first step in the management of leakage after
primary TE puncture?
A. Removal of the prosthesis with a red rubber catheter placement to allow the
site to narrow
B. Removal of the prosthesis to allow the site to close for revision puncture
C. Initiation of antifungal therapy
D. Replacement of the prosthesis with a larger device
E. Replacement of the prosthesis with a shorter device

5. Management of premature device failure includes


A. Systemic antifungal therapy
B. Topical antifungal therapy
C. Device replacement with another brand
D. Cleaning of the device with chlorhexidine
E. Abandoning TE fistula speech

318
BY DR. MOHAMMED ATIAA KAREEM ALNASHY
-------------------------------------------------------------------
CHAPTER 105

Vocal Rehabilitation Following Laryngectomy

1. E

2. C

3D

4. E

5B

319
BY DR. MOHAMMED ATIAA KAREEM ALNASHY
-------------------------------------------------------------------
CHAPTER ONE HUNDRED AND SIX: MANAGEMENT OF THE
IMPAIRED AIRWAY IN THE ADULT
--------------------------------------------------------------------------------------------
1. In a modified Mallampati class III airway, which of the following
structures are visible?
A. Uvula, faucial pillars
B. Uvula, faucial pillars, soft palate visible
C. Soft palate only
D. Hard palate only

2. The most common complication after tracheotomy is


A. Pneumomediastinum
B. Tracheal stenosis
C. Tube displacement
D. Tube obstruction
E. Hemorrhage

3. Which of the following statements is true?


A. A tracheoesophageal fistula usually occurs when the tracheotomy tube is tipped
anteriorly.
B. Chronic cough with eating and recurrent aspiration are common signs of a
tracheoinnominate fistula.
C. A low-volume, high-pressure tracheotomy tube cuff decreases the risk of tracheal
stenosis.
D. Sentinel bleeding may occur 3 days to 3 weeks before a tracheoinnominate blowout.
E. None of the above

4. Which of the following is true about percutaneous dilatational tracheotomy


(PDT)?
A. PDT should be avoided in obese patients and children.
B. Conventional open tracheotomy requires more time than PDT.
C. Endoscopic guidance during PDT may result in hypercarbia in patients with
head injuries.
D. Cadaver studies show that puncture site in the trachea vary greatly during PDT.
E. All of the above

5. Which of the following is an absolute contraindication for PDT?


A. Limited cervical spine extension
B. Positive end-expiratory pressure of greater than 15 cm H20
C. Need for emergent airway access
D. History of difficult intubation
E. Acute infection over tracheotomy site

320
BY DR. MOHAMMED ATIAA KAREEM ALNASHY
-------------------------------------------------------------------
CHAPTER 106
Management of the Impaired Airway in the Adult

1. C

2. E

3D

4. E

5. C

321
BY DR. MOHAMMED ATIAA KAREEM ALNASHY
-------------------------------------------------------------------
CHAPTER ONE HUNDRED AND SEVEN: ENDOSCOPY OF THE
TRACHEOBRONCHIAL TREE
--------------------------------------------------------------------------------------------

1. What are the limitations of fiberoptic bronchoscopy (FOB)?


A. Inability to visualize beyond the seventh-generation airways
B. Inability to visualize extrabronchial structures such as lymph nodes and or
blood vessels
C. Inability to sample peripheral lung nodules smaller than 2 cm
D. Inability to sample bronchoscopically low paraesophageal lymph nodes
E. Inability to sample the left atrial pressure

2. Which of the following statements regarding preparation and


anesthesia for bronchoscopy is true?
A. There are no required standard preprocedure laboratory studies such as
platelet counts and coagulation parameters, and it is acceptable to perform
biopsies with a platelet count of less than 50,000.
B. As an alternative to general anesthesia in the operating room, intravenous
conscious sedation with midazolam, fentanyl, and Diprivan can be safely
given in an off-site endoscopy suite with a lower level of monitoring.
C. Because there is no respiratory or hemodynamics compromise, lidocaine is
safe to use in nonallergic patients.
D. The choice of bronchoscope size and site of passage is not a major
consideration in a patient with normal trachea and vocal cords.
E. General anesthesia with paralytic agents need only be used in rigid
bronchoscopies.

3. Potential risks and complications that should be discussed with


patients include which of the following?
A. Fever is an unexpected sequelae of a diagnostic bronchoscopy and should
always prompt culture and initiation of antibiotic treatment.
B. Pneumothorax can occur only with the trans-bronchial use of needles,
forceps, and brushes.
C. Airway perforation during tumor or granulation debridement is more likely
with heat therapies such as argon plasma coagulation than with cryotherapy.
D. Airway edema and obstruction is equally likely after laser, electrocautery,
cryotherapy, and photodynamic therapy (PDT).
E. After PDT or endobronchial brachytherapy, there is a 3% to 25% risk of
eventual fatal hemoptysis.

322
BY DR. MOHAMMED ATIAA KAREEM ALNASHY
-------------------------------------------------------------------
4. Which of the following is true about stents that are available for
tracheobronchial airways use?
A. Stents are made of either metal or silastic (silicone and plastic).
B. Silicone stents are removable, but they are also more prone to migration.
C. The only self-expanding stents are metallic, and they can all be placed
without the need for rigid bronchoscopy or direct suspension laryngoscopy.
D. All metal stents are self-expanding and have the benefit of not requiring
balloon expansion.
E. Only the covered silastic stents are usable for covering tracheobronchial-
esophageal fistulas.

5. Which of the following is true regarding innovations in bronchoscopy?


A. Autofluorescence bronchoscopy requires the use of special bronchoscopes
and a photosensitizes
B. Endobronchial ultrasound aids in the localization of all the regional hilar
and mediastinal lymph nodes for transbronchial needle aspiration.
C. Three-dimensional reconstruction of multi-slice detector computed
tomography can generate "virtual bronchoscopy" fly-throughs in the airway
and obviate the need for many diagnostic bronchoscopies.
D. Endoscopic lung volume reduction uses valves and tissue glue to cause
regional atelectasis of hyperinflated lung segments.
E. High-dose local therapy with drug-eluting stents and bronchoscopic direct
intralesional injection of drugs do not require additional FDA approval as
long as they use existing approved FDA medications.

323
BY DR. MOHAMMED ATIAA KAREEM ALNASHY
-------------------------------------------------------------------
CHAPTER 107
Endoscopy of the Tracheobronchial Tree

1D
With slim and ultrathin bronchoscopes measuring less than 3 mm in diameter, a FOB
can be steered into the 12th to 15th generation bronchi. Endobronchial ultrasound can
provide a real-time image of transbronchial structures such as lymph nodes and blood
vessels and to help direct sampling of lymph nodes and other structures immediately
adjacent to the airway. The low paraesophageal lymph nodes are, however, situated away
from the carina and passage of the left mainstem bronchi; therefore, under normal
circumstances, it is better examined and sampled by way of upper GI endoscopic
ultrasonography. In addition to the sampling of regional lymph nodes, trans-bronchial
needle aspiration (TBNA) techniques can also be used in conjunction with
transbronchial biopsies and washes to sample peripheral lung nodules. Nodules <2 cm are
locatable with the help of fluoroscopy, CT scan, and, in the future, by electromagnetically
guided systems. However, the yield is definitely lower for smaller and more peripherally
located lesions. Historically, rigid TBNA needles were used to sample the left-atrial
pressure, although this is no longer performed as a primary indication.

2. A
Although it is advisable not to take undue risks when performing a bronchoscopy, there
is, in fact, no required standard set of preprocedure laboratory studies that will predict
or preclude hemorrhagic complications. Cancer patients undergoing cytotoxic treatment
may become pancytopenic with opportunistic infections requiring diagnosis; in these
cases, transfusion is attempted to maintain normal platelet counts, but when indicated,
lavage and careful biopsies may be performed. Sedation with propofol (Diprivan) is
considered deep sedation, requiring very close monitoring by qualified personnel, because
respiratory arrest can otherwise easily occur. General anesthesia (GA) with the use of
paralytic agents is generally reserved for rigid bronchoscopy, and rarely is rigid
bronchoscopy performed with them; however, GA with paralysis may also be required for
certain interventional procedures performed with a flexible fiberoptic bronchoscope, such
as laser or argon plasma coagulation debridement when the risk of misfire is increased
in a spontaneously breathing and coughing patient. Lidocaine in dosages in excess of 500
mg may be systemically absorbed in sufficient quantities so as to cause seizures and other
complications. The choice of bronchoscope type (rigid vs flexible), size and passage of
entry (oral vs nasal) depends on many parameters, including the stability of the patient's
neck and facial bones, size of the patient and hence his nares, and planned procedures.
For example, with flexible FOB, placement of endobronchial brachytherapy afterloading
catheters will be easier with a nasal route, whereas anticipated retrieval of a foreign body
would be more easily removed orally.

324
BY DR. MOHAMMED ATIAA KAREEM ALNASHY
-------------------------------------------------------------------
3. E
Low-grade fever can commonly occur after a diagnostic bronchoscopy and is most often
self-limited. Informing the patient ahead of time will help to alleviate much anxiety.
Although pneumothoraces occur most often with transbronchial biopsies, needle
aspirations or brushings of the peripheral lung with unintentional trauma to the visceral
pleura, it can occasionally happen in patients with severe GOPD and bullous lung
diseases, who may perform a Valsalva maneuver and cough vigorously during the
procedure. Airway perforation is a risk with interventional procedures, including by the
tip of the rigid bronchoscope, inadvertent passage of other firm instruments, or balloon
bronchoplasty through a false lumen because of the necrotic tumor debris that has
replaced the normal bronchial wall. Although laser, especially when set at a high wattage
setting and fired in a continuous mode, can definitely cause airway perforation, argon
plasma coagulation with a much shallower depth of penetration is much less likely to do
so. Certain tumor ablative therapies such as cryotherapy, PDT, and brachytherapy have
a delayed response such that critical airway narrowing should be managed by other
techniques. Conversely, PDT and brachytherapy can both have a prolonged effect, and
this may account for the 3% to 25% incidence of mostly delayed fatal hemoptysis. Lesions
in the right upper lobe take off, and the distal left mainstem, perhaps by its relationship
to the respectively pulmonary arteries, is most prone to this potential devastating
complication.

4. B
Although the earliest stents for tracheobronchial uses were made of hard polymers (the
Montgomery T-tube), stents are currently made from a range of materials, ranging from
silicone to various types of metal. There are also silicone stents with embedded stainless
steel support struts. Silicone stents are removable; because they are by nature completely
covered, they are, however, also more prone to migration and by their thickness also lead
to impaction of secretions. Although most silastic stents are incompressible and require a
rigid bronchoscope or a suspension laryngoscope for delivery and deployment, there is
now also available a compressible and self-expanding polyester-silicone stent. Most
metallic stents are self-expandable (SEMS), but not the earliest Palmaz and Gianturco
stainless steel stents that require balloon dilation for deployment. These earlier stents are
no longer used in the airways because of their tendency to perforate airways and lack of
covering that render them ineffective for stenting tumor infiltrated airways. Newer
SEMS come in both covered and uncovered versions, and the covered variants may be used
for covering over tracheobronchialesophageal fistulas and other causes of airway
perforation. The perfect airway stent has not been made, one that is easy to deploy,
removable when desired, but will not unintentionally migrate, and does not cause
granulation or promote infection. Future stents may be coated with special coverings that
will make them useful drug delivery devices for the local deposition of antineoplastic,
antifibrotic, antiinfective, or gene therapy.

325
BY DR. MOHAMMED ATIAA KAREEM ALNASHY
-------------------------------------------------------------------
5D
Endoscopic lung volume reduction (ELVR) will attempt to replicate surgical lung
volume reduction surgery (LVRS) by causing selected regional atelectasis of
hyperinflated lung segments. The FDA is currently favoring trials with only removable
valve devices (i.e., a reversible process). Use of existing approved drugs in combination
with approved devices will require further testing for safety and efficacy.
Autofluorescence bronchoscopy (AF) makes use of the properties of tissue
autofluorescence and does not require an exogenous photosensitizer. Advances in airway
imaging include the creation of 3D images and virtual fly-throughs, with the capability
of presenting even a retrograde view up the airway. However, false-positive results from
airway secretions and the present resolution is insufficient for it to replace diagnostic
bronchoscopy, and imaging cannot substitute for tissue sampling. Endobronchial
ultrasound (EBUS) helps to direct TBNA sampling of regional lymph nodes; however, it
can only do so for lymph node stations adjacent to the airway, hence the low
paraesophageal and lateral aortopulmonary lymph nodes are still not accessible by this
technique.

326
BY DR. MOHAMMED ATIAA KAREEM ALNASHY
-------------------------------------------------------------------
CHAPTER ONE HUNDRED AND EIGHT: DIAGNOSIS AND
MANAGEMENT OF TRACHEAL NEOPLASMS

1. Which two of the following tracheal tumors comprise most primary


tracheal neoplasms in adults?
A. Carcinoid and adenoid cystic carcinoma
B. Adenoid cystic carcinoma and squamous cell carcinoma
C. Granular cell tumor and squamous cell carcinoma
D. Papilloma and granular cell tumor
E. Carcinoid and leiomyoma

2. Which of the following techniques is the preferred treatment for


primary malignant tracheal tumors?
A. Radiation and chemotherapy
B. Chemotherapy alone
C. Photodynamic therapy
D. Stent placement
E. Complete surgical resection and consideration of radiation therapy

3. Which of the following treatments is the preferred management of


invasive well-differentiated thyroid carcinoma that invades the trachea?
A. Surgical resection of involved tracheal segment and primary anastomosis
B. Radiation therapy
C. Shave the tumor off the trachea then radiation therapy
D. Chemotherapy and radiation therapy
E. Stent placement

4. Which of the following treatments provide palliation to patients with


extrinsic tracheal compression?
A. Photodynamic therapy
B. Rigid bronchoscopy with core-out techniques
C. Stent placement
D. Cryosurgery
E. Laser therapy

5. Major disadvantages of expandable metal stents include


A. Requirement of rigid bronchoscopy to place
B. Stent migration, erosion, ingrowth of granulation tumor, and inability to
effectively clear secretions
C. Cannot be used if tumor involves the carina
D. Cannot be used if tumor has a luminal component
E. Need to be replaced every 6 months

327
BY DR. MOHAMMED ATIAA KAREEM ALNASHY
-------------------------------------------------------------------
CHAPTER 108
Diagnosis and Management of Tracheal Neoplasms

1. B

2. E

3. A

4. C

5. E

328
BY DR. MOHAMMED ATIAA KAREEM ALNASHY
-------------------------------------------------------------------
CHAPTER ONE HUNDRED AND NINE: UPPER AIRWAY
MANIFESTATIONS OF GASTROESOPHAGEAL REFLUX DISEASE

1. Which of the following regarding the upper esophageal sphincter


(UES) is true?
A. The upper esophageal sphincter is a complete muscular circle.
B. The cricopharyngeus receives input from centers in the nucleus
ambiguous and nucleus solitarius.
C. During UES contraction, the cricoid is displaced inferiorly and
posteriorly.
D. During sleep, UES pressure is increased.
E. The UES is innervated solely by the vagus nerve.

2. Which of the following substances decreases lower esophageal


sphincter tone?
A. Secretin
B. Cholecystokinin
C. Glucagon
D. Vasoactive intestinal peptide
E. All of the above

3. Which of the following conditions is associated with increased lower


esophageal sphincter resting pressure?
A. Hiatal hernia
B. CREST syndrome
C. Scleroderma
D. Isolated Raynaud's phenomenon
E. None of the above

329
BY DR. MOHAMMED ATIAA KAREEM ALNASHY
-------------------------------------------------------------------
4. Which of the following regarding manifestations of extraesophageal
reflux (EER) is true?
A. Pseudosulcus involves the free edge of the vocal fold and ends at the
vocal process.
B. Pachydermia laryngeus refers to thickening of the anterior larynx.
C. Stimulation of the larynx by aspirated secretions causes reflexive vocal
cord abduction.
D. Granuloma formation may indicate severe EER.
E. None of the above

5. Which of the following is true regarding GERD management?


A. Chewing gum 1 hour after meals increases acid contact time in patients
with reflux.
B. Antacids decrease LES resting pressure.
C. In a small double-blind placebo-controlled trial, short-term
omeprazole was equivalent to placebo in controlling patient symptoms.
D. Long-term H2-blocker therapy is as effective as proton pump inhibitors
in treating esophagitis.
E. Metoclopramide improves gastric motility and decreases LES pressure.

CHAPTER 109
Upper Airway Manifestations of Gastroesophageal Reflux Disease

1. B

2. E

3. E

4D

5. C

330
BY DR. MOHAMMED ATIAA KAREEM ALNASHY
-------------------------------------------------------------------

PART EIGHT
……………………………………………………………………………………………………………………
……………………………………………………………………………………………………………………
NECK

331
BY DR. MOHAMMED ATIAA KAREEM ALNASHY
-------------------------------------------------------------------
CHAPTER ONE HUNDRED AND TEN: DEEP NECK INFECTION
-------------------------------------------------------------------------------------------

1. A 48-year-old woman who had a small cutaneous nevus removed from


the lateral side of her neck is seen 2 days later with high fever,
confusion, and a 5-cm area of erythema surrounding the wound. The
area has indistinct borders, and crepitance is noted. What is the
appropriate initial management of this patient?
A. Antistaphylococcal antibiotics and overnight observation
B. Fine-needle aspiration and oral antibiotics
C. Hyperbaric oxygen therapy and broad-spectrum antibiotics
D. Emergent surgical exploration/debridement and broad-spectrum antibiotics
E. Bedside incision and drainage and discharge on oral antibiotics

2. A 7-year-old child is initially seen with fever and firm, painful


swelling of the bilateral floor of mouth 1 day after frenulectomy. Her
tongue is elevated, and she speaks with a characteristic breathy voice.
Which of the following is an important component in the initial
evaluation of this patient?
A. Fiberoptic laryngoscopy to evaluate the airway
B. Fine-needle aspiration
C. Ultrasound evaluation of the floor of mouth
D. Both computed tomography and magnetic resonance imaging of the neck
E. Panorex film

3. A patient has a buccal space infection develop that is not eliminated by


an initial course of broad-spectrum antibiotic therapy. What is the next
step in this patient's management?
A. Intraoral incision and drainage
B. Extraoral incision and drainage
C. Second course of a different antibiotic
D. Short-term course of steroids
E. Antifungal therapy

332
BY DR. MOHAMMED ATIAA KAREEM ALNASHY
-------------------------------------------------------------------
4. Which bacteria are most commonly cultured from deep space neck abscesses?
A. Aerobic gram-negative bacilli
B. Actinomyces israelii
C. Streptococci species
D. Staphylococci species
E. Pseudomonas species

5. What percentage of retropharyngeal abscesses occur in children


younger than 6 years of age?
A. 5%
B. 10%
C. 30%
D. 50%
E. 90%

CHAPTER 110
Deep Neck Infection

1D

2. A

3. B

4. C

5. E

333
BY DR. MOHAMMED ATIAA KAREEM ALNASHY
-------------------------------------------------------------------
CHAPTER ONE HUNDRED AND ELEVEN: BLUNT AND
PENETRATING TRAUMA TO THE NECK
--------------------------------------------------------------------------------------------

1. A 40-year-old man sustained a stab wound to zone III. He is hemodynamieally


stable but has an acute hypoglossal nerve paralysis. What is the next step?
A. Four-vessel angiogram
B. Computed tomography scan
C. Magnetic resonance imaging
D. Lateral soft tissue of the neck
E. Direct laryngoscopy

2. What region of the neck has the most difficult surgical access for exploration?
A. Base of skull region
B. Mid cervical region
C. Lower cervical region
D. Posterior neck triangle
E. Anterior-cervical triangle

3. The best incision to explore the carotid sheath for a unilateral, penetrating,
neck injury is
A. A modified Conley incision
B. Diagonal cervical incision along the anterior sternocleidomastoid muscle
C. A MacFee incision
D. An H incision
E. A Schobinger incision

4. The anatomic boundaries of zone III are from


A. The hyoid to base of skull
B. The superior border of the thyroid cartilage to the base of skull
C. The hyoid to the mandible angle
D. The mandible angle to the skull base
E. The clavicle to the cricoid

5. During endoscopy, what injured area is most commonly missed in evaluating


a penetrating neck-injury?
A. Esophageal inlet
B. Distal cervical esophagus
C. Nasopharynx
D. Vallecula
E. Supraglottic area

334
BY DR. MOHAMMED ATIAA KAREEM ALNASHY
-------------------------------------------------------------------
CHAPTER 111
Blunt and Penetrating Trauma to the Neck

1. A
A neurologic deficit signifies higher probability that a vascular injury is
present because of anatomic proximity.

2. A
Zone III is more difficult to explore surgically than zones II and I because of
the presence of the skull base and mandible.

3B
A diagonal incision along the anterior sternocleidomastoid muscle gives the
best exposure for carotid artery inspection.

4D
Severe injuries in zone III may not be evident initially on clinical
examination, because it can be masked laterally by the mandible.

5. A
Because of mucosa redundancy and proximal location of the esophageal inlet,
penetrating injuries can be missed when introducing the scope

335
BY DR. MOHAMMED ATIAA KAREEM ALNASHY
-------------------------------------------------------------------
CHAPTER ONE HUNDRED AND TWELVE: DIFFERENTIAL DIAGNOSIS
OF NECK MASSES
---------------------------------------------------------------------------------------------------

1. When taking random guided biopsies to look for an occult primary tumor,
which is not one of the most likely sites?
A. Nasopharynx around Rosenmüller fossa
B. Tonsil
C. Base of the tongue
D. Pyriform sinus
E. Buccal mucosa

2. Which of the following is not an indication for biopsy of a neck mass in a child?
A. Progressively enlarging nodes
B. Single asymmetric nodal mass
C. Stable bilateral, symmetric masses
D. Persistent nodal mass without antecedent signs of infection
E. Actively infectious conditions that do not respond to conventional antibiotics

3. You are called to see a 3-day-old, full-term infant who was delivered with the
assistance of forceps. The child has a palpable mass in the anterior neck in the
region of the sternocleidomastoid muscle. What is the appropriate management?
A. Heat, massage, and observation
B. Fine-needle aspiration
C. Open drainage
D. Two-week course of antistreptococcal antibiotics
E. Surgical exploration of the neck

4. A 13-year-old child is seen with a fever and painful swelling in the area of
the angle of the left mandible. Last week, she had an upper respiratory tract
infection. Ultrasonography reveals the area to be cystic. What would be
expected on aspiration of this lesion?
A. Serosanguineous fluid with abundant monocytes
B. Milky brown fluid that contains cholesterol crystals
C. Clear fluid with many lymphocytes
D. Inflammatory fluid with abundant neutrophils
E. Serous fluid with many bacteria

5. A 36-year-old man is seen with a right parotid mass. Which of the following
characteristics suggests malignancy?
A. Size >2 cm
B. Previous history of parotid mass
C. Pain
D. Intact cranial nerve exam
E. Family history of lymphoma

336
BY DR. MOHAMMED ATIAA KAREEM ALNASHY
-------------------------------------------------------------------
CHAPTER 112
Differential Diagnosis of Neck Masses

1. E

2. C

3. A

4B

5. C

337
BY DR. MOHAMMED ATIAA KAREEM ALNASHY
-------------------------------------------------------------------
CHAPTER ONE HUNDRED AND THIRTEEN: PRIMARY NEOPLASMS OF THE
NECK
--------------------------------------------------------------------------------------------

1. All of the following are potential etiologic factors in the development of


paragangliomas except
A. Familial syndromes, such as multiple endocrine neoplasia types IIA and IIB
B. Autosomal-dominant inheritance pattern modified by genomic imprinting
C. History of previous radiation exposure
D. Living at elevated altitudes
E. Conditions of chronic arterial hypoxemia, such as cyanotic heart disease

2. Symptoms or signs attributable to a carotid paraganglioma may include all


of the following except
A. Pulsatile tinnitus
B. Dysphagia
C. Hoarseness
D. Palpitations
E. Flushing

3. The cellular pattern of alternating regions of compact, spindle cells, called


Antoni type A areas, and more loosely arranged, hypocellular zones, called
Antoni type B areas, is characteristic of
A. Paragangliomas
B. Schwannomas
C. Neurofibromas
D. Fibrosarcomas
E. Synovial sarcomas

4. After a complete history and comprehensive physical examination of the


head and neck region, the next diagnostic step for a neck mass suspicious for
a cervical lymph node with squamous cell carcinoma metastatic from an
unknown primary tumor is
A. Incisional biopsy
B. Excisional biopsy
C. Fine-needle aspiration biopsy
D. Neck dissection
E. Close observation

5. The most common soft tissue sarcoma of the head and neck in children is
A. Angiosarcoma
B. Chondrosarcoma
C. Osteosarcoma
D. Rhabdomyosarcoma
E. Ewing's sarcoma

338
BY DR. MOHAMMED ATIAA KAREEM ALNASHY
-------------------------------------------------------------------
CHAPTER 113
Primary Neoplasms of the Neck

1. C
The etiology of paragangliomas is multifactorial and includes familial syndromes,
such as MEN types IIA and B; genomic imprinting; living at elevated altitudes; and
conditions causing chronic arterial hypoxemia, such as cyantoic heart disease.
Previous radiation exposure has not been described as an inciting agent in the
development of paragangliomas.

2A
As carotid paragangliomas enlarge, progressive symptoms attributable to cranial nerve
deficits of IX, X, XI, or XII may appear and may result in dysphagia, odynophagia,
or hoarseness. Functional paragangliomas only make up 1% to 3% of paragangliomas
but may be heralded by symptoms such as headaches, palpitations, flushing, or
perspiration because of neuropeptide secretion. Although carotid paragangliomas may
present as a pulsatile neck mass, pulsatile tinnitus is a symptom typically
characteristic of jugulotympanic paragangliomas.

3B
The histologic pattern of alternating regions containing compact, spindle cell Antoni
type A areas and more loosely arranged Antoni type B areas is characteristic of
schwannomas. Paragangliomas typically contain two types of cells: type I or chief
cells, and type II, or sustentacular cells. Neurofibromas typically demonstrate
histologically interlacing bundles of spindle cells. Fibrosarcomas present
histologically as fibroblastic proliferation of variable amounts of collagen and
reticulin forming a "herringbone" pattern. Synovial sarcomas typically demonstrate a
predominant spindle cell component, with cuboidal and columnar cells surrounding
glandular areas, and may have calcifications in up to 30% of cases.

4. C
Fine-needle aspiration (FNA) biopsy is a technique both sensitive and specific in the
diagnosis of squamous cell carcinoma metastatic to cervical lymph nodes. The need
for a complete history and physical examination cannot be understated, and FNA
should be the next step in the algorithm when evaluating a neck mass in an adult.

5D
The most common soft tissue sarcoma of the head and neck in children is
rhabdomyosarcoma. Head and neck rhabdomyosarcomas have the highest incidence
in the first decade, with another peak occurring in the second and third decade.

339
BY DR. MOHAMMED ATIAA KAREEM ALNASHY
-------------------------------------------------------------------
CHAPTER ONE HUNDRED AND FOURTEEN: LYMPHOMAS
PRESENTING IN THE HEAD AND NECK
--------------------------------------------------------------------------------------------

1. Which of the following represents a viral-associated lymphoma?


A. Tonsillar lymphoma and parvovirus
B. African Burkitt's lymphoma and Epstein-Barr virus
C. Hodgkin's lymphoma and HIV
D. B-cell lymphoma and parainfluenza virus
E. Sinonasal lymphoma and adenovirus

2. A patient has lymphoma of the right tonsil. This patient has a 20% to 30% chance
of having a synchronous or metachronous involvement of what other organ?
A. Brain
B. Spleen
C. Thyroid
D. Gastrointestinal tract
E. Kidney

3. A patient is seen with a stage II, low-grade lymphoma of the lingual tonsil.
What is the initial therapy for this patient?
A. Partial glossectomy with bilateral neck dissection
B. Three to six cycles of cyclophosphamide, doxorubicin, vincristine, and
prednisone (CHOP)-based chemotherapy, then radiation
C. Radiation therapy alone
D. Total glossectomy with bilateral radical neck dissection
E. Six cycles of CHOP-based chemotherapy alone

4. A patient with stage II diffuse large B-cell lymphoma is treated with six
cycles of CHOP-based chemotherapy followed by radiation. What is the
expected percentage of freedom from disease progression?
A. 80%
B. 60%
C. 40%
D. 20%
E. 10%

5. An 18-year-old woman is seen with a rapidly enlarging neck mass that is shown
on biopsy to be Burkitt's lymphoma. What is the initial therapy for this patient?
A. Chemotherapy alone
B. Induction chemotherapy followed by low-dose radiation therapy
C. Induction chemotherapy followed by high-dose radiation therapy
D. Radiation therapy alone
E. Surgical resection followed by radiation therapy

340
BY DR. MOHAMMED ATIAA KAREEM ALNASHY
-------------------------------------------------------------------
CHAPTER 114
Lymphomas Presenting in the Head and Neck

1B
2D
3.C
4B
5A

341
BY DR. MOHAMMED ATIAA KAREEM ALNASHY
-------------------------------------------------------------------
CHAPTER ONE HUNDRED AND FIFTEEN: RADIATION THERAPY
AND MANAGEMENT OF THE CERVICAL LYMPH NODES
--------------------------------------------------------------------------------------------

1. Which of the following sites of squamous cell carcinoma is most likely to


present with positive neck nodes?
A. Tonsillar fossa
B. Lip
C. Subglottic larynx
D. Nasal cavity
E. Glottic larynx

2. Although the ideal dose regimen for elective neck irradiation (ENI) has yet to
be clearly established, current studies support which of the following regimens?
A. 2000 to 2500 cGy in 3 to 4 weeks
B. 7000 to 8000 cGy in 4 to 5 weeks
C. 4500 to 5000 cGy in 4.5 to 5.5 weeks
D. 1500 to 2000 cGy in 4.5 to 5.5 weeks
E. 1500 to 2000 cGy in 2.5 to 3 weeks

3. Which of the following is a true statement regarding combined radiation


and surgical therapy for head and neck squamous cell cancer?
A. Preoperative doses of radiation are lower than postoperative doses.
B. Postoperative radiation is only directed at the bed of the primary tumor.
C. Sensitive structures such as the larynx and spinal cord are more easily
protected with preoperative versus postoperative radiation.
D. Radiation therapy is not of use with extracapsular spread.
E. High-dose preoperative radiation decreases the morbidity of surgical resection.

4. Which of the following is not true of hyperfractionation?


A. The total dose of radiation is increased.
B. The dose per fraction is decreased.
C. A single dose is given each day.
D. Overall treatment time is the same as conventional therapy.
E. Benefit is greater in primary tumor control than nodal control.

5. Which of the following statements regarding combined chemotherapy and


radiation therapy in head and neck cancer is true?
A. Overall 5-year survival is >75% with concurrent therapy.
B. Concurrent therapy increases survival by 8% to 12% in advanced cancers.
C. Adjuvant chemotherapy before radiation increases survival by >20%.
D. Neoadjuvant chemotherapy followed by radiation increases survival by 33%.
E. Paclitaxel is the most commonly used chemotherapeutic agent.

342
BY DR. MOHAMMED ATIAA KAREEM ALNASHY
-------------------------------------------------------------------
CHAPTER 115

Radiation Therapy and Management of the Cervical Lymph Nodes

1. A

2. C

3. A

4. C

5B

343
BY DR. MOHAMMED ATIAA KAREEM ALNASHY
-------------------------------------------------------------------
CHAPTER ONE HUNDRED AND SIXTEEN: NECK DISSECTION
--------------------------------------------------------------------------------------------

1. Which of the following is not of prognostic significance for recurrence after


radical neck dissection?
A. Presence of microscopic extracapsular extension
B. Presence of macroscopic extracapsular extension
C. Number of involved nodes
D. Level of nodal involvement
E. Presence of bilateral nodal disease.

2. On postoperative day 1 after neck dissection, a patient has increasing


fullness of the ipsilateral neck develop, and her wound drains put out 700
mL of yellowish fluid. Which of the following is true?
A. This complication occurs in 10% of neck dissections.
B. Conservative management with drainage and pressure dressings should
control the condition.
C. Early surgical exploration is indicated.
D. High-fat diet will help control the fluid accumulation.
E. Enteral feedings should be immediately discontinued.

3. Which of the following is true of selective neck dissection for oral cavity cancer?
A. Includes levels I to III
B. Also called infrahyoid neck dissection
C. The posterior border of the dissection is the anterior border of the
sternocleidomastoid.
D. If the oral tongue is involved, level V should be included in the specimen.
E. Contralateral neck dissection is indicated for N2a disease.

4. Which of the following is not true of radical neck dissection?


A. The spinal accessory nerve is sacrificed.
B. Levels I to V are resected with the specimen.
C. The internal jugular vein is preserved.
D. Does not include postauricular or suboccipital nodes
E. The sternocleidomastoid muscle is sacrificed.

5. Which of the following structures is contained in level V?


A. Precricoid (Delphian) node
B. Sentinel (Virchow's) node
C. Carotid bifurcation
D. Submandibular gland
E. Phrenic nerve

344
BY DR. MOHAMMED ATIAA KAREEM ALNASHY
-------------------------------------------------------------------
CHAPTER 116
Neck Dissection

1. E

2. C

3. A

4. C

5. B

345
BY DR. MOHAMMED ATIAA KAREEM ALNASHY
-------------------------------------------------------------------
CHAPTER ONE HUNDRED AND SEVENTEEN: SURGICAL
COMPLICATIONS OF THE NECK
--------------------------------------------------------------------------------------------

1. When planning a neck dissection incision, which is the best choice in a


patient who has undergone previous radiation therapy?
A. A modified MacFee incision because of the decreased risk of wound dehiscence
B. A triradiate incision because of the excellent exposure in a neck with postradiation
fibrosis and difficult landmarks
C. An apron flap because it is cosmetically most appealing
D. A triradiate incision because of the decreased vascularity after radiation therapy
E. An apron incision because of the decreased risk of wound dehiscence

2. Which of the following is not a reason to use 24 hours of perioperative


antibiotics when performing a neck dissection?
A. Studies have shown that longer courses of antibiotics do not affect the
incidence of postoperative wound infection.
B. The incidence of wound infection in neck dissection is significantly higher
than the incidences of wound infection in other clean head and neck cases.
C. Prospective, randomized trials have found a decreased incidence of wound
infections in patients who undergo neck dissection and are treated with 24
hours of antibiotics.
D. The cost of treatment of postoperative wound infections outweighs the cost
of prophylaxis.
E. The neck dissection may be combined with a procedure that leads to spillage
of oral flora into the wound.

3. Which of the following are true about radiation and chemotherapy?


A. Both treatments affect wound healing.
B. The only effects of these treatments that are relevant when planning surgery
are the effects on wound healing.
C. Timing of surgery after radiation and/or chemotherapy is an important
consideration.
D. Treatment time is the important factor affecting injury in late-responding
tissues, whereas fraction dose is the most important factor affecting injury
in early responding tissues.
E. Two of the above are correct.
F. Three of the above are correct.
G. All are correct.

346
BY DR. MOHAMMED ATIAA KAREEM ALNASHY
-------------------------------------------------------------------
4. Chylous fistula after neck dissection can be treated in all the following
ways except
A. Head elevation
B. Pressure dressings
C. Total parenteral nutrition
D. Instillation of doxycycline
E. Prevention
F. Subcutaneous somatostatin injections
G. Modified chain triglyceride enteral diet
H. Reoperation

5. 11th nerve syndrome


A. Always occurs after sacrifice of the spinal accessory nerve
B. Consists of a constellation of symptoms, including limited active
shoulder abduction and a constant dull ache, stiffness, or soreness
C. Does not occur with neck dissections that preserve the 11th nerve
D. All of the above

347
BY DR. MOHAMMED ATIAA KAREEM ALNASHY
-------------------------------------------------------------------
CHAPTER 117
Surgical Complications of the Neck

1. E
In the study by Yii and others, the only complication that was increased in a
comparison between triradiate, modified MacFee and apron incisions was the
incidence of wound dehiscence in previously radiated necks when the triradiate
incision was used. The authors suggest using the apron incision in these patients,
because they believe the exposure is better than the modified MacFee.

2. C
Although no prospective, randomized trials have shown a benefit of perioperative
wound infections, a number of retrospective analyses indicate a trend favoring the
use of a short course (24 hours) of antibiotics when performing a neck dissection.
These include a study of 192 patients undergoing neck dissection in whom three
times more wound infections occurred in those not receiving antibiotics, and a
study of 201 clean head and neck cases in which the rate of wound infection after
neck dissection was 13% vs 1% for all other procedures. Two quoted studies have
shown that shorter time courses of antibiotics were found to be as effective at
controlling wound infections in both clean head and neck cases and clean-
contaminated head and neck cases compared with longer courses of antibiotics. A
cost analysis study showed that treatment of three patients with wound infection
outweighed prophylaxis of 100 patients. Finally, it is well known that wound
infection rate increases dramatically when a procedure involving spillage of oral
secretions is performed.

3. E
Radiation therapy has both early effects (DNA damage and cell death) and late
effects (microvascular damage leading to capillary dilation and obliteration and
increased fibrous tissue) that impact wound healing, whereas chemotherapy affects
wound healing by its effect on WBCs. In 69 patients who underwent neck dissection
after chemoradiotherapy, complications included flap necrosis, need for
tracheotomy, nerve injury, and hypocalcemia. Timing of surgery after
chemoradiotherapy seems to be most advantageous in the range of 5 to 17 weeks
after treatment. Finally, treatment time seems to be an important factor in the
response of early-responding tissues, and fraction dose is important in the effect on
late-responding tissues.

348
BY DR. MOHAMMED ATIAA KAREEM ALNASHY
-------------------------------------------------------------------
4. G
As in many complications, prevention of injury to the thoracic duct is the best
form of treatment. Once a chyle fistula is recognized, it can be treated
conservatively with elevation of the head of bed, continued suction drains,
pressure dressings, replacement of fluids and electrolytes, and an enteral diet
restricted to medium-chain triglycerides or TPN. If the output is high (>600-
1000 mL/24 hr), reoperation is indicated. Doxycycline has been used as a
sclerosing agent, although caution is advised, because it is neurotoxic.
Subcutaneous somatostatin has also been reported in the treatment of both
chylothorax and chylous fistula of the neck.

5B
Eleventh nerve syndrome was first described by Ewing and Martin. This
syndrome includes a dull ache, stiffness or soreness, drooping of the shoulder,
aberrant scapular rotation, limited forward shoulder flexion, and limited
active shoulder abduction. Because this syndrome can be absent after radical
neck dissection in which the spinal accessory nerve is sacrificed, but present
after 11th nerve sparing neck dissections, it seems to have a more complex
explanation. One theory put forth is that adhesive capsulitis of the shoulder
may be a contributing factor after nerve-sparing neck dissections.

349
BY DR. MOHAMMED ATIAA KAREEM ALNASHY
-------------------------------------------------------------------

PART NINE
--------------------------------------------------------------------------------------------
--------------------------------------------------------------------------------------------
THYROID/ PARAT HYROID

350
BY DR. MOHAMMED ATIAA KAREEM ALNASHY
-------------------------------------------------------------------
CHAPTER ONE HUNDRED AND EIGHTEEN: DISORDERS OF THE
THYROID GLAND
--------------------------------------------------------------------------------------------

1. Otolaryngologic manifestations of hypothyroidism include all of the


following except
A. Sensorineural hearing loss and tinnitus
B. Sinus congestion
C. Enlargement of the tongue
D. Husky, raspy voice

2. Thyrotropin (TSH) levels are useful in all of the following clinical


scenarios except
A. A 58-year-old women previously diagnosed with hypothyroidism, to
establish the severity of her disease
B. Screening for hypothyroidism in a 24-year-old symptomatic woman
C. Guidance of thyroid replacement regimen in an elderly gentleman with
longstanding hypothyroidism
D. Establishing diagnosis in a 38-year-old man with palpitations, weight loss,
and increasing diaphoresis, who has normal T4 and T3 levels.

3. Methimazole and propylthiouracil may lead to remission of Graves' disease by


A. Decreasing the release of thyroid hormone
B. An immunosuppressive action
C. Decreasing thyroid hormone biosynthesis
D. Toxic effects on thyroid follicular cells

4. All of the following should be administered to a patient with thyroid


storm except
A. Supportive measures to maintain adequate CNS and cardiovascular function
B. Propranolol
C. High doses of glucocorticoids
D. Purified thyroglobulin to bind excess free T4
E. Inorganic iodine, orally or by nasogastric tube

5. You have just completed a total thyroidectomy and selective neck


dissection for a 48-year-old woman with papillary thyroid carcinoma
and metastatic disease in her neck. She is scheduled to undergo I131
treatment in 6 to 8 weeks. In the interim, you prescribe
A. Synthroid
B. Cytomel
C. Synthroid and Gytomel
D. Nothing

351
BY DR. MOHAMMED ATIAA KAREEM ALNASHY
-------------------------------------------------------------------
CHAPTER 118
Disorders of the Thyroid Gland

1B

2A

3B

4D

5B

352
BY DR. MOHAMMED ATIAA KAREEM ALNASHY
-------------------------------------------------------------------
CHAPTER ONE HUNDRED AND NINETEEN: MANAGEMENT OF
THYROID NEOPLASMS
--------------------------------------------------------------------------------------------

1. The primary blood supply for the superior parathyroid glands is


A. Superior thyroid artery
B. Inferior thyroid artery
C. Thyrocervical trunk
D. Carotid artery

2. Unlike other malignancies, which factor has an important prognostic


significance in patients with thyroid cancer
A. Gender
B. Regional metastases
C. Age
D. Histologic type

3. The most common form of thyroid carcinoma is


A. Follicular carcinoma
B. Medullary carcinoma
C. Papillary carcinoma
D. Anaplastic carcinoma

4. The primary method of spread for follicular carcinoma beyond the


initial disease site is
A. Regional metastases
B. Local extension
C. Distant metastases

5. Where does the recurrent laryngeal nerve (RLN) enter the laryngeal
framework?
A. Deep to the inferior thyroid artery
B. Lateral to the inferior constrictor muscles
C. Between the arch of the cricoid cartilage and the inferior cornu of the
thyroid cartilage
D. Through the cricothyroid muscle

353
BY DR. MOHAMMED ATIAA KAREEM ALNASHY
-------------------------------------------------------------------
CHAPTER 119
Management of Thyroid Neoplasms

1B
The superior parathyroid glands are derived from the fourth branchial pouch, and
the inferior parathyroid glands originate from
the third branchial pouch. The primary blood supply of the superior and inferior
parathyroid glands is the inferior thyroid artery. The inferior thyroid artery is a
branch of the thyrocervical trunk. Occasionally, the superior parathyroid glands
will also receive blood supply from the superior thyroid artery.

2. C
Although advanced age increases the likelihood for malignancy when evaluating a
patient with a head and neck lesion, age is a significant prognostic factor for
patients with thyroid carcinoma. Every prognostic classification, including the
AJCC TNM staging system, includes age at initial presentation as an important
variable in determining risk categorization.

3. C
Papillary carcinoma is the most common form of thyroid cancer, accounting for
60% to 70% of all cases. Follicular carcinomas account for approximately 10% to 15%
of all thyroid malignancies. Medullary carcinomas account for approximately 5%
of all thyroid carcinomas.

4B
Follicular carcinoma extends from the primary disease site mainly by local
extension. Unlike papillary and medullary carcinomas, follicular carcinomas are
less likely to metastasize to the cervical lymph nodes. The presence of cervical
lymph node disease should raise suspicion for significant local disease and visceral
invasion.

5. C
The RLN is found within a triangle defined by the trachea medially, the carotid
sheath laterally, and the undersurface of the retracted inferior thyroid pole
superiorly. The inferior thyroid artery has a variable relationship to the RLN and
may be deep or superficial to the inferior thyroid artery branches. The RLN enters
the laryngeal framework between the arch of the cricoid cartilage and the inferior
cornu of the thyroid cartilage after penetrating deep to the lowermost fibers of the
inferior constrictor muscle. The external branch of the superior laryngeal nerve
innervates the cricothyroid muscle.

354
BY DR. MOHAMMED ATIAA KAREEM ALNASHY
-------------------------------------------------------------------
CHAPTER ONE HUNDRED AND TWENTY: SURGICAL
MANAGEMENT OF PARATHYROID DISORDERS
--------------------------------------------------------------------------------------------

1. Most patients with primary hyperparathyroidism present with which of the


following?
A. Osteitis fibrosa cystica
B. Renal lithiasis
C. Neuromuscular syndrome
D. Osteoporosis
E. Minimally symptomatic hypercalcemia

2. The most reliable determinant for the presence of parathyroid carcinoma is


A. Fibrotic, septate glands
B. Periglandular fibrosis
C. Cervical soft tissue invasion
D. Lymphatic metastasis
E. Cellular atypia with mitosis

3. Which of the following biochemical profiles is found in patients with


primary hyperparathyroidism?
A. Increased S. calcium, increased S. Vitamin D, decreased urinary calcium
B. Increased S. calcium, increased S. phosphate, decreased S. Vitamin D
C. Decreased S. phosphate, increased urinary calcium, normal S. Vitamin D
D. Increased S. phosphate, increased urinary calcium, decreased Vitamin D
E. Increased S. calcium, increased S. phosphate, normal S. Vitamin D

4. Which of the following is the most reliable preoperative localizing study


used to predict the location of a single parathyroid adenoma?
A. Magnetic resonance imaging
B. Technetium 99m sestamibi scintigraphy
C. Computed tomography
D. High-resolution ultrasonography
E. Technetium 99m-thallium 201 subtraction imaging

5. Which of the following anatomic regions is most likely to harbor an ectopic


superior parathyroid adenoma?
A. Intrathyroid
B. Intrathymic
C. Carotid sheath
D. Anterior mediastinum
E. Retroesophageal

355
BY DR. MOHAMMED ATIAA KAREEM ALNASHY
-------------------------------------------------------------------
CHAPTER 120
Surgical Management of Parathyroid Disorders

1. E
At present, osteitis fibrosa cystica occurs in 1% of patients and only 10% to 20% are
initially seen with renal stones. Postmenopausal women with the disorder are at
greater risk for osteoporosis developing but do not represent the majority of
presenting manifestations. Some signs of muscle fatigue and malaise may be found
in as many as 40% of symptomatic patients.

2D
Adherence of glands to surrounding cervical soft tissue is common with
parathyroid carcinoma but may be found in adenoma with hemorrhage, resulting
in periglandular fibrosis with adherence and thyroid parenchyma involvement.
Broad separated fibrotic bands may be noted in both carcinoma and atypical
adenoma. Similarly, mitotic figures may also be seen in parathyroid adenoma and
hyperplasia, the absence of which does not eliminate the presence of carcinoma.
Metastases are the only certain sign of malignancy.

3. C
Hypercalcemia is the principal defining manifestation of primary
hyperparathyroidism. In contrast to patients with primary HPT, those with
familial hypocalciuric hypercalcemia will demonstrate low 24-hour urinary
calcium levels. Vitamin D levels are usually normal, and serum phosphate levels
are low in patients with primary hyperparathyroidism.

4B
Although both MRI and GT may be used as correlative adjuncts in localizing
hyperfunctional parathyroid glands in the reoperative setting, they are not
sufficient as an initial localizing study. Ultrasonography may not be effective in
localizing enlarged glands in the retroesophageal, retrotracheal, retrosternal, and
deep cervicothoracic inlet regions. Technetium 99m sestamibi is preferred over
subtraction imaging because of overall greater accuracy and ease of performance.

5. E
Ectopic locations for inferior parathyroid glands include an intrathymic location,
the anterior superior mediastinum, and within the carotid sheath. Although
ectopic superior glands may occupy an intrathyroidal location, they more
commonly will migrate to a retroesophageal position.

356
BY DR. MOHAMMED ATIAA KAREEM ALNASHY
-------------------------------------------------------------------
CHAPTER ONE HUNDRED AND TWENTY-ONE: PARANASAL
SINUSES: MANAGEMENT OF THYROID EYE DISEASE (GRAVES'
OPHTHALMOLOGY)
--------------------------------------------------------------------------------------------

1. Recent evidence suggests that Graves' ophthalmopathy is a result of


A. Autoimmune response directed at the extraocular muscle fibers
B. Immunomodulation mediated by the retrobulbar fibroblasts
C. Toxicity caused by fibroblast by-products of the altered metabolic
pathways in Graves' disease
D. Intravascular plaque deposits impairing the blood supply to the
extraocular muscles, resulting in ischemic damage and fibrosis

2. Which of the following is true according to recent evidence regarding


the epidemiology of Graves' ophthalmopathy?
A. Tobacco use increases the risk for goiter but has not been linked to
Graves' ophthalmopathy.
B. Men with Graves' disease have a lower rate of ophthalmopathy.
C. Women are three times more likely than men to have Graves' disease.
D. Patients of Asian descent with Graves' disease are more likely to have
ophthalmopathy develop than those of European descent.

3. Which of the following is true regarding the natural history and


clinical presentation of Graves' disease?
A. In most patients with undiagnosed Graves' disease, ophthalmopathy is
the presenting symptom.
B. There is a positive association between pre tibial myxedema and
ophthalmopathy in patients with Graves' disease.
C. Thyrotoxicosis aggravates eye disease in patients with Graves' disease.
D. Ophthalmoplagia is a temporary finding in Graves' disease, present
only in the rapid progression stage of the disease (after complete and
spontaneous return of eye movement function).

357
BY DR. MOHAMMED ATIAA KAREEM ALNASHY
-------------------------------------------------------------------
4. When planning surgical decompression of the orbit, the most
appropriate imaging study to obtain is
A. Thin-cut computed tomography scan of orbits
B. Orbital echography
C. Magnetic resonance imaging of orbits
D. Nuclear imaging using single photon emission-computed tomography
with 99mTc-DTPA and gallium-67

5. Orbital decompression, independent of the surgical method used, can


offer patients all of the following except
A. Reduction of proptosis
B. Improvement of vision from reduced optic neuropathy
C. Resolution of diplopia
D. Improved ocular motility

CHAPTER 121
Paranasal Sinuses: Management of Thyroid Eye Disease (Graves'
Ophthalmology)

1B

2. C

3B

4. A

5. C

358
BY DR. MOHAMMED ATIAA KAREEM ALNASHY
-------------------------------------------------------------------

PART TEN
--------------------------------------------------------------------------------------------
--------------------------------------------------------------------------------------------

GENERAL

359
BY DR. MOHAMMED ATIAA KAREEM ALNASHY
-------------------------------------------------------------------
CHAPTER ONE HUNDRED AND TWENTY-TWO: -ANATOMY OF THE
SKULL BASE, TEMPORAL BONE, EXTERNAL EAR, AND MIDDLE EAR
--------------------------------------------------------------------------------------------

1. The tympanic bone contributes to the formation of all of the following


except
A. External auditory meatus
B. Foramen lacerum
C. Styloid process
D. Eustachian tube
E. Glenoid fossa

2. Within the middle cranial fossa, the arcuate eminence of the superior
surface of the temporal bone corresponds to
A. Cochlea
B. Superior semicircular canal
C. Tegmen tympani
D. Geniculate ganglion

3. All of the following contribute to the sensory innervation of the


external ear except
A. Jacobsen's nerve
B. Great auricular nerve
C. Auriculotemporal nerve
D. Facial nerve
E. Vagus nerve

4. Which artery travels in the fallopian canal?


A. Anterior tympanic artery
B. Superior petrosal artery
C. Inferior tympanic artery
D. Superior tympanic artery
E. Jacobson's artery

5. In a middle cranial fossa procedure, which structure is considered the


principal surgical landmark?
A. Middle meningeal artery
B. Greater superficial petrosal nerve
C. Foramen lacerum
D. Arcuate eminence
E. Bill's bar

360
BY DR. MOHAMMED ATIAA KAREEM ALNASHY
-------------------------------------------------------------------
CHAPTER 122
Anatomy of the Skull Base, Temporal Bone, External Ear, and Middle Ear

1D

2B

3. A

4. B

5B

361
BY DR. MOHAMMED ATIAA KAREEM ALNASHY
-------------------------------------------------------------------
CHAPTER ONE HUNDRED AND TWENTY-THREE: NEURAL
PASTICITY IN OTOLOGY
-----------------------------------------------------------------------------------

1. When considering the neural projection system from the cochlea to


cortex, the terms "tonotopic" and "cochleotopic" are often used
interchangeably because
A. The only information to reach auditory cortex relates to tone frequency.
B. Processing of sound intensity and sound frequency are carried out
independently at the level of the cochlea.
C. There is an analogy with how light spectrum information is coded in
the retinotopic organization of the visual system.
D. The cochlea performs a place coding of sound frequency.
E. Cochlear hair cell damage can initiate a tonal tinnitus.

2. Which statement about cellular mechanisms of plasticity is correct?


A. The N-methyl-D-aspartate (NMDA) receptor is mainly associated with
adrenergic synapses.
B. Long-term potentiation (LTP) of synapses has only been experimentally
observed in neurons of hippocampus.
C. Hebbian strengthening of synapses requires the activation of cholinergic
neurons of the nucleus basalis.
D. LTP can result from alterations in both presynaptic and postsynaptic
mechanisms.
E. The NMDA receptor is slowly activated, because it is of the metabotropic
type.

3. Which of these comments concerning physiologic studies on auditory


system plasticity is correct?
A. High-frequency sensorineural hearing loss is always the result of apical hair
cell lesions in the cochlea
B. After partial cochlear deafferentation in the adult subject, reorganization
of tonotopic maps rapidly occurs in the cochlear nucleus.
C. In studies that use the aminoglycoside drug amikacin to damage the cochlea,
a typical result is a degeneration of hair cells at the base of the cochlea.
D. Plastic change after neonatal ablation of one cochlea results in a symmetric
ascending auditory pathway.
E. Lesions to primary auditory cortex can lead to hair cell damage in the
corresponding cochlear areas.

362
BY DR. MOHAMMED ATIAA KAREEM ALNASHY
-------------------------------------------------------------------
4. The cochleotopic (or tonotopic) projection system up to the cortex can
be considered the "main-line organization" of the auditory system
because
A. It functions to transfer to sensory cortex, as directly and efficiently as
possible, the cochlear pattern of neural activity that is caused by
acoustic stimulation.
B. Retinotopic pathways do not have such a clearly structured
organization.
C. Such a system allows information transfer between multiple sensory
modalities (e.g., touch, vision, hearing) at subcortical levels.
D. The sensory transduction of acoustic signals is carried out by cochlear
hair cells.
E. There is little or no processing of sound information until signals reach
auditory cortex.

5. Which of the following statements about age-related plasticity is


accurate?
A. Experimental studies have revealed that there is no significant
different between plasticity in the developing auditory system compared
with the mature system.
B. The long-term performance of congenitally deaf subjects with a
cochlear prosthesis is independent of their age at implantation.
C. In early development, the subcortical area of the auditory system seems
to be much more plastic (as judged by neural reorganization) that in
the adult subject.
D. In the adult animal, tonotopic map reorganization has been
experimentally demonstrated at all levels of the auditory system up to
the inferior colliculus.
E. Age-related plasticity has been clearly demonstrated in the auditory
system but does not seem to be the case in the visual system.

363
BY DR. MOHAMMED ATIAA KAREEM ALNASHY
-------------------------------------------------------------------
CHAPTER 123
Neural Plasticity in Otology

1. D
The main projection system within the auditory system is one in which the
topographic arrangement of cochlear afferent neurons is apparently maintained
throughout the system to cortex. It is simply because (spectral) sound frequency is
"place coded" along the cochlear length that a neuron connected up to a certain
cochlear position responds best to a certain frequency of sound. As noted in the text,
the interchangeability of these terms requires caution when the place coding of
sound frequency is disrupted (e.g., in cochlear pathology).

2D
Many mechanisms, both at the presynaptic site and associated with the postsynaptic
neuron, can produce an alteration in the efficacy of information transfer. Some of
these mechanisms are outlined in Figure 123-7.

3. C
A number of studies exploring plasticity in the auditory system have involved
making a total or partial cochlear deafferentation. Many studies have taken
advantage of the ototoxic effects of aminoglycoside antibiotics as an experimental
tool to cause cochlear hair cell lesions. A number of these aminoglycosides,
including amikacin, are very predictable in causing damage in basal cochlear
areas.

4. A
The visual, somatosensory, and auditory systems have a common organizational
feature. They each have a regular, topographically organized system of connections
that maintains patterns of neural activity generated at the sensory epithelium, up
to central cortical areas. This "main line organization"
allows a relatively accurate representation of the outside world at the cortical level.
It is mainly or only at this level where complex processing, memory storage and
retrieval, cross-modality comparisons, and so on can be carried out.

5. C
Age-related plasticity definitely exists in the auditory system, as it clearly does in
visual and other sensorimotor systems. In experimental studies of plasticity, it is
important to distinguish those that reveal plastic change in the adult subject and
those changes in a developing organism. In the auditory system, plasticity of
tonotopic map reorganization looks similar at the cortical level; however, in
subcortical regions, the age-related plasticity effects become very apparent.

364
BY DR. MOHAMMED ATIAA KAREEM ALNASHY
-------------------------------------------------------------------
CHAPTER ONE HUNDRED AND TWENTY-FOUR: TINNITUS AND
HYPERACUSIS
…………………………………………………………………………………………………………………….

1. Pulsatile tinnitus is uncommon in which clinical condition?


A. Pseudotumor cerebri
B. Otosclerosis
C. Acoustic neuroma
D. Dural arteriovenous fistula

2. Radiographic studies are helpful in diagnosing all of the following


conditions except
A. Pseudotumor cerebri
B. Carotid artery stenosis
C. Glomus jugulare
D. Dural arteriovenous fistula

3. Postoperative tinnitus after acoustic neuroma resection is most likely in


which scenario?
A. Preoperative tinnitus present; hearing preserved after surgery
B. Preoperative tinnitus present; hearing not preserved
C. Preoperative tinnitus absent; hearing preserved
D. Preoperative tinnitus absent; hearing not preserved

4. Clinicians should tell patients with chronic tinnitus


A. Because there is no cure for tinnitus, nothing can be done for them.
B. Because nothing can be done for tinnitus, the patient should just learn
to live with it.
C. There is a good chance that their tinnitus is the result of a brain tumor.
D. Even though there often is no cure for chronic tinnitus, effective
management strategies are available.

365
BY DR. MOHAMMED ATIAA KAREEM ALNASHY
-------------------------------------------------------------------
5. All of the following are common elements of effective tinnitus
management programs except
A. In-the-ear sound generators
B. Hearing aids
C. Stress reduction/relaxation therapy
D. Spending 15 to 20 minutes with each patient

6. Tinnitus severity
A. Is correlated with the matched loudness of the sound
B. Is correlated with the matched pitch of the sound
C. Is correlated with the patient's degree of sleep interference
D. Is the same for most patients

CHAPTER 124

Tinnitus and Hyperacusis

1. C

2. A

3B

4D

5D

6. C

366
BY DR. MOHAMMED ATIAA KAREEM ALNASHY
-------------------------------------------------------------------
CHAPTER ONE HUNDRED AND TWENTY-FIVE: - MANAGEMENT
OF TEMPORAL BONE TRAUMA
-------------------------------------------------------------------------------------------

1. All of the following are true of otic-capsule-disrupting temporal bone


fractures except
A. They almost always result in sensorineural hearing loss.
B. They have a much higher incidence of facial nerve palsies.
C. Recent studies show that only 10% to 15% of temporal bone fractures are otic-
capsule-disrupting.
D. There is a two to four times increase in the risk of cerebrospinal fluid fistula.

2. After re-examining published rates of facial nerve palsies associated with


temporal bone fractures and removing the associated sampling errors, the
true percentage of facial nerve palsies has been calculated to be
A. 3%
B. 5%
C. 7%
D. 10%
E. 15%

3. The most important factor in predicting facial nerve recovery after


temporal bone fracture is
A. Severity of injury
B. Timing of onset
C. Presence of associated infection
D. Presence of sensorineural hearing loss

4. All of the following are acceptable methods in evaluating suspected CNS


fistula after tympanomastoidectomy except
A. Evaluate draining fluid for β2 transferrin content
B. High resolution computed tomography (HRCT) with intrathecal contrast
C. Intrathecal fluorescein injection followed by examination of draining fluid
under a Wood's lamp
D. Skull base ultrasonography

5. If a suspected fistula is confirmed, the most appropriate next step should be


A. Repeat mastoidectomy with craniotomy and fistula repair
B. Placement of a lumbar drain
C. Prophylactic 14-day course of antibiotics with streptococcal/H influenzae
coverage
D. Conservative measures including bedrest with head elevation and stool
softeners

367
BY DR. MOHAMMED ATIAA KAREEM ALNASHY
-------------------------------------------------------------------
CHAPTER 125
Management of Temporal Bone Trauma

1. C

2. C

3B

4D

5D

368
BY DR. MOHAMMED ATIAA KAREEM ALNASHY
-------------------------------------------------------------------
CHAPTER ONE HUNDRED AND TWENTY-SIX: - OTOLOGIC
SYMPTOMS AND SYNDROMES
…………………………………………………………………………………………………………………….

1. Which of the following statements about bullous myringitis is true?


A. The infection is usually painless.
B. The etiology may be viral.
C. The presentation involves a sensorineural hearing loss (SNHL) alone.
D. The hearing loss is usually permanent.
E. The drainage is typically thick and mucopurulent.

2. Clear otorrhea from a ventilation tube may arise from


A. Bullous myringitis
B. Gustatory otorrhea
C. Spontaneous cerebrospinal fluid leak
D. Atypical tuberculosis
E. Chloroma formation

3. Aural fullness is usually not a symptom associated with


A. Eagle's syndrome
B. Patent eustachian tube
C. Perilymphatic fistula
D. Otitis media
E. Meniere's disease

4. Ramsay-Hunt syndrome does not usually include


A. Hearing loss
B. Otalgia
C. Vertigo
D. Facial paralysis
E. Dysarthria

5. The different diagnosis for sudden SNHL would not include


A. Cerebellopontine angle tumor
B. Immune mediated
C. Barotrauma
D. Head trauma
E. Graves' disease

369
BY DR. MOHAMMED ATIAA KAREEM ALNASHY
-------------------------------------------------------------------
CHAPTER 126
Otologic Symptoms and Syndromes

TB
Bullous myringitis is a painful infection of the tympanic membrane resulting
in a mixed hearing loss that typically resolves with treatment.

2. C
A spontaneous leak of cerebrospinal fluid may occur from arachnoid
granulations weakening the dura of the middle or posterior cranial fossa. If
persistent clear drainage occurs through a ventilation tube, the fluid should
be collected and tested for (3-2 transferrin.

3. A
Eagle's syndrome involves ear pain secondary to stretching or irritation of
the glossopharyngeal nerve from an elongated styloid process. The other
choices (b, c, d, e) are all associated with the symptom of aural fullness.

4. E
Ramsay-Hunt syndrome represents a viral polyneuropathy primarily
affecting the cochleovestibular and facial nerves. It occurs through
reactivation of latent varicella virus within the cranial nerve ganglia. It
rarely involves additional cranial nerves (V, IX, X, XI, XII).

5. E
Grave's disease is not associated with sudden SNHL. An uncommon
presentation of a CPA tumor is a sudden hearing loss that may or may not
recover with a course of steroids. Membranous labyrinthine injury from
barotrauma or head trauma without temporal bone fracture may also cause
sudden sensorineural hearing loss.

370
BY DR. MOHAMMED ATIAA KAREEM ALNASHY
-------------------------------------------------------------------
CHAPTER ONE HUNDRED AND TWENTY-SEVEN: - OTOLOGIC
MANIFESTATIONS OF SYSTEMIC DISEASE
…………………………………………………………………………………………………………………..

1. Which of the following disorders can mimic the symptoms and signs of
chronic otitis media?
A. Langerhans cell histiocytosis
B. Tuberculosis
C. Wegener's granulomatosis
D. All of the above
E. None of the above

2. What is the most common cause for the air-bone gap that is seen on
audiometric evaluation of patients with Paget's disease affecting the
temporal bone?
A. Malleus fixation
B. Stapes fixation
C. Obliteration of the round window
D. Resorption of the incus
E. All of above may occur in different patients
F. None of the above

3. What is the most common otologic manifestation of fibrous dysplasia


affecting the temporal bone?
A. Progressive narrowing of the external auditory canal
B. Obstruction of the eustachian tube with conductive hearing loss
C. Reddish mass behind an intact tympanic membrane
D. Facial nerve paralysis
E. Sensorineural hearing loss and vertigo caused by inner ear involvement

4. Which of the following clinical findings would suggest a diagnosis of


otosyphilis?
A. Positive Schwartze's sign (reddish hue to tympanic membrane on otoscopy)
B. Positive Brown's sign (blanching of tympanic membrane on pneumatic
otoscopy)
C. Positive Hennebert's sign (ocular deviation with pneumatic otoscopy)
D. All of the above
E. None of the above

371
BY DR. MOHAMMED ATIAA KAREEM ALNASHY
-------------------------------------------------------------------
5. The histopathologic report of granulation tissue removed at
tympanomastoidectomy indicates the presence of "chronic
inflammation, necrosis, granulomas with multinucleated giant cells,
vasculitis, and microabscesses." What is your diagnosis?
A. Tuberculosis
B. Wegener's granulomatosis
C. Langerhans' cell histiocytosis
D. Sarcoidosis
E. Syphilis

372
BY DR. MOHAMMED ATIAA KAREEM ALNASHY
-------------------------------------------------------------------
CHAPTER 127
Otologic Manifestations of Systemic Disease

1D
Langerhans cell histiocytosis, tuberculosis, and Wegener's granulomatosis
are all granulomatous diseases that can affect the middle ear and mastoid,
and, in so doing, they may closely mimic the symptoms and signs of chronic
otitis media.

2F
There is no consistent pathologic basis for the air-bone gap in Paget's disease.
Specifically, the apparent conductive hearing loss is not caused by a middle
ear lesion such as ossicular fixation or obliteration of the oval or round
windows. Therefore, attempts at surgical correction of the conductive hearing
loss are unlikely to be of benefit.

3. A
Progressive narrowing of the external auditory canal with conductive hearing
loss is the most common manifestation of fibrous dysplasia affecting the
temporal bone, occurring in about 80% of cases.

4. C
Positive Hennebert's sign, believed to be due to fibrous adhesions between the
stapes footplate and the membranous labyrinth, is seen as a result of inner
ear involvement by otosyphilis.

5B
This combination of histopathologic findings is characteristic of Wegener's
granulomatosis.

373
BY DR. MOHAMMED ATIAA KAREEM ALNASHY
-------------------------------------------------------------------
CHAPTER ONE HUNDRED AND TWENTY-EIGHT: NOISE-
INDUCED HEARING LOSS
……………………………………………………………………………………………………………………

1. Which one of the following has not been proposed as an anatomic


mechanism of noise damage?
A. Mechanical injury
B. Metabolic exhaustion
C. Ischemia
D. Ionic poisoning
E. Chronic bleeding

2. Otoacoustic emissions can be used clinically in patients with noise-induced


hearing loss (NIHL) to accomplish all but which one of the following?
A. To detect the initial stages of NIHL
B. To specify damage to the inner hair cell system
C. To estimate the configuration of the audiogram
D. To objectively monitor the progression of NIHL in instances of
continued exposure
E. To determine the frequency/amplifying characteristics of a
prescribed digital hearing aid

3. Synergistic effects are likely with chronic exposure to noise and all but
which one of the following?
A. Cisplatin
B. Carbon monoxide
C. Microwaves
D. Aminoglycosides
E. Whole-body or segmental vibration

4. Which one of the following nonauditory conditions has not be shown


to be associated with chronic exposure to noise?
A. Vertigo
B. Biologic stress
C. Emotional unrest
D. Hypertension
E. Gastrointestinal disease

374
BY DR. MOHAMMED ATIAA KAREEM ALNASHY
-------------------------------------------------------------------
5. Identify which one of the following is not a crucial aspect in the
regulatory control of sound levels in the workplace.
A. A hearing-conservation program if employees are exposed to
sounds >85 dBA
B. Sound levels measured on the dB SPL scale
C. The equivalent continuous sound level (Leq) principle
D. The time-weighted average
E. The equal-energy principle

375
BY DR. MOHAMMED ATIAA KAREEM ALNASHY
-------------------------------------------------------------------
CHAPTER 128
Noise-Induced Hearing Loss

1. E
The proposed anatomic substrates of NIHL include all of these possibilities except
chronic bleeding. Sound-induced mechanical damage includes such observations, at
either the high-power light microscope or scanning electron microscopy level, as
buckled supporting cells (e.g., pillar bodies that support the tunnel of Gorti) or bent
or broken stereocilia. Metabolic exhaustion caused by sound overexposure, which has
been evaluated using biochemical, histochemical, or immunohistochemical methods,
can occur during constant overstimulation, when cells have no opportunity to restore
the vital nutrients that power their intracellular machinery. Noise-induced ischemia
within the cochlea has been indexed by use of Laser-Doppler flowmetry measures
either through the round-window membrane or through the lateral bony wall of the
cochlea. Finally, ionic poisoning caused by the mixing of the disparate cochlear fluids
consisting of endolymph and perilymph has been shown to occur through the use of
in vivo tracers to demonstrate microbreaks in both the reticular lamina and apical
membranes of sensory and supporting cells in noise-damaged cochleas. Chronic
bleeding has never been observed in the acoustically over-stimulated cochlea probably
because of the unique pattern of blood supply to the inner ear, which, in the organ of
Corti is reduced to a system of microcapillaries and venules.

2B
Otoacoustic emissions (OAEs) can be used to accomplish all these efforts in the patient
with NIHL, except to specify damage to the inner hair cell system. There is
accumulating evidence, like that presented in Figure 128-5, which demonstrates that
OAEs detect cochlear dysfunction where NIHL can potentially occur before it has
been measured by other audiometric tests, including the clinical audiogram. In
addition, there are a number of lines of evidence (reviewed in 10) that OAEs are
predominately generated by the OHC system. Because OHCs represent the organ of
Corti component that is initially damaged in NIHL, OAEs make ideal measures with
which to identify the onset stages of this pathology. Thus, in NIHL, OHC
abnormalities should be the primary pathology. Therefore, OAEs are capable of
approximating the pattern of noise-induced hearing loss as measured by the pure-tone
audiogram. Furthermore, by indicating the pattern of OHC survival, they can be used
to predict which frequencies need to be amplified and which do not, thus optimizing
the fitting of a digital hearing aid to a particular patient with NIHL. Their
objectivity, simple set-up procedures, and rapid test times also make OAEs ideal for
monitoring the progression of hearing impairment in situations in which exposure to
noise continues.

376
BY DR. MOHAMMED ATIAA KAREEM ALNASHY
-------------------------------------------------------------------
3. C
It is well established that both noise damage and hearing loss can be augmented by
being exposed simultaneously (and, in some case, before and after) to certain agents.
These include the ototoxic drug cisplatin and the aminoglycosidic antibiotics. In
addition, a great deal of accumulating research has also shown such synergism
between noise and particular industrial chemicals such as the asphyxiant, carbon
monoxide. Furthermore, other stresses such as whole-body or segmental vibration have
also been shown to enhance the adverse effects of loud sounds. However, although
microwaves per se have been shown to have adverse effects on hearing, it is their
thermal rather than their mechanical properties that seem to produce any related
hearing loss.

4. A
Many studies of chronic noise exposure of the sort experienced by people who live near
airports that operate 24 hours a day have demonstrated noise-induced biologic and/or
psychological stress in such individuals. These effects include pathologic conditions
as emotional unrest, hypertension, and peptic ulcers. Although there is some evidence
using sophisticated rotary-chair testing of vestibular pathways that support the
sensitive compensatory processes of balance function, out-right vertigo is not a typical
complaint of the patient with NIHL.

5B
The regulatory control of sound in the workplace is promulgated by federal, state, and
local authorities. These basic regulations are essentially patterned after a ruling made
more than 20 years ago by OSHA that was subsequently enacted into law by Congress.
Part of the ruling states that if employees are exposed to >85 dB SPL in the workplace,
their employer must implement a hearing-conservation program that includes
baseline audiometric assessment, annual audiometric monitoring, and, if a hearing
loss is documented, the employee must be notified and educated about the use of
personal hearing protectors. As part of a hearing-conservation program, all workers
who toil in areas of high noise levels (i.e., >85 dBA) must wear ear protectors and
participate in a noise-education program on the hazardous effects of noise and the
correct fitting of personal hearing protectors. In identifying unsafe sound levels in
industry and in calculating the durations of safe exposure times, the concepts of the
time-weighted average and the principles of equal energy and equivalent continuous
sound level are paramount. However, the standard measure of sound level is in dBA
units rather than in sound pressure level (dB SPL) units that represent a linear scale
of measurement. The dBA scale is used to gauge the magnitude of occupational noise,
because it best estimates the configuration of the human threshold for hearing and
thus reduces the influence of sounds at very low and very high frequencies.

377
BY DR. MOHAMMED ATIAA KAREEM ALNASHY
-------------------------------------------------------------------
CHAPTER ONE HUNDRED AND TWENTY-NINE: - AUTOIMMUNE
INNER EAR DISEASE
…………………………………………………………………………………………………………………….

1. Which of the following autoimmune diseases involves the middle ear


more frequently than it affects the inner ear?
A. Systemic lupus erythematosus
B. Cogan's disease
C. Wegener's granulomatosis
D. Polyarteritis nodosa
E. Behcet's disease

2. Autoimmune inner ear disease is best diagnosed in a patient with


rapidly progressive hearing loss by
A. A documented clinical response to corticosteroids
B. A family history of autoimmune disease
C. Positive human leukocyte antigen (HLA) markers
D. Detection of specific circulating antibodies
E. The configuration of the audiogram

3. Autoimmune processes may involve which of the following structures?


A. The cochlear nerve
B. The vestibular labyrinth
C. The cochlea
D. The labyrinthine vasculature
E. All of the above

4. Patients with suspected autoimmune inner ear disease (AIED) require


an initial trial of prednisone for
A. 2 weeks
B. 4 weeks
C. 6 weeks
D. 8 weeks
E. 10 weeks

378
BY DR. MOHAMMED ATIAA KAREEM ALNASHY
-------------------------------------------------------------------
CHAPTER 129
Autoimmune Inner Ear Disease

1. C

2. A

3. E

4B

379
BY DR. MOHAMMED ATIAA KAREEM ALNASHY
-------------------------------------------------------------------
CHAPTER ONE HUNDRED AND THIRTY: VESTIBULAR AND
AUDITORY TOXICITY
…………………………………………………………………………………………………………………..

1. Which of the following ototoxic agents does not affect the basal turn of
the cochlea predominantly?
A. Gentamicin
B. Cisplatin
C. Arsenic
D. Neomycin
E. Amikacin

2. Which of the following drugs is most likely to cause permanent


sensorineural hearing loss in patients?
A. Erythromycin
B. Vancomycin
C. Furosemide
D. Cisplatin
E. Torsemide

3. Which of the following statements about aminoglycoside antibiotics is false?


A. Aminoglycoside antibiotics may cause permanent sensorineural hearing loss.
B. Permanent vestibular damage may result from aminoglycoside therapy.
C. Serum levels are predictive of aminoglycoside vestibular toxicity.
D. Oscillopsia is a symptom of severe vestibular damage from aminoglycoside
antibiotic treatment.
E. Intratympanic therapy with gentamicin carries a risk of permanent
sensorineural hearing loss.

4. Which of the following factors has the lowest predictive value for hearing
loss in patients undergoing therapy with aminoglycoside antibiotics?
A. Mutations involving connexin 26
B. Mutations of mitochondrial RNA
C. Renal insufficiency
D. Combined therapy with cisplatin
E. Septicemia

5. Which of the following drugs is most likely to damage the inner hair
cells of the organ of Corti?
A. Gentamicin
B. Furosemide
C. Cisplatin
D. Carboplatin
E. Vancomycin

380
BY DR. MOHAMMED ATIAA KAREEM ALNASHY
-------------------------------------------------------------------
CHAPTER 130: Vestibular and Auditory Toxicity

1. C
Arsenic. Each of the other drugs preferentially attack the hair cells of the
basal turn of the cochlea, causing high-frequency sensorineural hearing loss.

2D
Cisplatin. This drug is most likely to cause permanent sensorineural hearing
loss among the choices provided. Erythromycin, furosemide, and torsemide
can cause primarily reversible ototoxicity. Vancomycin usually not ototoxic
when given alone but can cause ototoxicity when given in combination with
aminoglycosides.

3. C
Blood levels of aminoglycosides are not predictive of vestibular damage. The
other options are all true.

4. A
Mutations involving connexin 26 can cause hearing loss but have not been
associated with increased sensitivity to ototoxicity from aminoglycosides.

5D
Carboplatin seems to selectively damage inner hair cells. The other drugs are
more likely to damage outer hair cells.

381
BY DR. MOHAMMED ATIAA KAREEM ALNASHY
-------------------------------------------------------------------
CHAPTER ONE HUNDRED AND THIRTY-ONE: - PHARMACOLOGIC
TREATMENT OF THE COCHLEA AND LABYRINTH
………………………………………………………………………………………………………………….

1. Ten days after using intratympanic gentamicin injections to treat


intractable vertigo in a 65-year-old patient with Meniere's disease, you
get a call that your patient is in the emergency department complaining
of severe nausea and vomiting. What is your reaction?
A. The patient is having severe intracerebral drug reaction to the
gentamicin and needs immediate intravenous steroids and fluid bolus to
maintain hemodynamic stability.
B. The patient is beyond the window of adverse drug reactions for
intratympanically delivered gentamicin. The triage team should be
notified that this is malingering behavior and a psychiatry consult
should be ordered.
C. The patient is experiencing acute vestibular deafferentation syndrome.
She should be reassured and offered vestibular rehabilitation as an
outpatient if symptoms do not resolve in the next 2 to 3 days.
D. The patient is experiencing visual-vestibular mismatch. Caloric testing
should be performed to document the severity of the reaction and to guide
future treatment.
E. The patient is having acute otitis media with central complications, a
known consequence of disturbing the middle ear resident flora with the
introduction of intratympanic gentamicin. Start antibiotics
immediately.

2. The primary problem with the use of intratympanic steroids for


Meniere's disease is that
A. There are no statistical data in randomized controlled clinical trials that
show that steroids are effective for any otologic indication related to
Meniere's disease.
B. Current steroid formulations are not suitable to intratympanic use,
because the acidity of the preparations induces stinging on injection.
C. Steroids decrease the host response to local pathogens, resulting in an
unacceptably high rate of postinjection otitis media.
D. Patients with Meniere's disease may have bilateral disease develop,
precluding the ability to treat both ears with the same strategy.
E. Researchers have not identified the exact mechanism of action
explaining how steroids produce their beneficial response.

382
BY DR. MOHAMMED ATIAA KAREEM ALNASHY
-------------------------------------------------------------------
3. The proven way to avoid anacusis when administering intratympanic
gentamicin treatment is
A. To use the titration strategy of dosing, so that if a patient starts to
experience hearing loss, the protocol can be immediately stopped
B. To administer intratympanic steroids at the first signs of hearing loss
C. To use the low-dose microcatheter perfusion systems as the delivery
method
D. To use frequent caloric testing to determine the moment that vestibular
ablation is achieved and to then promptly halt further treatment
E. None of the above

4. A patient is seen by you with severe tinnitus after going to a rock concert.
He is concerned because several family members have had a history of
hearing loss after severe noise exposure. He begs you for some kind of
treatment. Which of the following compounds could you use on an off-label
basis to try to prevent permanent noise-induced hearing loss?
A. Aspirin
B. Alcar
C. Riluzole
D. D-Methionine
E. All of the above

5. Neurotrophins have been shown to


A. Cause an acceleration of the apoptotic process
B. To effect a tonic suppression of the apoptotic process
C. To stimulate free radical production
D. To suppress free radical production
E. All of the above

383
BY DR. MOHAMMED ATIAA KAREEM ALNASHY
-------------------------------------------------------------------
CHAPTER 131
Pharmacologic Treatment of the Cochlea and Labyrinth

1. C
Acute vestibular deafferentation syndrome, also known as acute chemical
labyrinthine upset, is the consequence of unilaterally insulting the vestibular
apparatus. This phenomenon usually occurs 3 to 5 days after the injection.
Symptoms include vertigo, nausea, oscillopsia, and disequilibrium. Patients
can readily distinguish these symptoms from their typical Meniere's disease-
related symptoms. These symptoms become progressively worse until they
peak 1 week after onset. During this peak, patients usually require 2 to 3 days
of bed rest. Gradual resolution is achieved in 2 to 4 weeks in most patients.
Because acute vestibular deafferentation syndrome is an expected outcome of
therapy, some authors recommend that a vestibular rehabilitation team be
available to work with severely affected patients.

2. A
There is, indeed, no evidence in rigorous clinical studies (i.e., randomized and
controlled) documenting a clinical benefit of steroids in treating Meniere's
disease. There has been a stinging on injection documented by some patients
with methylprednisolone, and some practitioners recommend
coadministration with 0.1 mL of 1% lidocaine with 0.9 mL standard IV
methylprednisolone solution (40 mg/mL).141 There have been no reports of
IT steroids upsetting the inner ear flora. Because steroids are fairly benign,
treating patients with bilateral Meniere's disease with steroids is not the
relative contraindication that it is with IT gentamicin. Although it is true
that the mechanism of action of steroids in the inner ear are not yet fully
described, this is not a problem that would preclude steroid use in clinical
settings.

384
BY DR. MOHAMMED ATIAA KAREEM ALNASHY
-------------------------------------------------------------------
3. E
There is no way to completely prevent anacusis when gentamicin therapy is
used. Remember that the lowest dose recorded to cause total hearing loss was
only 0.24 mg.203 Even when using titration protocols, there is no evidence
showing that immediately stopping
therapy preserves or improves hearing outcomes. Although steroid injections
as salvage therapy may prove to be a useful therapy in the future, there are no
published studies yet documenting the efficacy of this intervention. The trend
toward the use of microdoses of gentamicin or delivering gentamicin through
sustained-release devices seems to be improving hearing outcomes.

4. E
All of these compounds are approved for use in humans and could potentially
be used on an off-label basis. Of these compounds, the one that has been shown
to improve outcomes after noise-induced trauma is riluzole. However,
intratympanic use of this compound has never been attempted in humans and
systemic application can cause significant side effects.

5. E
Although neurotrophins represent an exciting class of potential therapeutic
compounds, their wide range of actions are only beginning to be understood.
In fact, under certain pathologic conditions, neurotrophins can exacerbate,
rather than alleviate, injury. For a good review of this topic, see the review
article by Behrens and others (Neurotrophin-mediated potentiation of
neuronal injury, Microsc Res Tech 45[4-5]:276, 1999).

385
BY DR. MOHAMMED ATIAA KAREEM ALNASHY
-------------------------------------------------------------------

PART ELEVEN
……………………………………………………………………………………………………………………
……………………………………………………………………………………………………………………
INFECTIOUS PROCESSES

386
BY DR. MOHAMMED ATIAA KAREEM ALNASHY
-------------------------------------------------------------------
CHAPTER ONE HUNDRED AND THIRTY-TWO: INFECTIONS OF
THE EXTERNAL EAR
…………………………………………………………………………………………………………………….

1. A 36-year-old man is seen by you with a 3-day history of progressive


pain and itching in his right ear. This morning he found drainage from
that ear on his pillow. Your examination confirms your suspicion of
otitis externa with mild amount of creamy otorrhea. What is the best
treatment option?
A. Oral antibiotics
B. Debridement of canal and oral antibiotics
C. Debridement of canal and ototopical antibiotics
D. Debridement of canal, oral antibiotics, and ototopical antibiotics

2. A 42-year-old woman is seen with 6 days of ear pain and otorrhea. Over
the past 24 hours, her ipsilateral cheek has become swollen and red. On
examination, you see otitis externa with a moderate amount of creamy
otorrhea and a mild amount of erythema and edema to her ipsilateral
cheek. There is one enlarged preauricular lymph node. What is the best
treatment option?
A. Oral antibiotics
B. Debridement of canal and oral antibiotics
C. Debridement of canal and ototopical antibiotics
D. Debridement of canal, oral antibiotics, and ototopical antibiotics

3. A 56-year-old man is seen with a history of 3 months of itching and


pain in his right ear. He has been prescribed several otic drops for otitis
externa, but none have resolved his condition. On examination, you see
a red, mildly thickened EAG and conchal bowl with no otorrhea or
debris in canal. The skin in these areas has a slight amount of
desquamation. How will you treat him?
A. Oral antibiotics because patient has had ototopical therapy fail
B. Place an earwick and prescribe 14 days of Cortisporin
C. Topical clotrimazole cream
D. Topical corticosteroid cream

387
BY DR. MOHAMMED ATIAA KAREEM ALNASHY
-------------------------------------------------------------------
4. You are seeing a 52-year-old diabetic man with possible malignant
otitis externa. Your history and physical examination confirm your
suspicion. Your next steps include all of the following except
A. Normalization of any hyperglycemia
B. Culture of the EAG and frequent debridements
C. Technetium-99m and gallium-67 scans of the temporal bones
D. Initiation of IV antibiotics and obtain Infectious Disease consult
E. Proceed straight to mastoidectomy and tympanoplasty to remove
granulation tissue and necrotic bone

5. While evaluating a patient for right-sided hearing loss, you discover a


yellow-white mass that occludes the EAC. It does not appear to be
invading the canal walls. You see a similar finding on the right, but to
a lesser degree. What is the diagnosis?
A. External auditory canal cholesteatoma
B. Exostoses
C. Keratosis obliterans
D. Furunculosis
E. Cerumen impaction

388
BY DR. MOHAMMED ATIAA KAREEM ALNASHY
-------------------------------------------------------------------
CHAPTER 132
Infections of the External Ear

1. C

2. D

3. D

4. E

5. C

389
BY DR. MOHAMMED ATIAA KAREEM ALNASHY
-------------------------------------------------------------------
CHAPTER ONE HUNDRED AND THIRTY-THREE: CHRONIC OTITIS
MEDIA, MASTOIDITIS, AND PETROSITIS
…………………………………………………………………………………………………………………….

1. Which of the following theories on the pathogenesis of acquired aural


cholesteatoma does not exist?
A. Invagination of the tympanic membrane
B. Transdifferentiation
C. Basal cell hyperplasia
D. Epithelial ingrowth through a perforation
E. Squamous metaplasia of middle ear epithelium

2. Which following statements regarding infectious and noninfectious


complications of otitis media is true?
A. Complications are usually seen as a bullous myringitis.
B. They are characterized by a perforation of the lateral squamous layer.
C. Chronic perforation of the tympanic membrane, ossicular erosion, labyrinthine
erosion, and tympanosclerosis are major causes of hearing loss.
D. On physical examination there is usually a Schwartze's sign seen on the
tympanic membrane.
E. Chronic otitis media is a frequent sequela to cerumen impaction.

3. The diagnosis of petrous apicitis is suspected by


A. Scintigraphy
B. Plain x-ray
C. Surgical exploration
D. Clinical grounds and computed tomography
E. Tympanometry

4. Tympanosclerosis is associated with


A. Atherosclerosis of the internal carotid artery
B. Necrosis of the tympanic membrane
C. Cholesteatoma
D. History of otosclerosis
E. Recurrent bouts of acute otitis media

5. It has been observed that patients with a history of chronic otitis media
with effusion have
A. More sclerotic mastoids with decreased pneumatization compared with healthy subjects
B. Less sclerotic mastoids with decreased pneumatization compared with healthy subjects
C. More sclerotic mastoids with increased pneumatization compared with healthy subjects
D. Less sclerotic mastoids with increased pneumatization compared with healthy subjects.
E. More sclerotic mastoids with absent pneumatization compared with healthy subjects

390
BY DR. MOHAMMED ATIAA KAREEM ALNASHY
-------------------------------------------------------------------
CHAPTER 133
Chronic Otitis Media, Mastoiditis, and Petrositis

1B
There are four basic theories of the pathogenesis of acquired aural
cholesteatoma: (1) invagination of the tympanic membrane (retraction pocket
cholesteatoma); (2) basal cell hyperplasia; (3) epithelial ingrowth through a
perforation (the migration theory); and (4) squamous metaplasia of middle ear
epithelium. Transdifferentiation means converting one sort of cell into another
and has not been shown for acquired cholesteatoma.

2. C
The infectious and noninfectious complications of otitis media may result in
significant morbidity and complications, including acute and chronic mastoiditis,
petrositis, and intracranial infection. The noninfectious sequelae, including
chronic perforation of the tympanic membrane, ossicular erosion, labyrinthine
erosion, and tympanosclerosis, are major causes of hearing loss.

3D
Symptoms of petrositis usually are subtle. Typically, a patient who has had
previous mastoid surgery will complain of persistent infection and deep facial pain.
The diagnosis of petrous apicitis is suspected on clinical grounds, the most
appropriate diagnostic procedure is CT. High-resolution CT scanning usually
shows details of the petrous apex and provides important detail about potential
surgical routes.

4. E
Tympanosclerosis is a consequence of resolved otitis media or trauma and was often
seen after recurrent bouts of acute otitis media. There is no relation to otosclerosis,
but it may be present in cholesteatoma but is not associated with it.

5. A
It has been observed that patients with a history of chronic OME have more
sclerotic mastoids with decreased pneumatization than healthy subjects. Two
suggestions
have been made to explain this observation: the hereditary theory, which states that
children with hypoaeration of the mastoid are prone to OME, and the
environmental theory, which states that chronic OME results in hypop-
neumatization of the mastoid.

391
BY DR. MOHAMMED ATIAA KAREEM ALNASHY
-------------------------------------------------------------------
CHAPTER ONE HUNDRED AND THIRTY-FOUR: - COMPLICATIONS
OF TEMPORAL BONE INFECTIONS
……………………………………………………………………………………………………………………

1. A previously healthy 15-month-old girl is seen with a 1-day history of


high fevers, irritability, and left-sided facial paralysis. On
examination, you see a bulging tympanic membrane (TM) and diagnose
acute otitis media (AOM). What is causing the facial paralysis?
A. Acquired cholesteatoma erosion of the fallopian canal
B. Congenital dehiscent fallopian canal
C. Encephalitis from intracranial extension of infection
D. Viral infection causing Bell's palsy and edema of eustachian tube
mucosa with subsequent bacterial AOM

2. In performing a mastoidectomy for chronic otitis media complications,


the surgeon would benefit by
A. Directing the mastoidectomy directly toward the area of complication
first, then completing the remainder of the mastoidectomy
B. Using a diamond bur instead of a cutting bur
C. Always performing a canal-wall-down mastoidectomy
D. Avoid drilling too close to the tegmen, because this bone is extremely
thin and friable from the chronic infection

3. All of the following are characteristic of coalescent mastoiditis except


A. It usually evolves from acute O M and mastoiditis that persists for 2 to 4 weeks.
B. It is more common in children younger than 4 years old.
C. It tends to occur in patients with well-developed mastoid air cell systems.
D. It occurs in children who tend to have recurrent ear problems.

4. Masked mastoiditis
A. Is often associated with a sterile, serous otorrhea caused by the usual
history of multiple antibiotic therapies
B. Can most often be treated medically
C. Often occurs in patients who have not yet had antibiotic therapy
D. Is a disease entity in which patients experience chronic but not severe
postauricular pain after multiple courses of antibiotics
E. Is most often associated with a retracted or perforated TM

5. All of the following describe labyrinthine fistulas except


A. Most fistulas involve the lateral semicircular canal
B. The damaging effects are most often a result of a cholesteatoma
C. Patients experience whirling vertigo, diaphoresis, and extreme nausea
D. Principally diagnosed with history and fistula test

392
BY DR. MOHAMMED ATIAA KAREEM ALNASHY
-------------------------------------------------------------------
CHAPTER 134
Complications of Temporal Bone Infections

1B

2B

3D

4D

5D

393
BY DR. MOHAMMED ATIAA KAREEM ALNASHY
-------------------------------------------------------------------
CHAPTER ONE HUNDRED AND THIRTY-FIVE: INFECTIONS OF
THE LABYRINTH
……………………………………………………………………………………………………………….....

1. The most important common cause of congenital hearing loss in the United States
A. Treponema pallidum (congenital syphilis)
B. Rubella virus
C. Rubeola virus (measles)
D. Cytomegalovirus
E. Mumps virus

2. Congenital cytomegalovirus most commonly causes


A. Bilateral deafness
B. Unilateral deafness
C. Asymptomatic infection
D. Bilateral deafness and balance problems
E. Blindness but not hearing loss

3. The pathology of hearing loss from congenital viral infections occurs


primarily in the
A. Perilymphatic structures
B. Endolymphatic structures
C. Spiral ganglia
D. Vestibular ganglia
E. All of the above

4. Hearing loss from congenital rubella is most commonly


A. Bilateral involving all frequencies
B. Bilateral involving low frequencies
C. Bilateral involving high frequencies
D. Unilateral involving all frequencies
E. Unilateral involving low frequencies

5. Hearing loss from mumps is most commonly


A. Bilateral and congenital
B. Unilateral and congenital
C. Bilateral and acquired in childhood
D. Unilateral and acquired in childhood
E. Unilateral and acquired in adulthood

394
BY DR. MOHAMMED ATIAA KAREEM ALNASHY
-------------------------------------------------------------------
CHAPTER 135
Infections of the Labyrinth

1D
Since the introduction of the rubella vaccine, most congenital hearing loss in
developed countries occurs from cytomegalovirus. In most cases, the maternal
cytomegalovirus infection occurs in nonimmune women early in the pregnancy.
The maternal infection is usually asymptomatic. However, occasional cases of
congenital cytomegalovirus infection occur in immune mothers who apparently
developed a recurrent asymptomatic viremia from a latent infection. Treponema
pallidum (congenital syphilis) and rubella virus can cause occasional cases of
congenital hearing loss in the United States. Rubeola and mumps viruses cause
acquired hearing loss.

2. C
More than 99% of congenital cytomegalovirus infections are asymptomatic, with
virus detected in the infant's urine at birth. The virus disappears from the urine
over several months. However, occasional asymptomatically infected infants
subsequently have bilateral or unilateral hearing loss develop during the first
decade of life. The pathogenesis of this delayed hearing loss is unclear.

3. B
Most viruses that cause congenital hearing loss produce cochlear damage
involving endolymphatic structures. The specific endolymphatic structures
damaged depend on the virus. For example, cytomegalovirus tends to infect
cells in the stria vascularis and Reissner's membrane.

4. A
Congenital rubella remains a serious cause of congenital hearing loss in
developing countries that do not administer the rubella vaccine. Most hearing
loss in congenitally infected infants is bilateral and involves all frequencies.
In more than half the infants, the hearing loss is profound and permanent.

5D
Hearing loss from mumps is acquired during childhood mumps and is usually
unilateral. Hearing loss from mumps continues to represent a common cause
of acquired deafness in developing countries that do not administer the
mumps vaccine. The hearing loss usually develops toward the end of an
uncomplicated parotitis and is often profound.

395
BY DR. MOHAMMED ATIAA KAREEM ALNASHY
-------------------------------------------------------------------
CHAPTER ONE HUNDRED AND THIRTY-SIX: TYMPANOPLASTY
AND OSSICULOPLASTY
…………………………………………………………………………………………………………………….

1. Which of the following graft materials has been shown to have similar
success rates with temporalis fascia, but with significantly less surgical
time and without external incision?
A. Perichondrium
B. Vein
C. Alloderm
D. Autologous fat
E. Cartilage

2. In a lateral graft tympanoplasty, it is very important to do all of the


following except
A. Completely remove all of the squamous epithelium from the lateral
surface of the tympanic membrane (TM)
B. Be certain to avoid overlay of the graft onto the posterior canal wall
C. Position the graft to have the malleus lateral to the graft
D. Maintain the anterior tympanomeatal angle less than 90 degrees

3. When using a medial graft technique for repair of a total TM


perforation, the most common area for graft failure is
A. Anterosuperior
B. Anteroinferior
C. Posterosuperior
D. Posteroinferior
E. All areas with same risk

4. Bony fixation of the ossicles from infection is most commonly seen at


A. Stapes footplate
B. Incudostapedial joint
C. Head of the malleus
D. Incus in the attic

5. When desiring to place a partial ossicular reconstruction prosthesis


(PORP) in an ear that has a chronically retracted umbo, one can
A. Excise the malleus and stapes superstructure and proceed with a total
ossicular reconstruction prosthesis (TORP) placement
B. Conduct a cochleostomy closer to the umbo and place a TORP to achieve a
more favorable prosthesis position
C. Sever the tensor tympani tendon
D. Trim the length of the PORP and insert into the ear as is

396
BY DR. MOHAMMED ATIAA KAREEM ALNASHY
-------------------------------------------------------------------
CHAPTER 136
Tympanoplasty and Ossiculoplasty

1. C
2. B
3. A
4. C
5. C

397
BY DR. MOHAMMED ATIAA KAREEM ALNASHY
-------------------------------------------------------------------
CHAPTER ONE HUNDRED AND THIRTY-SEVEN: MASTOIDECTOMY
…………………………………………………………………………………………………………………….

1. Proper placement of the C-shaped incision used in simple mastoidectomy is


A. 1 cm anterior to the postauricular crease
B. 1 cm posterior to the postauricular crease
C. Directly in the postauricular crease
D. 3 cm posterior to the postauricular crease
E. 5 cm posterior to the postauricular crease

2. Which structure can be found along the floor of the antrum?


A. Superior semicircular canal
B. Horizontal semicircular canal
C. Posterior semicircular canal
D. Spine of Henle
E. Temporal line

3. Which of the following is best describes the facial recess?


A. The space between the incus and malleus
B. The space between the tympanic membrane and the chorda tympani
C. The junction of the middle fossa dura and the sigmoid sinus
D. The space between the facial nerve and the chorda tympani
E. A brief pause made by the surgeon to prepare for delicate surgery
around the facial nerve

4. Which of the following are advantages of the intact canal wall mastoidectomy?
A. More rapid healing postoperatively
B. Preservation of a self-cleaning ear
C. In-the-canal hearing aids are well tolerated.
D. No limitations on water activities
E. All of these

5. During mastoidectomy, the sigmoid sinus is injured. The bleeding is


ultimately controlled, and the procedure is completed. After
awakening, the patient reports visual changes and persistent headaches.
Which of the following is not indicated?
A. Magnetic resonance imaging
B. Magnetic resonance venography
C. Ophthalmology consultation
D. Immediate surgical exploration
E. All of these are indicated.

398
BY DR. MOHAMMED ATIAA KAREEM ALNASHY
-------------------------------------------------------------------
CHAPTER 137
Mastoidectomy

1B

2B

3D

4. E

5D

399
BY DR. MOHAMMED ATIAA KAREEM ALNASHY
-------------------------------------------------------------------

PART TWELVE
……………………………………………………………………………………………………………………
……………………………………………………………………………………………………………………

VESTIBULAR SYSTEM

400
BY DR. MOHAMMED ATIAA KAREEM ALNASHY
-------------------------------------------------------------------
CHAPTER ONE HUNDRED AND THIRTY-EIGHT: - ANATOMY OF
VESTIBULAR END ORGANS AND NEURAL PATHWAYS
…………………………………………………………………………………………………………………….

1. Which of the following statements regarding the efferent innervation


to the vestibular periphery is true?
A. Vestibular efferents originate in a small nucleus lateral to the facial genu.
B. Some of these 200 neurons project both ipsilaterally and contralaterally.
C. They make contact with both hair cells and afferents.
D. They are joined by cochlear efferents arising from the lateral superior
olivary nucleus.
E. All of the above

2. Which of the following statements regarding development of the


human vestibular system is false?
A. The otic placode develops at the seven-somite stage.
B. Hair cells differentiate before afferent nerve fibers arrive at the sensory epithelium.
C. Neural crest migrates at approximately 4 weeks to form the acousticofacial ganglion.
D. Semicircular canals form from three diverticula whose centers break down at
approximately 35 days.
E. The organ of Corti is the last portion of the labyrinth to reach adult form at
approximately 25 weeks.

3. Which of the following statements regarding inner ear fluids is true?


A. Endolymph resembles extracellular fluid by having high Na+ and low K+
content.
B. Perilymph is high in amino acids, especially glycine, compared with blood.
C. Vestibular endolymph is produced by cochlear stria vascularis cells that are
mitochondria poor.
D. Endolymph is resorbed in the endolymphatic sac.
E. Perilymph leaves the inner ear by drainage through the eustachian tube.

4. Which of the following depends on the integrity of the visual system?


A. Vestibuloocular reflex
B. Vestibulo colic reflex
C. Optokinetic reflex
D. Vestibulospinal reflex
E. None of the above

5. The vestibular nuclei do not project to which of the following targets?


A. Vestibular labyrinth
B. Thalamus and cortex
C. Cerebellum and precerebellar nuclei
D. Extraocular motor nuclei and nucleus prepositus
E. Reticular nuclei

401
BY DR. MOHAMMED ATIAA KAREEM ALNASHY
-------------------------------------------------------------------
CHAPTER 138:
Anatomy of Vestibular End Organs and Neural Pathways

1. E: All of the above. Answers a-d are all correct.

2 B: Afferent fibers arrive at the sensory epithelium before hair cells


differentiate. The remaining answers are all correct.

3 D: Endolymph is resorbed in the endolymphatic sac. It is produced by so-


called vestibular "dark" cells, which are mitochondria-rich, and it resembles
extracellular fluid by having low Na+ and high K+ content. Perilymph is low
in amino acids, especially glycine, compared with blood, and it leaves the
inner ear by drainage through venules and through the middle ear mucosa.

4. C: The optokinetic reflex depends on the integrity of the visual system,


because it relies on visual input from moving targets as a stimulus to move
the eyes. The other reflexes do not rely on visual input; in fact, they can
function in complete darkness.

5. A: The vestibular nuclei do not project to the labyrinth itself. There are
efferent vestibular neurons, which contain acetylcholine and other
transmitters and neuromodulators, that project to the labyrinth. The
vestibular nuclei project to all the remaining structures.

402
BY DR. MOHAMMED ATIAA KAREEM ALNASHY
-------------------------------------------------------------------
CHAPTER ONE HUNDRED AND THIRTY-NINE: PRINCIPLES OF
APPLIED VESTIBULAR PHYSIOLOGY
--------------------------------------------------------------------------------------------

1. Ewald's first law is most accurately represented by which statement?


A. Stimulation of a semicircular canal produces eye movements in the
plane of that canal.
B. Stimulation of a semicircular canal produces eye movements in a plane
orthogonal to that canal.
C. Stimulation of a semicircular canal produces eye movements that are
unpredictable.
D. Stimulation of a semicircular canal does not produce eye movements
under normal conditions.
E. Stimulation of a semicircular canal always produces horizontal eye
movements.

2. If the head is pitched nose up while rolling it toward the right in a


plane 45 degrees off the midsagittal plane, which semicircular canal is
most likely to be excited?
A. Right anterior canal
B. Right posterior canal
C. Left anterior canal
D. Left posterior canal
E. Left horizontal canal

3. A young woman complains that exposure of the left ear to loud sound
"makes the world flutter up and down." She most likely has
A. Left horizontal canal dehiscence
B. Right horizontal canal dehiscence
C. Left superior canal dehiscence
D. Right superior canal dehiscence
E. Left posterior canal dehiscence

403
BY DR. MOHAMMED ATIAA KAREEM ALNASHY
-------------------------------------------------------------------
4. A patient with unilateral vestibular impairment has left-beating
nystagmus (fast phase to the left) following the head-shake nystagmus
test. This most likely suggests a defect in which location?
A. Left vestibular apparatus
B. Right vestibular apparatus
C. Bilateral vestibular disease
D. Central nervous system
E. None of these are correct

5. Isolated loss of utricular nerve activity may result in which of the


following findings?
A. Head tilt toward the lesioned side
B. Disconjugate deviation eyes such that one pupil is elevated and one is depressed
C. A static conjugate counter-roll of the eyes rolling the superior pole of each
away from the intact utricle
D. None of these findings
E. All of these findings

CHAPTER 139
Principles of Applied Vestibular Physiology

1. A

2B

3. C

4B

5. E

404
BY DR. MOHAMMED ATIAA KAREEM ALNASHY
-------------------------------------------------------------------
CHAPTER ONE HUNDRED AND FORTY: EVALUATION OF THE
PATIENT WITH DIZZINESS
--------------------------------------------------------------------------------------------

1. Why does nystagmus change its direction after head shaking?


A. Nystagmus is generally unstable and can change direction without warning.
B. The neural integrator only has an effect for a limited period of time.
C. Position of the head can cause nystagmus to change direction.
D. Nystagmus never changes its direction after head shaking.
E. Adaptation of the vestibular pathways

2. After unilateral deafferentation, which is not true?


A. Patients tend to exhibit a head tilt to the weakened side.
B. Patients tend to exhibit ocular counter-rolling.
C. The effect of the velocity storage mechanism is prolonged.
D. Patients exhibit a combined horizontal/torsional nystagmus.
E. Patients have increased nystagmus in the dark.

3. Appropriate uses of posturography include which of the following


(choose all that apply)?
A. Detection of malingering patients
B. Determining duration of gentamicin therapy for Meniere's disease
C. Screening for most patients complaining of dizziness
D. Confirming a diagnosis of perilymphatic fistula
E. Following a course of physical therapy

4. Stereotypic eye movements for the most common form of benign positional
vertigo with the patient in the Dix-Hallpike position include
A. Downbeat vertical nystagmus with fast torsional movements toward the lower ear
B. Upbeat vertical nystagmus with fast torsional movements toward the lower ear
C. Downbeat vertical nystagmus with fast torsional movements toward the upper ear
D. Upbeat vertical nystagmus with fast torsional movements toward the upper ear
E. Horizontal nystagmus alone, with the direction depending on the cause

5. What is the mechanism of caloric stimulation of the labyrinth?


A. Heating of the horizontal canal
B. Heating of the anterior canal
C. Direct stimulation of vestibular afferents
D. Heating of the horizontal canal and direct stimulation of afferents
E. None of the above

405
BY DR. MOHAMMED ATIAA KAREEM ALNASHY
-------------------------------------------------------------------
CHAPTER 140
Evaluation of the Patient with Dizziness

1. E
Some forms of nystagmus are "unstable" and may oscillate, such as periodic
alternating nystagmus, but this is not the situation here. The neural
integrator is ineffective after a vestibular lesion but does not cause a reversal
of nystagmus. Tilting of the head can "dump" the velocity storage mechanism
and cause nystagmus to cease early, but the reason nystagmus automatically
changes its direction after head-shaking is because of adaptation of the
vestibular system. Subject to high levels of stimulation, the system gradually
adapts. This effects of this adaptation last longer than those from the initial
stimulus, causing an undershoot.

2. C
The effect of velocity storage is shortened after deafferentation. The
remainder of the answers are true.

3. A, E
There is no proven role for posturography in evaluating treatment with
gentamicin. It is a very inefficient modality in screening patients in general.
It is not an effective diagnostic tool for perilymphatic fistula, which is better
diagnosed by pressure changes in the inner ear and middle ear exploration.

4B
The most common cause of benign paroxysmal vertigo is debris in the
posterior canal. Canalithiasis and cupulolithiasis of the horizontal canal are
known and can cause the symptoms of choice e, but this is much less common.
Downbeat vertical nystagmus with a torsional component is typical of
superior semicircular canal dehiscence syndrome.

5D
Studies in environments where convection cannot occur because of lack of
gravity (such as in space) have shown that the caloric response still exists,
although somewhat attenuated. A direct effect of heating on the vestibular
apparatus, causing stimulation, is therefore understood to exist.

406
BY DR. MOHAMMED ATIAA KAREEM ALNASHY
-------------------------------------------------------------------
CHAPTER ONE HUNDRED AND FORTY-ONE: IMBALANCE AND
FALLS IN THE ELDERLY
--------------------------------------------------------------------------------------------

1. In which room of the home do elderly individuals fall the most often?
A. Bathroom
B. Kitchen
C. Bedroom
D. Living room
E. Utility room

2. What is the most primary body balance strategy that is frequently lost
first in aging patients who fall?
A. Ankle strategy
B. Head strategy
C. Hip strategy
D. Shoulder strategy
E. Stepping strategy

3. What is the most common balance complaint of elderly patients with


presbyastasis when seeing a physician?
A. Vertigo
B. Disequilibrium
C. Rapid heartbeat
D. Headache
E. Vision loss

4. Which of these senses does not play a role in balance in the elderly?
A. Vision
B. Vestibular
C. Proprioceptive
D. Auditory
E. Gustatory

407
BY DR. MOHAMMED ATIAA KAREEM ALNASHY
-------------------------------------------------------------------
5. Which of these tests is the least useful in assessing balance and/or gait
in an aging patient?
A. Clinical Test of Sensory Integration in Balance (CTSIB)
B. Fast Evaluation of Mobility, Balance and Fear (FEMBF)
C. Activity-Specific Balance Confidence Test (ABC)
D. Auditory Brainstem Response (ABR)
E. Modified Falls Efficiency Scale (MFES)

6. Which test in the ENG battery is the most sensitive to age related changes?
A. Optokinetic
B. Caloric
C. Pursuit
D. Head autorotation
E. Positional/positioning

408
BY DR. MOHAMMED ATIAA KAREEM ALNASHY
-------------------------------------------------------------------
CHAPTER 141
Imbalance and Falls in the Elderly

1. C
More than one third of all elderly falls in the home occur in the bedroom,
usually getting into or out of bed.

2. A
A result of aging is loss of lower body strategies, particular ankle strategies,
to compensate for sway. Instead, stepping strategies are used and because of
the amount of time to relocate the stepping foot, falls frequently ensue after
mild balance perturbations.

3B
Elderly patients most commonly are seen with complaints of disequilibrium
or loss of balance. True vertigo is primarily a result of a unilateral vestibular
loss, which is not usually an aging problem.

4. E
Gustatory sensations play no role in balance, whereas the other four listed do.

5D
Although an ABR test gives important information on auditory function in
the brainstem, it is not useful for assessing balance and/or gait.

6. C
The pursuit (tracking) test is the ENG test most susceptible to abnormalities
caused by age-related processes.

409
BY DR. MOHAMMED ATIAA KAREEM ALNASHY
-------------------------------------------------------------------
CHAPTER ONE HUNDRED AND FORTY-TWO: MENIERE'S
DISEASE AND OTHER PERIPHERAL VESTIBULAR DISORDERS
--------------------------------------------------------------------------------------------

1. With regard to transtympanic administration of gentamicin in the


treatment of Meniere's disease, which of the following is most accurate?
A. Hearing is always preserved.
B. Complete ablation of vestibular function is not always required for
control of symptoms.
C. Single transtympanic injections of gentamicin rarely result in
demonstrable vestibular changes.
D. The preferred administration technique is direct injection into the
peri/endolymph by way of the horizontal canal.
E. Injections should always be done before instituting any medical
management, because the medication use will not allow the clinician to
monitor the response to gentamicin.

2. Which is the test of choice for the detection of otologic syphilis?


A. Venereal Disease Research Laboratory (VDRL)
B. Rapid plasma reagin
C. Fluorescent treponemal antibody
D. Lumbar puncture
E. Pathogen isolation from a perilymph sample

3. Treatment of patients with suspected inner ear fistulae should consist


of the following except
A. Bed rest
B. Head elevation
C. Laxatives
D. Transtympanic steroid infusion
E. Monitoring of hearing and vestibular function

410
BY DR. MOHAMMED ATIAA KAREEM ALNASHY
-------------------------------------------------------------------
4. Which of the following statements is most likely to be true?
A. Positive middle ear pressure may cause nystagmus toward the ear with
the higher middle ear pressure.
B. Positive middle ear pressure may cause nystagmus toward the ear with
the lower middle ear pressure.
C. Positive middle ear pressure in one ear may cause nystagmus toward
either ear.
D. Positive middle ear pressure will most likely cause vertical, up-beating
nystagmus.
E. None of these are correct.

5. Which of these should be avoided in treatment of atmospheric inner


ear barotrauma?
A. Bed rest
B. Head elevation
C. Surgical exploration for selected cases with progressive symptoms
within 3 to 5 days
D. Recompression
E. Close monitoring of hearing and balance

CHAPTER 142
Meniere's Disease and Other Peripheral Vestibular Disorders

1B

2. C

3D

4. A

5D

411
BY DR. MOHAMMED ATIAA KAREEM ALNASHY
-------------------------------------------------------------------
CHAPTER ONE HUNDRED AND FORTY-THREE: CENTRAL
VESTIBULAR DISORDERS
--------------------------------------------------------------------------------------------

1. For a patient in an attack of acute vertigo, which of the following is


most suggestive of a central rather than peripheral cause?
A. Mixed horizontal-torsional nystagmus.
B. Nystagmus that increases with removal of visual fixation.
C. Unidirectional nystagmus that increases with rightward gaze and
decreases with leftward gaze.
D. Absence of a head thrust sign.

2. An elderly man awoke with vertigo and vomiting and is being evaluated
in the emergency department. Examination reveals direction-changing
nystagmus and profound gait imbalance. After determining that the
patient is otherwise stable, the next step in management is to
A. Administer vestibular suppressants and admit the patient for
observation
B. Begin a course of oral steroids and acyclovir and schedule outpatient
follow-up in 1 week
C. Prescribe vestibular suppressants and arrange for outpatient vestibular
rehabilitation
D. Obtain an emergent head computed tomography (CT) or magnetic
resonance imaging (MRI)
E. Perform a lumbar puncture and obtain an MR angiogram.

3. You are evaluating a 42-year-old woman with recurrent spontaneous


attacks of vertigo for the past 4 years that generally last several hours,
are associated with light sensitivity, improve with sleep, and
occasionally are associated with aural fullness or headache. There is a
history of motion sickness. Physical examination, audiogram, and
vestibular testing are normal. The most likely diagnosis is
A. Benign paroxysmal positioning vertigo (BPPV)
B. Vestibular migraine
C. Meniere's disease
D. Multiple sclerosis
E. Transient ischemic attacks

412
BY DR. MOHAMMED ATIAA KAREEM ALNASHY
-------------------------------------------------------------------
4. A 56-year-old man is being evaluated for progressive hearing loss and
imbalance over the past 3 years without any similar family history.
Examination reveals severe bilateral sensorineural hearing loss,
downbeat and gaze-evoked nystagmus, anosmia, and gait ataxia.
Taking an extensive history reveals no other diagnostic clues. To arrive
at the correct diagnosis, you should next
A. Perform an MRI of the head looking for iron accumulation around the
brainstem and cerebellum
B. Perform an MRI of the head with gadolinium looking for enhancement
along the eighth nerves
C. Perform brainstem auditory evoked responses (BAERs) looking for
slowing along the conduction pathways
D. Perform a CT scan of the head with contrast looking for a posterior
fossa tumor
E. Perform genetic testing for spinocerebellar ataxias

5. A 45-year-old pilot is seen with recurrent attacks of vertigo associated


with aural fullness and fluctuating unilateral low-frequency hearing
loss. He has been forced to stop working and now rarely leaves the
house, has lost interest in his hobbies, has difficulty sleeping, and has
had some crying spells. Appropriate management is to
A. Place him on a salt-restricted diet and start a diuretic
B. Perform intratympanic gentamicin therapy
C. Begin a migraine prophylactic medication
D. Refer him for psychotherapy
E. Aggressively treat both his Meniere's disease and his depression

413
BY DR. MOHAMMED ATIAA KAREEM ALNASHY
-------------------------------------------------------------------
CHAPTER 143
Central Vestibular Disorders

1D
A patient with acute vertigo from a peripheral cause will usually have a
positive head thrust sign (i.e., a rapid 15-degree passive rotation of the head
in one direction while the patient attempts to fixate on the examiners nose
elicits a corrective saccade that brings the gaze back to the fixation target).
This occurs because the slow phase of the high-frequency VOR is inadequate
to maintain fixation during head rotation and implies a unilateral loss of
horizontal semicircular canal function. A mixed horizontal-torsional
nystagmus is characteristic of an acute unilateral peripheral labyrinthine
lesion affecting all three semicircular canals, in which pure vertical or
torsional nystagmus would be more likely central. Peripheral nystagmus
characteristically increases with removal of visual fixation through Frenzel
lines or occlusive ophthalmoscopy. Peripheral nystagmus is also unilateral
and increases with gaze in the direction of the quick phases and decreases
with gaze in the direction of the slow phases (Alexander's law).

2. D
Cerebellar infarction or hemorrhage should be suspected in any patient with
acute vertigo when vascular risk factors (including age) are present. In one
study, as many as one-fourth of older patients seen in the emergency
department with acute isolated vertigo had cerebellar infarction. Central
ocular motor signs such as direction-changing or downbeat nystagmus and
profound gait imbalance increase this likelihood. When the brainstem is
affected, other cranial nerve symptoms and signs are generally present.
Patients should undergo emergent head imaging with non-contrast CT or
MRI, because an expanding cerebellar hematoma or swelling from a
cerebellar infarction can cause brainstem compression and death without
neurosurgical intervention.

414
BY DR. MOHAMMED ATIAA KAREEM ALNASHY
-------------------------------------------------------------------
3B
Vestibular migraine may be the most common cause of recurrent spontaneous
(nonpositional) vertigo among young and middle-aged people and is more
common is women. Often there is a remote or current history of headaches
that may occur during or independently from attacks of vertigo. Other
migrainous and even aural symptoms may accompany attacks. A history of
motion sensitivity is common among migraineurs, and a family history of
migraines (possibly never diagnosed) is typical. Distinguishing from
Meniere's disease can sometimes be difficult, but examination, audiogram,
and vestibular testing are generally normal, and diagnosis is based largely on
the appropriate history.

4. A
Superficial siderosis is a rare condition characterized by progressive
sensorineural deafness and ataxia and is due to hemosiderin deposition along
the leptomeninges, cranial nerves, subpial tissues, and spinal cord. It is caused
by recurrent subarachnoid bleeding, usually from an occult vascular
malformation, aneurysm, tumor, or previous intracranial surgery. The eighth
nerve, cerebellum, and olfactory bulb are particularly susceptible. T2-
weighted MRI scans reveal a margin of hypointensity surrounding the
brainstem and cerebellum reflecting the paramagnetic hemosiderin
deposition.

5. E
Because of the often disabling symptoms, patients with vestibular disorders
frequently become anxious or depressed. Dizziness has a more deleterious
effect when it is associated with a psychiatric disorder such as depression, and
treatment of the vestibular disorder may lead to a suboptimal response unless
the psychiatric disorder is also addressed. Thus, vestibular and psychiatric
disorders should be aggressively managed together. Depending on the
situation, this can be accomplished by explanation and reassurance to the
patient, medication management by the otolaryngologist, or referral to a
psychiatrist.

415
BY DR. MOHAMMED ATIAA KAREEM ALNASHY
-------------------------------------------------------------------
CHAPTER ONE HUNDRED AND FORTY-FOUR: SURGERY FOR
VESTIBULAR DISORDERS
-------------------------------------------------------------------------------------------

1. Which of the following statements about surgery for benign


paroxysmal positional vertigo is not true?
A. Singular neurectomy is only appropriate for posterior semicircular
canal disease.
B. Singular neurectomy is not effective if the condition is due to
cupulolithiasis as opposed to canalithiasis.
C. Posterior semicircular canal occlusion has widely replaced singular
neurectomy, because it is technically simpler and safer.
D. Surgery for BPPV is rarely required, because particle-repositioning
maneuvers are effective in most cases.
E. Although generally safe, posterior semicircular canal occlusion may
result in sensorineural hearing loss in the operated ear.

2. Any of the following options could be undertaken in an effort to relieve


ongoing episodic spells of vertigo after a retrolabyrinthine vestibular
neurectomy except
A. Middle fossa vestibular neurectomy
B. Transmastoid labyrinthectomy
C. Transcanal labyrinthectomy
D. Vestibular rehabilitation
E. Intratympanic gentamicin injections

3. When attempting to identify the offending ear that is causing episodic


vertigo of peripheral origin, which is the most reliable clinical
symptom or finding?
A. Unilateral tinnitus
B. Subjective aural fullness
C. A unilateral weakness of caloric response on vestibular testing
D. Asymmetric rotary chair responses
E. A long-standing profound sensorineural hearing loss in one ear

416
BY DR. MOHAMMED ATIAA KAREEM ALNASHY
-------------------------------------------------------------------
4. Which of these clinical scenarios is not compatible with a fluctuating
peripheral vestibular disorder and thus better treated with vestibular
rehabilitation rather than surgery?
A. Persistent disabling motion-provoked vertigo and chronic disequilibrium
after a significant vestibular crisis
B. Episodic spells of spontaneous vertigo associated with subjectively
fluctuating hearing loss, tinnitus, and fullness in one ear
C. Episodic spells of spontaneous vertigo in a patient who has normal caloric
responses and a profound hearing loss after a failed stapedectomy 10 years
ago
D. Intermittent disequilibrium, aural fullness, and a mixed hearing loss after
penetrating trauma to one ear
E. Persistent positional vertigo unresponsive to particle repositioning
maneuvers

5. When performing an endolymphatic sac operation, the surgeon should not


A. Counsel the patient that further procedures may be required to control
the vertigo
B. Decompress the sigmoid sinus
C. Violate the medial wall of the endolymphatic sac
D. Close the wound unless absolutely certain that the sac is properly
identified and fully decompressed
E. Skeletonize the facial nerve

417
BY DR. MOHAMMED ATIAA KAREEM ALNASHY
-------------------------------------------------------------------
CHAPTER 144
Surgery for Vestibular Disorders

1B
Singular neurectomy selectively sections the afferent innervation of the
posterior semicircular canal and, thus, is effective and appropriate for any
disorder afflicting that canal. Nevertheless, it is true that this procedure has
been largely supplanted by the technically simpler canal occlusion procedure.
Both operations are associated with the risk of hearing loss, although this
complication is quite rare with canal occlusion procedures. Particle
repositioning and other vestibular rehabilitation techniques are very
effective, and surgery of any kind is rarely required in this condition.

2D
If the dizziness is truly characterized as episodic spells of vertigo after any
posterior fossa vestibular neurectomy procedure, one must assume that the
vestibular division of the eighth cranial nerve has not
been fully transected. As a result, afferent information from the diseased
labyrinth is still being transmitted to the vestibular nucleus on the involved
side. Destruction of the vestibular end organ by any technique would be
expected to control ongoing vertigo, recognizing that either of the
labyrinthectomy procedures would result in complete hearing loss on the
involved side. If the hearing remained excellent, intratympanic gentamicin
injections would be the most conservative approach. Alternatively,
performing a more selective vestibular neurectomy distally in the internal
canal by the middle fossa approach could be considered. Vestibular
rehabilitation, although desirable for treating disequilibrium and/or motion-
provoked vertigo after any ablative vestibular operation, would not be
expected to provide any benefit in the setting described in this question.

3. E
The presence of a sensorineural hearing loss is a highly reliable indicator of
peripheral pathology, provided that the opposite ear has completely stable
hearing and no auditory symptoms. The patient with profound deafness and
episodic vertigo is the ideal candidate for labyrinthectomy. A unilateral
weakness of vestibular function may be confirmatory but is not absolute in
localizing the unhealthy ear in the absence of hearing loss. Likewise, the

418
BY DR. MOHAMMED ATIAA KAREEM ALNASHY
-------------------------------------------------------------------
presence of normal caloric responses should not dissuade the surgeon if
hearing loss is present. Tinnitus and fullness are nonspecific symptoms and,
again, may be confirmatory but are not sufficient to securely base a surgical
decision on. Rotary chair asymmetry suggests that an uncompensated
peripheral lesion is present but is of little help in lateralizing the lesion.

4. A
The symptom complex described in the first answer is the classic presentation
of the uncompensated but stable peripheral vestibular lesion. Surgical
treatment in this setting is almost never beneficial, and the patient should be
referred for a customized program of vestibular rehabilitation. Answers b and
c suggest two common variants of endolymphatic hydrops, Meniere's disease
and delayed ipsilateral hydrops, which are appropriately treated with
surgery, assuming medical therapy for hydrops was not successful. Answer d
suggests a perilymphatic fistula with perhaps subluxation or fracture of the
stapes footplate. Although further vestibular rehabilitation (customized
habituation exercises) may be helpful in refractory BPPV, this is a
fluctuating peripheral disorder and would be responsive to surgical therapy.

5. C
In any endolymphatic sac operation, the surgeon should remember that
although the operation is generally quite safe, the efficacy is questionable,
and any beneficial effect may be nonspecific. Thus, the patient should always
be counseled that the outcome is uncertain and that more definitive treatment
may be required. Safety is paramount, and it is appropriate to decompress the
sigmoid sinus and/or skeletonize the facial nerve to safely gain access to the
region of the sac. The literature suggests that no matter how the sac is
manipulated, the outcomes are similar if not identical. Therefore, any
procedure that violates the deep wall of the sac (posterior fossa dura) will
place the patient needlessly at risk for a CSF leak or meningitis. Likewise, if
the sac is extremely difficult to access, the surgeon should remember that any
salutary effect of the sac operation may well be a nonspecific result of the
general anesthetic or drilling in the temporal bone. Thus, one could never be
criticized for backing out if there is believed to be substantial danger to
vulnerable critical structures, particularly the dura, the posterior
semicircular canal, the sigmoid sinus, or the facial nerve.

419
BY DR. MOHAMMED ATIAA KAREEM ALNASHY
-------------------------------------------------------------------
CHAPTER ONE HUNDRED AND FORTY-FIVE: Vestibular and Balance
Rehabilitation Therapy: Program Essentials
--------------------------------------------------------------------------------------------

1. One of the two principle goals of vestibular and balance rehabilitation


therapy (VBRT) is
A. Advance the central vestibular compensation process
B. Eliminate symptoms associated with head and eye movements
C. Serve as an exposure therapy technique for patients with anxiety disorders
D. Improve the control of eye movements during head movement by use of
central preprogramming
E. Increase the speed of the eye movements during head rotation

2. After a sudden, stable vestibular insult, the initial symptoms are


significantly reduced within 72 hours as a result of
A. The patient remaining still and not aggravating the injury
B. Combination of the suppressive medication and the patient at bed rest
C. The naturally occurring process of tonic rebalancing at the level of the
vestibular nuclei
D. The combined process of adaptation and habituation
E. Use of significant and early substitution exercises

3. One of the primary reasons for determining whether a patient's symptoms


occur spontaneously or are provoked by head or eye movement is
A. To assist in the determination of the medication to be used in a
suppression format
B. As the first point of decision as to whether VBRT would be likely as a
primary management technique
C. As an indication that the lesion is unstable in nature
D. Both B and C
E. As the main indicator of central nervous system involvement

420
BY DR. MOHAMMED ATIAA KAREEM ALNASHY
-------------------------------------------------------------------
4. The following statements about the techniques commonly used in
VBRT are all true except
A. Habituation and adaptation both rely on repeated head movements to
give the desired effect.
B. The outcome of a VBRT program is equally as effective with both
individually customized exercises and generic exercises as long as the
patient is active.
C. Central preprogramming plays a role in the use of substitution exercises.
D. The basic goal of adaptation exercises is to improve the functionality of
the vestibuloocular reflex.
E. Maintenance activities are important with all the patients in a VBRT
program, but this is especially true for those with cerebellar involvement.

5. Which of the following patient groups is likely to benefit from a VBRT


program, but have a limited outcome?
A. Those with stable peripheral lesions reporting head movement-provoked
symptoms
B. Patients with purely spontaneous events of vertigo lasting several hours
per event
C. Patients with classic posterior canal benign paroxysmal positional vertigo
D. Patients with bilateral vestibular involvement with the primary goal of
improvement in gait
E. Those who are post head injury with peripheral and central involvement

421
BY DR. MOHAMMED ATIAA KAREEM ALNASHY
-------------------------------------------------------------------
CHAPTER 145
Vestibular and Balance Rehabilitation Therapy: Program Essentials

1. A
In VBRT, the main overall goal is to promote the naturally occurring central
compensation process. The other responses are subgoals or techniques by
which that can be accomplished.

2. C
The static phase of the central compensation process, tonic rebalancing,
occurs at the level of the vestibular nuclei and serves to significantly reduce
symptoms of vertigo after a stable peripheral system insult. This occurs
stimulated by the significant asymmetry in neural activity recognized by the
central nervous system and does not require any other external stimulus.

3D
Spontaneously occurring symptoms of dizziness are a strong indication of an
unstable peripheral or central lesion. An unstable lesion is a major indicator
as to why central system compensation has not gone to completion. Typically,
patients with significant spontaneous events are not able to use VBRT as the
primary form of management.

4B
Double-blinded control research has shown superior results in individually
customized VBRT vs a generic form. That said, the generic form, if designed
appropriately, will stimulate improvement but not to the degree as the
customized format.

5. E
Although appropriate given symptom complaints, patients with head injury as
the cause of their vestibular injury (peripheral and central) do not as a group
achieve the same degree of success with a VBRT program as other etiologies.

422
BY DR. MOHAMMED ATIAA KAREEM ALNASHY
-------------------------------------------------------------------

PART THIRTEEN
……………………………………………………………………………………………………………………
……………………………………………………………………………………………………………………
FACIAL NERVE

423
BY DR. MOHAMMED ATIAA KAREEM ALNASHY
-------------------------------------------------------------------
CHAPTER ONE HUNDRED AND FORTY-SIX: TESTS OF FACIAL
NERVE FUNCTION
--------------------------------------------------------------------------------------------

1. A Sunderland class I injury is also called


A. Axonotmesis
B. Axonotomy
C. Neuropraxia
D. Neurotomy
E. Neurotmesis

2. A nerve that has suffered a Sunderland class II injury can still produce
muscle contraction if electrically stimulated
A. Proximal to the lesion, within 3 days of the injury
B. Distal to the lesion, within 3 days of the injury
C. Proximal to the lesion, within 24 hours of the injury
D. Distal to the lesion, within 24 hours of the injury
E. Proximal to the lesion, within 1 week of the injury

3. Which of the following statements about facial nerve testing in Bell's palsy
is true?
A. Electroneurography is useless after more than 6 weeks have elapsed.
B. Electroneurography predicts outcome better than nerve excitability testing.
C. Electromyographic fibrillation potentials are a good prognostic sign.
D. Maximum stimulation testing yields useful prognostic information in cases
of partial paralysis.
E. Patients whose nerves become totally inexcitable may still recover completely.

4. In electromyographic facial nerve monitoring in acoustic tumor


surgery, which of the following statements is not true?
A. Spontaneous "train" activity may follow thermal stimulation.
B. The lower the threshold for stimulation at the brainstem at the end of the
procedure, the better the prognosis.
C. There is professional consensus that monitoring is beneficial.
D. Monitoring cannot be performed if the anesthesiologist uses paralytic agents.
E. Stimulus artifact sounds are useful in confirming proper equipment function.

5. The topognostic test most likely to be abnormal in the early phase of


Bell's palsy is
A. Schirmer's test
B. Stapedius reflex
C. Electrogustometry
D. Salivary flow
E. Salivary pH

424
BY DR. MOHAMMED ATIAA KAREEM ALNASHY
-------------------------------------------------------------------
CHAPTER 146
Tests of Facial Nerve Function

1. C
Class I injury is also called "conduction block" or "neuropraxia." Classes II
and III are called axonotmesis and neurotmesis, respectively. "Axonotomy"
and "neurotomy" are unrelated to the Sunderland classification.

2B
No paralyzed nerve can be successfully stimulated proximal to the lesion.
Distal stimulation, in a class II-V lesion, will produce muscle contraction
only until 3 to 4 days after the onset of the injury.

3. A
All tests baced on distal electrical stimulation (including NET, MST, and
ENOG) can yield useful prognostic data in cases of Bell's palsy but only when
paralysis is total, less than a month (6 weeks at the most) has elapsed, and
until excitability is lost or recovery begins. None of these tests has been shown
to be superior to the others. When excitability has been totally lost,
incomplete recovery is certain, and fibrillation potentials seen on needle
EMG are also harbingers of incomplete recovery.

4D
Somewhat surprisingly, it has been well documented that pharmacologic
paralysis sufficient to allow the anesthesiologist to control the patient's
ventilation will still permit EMG facial nerve monitoring. All the other
statements are true.

5. C
Electrogustometry has been shown to be abnormal in virtually all cases of
Bell's palsy (even with incomplete paralysis), which makes it nearly useless
in the very early stages of this disorder. If the stapedius reflex is present in a
case of complete paralysis of the facial muscles, one should doubt the
diagnosis of Bell's palsy and should consider imaging studies to rule out a
parotid or temporal bone lesion. Tests of salivary and lacrimal function have
been suggested as prognostic tests but have failed to demonstrate added value
after clinical data and electrical tests are available.

425
BY DR. MOHAMMED ATIAA KAREEM ALNASHY
-------------------------------------------------------------------
CHAPTER ONE HUNDRED AND FORTY-SEVEN: CLINICAL
DISORDERS OF THE FACIAL NERVE
--------------------------------------------------------------------------------------------
1. The most likely pathogenesis for Bell's palsy is
A. Epstein-Barr virus
B. Autoimmune ischemic neuropathy
C. Herpes simplex virus (HSV)
D. Varicella-zoster virus
E. Heterotopic viruses

2. In the interpretation of electrical testing of the facial nerve, it is


important to perform both electroneuronography (ENoG) and
electromyography (EMG) because
A. Desynchronization of motor units can cause artifactual decrease in the
compound action potential, but voluntary motor responses are preserved.
B. Desynchronization of motor units can cause a normal compound action
potential but decreased voluntary motor responses.
C. Recruitment of motor units can cause artifactual increase in the compound
action potential, but voluntary motor responses are preserved.
D. Recruitment of motor units can cause a normal compound action potential
but decreased voluntary motor responses.
E. EMG can detect injury of the nerve proximal to the testing site, whereas
ENoG cannot.

3. The most common cause of facial paresis and paralysis in infants is


A. Hemifacial microsomia
B. Subarachnoid hemorrhage
C. Congenital absence of depressor labii muscle
D. Birth trauma
E. Mobius syndrome

4. Facial paralysis of pregnancy is associated with


A. Preterm labor
B. Low birth weight
C. Perinatal abnormalities
D. Preeclampsia
E. First trimester pregnancy

5. The defining symptom of Melkersson-Rosenthal syndrome is


A. Fissured tongue
B. Bilateral facial paralysis
C. Recurrent facial paralysis
D. Rapid onset facial swelling
E. Orofacial edema

426
BY DR. MOHAMMED ATIAA KAREEM ALNASHY
-------------------------------------------------------------------
CHAPTER 147
Clinical Disorders of the Facial Nerve

1. C
Herpes simplex virus (HSV). HSV DNA has been detected in perineural fluid
of patients with Bell's palsy, whereas VZV DNA has not been recovered from
any. Conversely, VSV DNA was recovered from all the patients with Ramsay-
Hunt syndrome, whereas none had HSV-1 DNA.

2. A
Desynchronization can cause an artifactual depression of the CAP in the
presence of voluntary motor responses on EMG. The desynchronization
causes a "spreading out" of the CAP response, so that it is not clearly seen on
EnoG. This is extremely important if a patient is being considered for
surgical decompression that both tests are abnormal.

3. D
Birth trauma. Intrauterine injury is suspected, because the incidence is equal
between forceps, vaginal, and cesarean deliveries.

4D
Preeclampsia. Maternal facial paralysis is not associated with any fetal
abnormalities. It is most common in the third trimester of pregnancy and is
increased sixfold in preeclampsia.

5E
Orofacial edema. Edema of the lips, buccal area, and sometimes the
periorbital tissues of the face is the defining symptom in Melkersson-
Rosenthal syndrome. Facial paralysis and fissured tongue occur in half of
patients and the complete triad in only one fourth.

427
BY DR. MOHAMMED ATIAA KAREEM ALNASHY
-------------------------------------------------------------------
CHAPTER ONE HUNDRED AND FORTY-EIGHT: INTRATEMPORAL
FACIAL NERVE SURGERY
--------------------------------------------------------------------------------------------
1. A 45-year-old man with a 30-dB conductive left hearing loss and
recurrent facial paralysis seems to have a mass extending from the
geniculate ganglion to the mid-stapes region. Which surgical approach
is best?
A. Canal-wall-down mastoidectomy
B. Canal-up mastoidectomy
C. Translabyrinthine
D. Middle cranial fossa
E. Retrolabyrinthine

2. What is the principal advantage of the middle cranial fossa approach


to the facial nerve compared with the retrolabyrinthine or retrosigmoid
approach?
A. Lower incidence of sensorineural hearing loss
B. Lower incidence of postoperative infection
C. Access to the labyrinthine segment without impairing hearing
D. Less operative time
E. Less brain retraction

3. The major drawback of the transmastoid approach to the facial nerve is the
A. Difficult exposure of the stylomastoid foramen region
B. Incidence of postoperative conductive hearing loss
C. Incidence of postoperative sensorineural hearing loss
D. Limited access to the geniculate ganglion
E. Limited access to the middle-ear segment

428
BY DR. MOHAMMED ATIAA KAREEM ALNASHY
-------------------------------------------------------------------
4. A 35-year-old man is kicked by a horse and has a temporal bone
fracture with direct immediate facial nerve trauma restricted to the
area of the geniculate ganglion. Three weeks later pure-tone thresholds
are 85 dB with 8% word understanding, and there is marked vestibular
paresis on the affected side. The opposite side is normal. To explore and
manage the facial nerve, what is the best approach?
A. Translabyrinthine
B. Middle cranial fossa
C. Transmastoid
D. Transotic
E. Retrolabyrinthine

5. In performing end-to-end anastomosis of the facial nerve after


resecting the area of the geniculate ganglion for direct trauma, the
greatest risk to hearing loss occurs during
A. Resection of the damaged geniculate ganglion
B. Mobilization of the tympanic segment
C. Mobilization of the labyrinthine segment
D. The anastomosis
E. The application of the tissue glue

CHAPTER 148
Intratemporal Facial Nerve Surgery

1. D

2. C

3D

4. A

5. C

429
BY DR. MOHAMMED ATIAA KAREEM ALNASHY
-------------------------------------------------------------------

PART FOURTEEN
……………………………………………………………………………………………………………………
……………………………………………………………………………………………………………………
AUDITORY SYSTEM

430
BY DR. MOHAMMED ATIAA KAREEM ALNASHY
-------------------------------------------------------------------
CHAPTER ONE HUNDRED AND FORTY-NINE: COCHLEAR
ANATOMY AND CENTRAL AUDITORY PATHWAYS
--------------------------------------------------------------------------------------------
1. Which statement is true regarding the cochlear endolymph and perilymph?
A. Perilymph is contained within the scala media and exhibits a high K+
and a low Na+ ion concentration.
B. Endolymph is contained within the scala media and exhibits a high K+
and a low Na+ ion concentration.
C. Endolymph exhibits a negative electrical potential relative to perilymph.
D. Perilymph is contained within the scala tympani and exhibits a
positive electrical potential relative to endolymph.
E. Endolymph and perilymph communicate by way of the helicotrema.

2. Which cell type of the stria vascularis exhibits convoluted basolateral


cell membranes that contain ion transporting enzymes?
A. Intermediate
B. Basal
C. Melanocyte
D. Marginal
E. Interdental

3. Inner hair cells (IHC) and outer hair cells (OHC) show a different type
of nerve innervation. Which is the most correct description of their
innervation?
A. IHCs receive 90% of the afferent innervation, and their efferent nerve
fibers synapse on their afferent nerve fibers rather than on their cell body.
B. OHCs receive 90% of the afferent innervation, and their efferent nerve
fibers synapse on their afferent nerve fibers rather than on their cell body.
C. IHCs receive 90% of the afferent innervation, and their efferent nerve
fibers synapse on their cell body.
D. OHCs receive 90% of the afferent innervation, and their efferent nerve
fibers synapse on their cell body.
E. IHCs and OHCs are richly innervated by autonomic nerve fibers.

431
BY DR. MOHAMMED ATIAA KAREEM ALNASHY
-------------------------------------------------------------------
4. The first obligatory nerve relay center for CN VIII nerve afferent fibers
is which nucleus in the CNS?
A. Scarpa
B. Rosenthal
C. Spiral
D. Olivary
E. Cochlear

5. Which of the following statements is true for the hair cell stereocilia?
A. Stereocilia are true cilia-like structures, decrease in length toward the
cochlear apex, and do not contain mechanoelectrical transduction channels.
B. Stereocilia are microvilli-like structures, increase in length toward the
cochlear apex, and contain mechanoelectrical transduction channels.
C. Individual stereocilia are not connected to one another within the bundle.
D. Auditory hair cells in the adult mammal cochlea contain stereocilia and a
kinocilium.
E. Stereocilia are motile, because they contain an actin and myosin
cytoskeleton.

CHAPTER 149

Cochlear Anatomy and Central Auditory Pathways

1B

2D

3. A

4. E

5B

432
BY DR. MOHAMMED ATIAA KAREEM ALNASHY
-------------------------------------------------------------------
CHAPTER ONE HUNDRED AND FIFTY: MOLECULAR BASIS OF
AUDITORY PATHOLOGY
--------------------------------------------------------------------------------------------
1. Which of these statements regarding the categorization of
nonsyndromic deafness is correct?
A. DFNA designates autosomal-recessive nonsyndromic deafness.
B. DFNB designates autosomal-dominant nonsyndromic deafness.
C. DFN designates X-linked nonsyndromic deafness.
D. DFNMt designates mitochondrial nonsyndromic deafness.
E. All of these statements are correct.

2. Which of the following statements about outer hair cells is true?


A. There are roughly three times as many outer hair cells as inner hair cells.
B. Only -5% of auditory nerve fibers innervate outer hair cells.
C. Outer hair cells are more susceptible to noise damage than inner hair cells.
D. Outer hair cells are more susceptible to damage by aminoglycoside use than
inner hair cells.
E. All of these are true.

3. Which of the following is not commonly associated with Usher


syndrome type I?
A. Congenital deafness
B. Vestibular dysfunction
C. Congenital aural atresia
D. Retinitis pigmentosa
E. Genetic mutations

4. Which of the following statements regarding the homozygous Myo7a


mutant mouse is false?
A. It is frequently thought of as a model of Usher syndrome type IB.
B. It often develops degenerative disease of the retina.
C. It is often found to exhibit inner ear abnormalities.
D. It lacks melanosomes in cells of RPE.
E. All of the above statements are true.

433
BY DR. MOHAMMED ATIAA KAREEM ALNASHY
-------------------------------------------------------------------
5. Mutation of which gene may be associated with perilymphatic gusher
during stapes surgery?
A. Connexin 26
B. Pou3f4
C. KNCQ1
D. KCNE1
E. BSND

CHAPTER 150

Molecular Basis of Auditory Pathology

1. C

2. E

3. C

4B

5B

434
BY DR. MOHAMMED ATIAA KAREEM ALNASHY
-------------------------------------------------------------------
CHAPTER ONE HUNDRED AND FIFTY-ONE: ELECTROPHYSIOLOGIC
ASSESSMENT OF HEARING
--------------------------------------------------------------------------------------------
1. Absence of transient evoked otoacoustic emissions (TEOAEs) in a child
is most consistent with which of the following conditions?
A. Normal hearing
B. Sensorineural hearing loss
C. Conductive hearing loss
D. Auditory neuropathy
E. B and C
F. B and D

2. Historically, electrocochleography (ECoG) has been used to help with


the diagnosis of Meniere's disease. Which of the following measures
were considered consistent with the diagnosis of this disorder?
A. An abnormally large negative peak (Nl)
B. Absent ECoG in the face of normal hearing sensitivity
C. An enlarged SP:AP ratio
D. A reduced SP:AP ratio
E. None of the above

3. The auditory brainstem response is a measure of the synchronized


firing of neurons within the auditory pathways of the low brainstem.
Wave V of the auditory brainstem response (ABR) primarily reflects
neural activity from which of the following structures?
A. The auditory nerve
B. The cochlear nucleus
C. The superior olivary complex
D. The lateral lemniscal track
E. The inferior colliculus
F. The auditory mid brain
G. The auditory cortex

435
BY DR. MOHAMMED ATIAA KAREEM ALNASHY
-------------------------------------------------------------------
4. Although the click is the stimulus used most frequently to evoke the
ABR, this stimulus has a broad acoustic spectrum. Despite the relative
lack of frequency specificity, click-evoked ABR thresholds correlate
most strongly with audiometric thresholds in which of the following
frequency regions?
A. 250 to 500 Hz
B. 1000 Hz
C. 2000 to 4000 Hz
D. 4000 to 8000 Hz
E. None of the above

5. How does the electrically evoked auditory brainstem response (EABR)


as recorded from adult cochlear implant users differ from the
acoustically evoked version of this response (ABR) when it is recorded
from adult subjects with mild to moderate amounts of hearing loss?
A. The peaks of the EABR are typically larger in amplitude and shorter
in latency than the corresponding peaks of the ABR.
B. The peaks of the EABR are typically smaller in amplitude and longer
in latency than the corresponding peaks of the ABR.
C. The latency of wave V of the EABR changes significantly with
stimulation level, whereas the latency of wave V of the ABR does not.
D. The latency of wave V of the EABR does not change significantly with
stimulation level, whereas the latency of wave V of the ABR does.
E. A and D
F. B and D

436
BY DR. MOHAMMED ATIAA KAREEM ALNASHY
-------------------------------------------------------------------
CHAPTER 151
Electrophysiologic Assessment of Hearing

1. E
Both sensorineural hearing loss >25 to 30 dB HL and middle ear pathology
will result in absent transient otoacoustic emissions. Auditory neuropathy is
a condition characterized by an abnormal ABR in the face of normal
otoacoustic emissions. Similarly, subjects with normal hearing can be
expected to have measurable transient otoacoustic emissions.

2. C
Historically, an SP:AP ratio greater than approximately 0.4 has been
considered to be diagnostically significant for Meniere's disease.
Unfortunately, the sensitivity of this test is relatively low. Although the
figures vary to some extent, research has shown that only 60% to 70% of
individuals with confirmed Meniere's disease actually have an enlarged
SP:AP ratio.

3D
Although the individual peaks of the ABR represent activity from a range of
different generator sites, the neural generator site that contributes most
significantly to wave V is the lateral lemniscal tracks.

4. C
Click-evoked ABR thresholds correlate most strongly with the average of the
2000 Hz and 4000 Hz air conduction thresholds. Click-evoked ABR
thresholds are not accurate indicators of low frequency loss.

5. E
The peaks of the EABR are typically larger in amplitude and shorter in level
than the peaks of the ABR. In addition, as stimulation level is decrease, wave
V of the ABR is known to increase in
latency significantly. Because electrical stimulation bypasses the traveling
wave, the peaks of the EABR do not show a significant change in latency
with decreasing stimulus current levels.

437
BY DR. MOHAMMED ATIAA KAREEM ALNASHY
-------------------------------------------------------------------
CHAPTER ONE HUNDRED AND FIFTY-TWO: DIAGNOSTIC AND
REHABILITATIVE AUDIOLOGY
--------------------------------------------------------------------------------------------
1. The difference between speech detection threshold (SDT) and speech
reception threshold (SRT) is
A. SRT requires the listener to repeat the presented word.
B. SDT is usually 8 to 9 dB higher than the pure tone average (PTA).
C. SRT usually coincides with the PTA.
D. SDT can only be obtained with air conduction.
E. There is no difference between the terms.

2. Which of the following statements about masking is true?


A. Masking refers to the removal of all visual clues when evaluating
speech discrimination.
B. Masking should be used routinely with bone-conduction testing when
threshold levels between ears are asymmetric.
C. A masking dilemma frequently occurs when a patient has very severe
SNHL in one ear and normal hearing in the other ear.
D. Overmasking is a potential complication of audiometry that results
when a patient has tinnitus develop after exposure to a loud masking
signal.
E. All of these are true.

3. Which of the following is most correct regarding the use of the


stapedial reflex in the evaluation of facial paralysis?
A. It helps distinguish between neoplastic and nonneoplastic causes of
facial paralysis.
B. It helps distinguish between viral and nonviral causes of facial
paralysis.
C. It helps distinguish between a proximal and distal lesion of the facial
nerve.
D. It is an important prognostic tool in the evaluation of iatrogenic facial
nerve injury.
E. It is not useful in the evaluation of facial paralysis.

438
BY DR. MOHAMMED ATIAA KAREEM ALNASHY
-------------------------------------------------------------------
4. Which component of the ABR is the most robust and persists with
significant degrees of hearing loss?
A. Wave I
B. Wave II
C. Wave III
D. Wave IV
E. Wave V

5. Which of the following is a simple and straightforward audiometric


indicator of pseudohypoacusis?
A. Disagreement between the 500-, 1000-, and 2000-Hz PTA threshold and SRT
B. Bilateral absence of the stapedial reflex
C. Absence of evoked otoacoustic emissions
D. A normal ABR
E. An abnormal ABR

CHAPTER 152
Diagnostic and Rehabilitative Audiology

1. A

2B

3. C

4. E

5. A

439
BY DR. MOHAMMED ATIAA KAREEM ALNASHY
-------------------------------------------------------------------
CHAPTER ONE HUNDRED AND FIFTY-THREE: AUDITORY
NEUROPATHY
-----------------------------------------------------------------------------------------
1. Each of the following are characteristic findings of auditory
neuropathy except
A. Normal otoacoustic emissions and/or normal cochlear microphonics
B. Abnormal or absent auditory brainstem responses (ABRs)
C. Enhancement of the auditory nerve on post-gadolinium contrast Tl
magnetic resonance imaging scan
D. Absent stapedial reflexes
E. Poor speech recognition scores on audiogram

2. Which of the following is true regarding auditory neuropathy in


children?
A. Auditory neuropathy is often diagnosed by current newborn hearing
screening protocols.
B. Most children with auditory neuropathy are initially seen with an
associated peripheral neuropathy.
C. Currently, cochlear implantation is contraindicated in the auditory
rehabilitation of children with auditory neuropathy.
D. Neonatal hypoxia and hyperbilirubinemia are risk factors associated
with the development of auditory neuropathy.
E. Auditory neuropathy is transmitted to children by way of an X-linked
inheritance pattern.

3. In considering auditory rehabilitation for patients with auditory


neuropathy, which of the following statements is false?
A. When conducting a hearing aid trial, maximal benefit should be attempted
by decreasing background noise and improving the signal-to-noise ratio.
B. Hearing aids generally offer long-term successful auditory rehabilitation
for most patients with auditory neuropathy.
C. Hearing aids may damage the cochlea in patients with auditory neuropathy.
D. Cochlear implantation restores neural synchrony for patients with auditory
neuropathy.
E. The complication rates of cochlear implantation in auditory neuropathy are
comparable to standard cochlear implantation complication rates.

440
BY DR. MOHAMMED ATIAA KAREEM ALNASHY
-------------------------------------------------------------------
4. The human inner ear has some unique features in terms of the inner
hair cells (IHC) and the outer hair cells (OHC). The afferent
enervation to the OHC vs IHC can best be described as follows
A. IHC, 50%; OHC, 50%
B. IHC, 75%; OHC, 25%
C. IHC, 25%; OHC 75%
D. IHC, 5%; OHC 95%
E. IHC, 95%; OHC, 5%

5. There are characteristic evoked potential responses (cochlear


microphonic, CM; auditory brainstem response, ABR) for patients with
auditory neuropathy that are independent of the degree of hearing loss.
Which item is most likely to be observed in a child with auditory
neuropathy?
A. Phase-reversing CM absent and ABR normal
B. Phase-reversing CM present and ABR normal
C. Phase-reversing CM present and ABR abnormal
D. Phase-reversing CM absent and ABR abnormal

441
BY DR. MOHAMMED ATIAA KAREEM ALNASHY
-------------------------------------------------------------------
CHAPTER 153
Auditory Neuropathy

1. C
Auditory neuropathy is characterized by the following: (1) The patient must
complain of a hearing loss in at least some settings. (2) Patients have evidence
of normal outer hair cell function as demonstrated by normal otoacoustic
emissions and/or normal cochlear microphonics. (3) The patient demonstrates
auditory nerve dysfunction as displayed by abnormal auditory brainstem
response. (4) Patients with auditory neuropathy demonstrate poor speech
recognition scores that seem to be out of proportion to the degree of hearing
loss depicted by the pure tone thresholds. (5) Patients with auditory
neuropathy typically have absent middle ear stapedial reflexes. Normal
radiologic imaging studies of the brain and brainstem are also important in
making the diagnosis of auditory neuropathy. Auditory neuropathy is
currently a diagnosis of exclusion. There is no enhancement of the auditory
nerve on MRI in auditory neuropathy.

2D
Neonatal hypoxia and hyperbilirubinemia are risk factors associated with the
development of auditory neuropathy. This is a true statement, (a) Current
newborn hearing screening protocols frequently use otoacoustic emissions as
the first step in assessing newborns for hearing loss. Under this method, only
those children who fail otoacoustic emissions initially are pursued further
with ABR testing. Because children with auditory neuropathy will display
normal otoacoustic emissions, this screening method will miss the diagnosis
of auditory neuropathy. There are some institutions across the country that
are using auditory brainstem response as a part of the initial newborn
screening protocol for this reason, (b) Most children with auditory
neuropathy do not have an associated peripheral neuropathy. The peripheral
neuropathy is typically demonstrated in adult patients with auditory
neuropathy, (c) To date, there are many studies and case reports
demonstrating successful auditory rehabilitation through cochlear
implantation in children with auditory neuropathy. Although long-term
studies are not available to demonstrate durable results, initial data seem
promising that cochlear implantation will be a beneficial therapeutic
intervention for the auditory rehabilitation of children with auditory

442
BY DR. MOHAMMED ATIAA KAREEM ALNASHY
-------------------------------------------------------------------
neuropathy. The decision to perform cochlear implantation in children with
auditory neuropathy still represents clinical dilemma. These children
frequently demonstrate a large amount of residual hearing on pure tone
thresholds. However, when diligently observed for progress in speech
recognition and language acquisition, these children often demonstrate a
failure to make progress even with an adequate trial of amplification. It is at
this time that a cochlear implant should be seriously considered for these
children, (d) Neonatal hypoxia and hyperbilirubinemia are risk factors
associated with auditory neuropathy. It is not known precisely how these
insults contribute to the pathogenesis of auditory neuropathy, (e) Auditory
neuropathy is inherited both in an autosomal-recessive pattern and an
autosomal-dominant pattern. In the autosomal-dominant inheritance
pattern, the patients are more likely to have a slowly progressive hearing loss
and an associated peripheral neuropathy. The autosomal-recessive form
generally is seen in infancy, with profound hearing loss and no associated
peripheral neuropathy. The Otoferlin gene, which is localized on
chromosome 2, has been identified to be responsible for the nonsyndromic
recessive form of auditory neuropathy. At present, there is no genetic test
available to identify the presence of auditory neuropathy.

3B
The statement that hearing aids generally offer long-term successful auditory
rehabilitation for patients with auditory neuropathy is false. In general, it
has been a consistent finding that amplification has not provided successful
auditory rehabilitation for most cases of auditory neuropathy. Typically,
patients with auditory neuropathy will report frustration with amplification,
complaining that the sound is louder, and they can hear you but cannot
understand you. Hearing aids do sometimes improve the pure tone threshold
level; however, the speech recognition scores are often not improved, (a) If the
decision is made to conduct a hearing aid trial in cases of auditory
neuropathy, the audiologist should try to maximize benefit from the
amplification by use of directional microphones or person FM systems in an
attempt to decrease background noise and improve the signal to noise ratio,
(c) There is some controversy regarding the appropriate role of amplification
in the management of auditory neuropathy. Some authors will argue that
hearing aids are contraindicated because of the presence of intact outer hair
cells and the risk of noise-induced damage to these cells from amplification

443
BY DR. MOHAMMED ATIAA KAREEM ALNASHY
-------------------------------------------------------------------
systems. Because of this, when conducting a hearing aid trial, it is
recommended that hearing aids initially be fitted conservatively, with a low
maximum power output in an effort to preserve functioning outer hair cells.
Furthermore, otoacoustic emissions should be frequently monitored during
the amplification trial to assess any damage to the outer hair cells, (d) Even
though the auditory nerve may be damaged in patients with auditory
neuropathy, there is strong evidence to support that cochlear implantation
does provide reliable consistent nerve conduction, despite the presence of a
diseased or demyelinated nerve. This results in a restoration of neural
synchrony, as well as the promotion of neural survival, (e) Of the cases
reported in the literature, cochlear implantation for patients with auditory
neuropathy is associated with the same low complication rates as cochlear
implantation performed for other causes of hearing loss.

4. E
Because of various anatomic studies, especially Spoedlin (1996). His very
detailed drawings of surface preparations of human inner ears showed that
95% of the afferent fibers of the auditory nerve enervate the inner hair cells.
Each hair cell receives multiple fibers, whereas a single fiber may enervate
several different hair cells. This finding has resulted in many additional
anatomic and physiologic studies of the inner ear.

5. C
Patients with auditory neuropathy typically exhibit phase-reversing cochlear
microphonics and abnormal ABRs. Several examples are presented in the
chapter. See reference 4 in the book chapter (Berlin and others: Reversing
click polarity may uncover auditory neuropathy in infants, Ear Hear 19:37-
47, 1998) and Figures 153-3 and 153¬4, which show phase-reversing cochlear
microphonics and no neural potentials.

444
BY DR. MOHAMMED ATIAA KAREEM ALNASHY
-------------------------------------------------------------------
CHAPTER ONE HUNDRED AND FIFTY-FOUR: EVALUATION AND
SURGICAL MANAGEMENT OF CONDUCTIVE HEARING LOS
-------------------------------------------------------------------------------------------
1. Maximum conductive hearing loss occurs when
A. The incudostapedial joint is eroded behind an intact tympanic membrane.
B. The middle ear is filled with a thick effusion.
C. The tympanic membrane is completely perforated.
D. The round and oval windows are obliterated with otosclerosis.
E. The external canal is blocked by cerumen.

2. To optimize hearing with an ossicular prosthesis


A. Use a Cervital prosthesis
B. Use titanium prosthesis
C. Stage ossiculoplasty 6 to 9 months after removing cholesteatoma
D. Use cartilage interposed between the T M and the prosthesis
E. Place the prosthesis perpendicular to the tympanic membrane and
under minimal tension beneath the drum

3. The most commonly encountered ossicular abnormality in chronic


otitis media is
A. Malleus head fixation
B. Erosion of the head of the incus
C. Erosion of the lenticular process of the incus
D. Stapes superstructure erosion
E. Calcification of the lateral mallear ligament

4. In a 7-year-old girl, stapes fixation is encountered during a


mastoidectomy for cholesteatoma. Stapes manipulation should be done
A. Immediately with soft tissue sealing of the opening to the inner ear.
B. Staged after the ear heals (6-9 months)
C. Once the child is old enough to participate in the decision for surgery
D. Never. A hearing aid or bone-anchored hearing aid (BAHA) is a better
solution.

445
BY DR. MOHAMMED ATIAA KAREEM ALNASHY
-------------------------------------------------------------------
5. Ossiculoplasty should be considered when
A. The preoperative speech reception threshold (SRT) is greater than 30
dB or when the damaged ear is more than 15 dB less than the
contralateral ear.
B. The preoperative SRT is less than 30 dB and the opposite ear is greater
than 15 dB less than the contralateral ear.
C. The external auditory canal is occluded by large osteomas resulting in
a 15-dB conductive hearing loss
D. The tympanic membrane has failed a medial grafting.

446
BY DR. MOHAMMED ATIAA KAREEM ALNASHY
-------------------------------------------------------------------
CHAPTER 154
Evaluation and Surgical Management of Conductive Hearing Loss

1D
Obliteration of the round and oval windows does not allow a sound wave to
move into and through the cochlear fluids. This results in a 60-dB loss. The
hearing loss from middle ear effusion and perforations depends on the
thickness of the fluid and the size of the TM defect. Ossicular disruption
behind an intact TM results in a 55-dB loss.

2. E
The specific type and materials making up the prosthesis has little to do with
hearing results in middle ear reconstruction. Staging cholesteatoma surgery
can assist in hearing recovery but to a varying degree. Cartilage interposition
prevents extrusion. Perpendicular placement in respect to the TM with mild
tension on the head of the prosthesis ensures stability of the synthetic ossicle.

3. C
Incudostapedial joint erosion is the most common cause of ossicular erosion
associated with chronic otitis media. Malleus head fixation and incus head
erosion are rare. Calcification and superstructure erosion are less common
findings as well.

4. C
Reconstruction of the lateral ossicular chain is not problematic in patients of
any age. Manipulation of the stapes carries more significant risk, requiring
input from the child into the determination for surgical intervention.

5. A
See discussion in chapter.

447
BY DR. MOHAMMED ATIAA KAREEM ALNASHY
-------------------------------------------------------------------
CHAPTER ONE HUNDRED AND FIFTY-FIVE: SENSORINEURAL
HEARING LOSS: EVALUATION AND MANAGEMENT IN ADULTS
--------------------------------------------------------------------------------------------
1. Well-defined risks that enhance the likelihood of aminoglycoside
ototoxicity include all the following except
A. Presence of renal disease
B. Increased duration of therapy
C. Increased age
D. Malnutrition
E. Concomitant administration of loop diuretics

2. Factors influencing the development of noise-induced hearing loss


include all the following except
A. Intensity of offending sound
B. Duration of offending sound
C. Frequency of offending sound
D. Age of patient
E. Continuous versus intermittent sound

3. Which of the following treatment options for sudden sensorineural


hearing loss is most widely accepted and most likely to be effective?
A. Antiviral drugs
B. Anticoagulation
C. Steroid therapy
D. Carbogen therapy
E. Hypaque administration

448
BY DR. MOHAMMED ATIAA KAREEM ALNASHY
-------------------------------------------------------------------
4. Which of the following statements regarding sudden sensorineural
hearing loss and acoustic neuroma is a false statement?
A. Approximately 10% of patients with acoustic neuroma are initially seen
with sudden sensorineural hearing loss.
B. Approximately 1% of patients with sudden sensorineural hearing loss
have acoustic neuroma.
C. Recovery of hearing after steroid therapy indicates that acoustic
neuroma is not the etiology of the sudden hearing loss.
D. There is no relationship between tumor size and sudden sensorineural
hearing loss.
E. Gadolinium-enhanced magnetic resonance imaging is a more sensitive
test than auditory brainstem response for small acoustic neuromas.

5. Low-frequency sensorineural hearing loss is frequently seen in all of


the following disorders except
A. Benign intracranial hypertension
B. Endolymphatic hydrops
C. Presbycusis
D. Basilar migraine
E. Syphilis

449
BY DR. MOHAMMED ATIAA KAREEM ALNASHY
-------------------------------------------------------------------
CHAPTER 155
Sensorineural Hearing Loss: Evaluation and Management in Adults

1D
Well-defined risk factors for amino-glycoside-induced hearing loss have been
established and include (1) presence of renal disease; (2) longer duration of
therapy; (3) increased serum levels (either peak or trough levels); (4) advanced
age; and (5) concomitant administration of other ototoxic drugs, particularly
the loop diuretics.

2D
There is considerable variability in hearing loss among subjects with
identical exposure. Age, gender, race, and coexisting vascular disease have
been carefully studied, and when adequately controlled for other factors, they
have not been shown to correlate with susceptibility to NIHL.

3. C
Although the use of antiviral drugs would seem logical, no study to date has
demonstrated their effectiveness in sudden sensorineural hearing loss. Given
their low side effect profile and theoretical basis, many use antivirals in
SSNHL despite the absence of proven efficacy. There has never been a trial
showing benefit from anticoagulation, and this is not considered reasonable
therapy by most practitioners. Several large studies have shown benefit from
steroid treatment in selected subgroups of patients with SSNHL. Only
isolated reports have demonstrated benefit from carbogen or Hypaque.

4. C
Improvement in hearing after steroid therapy is frequently seen with acoustic
neuroma.

5. C
At least in the early stages of disease, hearing loss in benign intracranial
hypertension, endolymphatic hydrops, basilar migraine, and syphilis tends to
be primarily low frequency and fluctuating. Presbycusis is predominately a
high-frequency hearing loss.

450
BY DR. MOHAMMED ATIAA KAREEM ALNASHY
-------------------------------------------------------------------
CHAPTER ONE HUNDRED AND FIFTY-SIX: OTOSCLEROSIS
--------------------------------------------------------------------------------------------
1. Otosclerosis is a disease that
A. Is common to all
B. Occurs only in females
C. Is unique to the otic capsule
D. Is found at birth
E. Occurs mostly in males

2. Otosclerosis causes which type of hearing loss?


A. A purely conductive hearing loss
B. A progressive hearing loss
C. A purely sensorineural hearing loss
D. A profound hearing loss
E. All of the above

3. Otosclerosis typically
A. Has its onset of hearing loss in the fifth decade
B. Is more common in females than males by a ratio of 2:1
C. Occurs equally in males and females
D. Usually presents as a sudden hearing loss
E. Is associated with vertigo

4. On physical examination
A. A Schwartze sign is present in most patients.
B. The tympanic membrane is opaque.
C. Tuning forks are important in establishing the conductive component
of the hearing loss.
D. Patients may have blue sclera.
E. A white forelock is common.

5. The first widely used surgery to correct conductive hearing loss was
A. Stapedectomy by Shae
B. Stapes mobilization by Rosen
C. Small fenestra stapedectomy by Lempert
D. One stage tympanoplasty by Schuknecht
E. One stage fenestration by Lempert

451
BY DR. MOHAMMED ATIAA KAREEM ALNASHY
-------------------------------------------------------------------
CHAPTER 156
Otosclerosis

1. C
Otosclerosis is a unique process of changes in the bone of the otic capsule. The
lesions of the bone start as spongification, which progresses to sclerosis. The lesions
typically involve the otic capsule adjacent to the oval window and may spread
through the cochlea.

2. E
Otosclerosis usually causes a purely conductive hearing loss but may cause a mixed
conductive sensorineural hearing loss. Rarely, the loss is purely sensorineural with
no involvement of the stapes footplate. A significant number of patients undergoing
cochlear implants have a profound hearing loss due to advanced otosclerosis.

3B
In large series of patients with otosclerosis, about 70% are female. The onset is
usually in the early 20s but may be in the late 30s. It is not usual to find a history
that the female patient first noticed her hearing loss at the time of her first
pregnancy. The hearing loss is progressive and not associated with vertigo.

4. C
The Weber and Rinne are an important component in the clinical
evaluation of patients with otosclerosis. The 512 Hz tuning fork is used to establish
the conductive component of the hearing loss. The Weber will lateralize to the ear
with the greater conductive hearing loss. The Rinne will reveal bone conduction
greater than air conduction when the air bone gap is greater than 15 dB. When the
air bone gap is greater than 25 dB, the 1024 Hz fork will reverse. Tuning forks are
essential for confirming the audiometric findings. The positive Schwartze sign (a
red blush over the promontory) is seen in only about 10% of patients with active
otosclerosis. The tympanic membrane may be opaque, but it is usually clear. The
blue sclera is associated with osteogenesis imperfecta and the white forelock is seen
in Waardenburg's syndrome.

5. E
The one stage fenestration was first described and popularized by Julius Lempert
in the late 1930s. In the late 1800s, attempts were made to correct the hearing loss
with stapedectomy, but these attempts were associated with a high incidence of
meningitis and death. In 1953 Sam Rosen described the stapes mobilization
followed by John Shea's introduction of the stapedectomy.

452
BY DR. MOHAMMED ATIAA KAREEM ALNASHY
-------------------------------------------------------------------
CHAPTER ONE HUNDRED AND FIFTY-SEVEN: SURGICALLY
IMPLANTABLE HEARING AIDS
--------------------------------------------------------------------------------------------
1. List at least five limitations of traditional hearing aids that can
theoretically be overcome (or improved on) by implantable hearing
aids.

2. Why can an implanted hearing aid that directly drives the incus and/or
stapes generate louder perceived sound with less distortion using less
battery power than is possible with a traditional aid?

3. Why is the malleus neck cut during implantation of a Totally


Integrated Cochlear Amplifier (TICA) implantable hearing aid?

4. For what patients is an osseointegrated titanium fixture bone-


conduction aid (e.g., the Entific Bone-Anchored Hearing Aid
[BAHA]) appropriate?

5. What are (relative) contraindications for implantation and use of a


BAHA?

453
BY DR. MOHAMMED ATIAA KAREEM ALNASHY
-------------------------------------------------------------------
CHAPTER 157
Surgically Implantable Hearing Aids

1. Physical factors: insufficient gain, acoustic feedback, alteration of spectral


shape and phase, nonlinear distortion, occlusion effects, externally visible,
poor transduction efficiency (thus short battery life), lack of directionality.

2. Each conversion of energy from one physical domain to another incurs some
loss and distortion, so minimizing the number of conversions maximizes
transduction efficiency and minimizes distortion. With a conventional
hearing aid, acoustic waves in air impinge on a microphone and are
converted to an electric current; the current signal is amplified and drives
an electromagnetic speaker, creating acoustic waves in air again (but much
more intense); these waves then cause ossicular motion. Directly driving the
ossicular chain with an implantable aid obviates the conversion back into
air acoustic waves. Less amplification is required in the aid circuitry, and
the required incus (or stapes) motion is less than required of the speaker coil
in a conventional aid, so both distortion and power use can be minimized.

3. Just as feedback from the speaker to the microphone of a conventional


hearing aid can cause a squeal, the same can happen when the TIGA drives
the ossicular chain, making the tympanic membrane behave like a speaker
generating acoustic waves in air that feed back to the nearby TIGA
microphone in the ear canal. Gutting the malleus neck breaks this feedback
path but also compromises native conductive hearing.

4. (1) Any patient who uses a conventional bone conduction (BG) hearing-aid;
(2) air conduction (AG) hearing aid user with chronic otorrhea; (3) AG
hearing aid user experiencing too much discomfort because of chronic otitis
media/externa; (4) AG hearing aid user experiencing uncontrollable
feedback caused by a radical mastoidectomy or large meatoplasty; (5)
otosclerosis, tympanosclerosis, canal atresia with a contraindication to
repair, such as in an only hearing ear. Also, otosclerosis in combination with
2 through 4 above. (6) Patients with profound single-side sensorineural loss
may also benefit from ipsilateral BAHA use for contralateral routing of
sound.

5. (1) Pure tone average bone thresholds worse than 45 dB HL or word


discrimination score <60% in the target ear; (2) emotional instability,
development delay, or drug abuse; (3) age <5 years (NOTE: relative
contraindication, depending on skull thickness). Implantation of
osseointegrated fixtures in irradiated or otherwise diseased bone and in bone
<3-mm thick has a higher incidence of failure and device extrusion.

454
BY DR. MOHAMMED ATIAA KAREEM ALNASHY
-------------------------------------------------------------------

PART FIFTEEN
……………………………………………………………………………………………………………………
……………………………………………………………………………………………………………………
COCHLEAR IMPLANTS

455
BY DR. MOHAMMED ATIAA KAREEM ALNASHY
-------------------------------------------------------------------
CHAPTER ONE HUNDRED AND FIFTY-EIGHT: PATIENT EVALUATION
AND DEVICE SELECTION FOR COCHLEAR IMPLANTATION
--------------------------------------------------------------------------------------------
1. Which of the following statements is not true?
A. Up to 50% of all nonsyndromic sensorineural hearing loss can be
attributed to a mutation in a gap junction protein.
B. Genetic syndromal deafness is the leading cause of sensorineural
hearing loss.
C. Auditory neuropathy is a hearing disorder in which normal cochlear
outer hair cell function is present in conjunction with abnormal
auditory neural responses, resulting in poor neural synchrony.
D. Prenatal infection with TORCH organisms can result in reduced
ganglion cell counts and abnormal positions of the facial nerve.
E. Bilateral temporal bone fractures resulting in deafness can be
rehabilitated with cochlear implantation.

2. Current adult selection criteria for cochlear implantation include all


of the following except
A. Severe or profound hearing loss with a pure-tone average (PTA) of 50 dB
B. Aided scores on open-set sentence tests of <50%
C. No evidence of central auditory lesions
D. One- to three-month preoperative trial of hearing aid use

3. Which of the following is a contraindication to cochlear implantation?


A. An adult with prelingual onset of severe-to-profound hearing loss
B. Age >65 years
C. A narrow internal auditory canal
D. A child with chronic suppurative otitis media
E. Auditory neuropathy

4. Which of the following is a reasonable expectation for a child with the


onset of deafness of less than 1 year who is implanted between the ages
of 4 and 5 years?
A. Communication skill development at rates similar to normal-hearing peers
B. Attendance at a school with minimal support services
C. Improvement in speech perception with good closed-set performance but
limited open-set abilities
D. Reduced dependence on visual cues for communication

456
BY DR. MOHAMMED ATIAA KAREEM ALNASHY
-------------------------------------------------------------------
5. Factors that affect cochlear implant performance in children include
all of the following except
A. Age at implantation
B. Hearing experience
C. Presence of other disabilities
D. Parent and family support
E. Motivation to hear

6. Which of the following current speech-processing strategies is unique


to the MED-EL system?
A. Advanced combination encoder strategy
B. Spectral peak extraction
C. N-of-m pulsatile strategy
D. Continuous interleaved sampling
E. Pulses with the high-resolution strategy.

457
BY DR. MOHAMMED ATIAA KAREEM ALNASHY
-------------------------------------------------------------------
CHAPTER 158
Patient Evaluation and Device Selection for Cochlear Implantation

1B
Genetic syndromal deafness represents a small proportion of all significant hearing
loss. Studies indicate that up to 50% of all NSHL cases are due to a mutation in a
single gene encoding connexin 26 (Gx26). The gene coding for Gx26 (gap junction
protein p2 or GJB2) is located at locus DFNB1 on human chromosome 13ql2. The
diagnosis of auditory neuropathy/ auditory dyssynchrony (AN/D) has been
specified as a hearing disorder in which normal cochlear outer hair cell function
is found in conjunction with absent or abnormal auditory neural responses, which
is indicative of poor neural synchrony. Prenatal infection with TORCH organisms
(toxoplasmosis, syphilis, rubella, cytomegalovirus [CMV] and herpes) is commonly
associated with deafness. This spectrum of infections can result in reduced
ganglion cell counts, cognitive dysfunction, and abnormal position of the facial
nerve. Bilateral temporal bone fractures resulting in deafness can be rehabilitated
with cochlear implants.

2A
Current adult selection criteria in the most recent clinical trials include: (1) severe
or profound hearing loss with a pure-tone average (PTA) of 70 dB HL,
(2) use of appropriately fit hearing aids or a trial with amplification,
(3) aided scores on open-set sentence tests of <50%, (4) no evidence of central auditory
lesions or lack of an auditory nerve, and (5) no evidence of contraindications for
surgery in general or cochlear implant surgery in particular. In addition, cochlear
implant centers generally recommend at least 1 to 3 months of hearing aid use,
realistic expectations by the patient and family members, and willingness to comply
with follow-up procedures as defined by the center.

3. C
Although the average postoperative scores for individuals with prelin-gual hearing
loss are generally lower than those with postlingual hearing loss, there have been
significant preoperative to postoperative improvements in speech perception
reported for this group. Therefore, adults with prelingual onset of severe-to-
profound hearing loss may be appropriate candidates for cochlear implantation.
Audiologic results for cochlear implant users ages 65 to 80 years indicate
significant improvements for both preoperative and postoperative comparisons and
for varied speech stimulus presentation levels. Therefore, increased age is not a
contraindication for cochlear implant candidacy. When congenital or acquired
narrow internal auditory canals are identified on preoperative CT scanning,
primary afferent innervation may be lacking, and cochlear implantation is
therefore contraindicated. Cochlear implantation was initially viewed as
contraindicated in young children with chronic suppurative otitis media (CSOM)
because of the potential risk of infection. However, selective retrospective studies
have shown that the prevalence and severity of OM does not increase after

458
BY DR. MOHAMMED ATIAA KAREEM ALNASHY
-------------------------------------------------------------------
implantation, leading surgeons to advocate cochlear implantation if the ear is dry
at the time of implantation. The diagnosis of auditory neuropathy does not
preclude a child from cochlear implant candidacy.

4. D
For children implanted between 4 and 5 years, expectations include improvement
in speech perception with excellent closed-set performance and varied open-set
abilities, improvements in speech production, use of hearing to support
improvements in language, and reduced dependence on visual cues for
communication.

5. E
The most common preimplant factors that affect performance for children include
age at implantation, hearing experience (age at onset of profound hearing loss,
amount of residual hearing, progressive nature of the hearing loss, aided levels,
consistency of hearing aid use), training with amplification (in the case of some
residual hearing), presence of other disabilities, and parent and family support.
Postimplant factors that contribute to performance levels include length of
cochlear implant use, rehabilitative training, and family support.

6. C
With the n-of-ra pulsatile strategy, n is the number of electrodes stimulated out of
a total of m electrodes available. The goal of the n-of-m strategy is to transmit the
most prominent and important spectral cues of the input signal envelope as rapidly
as possible. The important spectral information is sent by designated electrodes that
are tonotopically organized (i.e., high- and low-frequency information to basal and
apical electrodes, respectively). The greater the n, the more spectral information
may be provided, given that the electrodes are able to be perceived as independent
stimulation channels. N-of-m is available with the Med-El device. Pulses with the
High Resolution (HiRes) strategy are available with the HiRes 90K device
manufactured by the Advanced Bionics Corporation. Continuous interleaved
sampling (CIS) is a speech processing strategy that has been implemented in
cochlear implant devices in recent years. Clarion, Nucleus, and MED-EL
implement a version of the CIS speech-processing strategy in their respective
devices. Spectral peak extraction, or SPEAK, is implemented in the Nucleus device.
The Advanced Combination Encoder (ACE) strategy was designed for the Nucleus
device to incorporate the spectral representation benefits of SPEAK with a high
rate CIS.

459
BY DR. MOHAMMED ATIAA KAREEM ALNASHY
-------------------------------------------------------------------
CHAPTER ONE HUNDRED AND FIFTY-NINE: MEDICAL AND
SURGICAL CONSIDERATIONS IN COCHLEAR IMPLANTS
--------------------------------------------------------------------------------------------
1. The medical evaluation of a cochlear implant (CI) candidate includes
A. General health
B. Imaging studies (computed tomography or magnetic resonance imaging)
C. Determination of appropriate expectations
D. Degree and duration of hearing loss
E. All of the above

2. The most frequent complication of CI surgery involves


A. Skin flap
B. Mastoid cavity infection
C. Facial nerve paresis
D. Perilymph fistula
E. Meningitis

3. Selection criteria for pediatric CI candidates include all of the


following except
A. Family support and appropriate expectations
B. Oral habilitation opportunities
C. Medical clearance for general anesthesia
D. No benefit from hearing aids
E. Physiologic age >12 months in most cases

4. Recurrent otitis media in a young child


A. Is an absolute contraindication to CI
B. Frequently leads to meningitis in implanted children
C. Must be controlled at the time of surgery
D. Cannot be treated with ventilation tubes if CI is being considered
E. Becomes more frequent after CI

5. Most cochlear ossification


A. Precludes CI
B. Is associated with bad results in CI
C. Fills the basal turn of the cochlea
D. Is limited to the round window area
E. Is associated with bent or broken CI electrodes

460
BY DR. MOHAMMED ATIAA KAREEM ALNASHY
-------------------------------------------------------------------
CHAPTER 159
Medical and Surgical Considerations in Cochlear Implants

1. E

2. A

3D?

4. C

5. D

461
BY DR. MOHAMMED ATIAA KAREEM ALNASHY
-------------------------------------------------------------------
CHAPTER ONE HUNDRED AND SIXTY: COCHLEAR IMPLANTS:
RESULTS, OUTCOMES, AND REHABILITATION
--------------------------------------------------------------------------------------------
1. Compared with cochlear implantation of the better-hearing ear,
performance after implantation of the poorer hearing ear shows what
differences?
A. Implantation of the better-hearing ear leads to better outcomes.
B. Implantation of the worse-hearing ear leads to better outcomes.
C. Implant performance with either ear is statistically equivalent.
D. Implantation of the poorer hearing ear should be performed only in
cases of deafness caused by meningitis.

2. Which of the following is most important variable for implant


performance in children?
A. Type of multichannel device chosen
B. Implantation of the better-hearing ear over the worse-hearing ear
C. Intelligence quotient (IQ) of child
D. Duration of deafness

3. After cochlear implantation in children, the rate of language development


A. Is slower in implanted children than normal-hearing children
B. Is equal in implanted children than normal-hearing children
C. Is faster in implanted children than normal-hearing children
D. Is dependent on age at implantation

4. In terms of cost-effectiveness, cochlear implantation is


A. Not cost-effective but covered by insurance companies
B. Not cost-effective but too beneficial to deny
C. Highly cost-effective with a cost per quality-adjusted life-year analysis
D. Only cost-effective in adults with post-lingual deafness

5. Cochlear implantation in the elderly


A. Should not be performed because of the increased risk of morbidity
B. Should only be performed before the age of 65 years
C. Is not cost-effective and should not be performed
D. Is cost-effective based on large gains in health-related quality of life

462
BY DR. MOHAMMED ATIAA KAREEM ALNASHY
-------------------------------------------------------------------
CHAPTER 160
Cochlear Implants: Results, Outcomes, and Rehabilitation

1. C

2D

3B

4. C

5D

463
BY DR. MOHAMMED ATIAA KAREEM ALNASHY
-------------------------------------------------------------------

PART SIXTEEN
……………………………………………………………………………………………………………………
……………………………………………………………………………………………………………………
SKULL BASE

464
BY DR. MOHAMMED ATIAA KAREEM ALNASHY
-------------------------------------------------------------------
CHAPTER ONE HUNDRED AND SIXTY-ONE: DIAGNOSTIC AND
INTERVENTIONAL NEURORADIOLOGY
--------------------------------------------------------------------------------------------
1. For which condition is magnetic resonance imaging (MRI) not the
imaging modality of choice?
A. Conductive hearing loss
B. Sensorineural hearing loss
C. Intracranial meningeal disease
D. Perineural spread of tumor

2. The ideal embolic agent for embolizing a chemodectoma is


A. Gelfoam
B. A tissue adhesive ("glue")
C. Polyvinyl alcohol foam (PVA)
D. Metallic coils

3. One or more of the following rationales make valuable the addition of


cerebral blood flow (CBF) studies to the balloon occlusion test (BOT)
A. It is important to define areas of ischemia and infarction preoperatively.
B. The BOT is only "positive" when CBF drops to less than 20 mL/100 g brain
tissue per minute during temporary carotid occlusion.
C. CBF studies can determine the potential for clot propagation and
embolization after vascular occlusion.
D. CBF studies may define the risk of cerebral ischemia from decreased cardiac
output or hypotension following permanent vascular occlusion.

4. One or more of the following are true of the imaging of a glomus


jugulare tumor
A. The tumor produces bone destruction simulating a malignant tumor.
B. It is always a very vascular tumor.
C. A large tumor may have feeding arteries from multiple major intracranial
and extracranial vessels.
D. Preoperative embolization may be valuable to decrease blood loss at surgery.
E. Multiple chemodectomas may be detected on MRI studies.
F. All of the above

465
BY DR. MOHAMMED ATIAA KAREEM ALNASHY
-------------------------------------------------------------------
5. One or more of the following statements regarding intracranial
meningiomas is true
A. Fifty percent of meningiomas are located over the convexities and in
the parasagittal and parafalcine regions; 40% originate from sites along
the skull base or tentorium; 10% are in other locations.
B. Meningiomas are always hypervascular at angiography, although they
may appear to be less vascular on computed tomography and MRI.
C. Meningiomas should be embolized preoperatively to decrease blood loss
at surgery.
D. A provocative injection in a feeding artery with 1% lidocaine may avoid
inadvertent cranial nerve palsy with embolization.

466
BY DR. MOHAMMED ATIAA KAREEM ALNASHY
-------------------------------------------------------------------
CHAPTER 161
Diagnostic and Interventional Neuroradiology

1. A
MRI is the modality of choice for evaluating sensorineural hearing loss,
because it is the most sensitive detector of tumors and other diseases affecting
the internal auditory canal and the cerebellopontine angle, and it is able to
detect abnormal signal intensities from the parenchyma, such as the
involvement of the brainstem by multiple sclerosis. It is able to demonstrate
enhancement of the meninges without obscuration by the contiguous bony
structures, which hampers CT in such detection. It can demonstrate
enhancement of the cranial nerves passing through the bony foramina,
indicative of perineural spread of tumor, especially if fat-suppression
techniques are used. The contiguity of bone again is the detriment to the use
of CT. However, CT is the modality of choice when evaluating the middle ear
and the ossicles for a condition producing conductive hearing loss. The spatial
resolution of CT is superior to MRI, so that small bony structures are far
better evaluated with CT than MRI. In addition, air in the middle and outer
ears is a natural contrast agent for CT.

2. C
PVA is the perfect choice for embolizing a vascular tumor such as a
chemodectoma. It is very easy to use, coming from the manufacturer as dried
particles of well-defined sizes from which to select. Small particles
(approximately 150 microns) will block the small arteries in the tumor bed,
not just the larger feeding arteries that coils can only do. Vascular
recanalization will take weeks to months or might not occur; Gelfoam breaks
down in 72 hours, and so if surgery is delayed, the effect of the embolization
may be lost. Gelfoam is usually hand-cut on the table, and so is less easy to
use than PVA. Finally, a tissue adhesive is like water in that it will flow into
the smallest tributaries, such as the tiny feeders to the cranial nerves. There
is usually no reason to subject a patient to the risk of cranial nerve palsy for
a preoperative embolization, especially when PVA is universally available.

467
BY DR. MOHAMMED ATIAA KAREEM ALNASHY
-------------------------------------------------------------------
3. B, D
The "passage" of the BOT relies on the status of collateral circulation to the
hemisphere fed by the vessel being temporarily occluded. If there are no
neurologic deficits during the test, the only thing the performer knows is that
the blood flow must be above the threshold to produce a deficit, 20 mL/100
g/min. If the flow was 22 mL, the patient might be at risk for a postoperative
stroke developing after permanent vascular occlusion if there were
superimposed hypotension of decreased cardiac output. A CBF study is the
only way to make such a determination. Although quantitative CBF studies
can define ischemic or infracted tissue simply by the blood flow numbers, a
much simpler way to define infarction is MRI with diffusion-weigh ted
imaging. CBF studies will not define the potential for clot propagation, which
may be a major reason for postoperative strokes even with a "negative"
preoperative BOT.

4. F
Although the glomus jugulare tumor is histologically "benign," it produces
irregular bone destruction that simulates a more aggressive, even malignant,
tumor. It is always hypervascular; if, on angiography, the tumor in question
is not vascular, it is not a chemodectoma. Small tumors may be fed primarily
from the ascending pharyngeal artery, but a large tumor will also receive
supply from the meningeal arteries, the stylomastoid branch of the occipital
artery, the posterior auricular artery, and the tentorial branch of the internal
carotid artery. Preoperative embolization is an excellent technique to
decrease surgical blood loss. Surgery is made more difficult by the close
quarters of the bony skull base. Embolization is more difficult if multiple
feeders are present. Finally, MRI of the entire head and neck is an excellent
way to detect multiple chemodectomas present in approximately 10% of
patients.

5. A, D
The incidence by location is true, according to numerous sources (see text and
references). Although many meningiomas are hypervascular, those in certain
locations are notorious for not being vascular at angiography; the suprasellar
meningioma is an example. Embolization caries the risk of stroke and cranial
nerve palsy.

468
BY DR. MOHAMMED ATIAA KAREEM ALNASHY
-------------------------------------------------------------------
CHAPTER ONE HUNDRED AND SIXTY-TWO: TEMPORAL BONE
NEOPLASMS AND LATERAL CRANIAL BASE SURGERY
--------------------------------------------------------------------------------------------
1. The smallest acceptable procedure to remove a squamous cell carcinoma
localized to the osseous external auditory canal is
A. Localized resection of the skin of the external auditory canal with
frozen-section margins
B. Sleeve resection of the external auditory canal, including the tympanic
membrane
C. Lateral temporal bone resection
D. Subtotal temporal bone resection
E. Total temporal bone resection

2. The Fisch type A infratemporal fossa dissection is designed to permit


A. Exploration and resection of the petrous apex, clivus, and superior
infratemporal fossa
B. Resection of squamous cell carcinomas of the external auditory canal
invading the mesotympanum
C. Access in resection of lesions involving the jugular bulb and vertical
petrous carotid and posterior infratemporal fossae
D. All of the above
E. None of the above

3. Rhabdomyosarcoma of the temporal bone is optimally treated with


A. Lateral temporal bone resection
B. Total temporal bone resection with complete tumor removal
C. Subtotal temporal bone resection alone for surgical debulking
D. Chemotherapy and radiotherapy with surgical debulking or total tumor
removal when possible
E. Chemotherapy and radiotherapy without surgery

469
BY DR. MOHAMMED ATIAA KAREEM ALNASHY
-------------------------------------------------------------------
4. High-grade neoplasms extending from the external auditory canal to
involve the medial mesotympanum are best managed with
A. Resection of the external auditory canal with mastoidectomy to remove
tumor in the mesotympanum, followed by radiotherapy
B. Subtotal temporal bone resection followed by radiotherapy
C. Concomitant cisplatin/5-fluorouracil chemotherapy and electron beam
radiotherapy
D. Extended temporal bone resection with sacrifice of the carotid artery
and facial nerve followed by radiation therapy
E. Stereotactic radiosurgery

5. Which surgical approach is usually most appropriate to remove glomus


tympanicum tumors (paragangliomas) whose borders are not entirely
visible through the tympanic membrane?
A. Transtympanic
B. Transcanal
C. Fisch type A
D. Lateral temporal bone resection
E. Extended facial recess

CHAPTER 162
Temporal Bone Neoplasms and Lateral Cranial Base Surgery

1. C

2. C

3D

4B

5. E

470
BY DR. MOHAMMED ATIAA KAREEM ALNASHY
-------------------------------------------------------------------
CHAPTER ONE 163: EXTRA-AXIAL NEOPLASMS INVOLVING
THE ANTERIOR AND MIDDLE CRANIAL FOSSA
--------------------------------------------------------------------------------------------
1. The 5-year survival for both squamous cell carcinoma and
adenocarcinoma is
A. 15% to 25%
B. 30% to 40%
C. 40% to 50%
D. 50% to 70%
E. 70% to 85%

2. Perineural spread is found in two-thirds of which histologic subtype of


anterior skull base tumor?
A. Squamous cell carcinoma
B. Adenoid cystic
C. Esthesioneuroblastoma
D. Sinonasal undifferentiated carcinoma
E. Rhabdomyosarcoma

3. Which tumor type has the highest 5-year survival percentage?


A. Squamous cell carcinoma
B. Sarcoma
C. Sinonasal undifferentiated sinonasal carcinoma
D. Mucosal melanoma
E. Adenocarcinoma

4. Which American Joint Commission of Cancer (AJCC) 2002 stage of


maxillary sinus cancer is described as: Tumor invasion of posterior
maxillary sinus, subcutaneous tissue, floor or medial wall of orbit,
pterygoid fossa, and ethmoid sinuses?
A. T1
B. T2
C. T3
D. T4a
E. T4b

471
BY DR. MOHAMMED ATIAA KAREEM ALNASHY
-------------------------------------------------------------------
5. Which AJCC 2002 stage of nasal cavity and ethmoid sinus is described
as: Tumor with extension into the anterior orbital contents, minimal
extension into anterior cranial fossa, pterygoid plates, and sphenoid,
frontal sinus?
A. T1
B. T2
C. T3
D. T4a
E. T4b

CHAPTER 163

Extra-Axial Neoplasms Involving the Anterior and Middle Cranial Fossa

1D

2B

3B

4. C

5D

472
BY DR. MOHAMMED ATIAA KAREEM ALNASHY
-------------------------------------------------------------------
CHAPTER 164: SURGERY OF THE ANTERIOR AND MIDDLE CRANIAL BASE
--------------------------------------------------------------------------------------------
1. Which cranial nerves pass through the superior orbital fissure?
A. III, IV, V-l, and VI
B. II, III, IV, and VI
C. III, IV, V-l, and V2
D. III, IV, V-2, and VI
E. II, III, IV, and V-2

2. The maxillary branch of the trigeminal nerve (V2) travels through


which foramen in the floor of the middle cranial fossa?
A. Superior orbital fissure
B. Foramen rotundum
C. Foramen ovale
D. Foramen spinosum
E. Foramen lacerum

3. Brain relaxation techniques that may be used during anterior skull


base surgery include
A. Mannitol
B. Cerebral spinal fluid (CSF) drain
C. Hyperventilation
D. Corticosteroids
E. All of the above

4. In the early postoperative period after a combined transfacial/frontal


craniotomy approach, the patient experiences progressive neurologic
deterioration, confusion, and obtundation after blowing his nose. The
next course of action should be
A. Reduce inhaled oxygen concentration
B. Increase CSF drainage rate
C. Stat head computed tomography scan
D. Administer a benzodiazepine
E. All of the above

5. The frontal branch of the facial nerve runs deep to which of the
following structures?
A. Temporal parietal fascia
B. Superficial layer of the deep temporal fascia
C. Deep layer of the deep temporal fascia
D. Periosteum of the zygomatic arch
E. Temporal fat pad

473
BY DR. MOHAMMED ATIAA KAREEM ALNASHY
-------------------------------------------------------------------
CHAPTER 164
Surgery of the Anterior and Middle Cranial Base

1. A

2B

3. E

4. C

5. A

474
BY DR. MOHAMMED ATIAA KAREEM ALNASHY
-------------------------------------------------------------------
CHAPTER ONE HUNDRED AND SIXTY-FIVE: EXTRA-AXIAL
NEOPLASM OF THE POSTERIOR FOSSA
--------------------------------------------------------------------------------------------
1. Which of the following statements regarding neurofibromatosis and
acoustic tumors is true?
A. Bilateral acoustic tumors are diagnostic of NF-1.
B. Bilateral acoustic tumors are diagnostic of NF-2.
C. Bilateral optic meningiomas are diagnostic of NF-1.
D. Cafe-au-lait spots are characteristic of NF-2.
E. Acoustic tumors cannot occur in NF-1.

2. Which of the following statements regarding cerebellopontine angle


meningiomas is true?
A. They are usually centered on the porous acusticus.
B. They are usually eccentric to the porous acusticus.
C. Hyperostosis is uncommon.
D. They typically arise from the meninges covering VIII.
E. They typically arise from the dural-glial junction.

3. Which of the following statements regarding the translabyrinthine


approach is true?
A. Large tumors are inadequately exposed with this approach.
B. The facial nerve in the internal auditory canal (IAC) fundus is
typically obscured in this approach.
C. Bill's bar helps localize the facial nerve in the porous acusticus.
D. The dura is best opened after exposing the distal IAC but before
uncovering the posterior fossa dura.
E. The superior vestibular nerve is posterior to the facial nerve in the IAC.

4. The characteristic symptoms of (CPA) malignancies are


A. Rapid symptom progression in multiple CPA nerves
B. Sudden sensorineural hearing loss
C. Disequilibrium
D. Retroorbital pain
E. Retroauricular pain

475
BY DR. MOHAMMED ATIAA KAREEM ALNASHY
-------------------------------------------------------------------
CHAPTER 165

Extra-Axial Neoplasm of the Posterior Fossa

1B

2. B

3. E

4. A

476
BY DR. MOHAMMED ATIAA KAREEM ALNASHY
-------------------------------------------------------------------
CHAPTER 166: AUDITORY BRAINSTEM IMPLANTS
--------------------------------------------------------------------------------------------
1. The target region for the auditory brainstem implant is the
A. Interstitial nucleus of Cajal
B. Dorsal and ventral cochlear nuclei
C. Zona inserta of the cochlear nerve
D. Roof of the fourth ventricle
E. Superior and medial vestibular nuclei

2. The optimum surgical approach for the auditory brainstem implant is the
A. Middle cranial fossa
B. Retrosigmoid
C. Suboccipital
D. Transcochlear
E. Translabyrinthine

3. In contrast to traditional multichannel cochlear implants, the


multichannel auditory brainstem implant
A. Has a substantially higher complication rate
B. Cannot realize significant speech discrimination
C. Achieves maximum results more quickly
D. Is under investigational FDA protocol
E. Causes no nonauditory sensations

4. Users of the multichannel auditory brainstem implant have


experienced all of the following except
A. Variations in pitch sensations across electrodes
B. Open-set speech discrimination
C. Auditory sensations
D. Extra-auditory sensations
E. All have been experienced

5. Which of the following statements regarding open-set speech discrimination


after multichannel auditory brainstem implantation is true?
A. Average auditory brainstem implantation performance equals that of
multichannel cochlear implants.
B. Only closed-set discrimination has been achieved.
C. When open-set speech discrimination occurs, it is evident within the first 3
months after hook-up.
D. Ultimate performance also depends on learning or brain accommodation.
E. It correlates positively with nonauditory effects

477
BY DR. MOHAMMED ATIAA KAREEM ALNASHY
-------------------------------------------------------------------
CHAPTER 166
Auditory Brainstem Implants

1. B

2. E

3. D

4. E

5D

478
BY DR. MOHAMMED ATIAA KAREEM ALNASHY
-------------------------------------------------------------------
CHAPTER ONE HUNDRED AND SIXTY-SEVEN: TRANSNASAL
ENDOSCOPIOASSISTED SURGERY OF THE SKULL BASE
--------------------------------------------------------------------------------------------
1. For masses involving midline skull base structures such as the clivus and sella
regions, the most ideal endoscopic approach for resection is which of the
following?
A. Transmaxillary
B. Transseptal
C. Transnasal direct
D. Transethmoidal
E. Transpalatal

2. All of the following structures are located within the pterygopalatine fossa except
A. Pterygopalatine ganglion
B. Vidian nerve
C. Internal maxillary artery
D. The maxillary nerve
E. The anterior ethmoidal artery

3. All of the following statements regarding transnasal endoscopically assisted


approaches for repair of cerebral spinal fluid (CSF) fistulas are true except
A. They allow for precise localization of bony and tissue defects.
B. They allow for direct repair of bony and tissue defects.
C. They allow for visualization of intranasal fluorescein after intrathecal
injection.
D. They may involve repair with free mucoperiosteal grafts from the inferior or
middle turbinate or the nasal septum.
E. They require total middle turbinate resection for access.

4. Profuse bleeding is encountered when the external clival dura is incised during
a surgery when a transnasal endoscopically assisted approach is used to access
the clivus. The source of this bleeding is most likely
A. The cavernous sinus
B. The basilar venous plexus
C. The internal carotid artery
D. The vertebral artery

5. A patient undergoes resection of a skull base mass with a transnasal


endoscopically assisted approach and complains of visual changes in the recovery
room and is ultimately found to have optic nerve damage. The injury to the optic
nerve most likely accompanied injury in which of the following areas
A. The optic foramen
B. The anterior ethmoid air cells
C. The lamina papyracea
D. The superior-lateral sphenoid sinus wall
E. The inferior-lateral sphenoid sinus wall

479
BY DR. MOHAMMED ATIAA KAREEM ALNASHY
-------------------------------------------------------------------
CHAPTER 167
Transnasal Endoscopic-Assisted Surgery of the Skull Base

1B
The transseptal approach was conceived to provide midline access to the
sphenoid sinus region through the nasal septum. This midline access avoids
damage to the structures in the nasal cavity and the carotid artery and optic
nerve along the lateral wall of the sphenoid sinus. It is particularly useful to
access the clivus, sella, and parasellar regions, because these are all midline
structures. The transmaxillary approach is ideal for approaching lesions
involving the medial portion of the maxillary sinus, the pterygopalatine
fossae, or zygomatic fossae. The transnasal direct approach is ideal for lesions
involving the roof of the nasal cavity without involvement of the ethmoid
sinus, lesions of the nasopharynx, and some lesions involving the sphenoid
sinus. The transethmoidal approach is indicated for lesions extending into or
involving the ethmoid and sphenoid sinuses.

2. E
The anterior ethmoidal artery, a terminal branch of the ophthalmic artery,
exits the ethmoid foramen at or just superior to the frontal ethmoid suture
and enters the anterior cranial fossa as the lateral edge of the cribriform
plate. It supplies the mucosa of the anterior and middle ethmoid air cells and
the dura covering the cribriform plate and the planum sphenoidale. The
pterygopalatine fossa is situated between the posterior wall of the maxillary
sinus anteriorly and the pterygoid process of the sphenoid posteriorly. It
contains the pterygopalatine ganglion, which receives the Vidian nerve, the
maxillary nerve as it leaves the foramen rotundum, the internal maxillary
artery and its two terminal branches, the posterior lateral nasal artery and
the septal artery.

480
BY DR. MOHAMMED ATIAA KAREEM ALNASHY
-------------------------------------------------------------------
3. E
The transnasal endoscopically assisted approach for repair of CSF fistula has
many advantages over craniotomy. It allows for precise localization of defects
and direct repair with minimal morbidity. The type of repair depends on the
size and location of the defect. Small defects may be covered with
mucoperiostcal grafts from the middle or inferior turbinates. When located
in the fovea ethmoidalis, an ethmoidectomy is required for identification.
Total middle turbinate resection is indicated specifically for repair of defects
in the cribriform plate.

4B
The basilar venous plexus is located between the two layers of the dura of the
upper clivus and is related to the dorsum sella and the posterior wall of the
sphenoid sinus. It forms interconnecting venous channels between the
inferior petrosal sinuses laterally, the cavernous sinuses superiorly, and the
marginal sinus and epidural venous plexus inferiorly. It is the largest
communicating channel between the paired cavernous sinuses. Although
midline transfacial approaches to midline skull base structures are
advantageous for their direct access, they are restricted by critical
neurovascular structures such as the internal carotid artery, optic nerve,
cavernous sinus, and the basilar venous plexus. Large lesions often compress
the basilar venous plexus, but profuse bleeding can occur when the external
layer of clival dura is incised and the plexus is not compressed. Bleeding can
usually be controlled with Surgicel, but this area must be approached with
caution.

5D
Most orbital complications stem from direct injury to the optic nerve or the
extraocular muscles or from bleeding within the bony orbit. Direct or indirect
damage to the optic nerve usually occurs at the superolateral sphenoid sinus
wall or in the posterior ethmoid cells. The location of the optic nerves along
the superolateral wall of the sphenoid sinus must be appreciated when
performing procedures through the sphenoid sinus to avoid injury.

481
BY DR. MOHAMMED ATIAA KAREEM ALNASHY
-------------------------------------------------------------------
CHAPTER ONE HUNDRED AND SIXTY-EIGHT: INTRAOPERATIVE
MONITORING OF CRANIAL NERVES IN NEUROTOLOGIC SURGERY
--------------------------------------------------------------------------------------------
1. When using auditory brainstem response (ABR) monitoring in the
operating room, what should be done when at the time of the craniotomy?
A. Switch to the use of analog rather than digital filtering
B. Change the filter settings to a wider frequency range
C. Increase the rate of stimuli for eliciting the ABR
D. Obtain a control ABR from the contralateral ear
E. Obtain a new intraoperative baseline

2. One of the electrodes for facial nerve monitoring is typically placed in


the orbicularis oculi near the lateral canthus. How can a response from
CN VI be distinguished from CN VII?
A. CN VI will have a longer latency and higher amplitude.
B. CN VI will have a longer latency and lower amplitude.
C. CN VI will have a shorter latency and higher amplitude.
D. CN VI will have a shorter latency and lower amplitude.
E. If placed correctly, the electrode should only record signals from CN VII.

3. In large vestibular schwannomas, which of the following was found to


be related to poor postoperative facial nerve outcome?
A. Large-amplitude spontaneous activity
B. Change in pattern of electromyography (EMG) from responsive to silent
C. Occurrence of "burst" activity
D. Low-amplitude train activity
E. "Popcorn" activity

4. Methods based on recording compound nerve action potentials (CNAP)


have what potential advantage over EMG monitoring?
A. They are more sensitive.
B. They can be used even if the patient is paralyzed.
C. They are insensitive to artifact from bipolar cautery.
D. They are insensitive to artifact from anesthetic agents.
E. They can more accurately detect manipulation of the nerve.

5. Surgical manipulation near which of the following nerves has the


potential to cause bradycardia?
A. CN V
B. CN VI
C. CN VII
D. CN VIII
E. CN IX

482
BY DR. MOHAMMED ATIAA KAREEM ALNASHY
-------------------------------------------------------------------
CHAPTER 168
Intraoperative Monitoring of Cranial Nerves in Neurotologic Surgery

1. E

2D

3B

4B

5A

483
BY DR. MOHAMMED ATIAA KAREEM ALNASHY
-------------------------------------------------------------------
CHAPTER ONE HUNDRED AND SIXTY-NINE: RADIATION
THERAPY OF THE CRANIAL (SKULL) BASE
--------------------------------------------------------------------------------------------
1. The ability to change the target of a beam instantly is a characteristic
of which method of radiation therapy?
A. Linear-accelerator (LINAG)-based
B. Gamma knife unit
C. Proton beam radiotherapy
D. Cyberknife (photon beam)
E. Intensity-modulated radiation therapy (IMRT)

2. Which type of radiosurgery is based on the underlying principle that each


broad radiation beam is divided into a number of smaller "beam-lets," which
are then added to form a dose distribution that is tailored to the shape of the
target?
A. Linear-accelerator (LINAG)-based
B. Gamma knife unit
C. Proton beam radiotherapy
D. Cyberknife (photon beam)
E. IMRT

3. For which of the following tumors should complete surgical resection


always be followed by adjuvant radiation therapy?
A. Juvenile angiofibroma
B. Esthesioneuroblastoma
C. Nasopharyngeal carcinoma
D. Temporal bone paraganglioma
E. Meningioma

4. What is the best treatment option for a patient with early-stage


nasopharyngeal carcinoma?
A. Radiation therapy only
B. Complete surgical resection only
C. Complete surgical resection followed by radiation therapy
D. Preoperative chemotherapy followed by complete surgical resection
E. Chemotherapy and radiation therapy

5. High-dose conformal radiotherapy to the skull base, as may be required to treat


skull base chordomas, can result in which of the following complications?
A. Optic neuropathy
B. Memory loss
C. Hypoadrenalism
D. Right-sided hemiparesis
E. Left-sided hemiparesis

484
BY DR. MOHAMMED ATIAA KAREEM ALNASHY
-------------------------------------------------------------------
CHAPTER 169
Radiation Therapy of the Cranial (Skull) Base

1D

2. E

3B

4. A

5. C

485
BY DR. MOHAMMED ATIAA KAREEM ALNASHY
-------------------------------------------------------------------

PART SEVENTEEN
……………………………………………………………………………………………………………………
……………………………………………………………………………………………………………………
PEDIATRIC OTOLARYNGOLOGY

486
BY DR. MOHAMMED ATIAA KAREEM ALNASHY
-------------------------------------------------------------------
CHAPTER 170: GENERAL CONSIDERATIONS
--------------------------------------------------------------------------------------------
1. Airway obstruction in newborns may cause rapid ventilatory fatigue because
A. Their diaphragm is low in type I muscle fibers.
B. Their diaphragm is low in type II muscle fibers.
C. Of relative low compliance of the chest wall
D. Of their relatively low basal metabolic rate
E. They have a low rest tone while sleeping.

2. The laryngeal chemoreflex (LCR) causing laryngospasm is most sensitive to


A. Water
B. Saline
C. Acid
D. Base
E. Pressure

3. Because newborn cardiac muscle has fewer contractile fibers and more
connective tissue, cardiac output is most dependent on which of the following
A. Preload
B. Afterload
C. Rate
D. Systolic pressure
E. Diastolic pressure

4. Often the first sign of hypoxia in a neonate is


A. Tachycardia
B. Bradycardia
C. Hypotension
D. Hypertension
E. Apnea

5. Of the following physical signs, which is the best estimate that a young
infants blood volume is adequate?
A. Heart rate
B. Mean arterial blood pressure
C. Color
D. Temperature
E. Percent hemoglobin saturation

487
BY DR. MOHAMMED ATIAA KAREEM ALNASHY
-------------------------------------------------------------------
CHAPTER 170
General Considerations

1. A
When newly born, infants have a low ratio of type I to type II muscle fibers in their
diaphragm. This gives a higher proportion of fatigable type II fibers. If the work of
breathing increases, they may soon fatigue, faster than older children. This is
compounded by a high compliance of the chest wall decreasing the efficiency at which
ventilation occurs.

2. A
The laryngeal chemoreflex (LCR) causing laryngospasm seems to be most sensitive to
water and is ablated by saline application. Acid, base, and pressure can also induce
the reflex and may be important in the cycle of laryngospasm, airway obstruction,
hypoxia, bradycardia, and death seen in SIDS.

3. C
The usual Starling curves of contractility we are familiar with in the adult
cardiovascular physiology do not hold true for the neonatal heart. Cardiac output is
rate dependent in the neonatal heart. Bradycardia invariably equates with reduced
cardiac output. Because of the differences in compliance and contractility in the
neonatal heart, increased contractility is not possible to maintain cardiac output
during bradycardia. The low compliance of the relaxed ventricle limits the size of the
stroke volume and, therefore, increases in preload are not as important in neonatal
physiology as is heart rate.

4B
Because of a relatively high metabolic rate seen in neonates and the relative low
reserve for gas exchange as described previously, hypoxemia can develop rapidly, and
the first sign is usually bradycardia. During surgery, any unexplained episode of
bradycardia should be initially treated with oxygen and increased ventilation. During
hypoxemia, neonatal pulmonary vasoconstriction and hypertension occur more
dramatically than in adults. This can shift them back into fetal circulation,
compounding the problem.

5B
Because the total blood volume of an infant is small, significant blood loss can accompany
relatively minor surgical blood loss. It has been observed during exchange transfusions
that withdrawal of blood parallels a decline in systolic blood pressure and cardiac output.
This is reversible to normal parameters with replacement of the same blood volume
removed. Changes in arterial blood pressure with normal heart rates are thus proportional
to the degree of hypovolemia. A newborn's ability to adapt the intravascular volume to
the available blood volume is limited because of less efficient control of capacitance
vessels and immature or ineffective baroreceptors. The infant's systolic arterial blood
pressure is closely related to the circulating blood volume. Blood pressure is then an
excellent guide to the adequacy of blood or fluid replacement during anesthesia, a fact
that has been confirmed by extensive clinical experience.

488
BY DR. MOHAMMED ATIAA KAREEM ALNASHY
-------------------------------------------------------------------
CHAPTER 171: - DEVELOPMENTAL ANATOMY
--------------------------------------------------------------------------------------------
1. What are the contents of the carotid sheath?
A. The common carotid artery (including the internal and external carotid
arteries); cranial nerves IX, X, XI, and XII; and the ansa cervicalis
B. The internal jugular vein; the common carotid artery (including the internal
and external carotid arteries); and cranial nerves X, XI, and XII
C. The common carotid artery (including the internal and external carotid
arteries) and cranial nerves IX, X, and XI
D. The common carotid artery (including the internal and external carotid
arteries); the internal jugular vein; and cranial nerves IX, X, and XI

2. What is the most reliable way to differentiate the internal from the
external carotid artery in the neck?
A. The internal carotid artery has no branches in the neck.
B. The internal carotid artery lies anterior to the external carotid artery.
C. The external carotid artery has no branches in the neck.
D. The external carotid artery lies anterior to the internal carotid artery.

3. What are the branches of the thyrocervical trunk?


A. Superior thyroid, inferior thyroid, and suprascapular arteries
B. Superior and inferior thyroid arteries and cervical artery
C. Inferior thyroid, ascending cervical, transverse cervical, and suprascapular
arteries
D. Ascending, transverse, and descending cervical arteries

4. Which portions of the ossicular chain derive from the first branchial arch?
A. Stapes
B. Short processes of the malleus and incus
C. Long processes of the malleus and incus
D. Short process of the malleus and long process of the incus

5. Which portions of the ossicular chain derive from the second branchial arch?
A. Stapes
B. Stapes suprastructure and long processes of the malleus and incus
C. Stapes suprastructure and short processes of the malleus and incus
D. Malleus and incus

489
BY DR. MOHAMMED ATIAA KAREEM ALNASHY
-------------------------------------------------------------------
6. What are the embryologic origins of the laryngeal cartilages?
A. First branchial arch
B. Second branchial arch
C. Third branchial arch
D. Fourth, fifth, and sixth branchial arches

7. What are the clinical features of hemifacial microsomia?


A. Mandibular hypoplasia, microtia, and aural atresia
B. Zygomatic and mandibular hypoplasia
C. Zygomatic hypoplasia, microtia, and aural atresia
D. Orbital and zygomatic hypoplasia

8. What is the pathogenesis of hemifacial microsomia?


A. Positional plagiocephaly
B. In utero vascular injury
C. Abnormalities of fusion of the branchial arches
D. Unknown

9. Complete injury of the accessory nerve in the right supraclavicular


fossa results in
A. Inability to turn the head to right
B. Inability to turn the head to the left
C. Inability to turn the head to the left and shrug the right shoulder
D. Inability to shrug the right shoulder

10. Which of the following masses present as midline masses of the neck?
A. Branchial cyst and carotid body tumor
B. Branchial cyst and thyroglossal duct cyst
C. Thyroglossal duct cyst and dermoid cyst
D. Pharyngocele and laryngocele

490
BY DR. MOHAMMED ATIAA KAREEM ALNASHY
-------------------------------------------------------------------
CHAPTER 171
Developmental Anatomy

1. A
The carotid sheath begins at the base of the skull surrounding the carotid and
jugular canal and thus receives the vessels and 9, 10, and 11 cranial nerves. The
sheath also includes the hypoglossal canal and the emerging 12th nerve.

2. A
This is straightforward descriptive anatomy.

3. C
This is also straightforward anatomy, although you should be aware that on
occasion the transverse cervical arises as an independent branch from the third
part of the subclavian artery.

4B
Answer a is incorrect, because the stapes is derived from the second arch; c is
incorrect, because the long process of the malleus and incus are from the second
arch; d is also incorrect, because although the malleus is from the first arch, the
long process of the incus is from the second.

5B
Straightforward descriptive anatomy.

6. D
Straightforward embryology description.

7. A
Straightforward clinical description.

8B
Straightforward clinical description.

9. D
Injury in the supraclavicular fossa only involves the distal part of XI, thus it has
no effect on the sternomastoid muscle and only affects the trapezius on the
paralyzed side.

10.C
Only thyroglossal cysts and dermoid present as midline masses of the neck.
Branchial cysts are in the anterior triangle and follow the anterior border of the
sternomastoid muscle. Pharyngoceles, laryngoceles, and carotid body tumors
present as masses in the carotid triangle.

491
BY DR. MOHAMMED ATIAA KAREEM ALNASHY
-------------------------------------------------------------------
CHAPTER ONE HUNDRED AND SEVENTY-TWO: ANESTHESIA
--------------------------------------------------------------------------------------------
1. The premedication drug of choice for children ages 8 months to 8 years is
A. IV midazolam
B. IV diazepam
C. IM ketamine
D. Rectal methohexital
E. Oral midazolam

2. During tympanoplasty, which inhalational agent should be turned off


before closure of the middle ear or even avoided entirely?
A. Nitrous oxide
B. Sevoflurane
C. Halothane
D. Isoflurane
E. Desflurane

3. Which inhalational agent has an irritant effect on the airway, making it


contraindicated for airway procedures such as bronchoscopy?
A. Nitrous oxide
B. Sevoflurane
C. Halothane
D. Isoflurane
E. Desflurane

4. What receptor type has been implicated in postoperative nausea and


vomiting?
A. Dopamine
B. Acetylcholine
C. Histamine
D. Serotonin
E. All of the above

5. In premature infants, elective procedures should be delayed until 55 weeks


to lower the risk of which complication of anesthesia?
A. Cardiopulmonary arrest
B. Malignant hyperthermia
C. Apnea
D. Bronchospasm
E. All of the above

492
BY DR. MOHAMMED ATIAA KAREEM ALNASHY
-------------------------------------------------------------------
CHAPTER 172
Anesthesia

1. E

2. A

3. E

4. E

5. C

493
BY DR. MOHAMMED ATIAA KAREEM ALNASHY
-------------------------------------------------------------------
CHAPTER 173: CHARACTERISTICS OF NORMAL AND ABNORMAL
POSTNATAL CRANIOFACIAL GROWTH AND DEVELOPMENT
--------------------------------------------------------------------------------------------
1. Craniofacial growth is believed to be
A. Genetically programmed
B. Mediated primarily by cartilage
C. Controlled entirely by a feedback system between the frontal matrix and bone
and cartilage
D. Concentrated in growth centers
E. A complex multifactorial phenomenon

2. Which synchondrosis is the principle growth cartilage of the basicranium


during childhood?
A. Sphenooccipital
B. Sphenoethmoidal
C. Intersphenoidal
D. Sphenofrontal
E. Frontoethmoidal

3. Craniofacial abnormalities associated with Down syndrome include all of


the following except
A. Brachycephaly
B. Midface hypoplasia
C. Wide and long palate
D. Reduced interorbital distance
E. Small ears

4. Early sphenooccipital closure resulting in a short cranial base, small


foramen magnum and microcephaly with frontal bossing, sunken bridge of
the nose, and midface hypoplasia is seen in which of the following entities?
A. Down syndrome
B. Treacher-Collins syndrome
C. Isolated Pierre Robin sequence
D. Stickler syndrome
E. Achondroplasia

5. Which of the following statements about open-mouth posture (OMP) in


children is correct?
A. It alters dentofacial growth.
B. It is an indication for adenoidectomy.
C. It is associated with a posterior crossbite.
D. It indicates nasal airway blockage.
E. It may be a normal part of development.

494
BY DR. MOHAMMED ATIAA KAREEM ALNASHY
-------------------------------------------------------------------
CHAPTER 173
Characteristics of Normal and Abnormal Postnatal Craniofacial Growth and
Development

1. E

2. A

3. C

4. E

5. E

495
BY DR. MOHAMMED ATIAA KAREEM ALNASHY
-------------------------------------------------------------------
CHAPTER 174: VASCULAR TUMORS AND MALFORMATIONS OF THE HEAD AND NECK
--------------------------------------------------------------------------------------------
1. Which of the following statements regarding hemangioma is true?
A. Hemangioma is always present at birth.
B. Hemangioma will grow with the child.
C. There is an equal distribution of hemangioma between boys and girls.
D. Hemangioma is more common in African Americans.
E. Hemangioma grows rapidly during the first 6 to 8 months of life.

2. Which of the following statements regarding the management of


hemangioma is true?
A. All hemangiomas respond to corticosteroid treatment.
B. Systemic interferon is the first line of medical treatment for hemangioma.
C. Combined treatment with corticosteroid and interferon is recommended in
the management cervicofacial hemangioma.
D. The usual dosage of corticosteroid is 1 mg/kg/day.
E. The empiric dose for IFN is 2 to 3 million units/m2.

3. Which of the following statements regarding subglottic hemangioma is incorrect?


A. Subglottic hemangioma usually presents in the first 6 months of life.
B. Diagnosis of subglottic hemangioma is based on findings at the time of
laryngoscopy, and often biopsy is not required.
C. The most common presentation is a left-sided subglottic hemangioma.
D. There is a higher risk of having a subglottic hemangioma in children with
cervicofacial hemangioma in the "beard" distribution.
E. All children with subglottic hemangiomas will response to systemic
corticosteroids if treated long enough.

4. Which of the following statements regarding vascular malformation is true?


A. Vascular malformation often present by 6 months of age.
B. There is a rapid growth followed by slow resolution in the life cycle of
vascular malformation.
C. Vascular malformations are more common in girls.
D. Vascular malformations are divided into slow-flow or fast-flow lesions.
E. Systemic steroid is the first line of medical treatment.

5. Which of the following statements regarding vascular malformation is incorrect?


A. Lymphatic malformation grows commensurately with the child.
B. The two strategies for the management of lymphatic malformations are
sclerotherapy and surgical resection.
C. The 19th century term for "capillary malformation" is "port-wine stain."
D. Ligation or proximal embolization of feeding vessels is the treatment choice
for AVM.
E. Venous malformations are easily compressible and expand when the affected
area is dependent or after a Valsalva maneuver.

496
BY DR. MOHAMMED ATIAA KAREEM ALNASHY
-------------------------------------------------------------------
CHAPTER 174
Vascular Tumors and Malformations of the Head and Neck

1. E
Hemangioma grows rapidly (the proliferative phase) during the first 6 to 8
months of life followed with slow regression (the involution phase).

2. E
The empiric dose for IFN is 2 to 3 million units/m2, injected subcutaneously
every day.

3.E
The usual dosage of systemic corticosteroid is 2 to 3 mg/kg/day of prednisone.
If no response is seen in 7 to 10 days, steroids should be tapered and stopped.

4D
It is clinically useful to separate the vascular malformations into "slow-flow"
(capillary, venous, lymphatic, or combined form) or "fast-flow"
(arteriovenous fistula [AVF] and arteriovenous malformation [AVM])
lesions.

5D
Embolization must be of the nidus, or epicenter, of the AVM. There is no
place for ligation or proximal embolization of feeding vessels. This will lead
to rapid recruitment of flow from nearby arteries and denies access for
embolization.

497
BY DR. MOHAMMED ATIAA KAREEM ALNASHY
-------------------------------------------------------------------
CHAPTER ONE HUNDRED AND SEVENTY-FIVE: CRANIOFACIAL
SURGERY FOR CONGENITAL AND ACQUIRED DEFORMITIES
-------------------------------------------------------------------------------------------
1. Premature fusion of the sagittal suture results in which craniofacial
abnormality?
A. Brachycephaly
B. Acrocephaly
C. Trigonocephaly
D. Scaphocephaly
E. Plagiocephaly

2. Which of the following syndromes exhibits autosomal-dominant


inheritance and often results in brachycephaly, midface hypoplasia,
class III malocclusion, and normal intelligence?
A. Apert's syndrome
B. Pfeiffer's syndrome
C. Crouzon's syndrome
D. Jackson-Weiss syndrome
E. Saethre-Chotzen syndrome

3. Which of the following is caused by mutations in genes that encode


type II and/or type XI collagen?
A. Carpenter syndrome
B. Stickler syndrome
C. Velocardiofacial syndrome
D. Treacher-Collins syndrome
E. Facio-oculo-auriculo-vertebral spectrum

4. Delaying surgical intervention is acceptable in which of the following cases?


A. Corneal exposure
B. Increased intracranial pressure
C. Kleeblattschadel (clover-leaf skull)
D. Maxillary hypoplasia
E. None of the above

5. Distraction osteogenesis
A. Refers to creating a controlled, rapid fracture between bony segments
B. Induces the formation of new bone between distracted segments
C. Is a new technique in orthopedics that seems promising for adaptation to
craniofacial surgery
D. Requires external hardware to execute the fracture
E. Allows for two-dimensional skeletal lengthening

498
BY DR. MOHAMMED ATIAA KAREEM ALNASHY
-------------------------------------------------------------------
CHAPTER 175
Craniofacial Surgery for Congenital and Acquired Deformities

1D

2. C

3B

4D

5B

499
BY DR. MOHAMMED ATIAA KAREEM ALNASHY
-------------------------------------------------------------------
CHAPTER 176: CLEFT LIP AND PALATE
--------------------------------------------------------------------------------------------
1. What forms the primary palate?
A. It forms as an outgrowth of the incisive foramen.
B. Fusion of the palatine shelves
C. Fusion of the medial nasal prominences
D. Fusion of the lateral nasal prominences
E. Fusion of the maxillary prominences

2. The velopharyngeal sphincter is composed of all of the following except


A. Levator veli palatine
B. Palatopharyngeus
C. Superior pharyngeal constrictor
D. Middle pharyngeal constrictor
E. Muscularis uvulae

3. According to the author, the most important aspect of cleft lip repair is
A. Reorientation and reconstitution of orbicularis oris around the entire
oral cavity
B. Creating a philtral ridge height of at least 12 mm
C. Using a lip adhesion preliminary procedure to provide sufficient tissue
for reconstruction
D. Complete correction of all nasal deformity during the initial procedure
E. None of the above

4. The most common complication after repair of a cleft palate is


A. Postoperative bleeding
B. Oronasal fistula
C. Velopharyngeal insufficiency
D. Wound infection
E. Airway obstruction

5. Some centers advocate a staged approach to cleft palate repair to


improve outcomes of which goal of palatoplasty?
A. Development of functional occlusion
B. Preservation of midface growth
C. Creation of competent velopharyngeal valve for swallowing
D. Creation of competent velopharyngeal valve for speech
E. All of the above

500
BY DR. MOHAMMED ATIAA KAREEM ALNASHY
-------------------------------------------------------------------
CHAPTER 176
Cleft Lip and Palate

1. C

2. D

3. A

4. C

5. B

501
BY DR. MOHAMMED ATIAA KAREEM ALNASHY
-------------------------------------------------------------------
CHAPTER 177: VELOPHARYNGEAL DYSFUNCTION
--------------------------------------------------------------------------------------------

1. Which of the following statements regarding velocardiofacial


syndrome is false?
A. The internal carotid arteries may be found in an aberrant medial
position in the oropharynx.
B. All patients with velocardiofacial syndrome have cardiac anomalies.
C. Pharyngeal hypotonia is a feature of velocardiofacial syndrome.
D. Microdeletions of the qll region of chromosome 22 have been found in
patients with velocardiofacial syndrome.
E. Microdeletions for velocardiofacial syndrome may overlap with those
of DiGeorge syndrome.

2. Which of the following statements regarding velopharyngeal closure


patterns is true?
A. Maximum lateral wall motion usually occurs above the levator eminence
B. The vertical pattern is the most common.
C. Closure patterns are described by the shape of the gap while closing and the
relative contribution of the pharyngeal walls.
D. Studies have shown that vertical closure length is short.
E. Pharyngeal wall motion is rated by its movement to the midline.

3. Which of the following statements regarding submucous clefts is true?


A. A bifid uvula is diagnostic of a submucous cleft.
B. The presence of a submucous cleft palate is a strict contraindication to
adenoidectomy.
C. A submucous cleft has a bifid uvula, muscular diastasis of the soft
palate, and loss of the anterior nasal spine.
D. Almost all children with submucous cleft are symptomatic for
velopharyngeal insufficiency.
E. A flat soft palate or absence of the bulge from uvula contraction during
palatal elevation is a finding of occult submucous clefts.

502
BY DR. MOHAMMED ATIAA KAREEM ALNASHY
-------------------------------------------------------------------
4. Which of the following statements is false?
A. Hypernasality may be the result of mislearning and not true velopharyngeal
insufficiency.
B. Nasal emissions may occur with select phonemes.
C. Palatal lifts elevate the soft palate and can be used when palate length is
adequate.
D. The conditions referred to in A and B are best treated with speech therapy.
E. Nasometers measure airflow orally versus nasally and display a ratio of the
two.

5. Which of the following statements is false?


A. The sphincter pharyngoplasty involves the palatopharyngeus posterior
tonsillar pillar muscles.
B. Pharyngeal flaps are effective in reducing hypernasality, have been
used for decades to correct velopharyngeal insufficiency, and are often
tailored to match the gap size.
C. Posterior-wall augmentation has many theoretic advantages; in
practice, it is useful for small, central gaps of 3 mm or less.
D. Apnea is most common in sphincter pharyngoplasty surgery.
E. Superiorly based pharyngeal flaps involve a myomucosal flap using the
constrictor muscle.

CHAPTER 177
Velopharyngeal Dysfunction

1B

2. C

3. E

4. E

5D

503
BY DR. MOHAMMED ATIAA KAREEM ALNASHY
-------------------------------------------------------------------
CHAPTER 178: CONGENITAL MALFORMATIONS OF THE NOSE
--------------------------------------------------------------------------------------------

1. The embryogenesis of choanal atresia generally is believed to be caused by


A. Failure of nasal canalization in the 10th intrauterine week
B. Failure of rupture of the nasobuccal membrane in the fifth to sixth
intrauterine week
C. Nasal placode invagination in the third to fourth intrauterine week
D. Neural crest maldevelopment in the 12th intrauterine week
E. Failure of ectoderm migration in the eighth week

2. A patient with a single upper incisor and narrow bony nasal pyriform
aperture most likely has a form of
A. CHARGE association
B. Holoprosencephaly
C. Down syndrome
D. Hydrocephalus
E. Goldenhar syndrome

3. Which of the following is not an anatomic deformity in choanal atresia?


A. Narrow nasal cavity
B. Medial obstruction caused by thickening of the vomer
C. Lateral bony obstruction in the medial pterygoid plate
D. Membranous obstruction
E. Bony obstruction

4. Pyriform aperture stenosis typically presents


A. Immediately after birth
B. With the first feeding
C. During the first few months of life
D. When the first teeth erupt
E. During adolescence

5. Dermoids are composed of which germinal layers?


A. Mesoderm and ectoderm
B. Endoderm and mesoderm
C. Endoderm, mesoderm, and ectoderm
D. Mesoderm only
E. Ectoderm only

504
BY DR. MOHAMMED ATIAA KAREEM ALNASHY
-------------------------------------------------------------------
CHAPTER 178
Congenital Malformations of the Nose

1. B

2. D

3. C

4. C

5. A

505
BY DR. MOHAMMED ATIAA KAREEM ALNASHY
-------------------------------------------------------------------
CHAPTER 179: PEDIATRIC CHRONIC SINUSITIS
--------------------------------------------------------------------------------------------

1. Children are more likely to have sinusitis than adults because


A. They have an immature immune system.
B. Have more upper respiratory tract viral infections
C. Smaller ostia to the sinuses
D. All of the above
E. None of the above

2. Which of the following is true regarding imaging of sinuses in children?


A. Plane films are valid form of imaging.
B. Plane films are required to make the diagnosis of sinusitis.
C. Computed tomography (CT) scans are required to make the diagnosis of
sinusitis in children.
D. The diagnosis of sinusitis is clinical and does not require radiologic
confirmation.
E. CT scans are best obtained during an acute infection.

3. Medical management is
A. Not frequently effective
B. Directed toward more resistant bacteria
C. Usually targeted toward specific bacteria, and broad-spectrum coverage
is not warranted
D. Universally effective
E. Best provided with IV therapy

4. Absolute indications for endoscopic sinus surgery include all of the


following except
A. Complete nasal airway obstruction from polyps
B. Antrochoanal polyps
C. Chronic rhinosinusitis that persists despite maximum medical management
D. Mucoceles
E. Orbital abscess

5. Which of the following is true regarding facial growth in children


after endoscopic sinus surgery?
A. Endoscopic sinus surgery does not alter facial growth.
B. Endoscopic sinus surgery results in retarded midface growth.
C. The effects of facial growth are not known and surgery should not be performed.
D. Piglet models show retarded facial growth, and it therefore is assumed that the
same will be true in children
E. Facial plastic surgeons have shown their mid-face growth retardation.

506
BY DR. MOHAMMED ATIAA KAREEM ALNASHY
-------------------------------------------------------------------
CHAPTER 179
Pediatric Chronic Sinusitis

1. D
Age is clearly one of the most significant factors in pediatric sinusitis.
Because of their immature immune system, they are more likely to have upper
respiratory tract viral infections and associated acute sinusitis. There is a
strong association between sinusitis and respiratory viral infections. Viral
infections are thought to cause significant ciliary dysfunction by decreasing
the ciliary beat frequency or destroying the ciliary blanket. This results in
edema, which obstructs the ostium and increases the chance of establishing a
bacterial infection of the sinuses. This edema will interrupt the drainage of
the anterior ethmoid sinuses and maxillary sinuses and predispose the patient
to acute and chronic sinusitis.

2D
It is now clear that plain films do not adequately image the pediatric sinuses.
In the setting of acute sinusitis, we would expect plain films and CT scans to
be positive. Gwaltney's and Glasier's work showed a high incidence of
opacification of the anterior ethmoid and maxillary sinuses with acute
rhinovirus infections. For assessing the status of sinuses, the coronal CT
remains the image method of choice. In general, sinusitis is a clinical
diagnosis, and radiographic imaging is not necessary in children to confirm
the diagnosis. CT scans should be obtained when both the parents and the
surgeon believe surgical intervention is warranted. The CT scan is used
primarily to look for anatomic abnormalities that would increase the risk of
surgical complications and to help document the presence of disease. The CT
scan should been obtained after a trial of maximum medical management
that would include broad-spectrum antibiotics and topical nasal steroid
sprays for at least 4 weeks. The CT scan should be obtained at the end of this
course of management.

507
BY DR. MOHAMMED ATIAA KAREEM ALNASHY
-------------------------------------------------------------------
3N
Chronic sinusitis is associated with more resistant bacteria and therefore will
need to be treated with broader spectrum antibiotics. For the most resistant
strains of pneumococcus, File has found the newer form of
amoxicillin/clavulanate (AMX/CA) 2000/125 mg and the fluoroquinolones
were highly active against these cultured isolates from patients with
community-acquired respiratory tract infection. These drugs, however,
should be saved for the most resistant infections.

4. C
Absolute indications include (1) complete nasal airway obstruction in cystic
fibrosis caused by massive polyposis or closure of the nose by medialization
of the lateral nasal wall; (2) antrochoanal polyp; (3) intracranial
complications; (4) mucoceles and mucopyoceles; (5) orbital abscess; (6)
traumatic injury to the optic canal; (7) dacryocystorhinitis caused by sinusitis
and resistant to medical treatment; (8) fungal sinusitis; (9) some
meningoencephaloceles; and (10) some neoplasms. Relative or possible
indications, which include most patients, are (1) chronic rhinosinusitis that
persists despite optimal medical management and after the exclusion of any
systemic disease.

5A
Bothwell and others sought to determine whether functional endoscopic sinus
(FES) surgery performed in children with chronic rhinosinusitis alters
facial growth. Sixty-seven children with a mean age of 3.1 years at
presentation were evaluated for facial growth 10 years later at a mean age of
13.2 years. In this group, there were 46 children who underwent FES surgery
and 21 children who did not undergo FES surgery and acted as a control.
Quantitative anthropomorphic analysis was performed with 12 standard
facial measurements on both groups. A facial plastic expert performed
blinded qualitative facial analysis on standardized photographs. Both
quantitative and qualitative analyses showed no trends or statistical
significance in changes of facial growth between children who underwent
FES surgery and those who had chronic sinusitis but did not undergo FES
surgery. Their data also showed no deviations, or trends toward deviation,
from the standard norms in children. They concluded there was no evidence
that FES surgery affected facial growth in children.

508
BY DR. MOHAMMED ATIAA KAREEM ALNASHY
-------------------------------------------------------------------
CHAPTER 180: SALIVARY GLAND DISEASE
--------------------------------------------------------------------------------------------

1. Which of the following inflammatory processes of the salivary glands


is least likely to occur?
A. Cat scratch disease
B. Atypical mycobacterial infections
C. Mumps
D. Reactive lymph node hyperplasia
E. Suppurative bacterial sialadenitis

2. The most common organism responsible for suppurative bacterial


sialadenitis is
A. Pseudomonas aeruginosa
B. Streptococcus pneumoniae
C. Escherichia coli
D. Bacteroides melaninogenicus
E. Staphylococcus aureus

3. The most common salivary gland neoplasm in young children is


A. Mucoepidermoid carcinoma
B. Lymphangioma
C. Lymphoma
D. Acinic cell carcinoma
E. Hemangioma

4. The most common malignant salivary gland neoplasm in children is


A. Lymphoma
B. Mucoepidermoid carcinoma
C. Squamous cell carcinoma
D. Warthin's tumor
E. Rhabdomyosarcoma

5. Treatment options for excessive salivation/chronic drooling include all


of the following except
A. Bilateral submandibular duct rerouting
B. Bilateral submandibular gland excision with parotid duct ligation
C. Sublingual gland excision
D. Bilateral parotid duct and submandibular duct ligation
E. Use of glycopyrrolate

509
BY DR. MOHAMMED ATIAA KAREEM ALNASHY
-------------------------------------------------------------------
CHAPTER 180
Salivary Gland Disease

1. C
Extensive use of the mumps vaccination has lead to a significant decline in the
number of reported cases of mumps in the United States. Although not common,
the other inflammatory processes listed occur far more often than mumps.

2. E
The most common organisms leading to acute bacterial sialadenitis are
Staphylococcus aureus and Streptococcus viridans. The use of appropriate
antimicrobial agents against these organisms is essential. The use of hydration,
massage, sialogogues, and warm compresses is also beneficial to treat this process.

3. E
Hemangioma is the most common neoplasm found in the parotid gland in the
pediatric population. These are usually discovered at birth or shortly after
birth. Physical findings include facial asymmetry and a fluctuant mass. The
hemangioma generally grows very rapidly over the third to twelfth month of
life until reaching a plateau. The lesions then remain stable for a period of
time before a slow but progressive involution. Ninety percent of hemangiomas
involute by the age of 9 years.

4B
Mucoepidermoid carcinoma is the most common malignant epithelial
salivary gland neoplasm, accounting for approximately 50% of salivary gland
malignancies. Most of these are low-grade lesions that have a good prognosis.
The treatment is surgical excision (superficial parotidectomy), including a
generous cuff of normal salivary gland tissue around it, with preservation of
the facial nerve. The facial nerve is preserved in all cases, unless the tumor is
grossly invading the nerve.

5. C
Sublingual gland excision is not a realistic treatment option for children with
excessive salivation. A good medical option is a trial of glycopyrrolate.
Surgical options include bilateral parotid duct ligation with submandibular
gland excision, bilateral parotid duct and submandibular duct ligation, or
submandibular duct rerouting.

510
BY DR. MOHAMMED ATIAA KAREEM ALNASHY
-------------------------------------------------------------------
CHAPTER ONE HUNDRED AND EIGHTY-ONE: PHARYNGITIS AND
ADENOTONSILLAR DISEASE
--------------------------------------------------------------------------------------------

1. Cultures taken from deep neck abscesses reveal


A. Staphylococcus aureus
B. Streptococcus pyogenes
C. Bacteroides
D. Polymicrobial infections (aerobic and anaerobic)
E. Pseudomonas aeruginosa

2. Which of the following statements regarding chronic adenotonsillar


hypertrophy is false?
A. Adenotonsillar hypertrophy has been associated with second-hand
smoke exposure.
B. Adenotonsillar hypertrophy has a significant association with inhalant
allergies in children younger than 3 years of age.
C. (β-lactamase-producing bacteria play a significant role in
pathologically enlarged tonsils and adenoids.
D. When studying the bacteriology of adenotonsillar hypertrophy, core
samples tend to be more accurate than surface cultures.
E. Adenotonsillar hypertrophy with airway obstruction has significantly
increased as an indication for adenotonsillar surgery in recent years.

3. Performance of adenoidectomy in the presence of an occult submucous


cleft palate may result in
A. Chronic eustachian tube dysfunction
B. The development of dental malocclusion
C. Speech articulation problems, primarily related to the nasal sounds
[M], [N], and [NG]
D. Velopharyngeal insufficiency
E. Chronic rhinosinusitis

511
BY DR. MOHAMMED ATIAA KAREEM ALNASHY
-------------------------------------------------------------------
4. Postoperative admission after adenotonsillectomy is indicated in all
patients except those
A. Younger than 3 years of age
B. With a history of snoring
C. Who live more than 90 minutes from the hospital
D. With a history of asthma
E. With a history of an underlying bleeding disorder

5. Before undergoing adenotonsillectomy, all patients with Down


syndrome should undergo which of the following tests?
A. Pulmonary function
B. Magnetic resonance imaging of the head and neck
C. Flexion and extension radiographs of the cervical spine
D. Immunoglobulin levels, including IgG subclasses
E. Rhinomanometry

CHAPTER 181
Pharyngitis and Adenotonsillar Disease
1D
2B
3D
4B
5.C

512
BY DR. MOHAMMED ATIAA KAREEM ALNASHY
-------------------------------------------------------------------
CHAPTER ONE HUNDRED AND EIGHTY-TWO: OBSTRUCTIVE
SLEEP APNEA IN CHILDREN
--------------------------------------------------------------------------------------------

1. The diagnosis of obstructive sleep apnea in children is best made by


A. Careful observation of sleep pattern by the caregivers
B. Polysomnography in a pediatric sleep laboratory
C. Assessment of tonsil and adenoid size
D. Home sleep monitoring tests
E. The presence of daytime somnolence

2. Which of the following is a clinical predictor of high risk for


respiratory compromise after adenotonsillectomy for obstructive sleep
apnea syndrome (OSAS)?
A. Severe OSAS on polysomnography
B. Young age
C. Cerebral palsy
D. Down syndrome
E. All of the above

3. A child with severe snoring is observed by skilled health providers to


have no apneic pauses during sleep. This observation
A. Effectively rules out OSAS in this child
B. Cannot exclude the diagnosis of OSAS, because obstructive hypoventilation
may be present
C. Is evidence that adenotonsillectomy should be performed
D. Is helpful only in children older than age 5
E. Combined with clinical measures of tonsil size can predict the need for
adenotonsillectomy

4. All of the following statements about primary snoring are true except
A. Risk factors include adenotonsillar hypertrophy, obesity, decreased
nasal patency, and passive smoke exposure.
B. Primary snoring does not seem to progress to OSAS.
C. Primary snoring can be distinguished from OSAS by a careful history
and physical examination.
D. Currently, treatment is not recommended for primary snoring.

5. The most common first-line treatment for pediatric OSAS is


A. Continuous positive airway pressure (CPAP)
B. Adenotonsillectomy
C. Tracheotomy
D. Supplemental oxygen

513
BY DR. MOHAMMED ATIAA KAREEM ALNASHY
-------------------------------------------------------------------
CHAPTER 182
Obstructive Sleep Apnea in Children

1B
The "gold standard" for diagnosis of OSAS in children is polysomnography in a
pediatric sleep laboratory. Tonsil and adenoid size does not directly predict the
presence of OSAS. Daytime somnolence is unusual in children with OSAS. History
and clinical examination do not distinguish between primary snoring and OSAS
in children. Home testing for OSAS in children is not widely used and needs to be
validated against studies in the sleep laboratory.

2. E
Several retrospective studies have detailed clinical risk factors for respiratory
compromise after adenotonsillectomy for OSAS. These risk factors include young
age, severe OSAS on sleep study, craniofacial anomalies, neuromotor disease, and
chromosomal abnormalities.

3B
Sleep-related upper-airway obstruction in children may manifest as obstructive apnea
or obstructive hypoventilation. Obstructive hypoventilation results from continuous
partial airway obstruction, which leads to paradoxical respiratory efforts,
hypercarbia, and often hypoxemia. Diagnosis of obstructive hypoventilation in
children requires end-tidal G02 monitoring during polysomnography. Despite the
absence of complete airway obstruction during sleep, children with obstructive
hypoventilation are at risk for all of the reported complications of OSAS. Clinical
measures of tonsil size alone do not predict the need for surgery.

4. C
Numerous studies have demonstrated that OSAS cannot be distinguished from PS
in children on the basis of clinical history and physical examination alone.
Overnight polysomnography in a sleep laboratory is the current "gold standard" for
differentiating childhood OSAS from PS. An accurate diagnosis of OSAS will
ensure that appropriate treatment is provided when needed and will avoid
unnecessary surgery in patients with PS.

5B
Adenotonsillectomy is the most common treatment of childhood OSAS and is usually
curative, especially in otherwise healthy children. Tracheotomy is sometimes
indicated for the management of severe OSAS in children with complicated anatomic
or neuromotor issues. Children who fail to respond to or are not candidates for surgical
intervention can also often be managed successfully with nasal continuous (CPAP) or
bilevel positive airway pressure. Nocturnal oxygen supplementation has been studied
as a temporary treatment for hypoxemia associated with OSAS until definitive
therapy can be provided. Supplemental oxygen therapy may suppress hypoxic
ventilatory drive and lead to significant hypercarbia in some OSAS patients, so it must
be administered with caution and initiated in a monitored setting.

514
BY DR. MOHAMMED ATIAA KAREEM ALNASHY
-------------------------------------------------------------------
CHAPTER ONE HUNDRED AND EIGHTY-THREE: PEDIATRIC
HEAD AND NECK MALIGNANCIES
--------------------------------------------------------------------------------------------

1. Certain populations of children are at increased risk for malignancies.


These include
A. Down syndrome
B. Patients who have received radiotherapy
C. Bloom syndrome
D. Hemihypertrophy
E. All of the above

2. Tissue from a biopsy of a suspected childhood tumor is best handled by


A. Placing the tissue in formalin
B. Placing the tissue in Karnofsky's solution/EM fixative
C. Snap freezing
D. Placing the tissue in a sterile container and delivering it fresh to the
pathologist

3. Routine elective neck dissection for pediatric patients with salivary


gland malignancies is
A. Always recommended
B. Recommended only if there are suspicious nodes on CT
C. Not recommended
D. Recommended depending on tumor subtype

4. Class I monostotic Langerhans' cell histiocytosis (LCH) can be treated with


A. Low-dose radiotherapy
B. Curettage
C. Steroid injection
D. Biopsy
E. All of the above

5. A post-liver transplant pediatric patient is seen with new enlargement


of the tonsils and/or adenoids. The otolaryngologist should
A. Schedule the patient for T & A
B. Observe the patient for 6 weeks
C. Start decreasing the immunosuppressive drugs
D. Perform an FNA

515
BY DR. MOHAMMED ATIAA KAREEM ALNASHY
-------------------------------------------------------------------
CHAPTER 183
Pediatric Head and Neck Malignancies

1. E
All of the patient situations listed put children at increased risk for
childhood malignancies developing on the basis of altered immune system
capabilities.

2D
Fresh tissue, with its mRNA, is necessary for the pathologist to perform
molecular genetic analysis, cytogenetics, and cell culture. This is critical in
the workup of pediatric small round blue cell tumors.

3. C
Cervical metastases are rare in pediatric salivary gland malignancies. The
presence of cervical adenopathy in a pediatric patient is more likely to reflect
reactive hyperplasia of cervical lymph nodes. Therefore, routine elective neck
dissection is not recommended. Neck dissection is performed only in the
setting of obvious nodal extension or FNA-proven metastases.

4. E
Monostotic, local LCH can be treated by all of the methods listed.
Interestingly, spontaneous resolution can also occur. Chemotherapy is
reserved for multisystem or widespread disease.

5. A
Removal of the tonsils and adenoids is often curative in the posttransplant
pediatric patient with PTLD affecting the tonsils and adenoids. The rate of
PTLD is highest in liver, heart, and heart-lung transplant recipients because
of the increased immunosuppression needed in these patients. Therefore,
observation is never warranted.

516
BY DR. MOHAMMED ATIAA KAREEM ALNASHY
-------------------------------------------------------------------
CHAPTER ONE HUNDRED AND EIGHTY-FOUR: DIFFERENTIAL
DIAGNOSIS OF NECK MASSES
--------------------------------------------------------------------------------------------

1. Second branchial cleft cysts are typically found


A. Near the angle of the mandible
B. In the posterior cervical triangle
C. High in the neck and deep to the anterior border of the sternocleidomastoid
muscle
D. Near the upper pole of the thyroid gland

2. Which radiologic study is the most efficient for identifying a normal


thyroid gland before performing excision of the thyroglossal duct cyst?
A. Technetium-99 scan
B. Iodine-131 scan
C. Magnetic resonance imaging with gadolinium
D. Ultrasonography of the neck

3. In most cases of cutaneous hemangiomas of the neck, the following are


acceptable forms of treatment except
A. Laser therapy
B. Radiation therapy
C. Watchful waiting
D. Surgical excision

4. Thymic cysts develop as a derivative of which embryologic structure


A. Second branchial arch
B. Fourth pharyngeal pouch
C. Third pharyngeal pouch
D. Third branchial arch

5. After the neonatal period, suppurative lymphadenopathy may occur as


the result of infection with any of the following organisms except
A. Pseudomonas species
B. Staphylococcus aureus
C. Haemophilus influenzae
D. Group A β-streptococcus

517
BY DR. MOHAMMED ATIAA KAREEM ALNASHY
-------------------------------------------------------------------
6. The diagnosis of cat-scratch disease is best made with which of the
following?
A. Culture of infected tissue for the bacterium, Bartonella henselae
B. History of superficial scratch by a cat
C. Serologic testing for Bartonella henselae
D. Biopsy of infected nodes looking for viral inclusions

7. The best treatment of neck lymph nodes infected by Mycobacterium


tuberculosis is
A. Surgical excision
B. Antituberculous chemotherapy with two drugs
C. Antituberculous chemotherapy with one drug
D. Incision of the infected nodes and curettage

8. All of the following clinical signs are seen acutely with Kawasaki's
disease except
A. Coronary artery aneurysms
B. Erythema, edema, and desquamation of hands and feet
C. Nonpurulent cervical adenopathy
D. Thrombocytosis

9. All of the following have been reported to cause drug-induced


lymphadenopathy except
A. Allopurinol
B. Pyrimethamine
C. Phenytoin
D. Chlorpromazine

10. All of the following may be seen with nasopharyngeal carcinoma except
A. Unilateral otitis media with effusion
B. Neck mass
C. Positive mono spot test
D. Elevated titers of Epstein-Barr virus types 2 and 3

518
BY DR. MOHAMMED ATIAA KAREEM ALNASHY
-------------------------------------------------------------------
CHAPTER 184
Differential Diagnosis of Neck Masses

1. C
First branchial derivatives are found along the mandible and third
derivatives near the upper pole of the thyroid gland.

2D
Ultrasonography of the neck is the most practical and economical of the
studies listed above to identify normal thyroid tissue.

3B
All of the modalities are useful. Treatment of cutaneous hemangiomas may
result in later malignant transformation.

4. C

5A
Pseudomonas is a virulent organism in the neonatal period but is not a
pathogen in infants and children with normal immune systems. The other
three organisms are all common pathogens in infants and children.

6. C
Serologic testing for Bartonella DNA is the best method for confirmation.
Bartonella henselae is difficult to culture, and viral inclusions within
biopsied nodes are nonspecific.

7. B
The treatment of Mycobacterium tuberculosis is with two-drug therapy. Use
of one drug fosters resistance, and surgery is not a usual option.

8. A
Coronary artery aneurysm is a late sequela of Kawasaki's disease.

9. D

10. C
While patients have elevated titers to Epstein Barr virus, a mono spot test
should be negative. Nasopharyngeal carcinoma typically metastasizes to the
neck and may produce unilateral otitis media with effusion.

519
BY DR. MOHAMMED ATIAA KAREEM ALNASHY
-------------------------------------------------------------------
CHAPTER 185: CONGENITAL DISORDERS OF THE LARYNX
--------------------------------------------------------------------------------------------

1. A child who has been identified with a laryngeal web should undergo
which type of evaluation prior to surgical repair?
A. Hearing evaluation
B. Coagulation studies
C. Renal ultrasound
D. Cardiac evaluation
E. Flexion and extension neck X-rays

2. At the time of endoscopy, a 2-month-old infant with stridor and aspiration


symptoms is found to have a laryngeal cleft that extends through the cricoid
cartilage but not into the cervical trachea. What is the proper staging of this
anomaly?
A. Armitage type 1C
B. Evans type II
C. Benjamin/Inglis type II
D. Myer/Cotton type LIII
E. All of the above

3. Which of the following is frequently associated with laryngomalacia?


A. Nasal septal deviation
B. Developmental delay
C. Gastroesophageal reflux disease
D. Auricular deformities
E. Disorders of the pituitary gland

4. Which congenital laryngeal anomaly typically presents with symptoms


and signs of aspiration?
A. Laryngomalacia
B. Laryngeal web
C. Saccular cyst
D. Bifid epiglottis
E. Laryngeal cleft

5. Which congenital anomaly has been associated with sudden infant death?
A. Thyroglossal duct cyst (in vallecula)
B. Laryngeal cleft
C. Laryngomalacia
D. Bilateral vocal cord paralysis
E. Unilateral vocal cord paralysis

520
BY DR. MOHAMMED ATIAA KAREEM ALNASHY
-------------------------------------------------------------------
CHAPTER 185
Congenital Disorders of the Larynx

1D
Cardiac evaluation. Laryngeal webs are commonly associated with the
chromosome deletion 22qll. Microscopic and submicroscopic deletions of this
chromosome cause a wide range of phenotypes, including DiGeorge syndrome,
velocardiofacial syndrome, conotruncal anomaly face syndrome, and sporadic
or familial heart defects. Features related to these syndromes include cardiac
defects, abnormal facies, thymus hypoplasia, cleft palate, and hypocalcemia
(CATCH-22). A cardiac defect should be ruled out before surgery to minimize
the risks of the procedure.

2. E
All of the above. The staging system for laryngeal clefts and
laryngotracheoesophageal clefts has evolved over the past few decades. The
most commonly used staging systems are the Benjamin/Inglis and
Myer/Cotton system. When describing intraoperative findings, the surgeon
should specify which staging system is used.

3. C
Gastroesophageal reflux disease. Laryngomalacia is commonly associated
with GERD. There is some debate whether the laryngomalacia causes the
GERD or the GERD contributes to the laryngomalacia (the proverbial
chicken and egg problem.) The presence of GERD should be considered and
possibly treated in all patients with laryngomalacia.

4. E
Laryngeal and laryngotracheoesophageal clefts. Children with laryngeal
clefts typically have significant aspiration problems, because the
liquids/foods they consume pass into the trachea and bronchi through the
cleft.

5A
Thyroglossal duct cyst in the valleculas Autopsy reports of several infants
who died in their beds have revealed a thyroglossal duct cyst at the foramen
cecum.

521
BY DR. MOHAMMED ATIAA KAREEM ALNASHY
-------------------------------------------------------------------
CHAPTER 186: MANAGING THE STRIDULOUS CHILD
--------------------------------------------------------------------------------------------

1. Management of the difficult to extubate neonate would include


A. Antireflux treatment
B. Corticosteroids
C. Cricoid split
D. Nebulized racemic epinephrine

2. Tracheotomy complications may be prevented by


A. Stay sutures in the trachea
B. Antibiotic coverage at surgery
C. Proper trach tube fitting
D. Inserting an extra-long tube initially

3. Vocal cord paralysis


A. Is more often bilateral
B. Is more commonly found in males
C. May be associated with paradoxical movement of the vocal cords
D. May be diagnosed with ultrasonography

4. Vocal cord paralysis is more often associated with


A. Birth trauma
B. Malignant disease
C. Arnold-Chiari and hydrocephalus
D. Familial disease

5. Subglottic hemangiomas
A. Are more common in males
B. Are associated with cervicofacial hemangiomas
C. Are normally present at birth
D. May be treated with steroids

522
BY DR. MOHAMMED ATIAA KAREEM ALNASHY
-------------------------------------------------------------------
CHAPTER 186
Managing the Stridulous Child
1. A, B, C
2. A, B
3. A, B, C, D
4. A, C
5 B, D

523
BY DR. MOHAMMED ATIAA KAREEM ALNASHY
-------------------------------------------------------------------
CHAPTER 187: GLOTTIC AND SUBGLOTTIC STENOSIS
--------------------------------------------------------------------------------------------

1. Laryngeal stenosis
A. Is most commonly associated with iatrogenic injury
B. Is most often found in the subglottis
C. Is associated with necrosis, ulceration, and perichondritis
D. Always occurs with prolonged intubation

2. Important factors in the development of laryngeal stenosis include


A. Duration of intubation
B. Tube size
C. Patient age
D. Tube movement
E. All of the above

3. Symptoms of laryngeal stenosis


A. Relate to airway, voice, and feedings
B. Are not progressive
C. Are best evaluated with rigid endoscopy
D. Are best evaluated with flexible endoscopy

4. Repair of laryngeal stenosis


A. Should be attempted through endoscopic means as a first alternative
B. Is individualized according to site, severity, and circumstances of the
individual patient
C. Is performed to achieve early decannulation with a good voice
D. Should be performed in all individuals

5. The type of open repair performed depends on


A. The degree of cartilaginous support remaining
B. The length of the stenosis
C. The degree of glottic involvement
D. Grade of stenosis
E. All of the above

524
BY DR. MOHAMMED ATIAA KAREEM ALNASHY
-------------------------------------------------------------------
CHAPTER 187
Glottic and Subglottic Stenosis
1. A, B, C
2. E
3. A, C
4. B, C
5. E

525
BY DR. MOHAMMED ATIAA KAREEM ALNASHY
-------------------------------------------------------------------
CHAPTER 188: GASTROESOPHAGEAL REFLUX AND
LARYNGEAL DISEASE
--------------------------------------------------------------------------------------------

1. Which of the following statements regarding GER in children is true?


A. GER disease is rare in children.
B. Postprandial GER is physiologic.
C. Silent GER can induce extra-gastrointestinal symptoms.
D. Heartburn is infrequent in children.
E. Major complications of GER are exceptional in children.

2. Which of the following laryngeal pathologic conditions is frequently


associated with GER?
A. Laryngomalacia
B. Posterior laryngeal cleft
C. Congenital subglottic stenosis
D. Acquired subglottic stenosis
E. Vallecular cyst

3. The management of GER always includes


A. Healthy diet and lifestyle
B. Small meals at regular times
C. Raising the head of the bed
D. Prokinetic drugs
E. Proton pump inhibitors

526
BY DR. MOHAMMED ATIAA KAREEM ALNASHY
-------------------------------------------------------------------
CHAPTER 188
Gastroesophageal Reflux and Laryngeal Disease

1. B, C, D
GER is frequent in children; at least 20% of children experience clinical GER
disease. Postprandial GER can be considered physiologic. In this particular
case, pH monitoring is normal. In contrast with GER in adults, heartburn is
not a frequent sign of GERD in children. However, major complications,
such as obstructive apnea, choking, and failure to thrive are not exceptional.

2. A, B, D
GER is frequently associated with laryngeal diseases. Esophageal motor
disorders are frequently associated with laryngomalacia, probably because of
the neurologic dysfunction that determines laryngomalacia. Posterior
laryngeal clefts include malformations of the tracheal muscular posterior
wall. GER seems to be important in the development of acquired subglottic
stenosis but not in pure congenital stenoses.

3. A, B, C
The first steps of GER treatment are always diet measures and lifestyle
assessment. Antireflux or antacid drugs are the second line of treatment.

527
BY DR. MOHAMMED ATIAA KAREEM ALNASHY
-------------------------------------------------------------------
CHAPTER ONE HUNDRED AND EIGHTY-NINE: ASPIRATION AND
SWALLOWING DISORDERS
--------------------------------------------------------------------------------------------

1. Normal swallowing includes which of the following stages?


A. The oral stage is the automatic sucking reflex in infants.
B. The sucking reflex disappears after 1 month.
C. A pharyngeal stage
D. An esophageal stage taking about 0.5 seconds
E. An esophageal stage taking about 5 seconds

2. Causes of swallowing disorders include


A. Esophageal atresia
B. Posterior laryngeal cleft
C. Unilateral laryngeal palsy
D. Tracheoesophageal fistula
E. Laryngomalacia

CHAPTER 189
Aspiration and Swallowing Disorders

528
BY DR. MOHAMMED ATIAA KAREEM ALNASHY
-------------------------------------------------------------------
CHAPTER 190: VOICE DISORDERS
--------------------------------------------------------------------------------------------

1. Unilateral vocal cord paralysis in children with a weak, breathy, and


hoarse voice is initially managed by
A. Gelfoam injection
B. Fat injection
C. Isshiki thyroplasty
D. Speech therapy

2. Posterior glottic stenosis and cricoarytenoid fixation


A. Are usually due to airway trauma
B. Usually results in persistent voice disorder even after surgical correction
C. Are easily diagnosed by flexible fiberoptic laryngoscopy
D. Are treated by arytenoidectomy to improve the airway and voice

3. Recurrent respiratory papillomatosis usually


A. Presents with hoarseness and/or airway obstruction
B. Is usually diagnosed after age 5
C. Is associated mainly with human papillomavirus types 9 and 16
D. Has good long-term prognosis for voice

4. Vocal cord granulomas are usually


A. Secondary to external cervical trauma
B. Can be caused by gastroesophageal reflux disease (GERD)
C. Treated surgically using C02 laser
D. Operated on to improve the voice quality

5. Epidermoid cysts of vocal cord


A. Are usually bilateral
B. Improve with speech therapy
C. Are often mistaken for vocal cord nodules
D. Stroboscopy shows decreased vibrating amplitude and complete glottic
closure

529
BY DR. MOHAMMED ATIAA KAREEM ALNASHY
-------------------------------------------------------------------
CHAPTER 190
Voice Disorders

1B
Unilateral vocal cord paralysis rarely requires airway intervention. Vocal
cord paralysis spontaneous recovers over 6 to 12 months by contralateral
compensation. Recovery is hastened by speech therapy. Only few patients with
persistent dysphonia or aspiration will need surgery such as vocal cord
injection, medialization, or reinnervation.

2B
Posterior glottic stenosis in children is most commonly secondary to airway
trauma from intubation. To distinguish cricoarytenoid fixation from vocal
cord paralysis, palpation of the cricoarytenoid joint at rigid endoscopy is
necessary. Arytenoidectomy should be avoided to prevent aspiration,
deterioration of voice, and difficulty with future airway repair. The surgical
procedure of choice is anterior and posterior cricoidotomy with posterior
graft. The postoperative voice is functional; however, persistent hoarseness
and breathiness are common.

3. A
Recurrent respiratory papillomatosis (RRP) is usually diagnosed between
ages 2 and 3 years and is most often associated with human papillomavirus
types 6 and 11. RRP often involves the laryngeal surface of the epiglottis, the
upper and lower margins of the ventricles, and the undersurface of the vocal
cords, thus resulting in symptoms of hoarseness and airway obstruction. The
need for repeated procedures usually causes scarring of the vocal cords and
long-term deterioration of the voice.

4B
Vocal cord granulomas are often secondary to intubation trauma and are
causally related to GERD. Most granulomas resolve with medical
management of GERD. Surgical removal of vocal cord granuloma is reserved
for lesions causing airway obstruction. Regardless of the surgical technique
chosen, recurrences are common.

5. C
Epidermoid cysts are unilateral but occur with associated edema of the
contralateral vocal cord and thus are often mistaken for vocal nodules.
Laryngeal stroboscopy shows decreased vibrating amplitude and incomplete
glottic closure. Epidermoid cysts do not resolve spontaneously and require
microsurgical excision.

530
BY DR. MOHAMMED ATIAA KAREEM ALNASHY
-------------------------------------------------------------------
CHAPTER ONE HUNDRED AND NINETY-ONE: CONGENITAL
DISORDERS OF THE TRACHEA
--------------------------------------------------------------------------------------------

1. What is the classic presentation of an infant with tracheal stenosis?


A. Inspiratory stridor
B. Expiratory stridor
C. Cough and feeding difficulty
D. Biphasic stridor with a marked expiratory component
E. Hoarseness

2. Primary tracheomalacia may present in which group(s) of patient(s)?


A. Premature infants
B. Infants with connective tissue disorders
C. Healthy full-term infants
D. A and B
E. All of the above

3. Which of the following is the most common presentation of


tracheoesophageal fistula?
A. Pure atresia
B. Esophageal atresia with proximal esophagotracheal fistula
C. Esophageal atresia with distal esophagotracheal fistula
D. Esophageal atresia with double fistula
E. H-type fistula without true esophageal atresia

4. Which of the following condition(s) is/are often seen after repair of


tracheoesophageal fistula?
A. Tracheomalacia
B. Esophageal dysmotility
C. Gastroesophageal reflux
D. None
E. A, B, C

5. Which of the following is the most common form of vascular


compression of the tracheobronchial tree?
A. Innominate artery compression
B. Double aortic arch
C. Right aortic arch
D. Anomalous subclavian artery
E. Pulmonary artery sling

531
BY DR. MOHAMMED ATIAA KAREEM ALNASHY
-------------------------------------------------------------------
CHAPTER 191
Congenital Disorders of the Trachea

1B
Most infants with tracheal stenosis have stridor and respiratory symptoms in
the prenatal period. Classically, biphasic stridor with a marked expiratory
component is described. Associated symptoms may include cough, wheezing,
apnea, croup, and feeding difficulty.

2. E
Primary tracheomalacia can be seen in premature infants, in infants with
connective tissue disorders, and in otherwise healthy full-term infants. It is
a weakness of the tracheal wall, resulting in marked exaggeration of
movement with respiration. The clinical presentation typically includes
expiratory stridor, often reminiscent of asthmatic wheezing, and varying
degrees of respiratory distress.

3. C
The most common presentation of tracheoesophageal fistula is proximal
esophageal atresia with distal tracheoesophageal fistula.

4. E
Postoperatively, patients often have symptoms of tracheomalacia, esophageal
dysmotility, and gastroesophageal reflux. Tracheomalacia is related to
tracheal cartilages having an indented semicircular rather than a normal
horseshoe shape and a flaccid posterior membranous wall. These symptoms
often resolve with time.

5A
Innominate artery compression is the most common form of vascular
compression. It occurs when the artery arises more to the left than usual,
passing from left to right and compressing the anterior wall of the trachea.

532
BY DR. MOHAMMED ATIAA KAREEM ALNASHY
-------------------------------------------------------------------
CHAPTER ONE HUNDRED AND NINETY-TWO: TRACHEAL STENOSIS
-------------------------------------------------------------------------------------------

1. The trachea develops from


A. An evagination of foregut mesenchyme
B. An invagination of embryonic ectoderm
C. Direct extension of midgut endoderm
D. The fourth pharyngeal arch
E. The sixth pharyngeal arch

2. All of the following factors are thought to contribute to laser therapy


failure except
A. Circumferential scarring
B. Mature fibrosis
C. Fibrosis greater than 1 cm in vertical length
D. Tracheomalacia
E. Severe bacterial infection in the presence of a tracheostomy

3. If rib cartilage is unavailable to be used as a tracheal graft material,


what other autologous material may be used?
A. Periosteum
B. Esophagus
C. Pericardium
D. Dura
E. All of the above

4. According to the author, resection of greater than three to five


cartilaginous rings in children necessitates
A. Wedge resection
B. Slide tracheoplasty
C. Laryngeal release
D. Perihilar release
E. Dissection of pulmonary vasculature

5. All of these methods are acceptable for dealing with long segment
tracheal stenosis in children except
A. Slide tracheoplasty
B. Augmentation with autologous graft
C. Cadaveric human tracheal homograft
D. Free tracheal autograft
E. Synthetic tracheal prostheses

533
BY DR. MOHAMMED ATIAA KAREEM ALNASHY
-------------------------------------------------------------------
CHAPTER 192
Tracheal Stenosis
1. A
2. B
3. E
4. C
5. E

534
BY DR. MOHAMMED ATIAA KAREEM ALNASHY
-------------------------------------------------------------------
CHAPTER ONE HUNDRED AND NINETY-THREE: CAUSTIC INGESTION
--------------------------------------------------------------------------------------------

1. Which of the following is most likely to lead to severe esophageal


injury in children even when swallowed in small amounts?
A. Crystalline NaOH drain cleaner
B. Hydrochloric acid drain cleaner
C. Liquid NaOH drain cleaner
D. Sulfuric acid
E. Concentrated ammonia

2. Which of the following signs is most likely to be associated with severe


esophageal injury after ingestion of liquid NaOH?
A. Drooling
B. Fever
C. Oral mucosal burns
D. Stridor
E. Cough

3. An 18-month-old child is seen with dysphagia in the emergency


department at 10:00 PM. He appears in no distress, but according to his
parents has refused all food and drink for the past 14 hours. Results of
his examination are normal, and you decide to order a chest radiograph
to check for a foreign body. A small metallic disk in the midesophagus
looks suspiciously like a battery. What is the best plan of action?
A. Observation with recheck in the morning after another chest radiograph
B. Removal by endoscopy in the morning as an elective procedure
C. Immediate removal in the operating room that evening by rigid endoscopy
D. Attempted removal with a small transoral Foley catheter placed distal to the
disk and then slowly pulled out
E. Immediate fiberoptic esophagoscopy with the patient under sedation

4. Esophagoscopy of a young child witnessed to have swallowed liquid


sodium hydroxide reveals a circumferential exudative mucosal burn at
the midesophageal level. What is the most accurate stage of this lesion?
A. First degree
B. Second degree
C. Third degree
D. Fourth degree

535
BY DR. MOHAMMED ATIAA KAREEM ALNASHY
-------------------------------------------------------------------
5. Esophagoscopy of a teenager who drank a small amount of liquid drain
cleaner reveals a midesophageal third-degree burn. What is probably
the most important pharmacologic therapy for this injury?
A. Broad-spectrum antibiotics
B. Pharmacologic therapy has not been proven effective
C. 60 mg or oral prednisone for 2 weeks and then taper
D. Aggressive antireflux medications (e.g., omeprazole)
E. Penicillamine

6. Esophagoscopy of a lye burn reveals blackened, necrotic mucosa


through the full length of the esophagus. What additional procedures
should be performed?
A. Gastroscopy with a fiberoptic scope to see whether debridement of
necrotic stomach tissue is necessary
B. Placement of a gastrostomy tube and trans-esophageal/gastric string for
future retrograde dilations
C. Strong consideration of a thoracotomy to rule out transmural necrosis
of the esophagus
D. All of the above
E. A and B

7. At approximately what pH is the threshold for alkalinity severe


enough to cause rapid mucosal necrosis?
A. 13
B. 10
C. 12.5
D. 13.5

8. What type of injury is characteristic of alkaline burns?


A. Granulomatous inflammatory necrosis
B. Liquefactive necrosis
C. Coagulative necrosis
D. Ischemic necrosis

536
BY DR. MOHAMMED ATIAA KAREEM ALNASHY
-------------------------------------------------------------------
CHAPTER 193
Caustic Ingestion

1. C
Liquid NaOH quickly comes in contact with mucosa distally. Due to its
alkaline nature, it leads to a mucosa injury in seconds.

2D
Stridor indicates laryngeal edema and is evidence that the alkaline substance
passed to or (generally) beyond the level of the cricoid, and thus the
esophageal inlet.

3. C
It is best to immediately remove any coin-shaped object that has the potential
to be a battery, because injury from a battery is progressive and quite
destructive. Direct observation with airway protection during removal is
paramount for safety and prevention of further injury to a weakened
esophageal wall.

4. C
At a minimum, this represents a circumferential burn. The exact depth may
be difficult to immediately determine.

5D
Esophageal reflux must be maximally controlled in patients with esophageal
burns from ingested agents to minimize stricture formation.

6. D
Such a severe burn can extend into the mediastinum and stomach, with
complete necrosis of the esophagus.

7. C
This is based on work by Vancura and others (Toxicity of alkaline solutions,
Ann Emerg Med 9:118, 1980).

8B
This prevents a protective eschar that might limit extent of injury from
forming. This leads to a deeper, more severe injury per amount consumed
when compared to similar acid burns.

537
BY DR. MOHAMMED ATIAA KAREEM ALNASHY
-------------------------------------------------------------------
CHAPTER 194: FOREIGN BODIES OF THE AIRWAY AND ESOPHAGUS
--------------------------------------------------------------------------------------------

1. Which of the following statements regarding bronchial foreign bodies is true?


A. Bronchial foreign bodies are more prevalent than esophageal foreign bodies.
B. The three phases of foreign body aspiration are the initial, asymptomatic, and
complication phases.
C. Radiographs are essential in ruling out a bronchial foreign body.
D. Hyperinflation of the affected lung from a check-valve effect is a late finding.
E. Optical forceps aid the endoscopist by increasing the ventilatory capacity of the
bronchoscope.

2. Which of the following statements regarding esophageal foreign bodies is false?


A. Coins are the most common esophageal foreign body.
B. There is an increased probability of an esophageal anomaly in a patient with
multiple foreign bodies of the esophagus.
C. Most esophageal foreign bodies lodge at the gastroesophageal junction.
D. An esophageal foreign body may cause an acute airway obstruction.
E. An esophageal foreign body may migrate to an extraluminal position.

3. Regarding esophageal perforations, which of the following statements is false?


A. Esophageal perforation may occur from pill ingestion.
B. Esophageal perforations may occur from the length of time an object is lodged.
C. Perforation of the esophagus is a risk of esophageal foreign body removal.
D. Fever, tachycardia, and pain may be signs of an early perforation.
E. All esophageal perforations require open drainage to manage adequately.

4. Disk batteries
A. May cause injury to the esophageal mucosa in as little as 1 hour
B. Can be removed within 24 to 48 hours after ingestion
C. Are commonly aspirated into the bronchi
D. Cannot be localized with radiographs in suspected ingestions
E. Greater than 15 mm have a greater chance to pass through the gastrointestinal
tract

5. Which of the following regarding foreign body emergencies of the airway and
esophagus are true?
A. Impaction of a disk battery within the esophagus is not a true emergency.
B. The Heimlich maneuver should only be performed on an esophageal foreign
body impaction where the airway is not obstructed.
C. Public education regarding choking emergencies has not decreased mortality of
acute airway obstruction.
D. Food objects do not cause acute airway obstruction.
E. The Heimlich maneuver should be performed with back blows and chest thrusts
in children younger than 1 year of age.

538
BY DR. MOHAMMED ATIAA KAREEM ALNASHY
-------------------------------------------------------------------
CHAPTER 194
Foreign Bodies of the Airway and Esophagus

1. B
Esophageal foreign bodies are more common. The three phases of foreign body
aspiration are the initial, asymptomatic, and complication phases. Radiographs
may aid in the diagnosis of bronchial foreign body but cannot absolutely rule out
the presence of a foreign object. Hyperinflation of the affected lung is an early
finding. Optical forceps aid in visualization during bronchial foreign body removal
but can actually decrease the ventilatory capacity of the bronchoscope because of
the increased diameter of the forceps.

2. C
Coins are the most common esophageal foreign body. Multiple objects found in the
esophagus are associated with an esophageal anomaly in up to 80% of patients. Most
esophageal foreign body impactions occur in the cervical esophagus below the
cricopharyngeus muscle. Airway obstruction can result from direct compression of
the trachea anteriorly by the object or by inflammation. A longstanding esophageal
foreign body may migrate to an extraluminal position.

3. E
Esophageal perforations may be caused by the object itself, the length of time the
object is lodged, or by the attempt at removal. Pill ingestion has been reported to
result in esophageal perforation. Fever, tachycardia, and increased pain may
indicate an early esophageal perforation. Open drainage may be necessary to treat
an esophageal perforation but is not always indicated.

4. A
Disk battery ingestions that impact in the esophagus may cause esophageal injury
in as quickly as 1 hour and need to be removed as quickly as possible. Disk batteries
are usually ingested and rarely are seen as a bronchial foreign body. Radiographs
are helpful to localize disk battery impaction within the esophagus. Disk batteries
<15 mm in size may traverse through the gastrointestinal tract.

5. E
Disk battery impaction within the esophagus is an emergency, and the battery
should be removed as quickly as possible. Public education has decreased mortality
from acute airway obstruction from foreign bodies. Food and nonfood objects may
cause acute airway obstruction. The Heimlich maneuver should only be performed
in complete airway obstruction from a foreign object; in children younger than 1
year, it should be performed with back blows and chest thrusts.

539
BY DR. MOHAMMED ATIAA KAREEM ALNASHY
-------------------------------------------------------------------
CHAPTER ONE HUNDRED AND NINETY-FIVE: INFECTIONS OF
THE AIRWAY
--------------------------------------------------------------------------------------------

1. What is the narrowest portion of the upper respiratory tract in a child?


A. Glottis
B. Subglottis
C. Supraglottis
D. Trachea
E. Oropharynx

2. Inspiratory stridor, hoarseness, and barking cough are the cardinal


symptoms of an illness most often caused by which organism?
A. Influenza A and B
B. Haemophilus influenzae type B
C. Parainfluenza virus 1 and 2
D. Moraxella catarrhalis
E. Respiratory syncytial virus

3. The "steeple sign" seen the AP view of radiographs of the upper airway
is associated with which illness?
A. Epiglottitis
B. Supraglottitis
C. Peritonsillar abscess
D. Retropharyngeal abscess
E. Croup

4. If supraglottitis is suspected, what should be the first course of action?


A. Establish IV access
B. Perform nasopharyngolaryngoscopy to confirm diagnosis
C. Culture the surface of the epiglottis
D. Establish airway through endotracheal or nasotracheal intubation
E. Perform emergency tracheotomy

5. A recent report found Moraxella catarrhalis to be the most frequently


implicated pathogen in which of the following disease entities?
A. Epiglottitis
B. Bacterial tracheitis
C. Spasmodic croup
D. Retropharyngeal abscess
E. Laryngotracheitis

540
BY DR. MOHAMMED ATIAA KAREEM ALNASHY
-------------------------------------------------------------------
CHAPTER 195
Infections of the Airway

1. B

2. C

3. E

4. D

5. B

541
BY DR. MOHAMMED ATIAA KAREEM ALNASHY
-------------------------------------------------------------------
CHAPTER 196: RECURRENT RESPIRATORY PAPILLOMATOSIS
--------------------------------------------------------------------------------------------------

1. Although most of the human papillomavirus (HPV) genetic types can infect
respiratory mucosa, which of the following pairs of HPV types is most
closely associated with recurrent respiratory papillomatosis?
A. HPV 31, HPV 33
B. HPV 6, HPV 11
C. HPV 16, HPV 18
D. HPV 5, HPV 10
E. HPV 79, HPV 84

2. Which of the following statements concerning recurrent respiratory


papillomatosis is true?
A. Once RRP goes into remission, the patient is cured for life.
B. Cesarean section delivery always prevents maternal-fetal transmission.
C. RRP is a benign disease and poses no malignant risk.
D. RRP of adults is generally more aggressive than RRP in children.
E. There is a close association between maternal condylomata and the
development of recurrent respiratory papillomatosis (RRP) in her offspring.

3. In which of the following anatomic regions does RRP pose the worst prognosis?
A. Pulmonary
B. Larynx
C. Soft palate
D. Oral vestibule
E. Nasal vestibule

4. Which of the following therapeutic modalities has consistently been


shown to eradicate RRP-HPV?
A. C02 laser
B. Microdebrider
C. Photodynamic therapy
D. No single modality has been consistently shown to efficiently eradicate
RRP-HPV.
E. Cidofovir
F. Interferon

5. All of the following signs and/or symptoms are often associated with the new
diagnosis of laryngeal papillomatosis in a child except
A. Dysphonia
B. Stridor
C. Respiratory distress
D. Cough
E. Snoring

542
BY DR. MOHAMMED ATIAA KAREEM ALNASHY
-------------------------------------------------------------------
CHAPTER 196
Recurrent Respiratory Papillomatosis

1B
HPV types 16 and 18 have been associated with malignancies of the aerodigestive
tract and the cervix. HPV types 6 and 11 are most closely associated with RRP and
are generally not associated with malignancies. However, RRP can undergo
malignant degeneration. HPV types 31 and 33 are somewhere between HPV 6/11
and 16/18 in malignant potential.

2. E
HPV types 6 and 11 are associated with greater than 90% of genital condylomata,
and there is a strong association between maternal condylomata and the
transmission to her offspring. Overt condylomata are seen in more than 50% of
mothers who give birth to children with RRP. There has been no demonstration of
the eradication of HPV by any treatment modality. HPV has been demonstrated
even in histologically normal mucosa, and RRP may recur at any age after
remission. Cesarean-section delivery does not completely eliminate the risk of
maternal-fetal transmission, suggesting that there may be placental transmission
in some rare cases. Malignant degeneration has been shown to occur rarely in
patients with RRP. In a reported series of 244 patients with RRP, 4 underwent
documented malignant transformation (1.6%). In general, the younger the age at
diagnosis, the more aggressive the RRP clinical course should be.

3. A
The finding of pulmonary dissemination of RRP is a grave development. The
clinical course of pulmonary spread of RRP is insidious and may progress over
years but eventually manifests in respiratory failure from destruction of lung
parenchyma. Furthermore, pulmonary dissemination is anecdotally associated
with a higher risk of malignant transformation of RRP. The other sites listed pose
no special consideration for prognosis.

4D
Although multiple methods of treatment have been tested, and many adjuvant
therapies are in ongoing clinical trials, no single modality or combination of
modalities has been consistently shown to eradicate RRP HPV or to guarantee
long-term remission.

5. E
The cardinal triad of new-onset laryngeal RRP includes relentlessly progressive
hoarseness accompanied by the development of inspiratory stridor, progressing to
respiratory distress. Cough is a frequently associated symptom. Snoring, in contrast,
is more a manifestation of upper airway (nasopharyngeal/ oropharyngeal)
obstruction.

543
BY DR. MOHAMMED ATIAA KAREEM ALNASHY
-------------------------------------------------------------------
CHAPTER ONE HUNDRED AND NINETY-SEVEN: EARLY
DETECTION AND DIAGNOSIS OF INFANT HEARING IMPAIRMENT
--------------------------------------------------------------------------------------------

1. All of the following statements supporting the use of universal newborn


hearing screening are true except
A. Fifty percent of children with moderate to profound hearing loss
exhibit no risk factors for hearing loss.
B. Infants with hearing loss enrolled in appropriate early intervention
have significantly fewer developmental delays.
C. There is a significant delay in identification of deaf infants without
risk factors for hearing loss.
D. Congenital hearing loss accounts for more than half the cases of
significant hearing loss in children 3 to 17 years of age.
E. Reliance on physician observation and/or parental recognition has not
been successful in the past in detecting significant hearing loss in the
first year of life.

2. Auditory brain stem response (ABR) morphology is affected by which


of the following?
A. Sleep
B. Attention
C. Sedation
D. Age
E. Gender of child

3. Which of the following correctly describes a reason why ABR might


be preferred over otoacoustic emission (OAE)?
A. OAE frequently misses hearing losses in which hearing is normal at
some frequency.
B. OAE may miss inner hair cell and eighth nerve hearing losses.
C. ABR takes less time to perform.
D. ABR is unaffected by neurologic pathology.
E. ABR provides information over a broader frequency range.

544
BY DR. MOHAMMED ATIAA KAREEM ALNASHY
-------------------------------------------------------------------
4. Most cases of genetic deafness exhibit which inheritance pattern?
A. Autosomal recessive
B. Autosomal dominant
C. X-linked
D. Mitochondrial
E. Paternal

5. Which of the following is a considered a contraindication to cochlear


implantation?
A. Age <1 year
B. Vestibular nerve aplasia
C. Dilated vestibular aqueduct
D. All of the above
E. None of the above

CHAPTER 197
Early Detection and Diagnosis of Infant Hearing Impairment
1. D

2D

3B

4. A

5B

545
BY DR. MOHAMMED ATIAA KAREEM ALNASHY
-------------------------------------------------------------------
CHAPTER 198: CONGENITAL MALFORMATIONS OF THE INNER EAR
--------------------------------------------------------------------------------------------

1. When do most malformations of the inner ear occur?


A. When formation of the membranous labyrinth is interrupted during the first
trimester
B. When formation of the organ of Corti is interrupted during the second trimester
C. When maturation of the sensory epithelium is interrupted during the third
trimester
D. When formation of the otic capsule is interrupted during the first trimester
E. When ossification of the otic capsule is interrupted during the second trimester

2. Teratogenic influences known to affect inner ear organogenesis include


all of the following except
A. Rubella
B. Cytomegalovirus
C. Thalidomide
D. Radiation exposure
E. All of the above are known to affect inner ear organogenesis.

3. What is the most frequent histopathologic finding in congenital deafness?


A. Cochlear aplasia
B. Complete labyrinthine aplasia
C. Complete membranous labyrinthine dysplasia
D. A deformed inner ear in which the cochlea and vestibule are confluent,
forming an ovoid cystic space without internal architecture
E. Incomplete development of the pars inferior

4. What is the most common radiographically detectable malformation of


the inner ear?
A. Complete absence of the cochlea
B. Enlargement of the vestibular aqueduct
C. Aberrant semicircular canals
D. Congenitally large internal auditory canal (IAC)
E. Cochlear aqueduct dilation

5. Which procedure is indicated as a first attempt in transotic


cerebrospinal fluid (CSF) leakage when hearing is poor?
A. Tympanotomy with overlaying of connective tissue graft at the site of leakage
B. Posterior fossa craniotomy with placement of muscle plug in the IAC
C. Direct approach with tympanotomy, removal of footplate, and obliteration
of vestibule
D. Hypotympanic approach to ablation of cochlear aqueduct
E. Closure of eustachian tube

546
BY DR. MOHAMMED ATIAA KAREEM ALNASHY
-------------------------------------------------------------------
CHAPTER 198

Congenital Malformations of the Inner Ear

1. A

2. E

3. E

4B

5. C

547
BY DR. MOHAMMED ATIAA KAREEM ALNASHY
-------------------------------------------------------------------
CHAPTER ONE HUNDRED AND NINETY-NINE A: RECONSTRUCTION
SURGERY OF THE EAR: MICROTIA RECONSTRUCTION
--------------------------------------------------------------------------------------------

1. Which of the following statements regarding microtia is false?


A. Approximately half of patients have an associated congenital syndrome.
B. Unilateral cases outnumber bilateral cases.
C. There is a left ear predominance, and girls are affected more often than boys
at roughly a 2.5:1 ratio.
D. Certain populations, such as Navaho Indians and Japanese, may have a
higher incidence.
E. Incidence is 1 in 10,000 to 20,000 live births.

2. Which of the following ear embryology statements is true?


A. Most of the central ear is derived from hillocks 4 and 5.
B. The external ear begins to form during the eighth week of gestation.
C. The lobule seems to be one of the first parts of the ear to develop.
D. The auricle begins as six small buds of mesenchyme surrounding the
dorsal end of the second pharyngeal cleft.
E. The first pharyngeal arch forms about 85% of the auricle.

3. Which of the following anatomic statements is false?


A. Ear protrusion from the mastoid is 1.5 to 2.0 cm, creating an angle of
15 to 20 degrees.
B. The ear is in a slightly more vertical orientation than the nasal dorsum.
C. Normal ear height at maturity is 5.5 to 6.5 cm and is attained at ages 15
for boys and 13 for girls.
D. The horizontal width of the ear is achieved at a later age than the
ventral height.
E. The distance from the lateral palpebral fissure to the root of the helix
should approximate the length of the normal ear at maturity

548
BY DR. MOHAMMED ATIAA KAREEM ALNASHY
-------------------------------------------------------------------
4. In classic microtia reconstruction, which of the following describes the
correct staging order?
A. Cartilage implantation, posterior skin graft, lobule transfer, tragus
reconstruction
B. Tragus reconstruction, cartilage implantation, posterior skin graft, lobule
transfer
C. Lobule transfer, tragus reconstruction, cartilage implantation, posterior
skin graft
D. Lobule transfer, cartilage implantation, posterior skin graft, tragus
reconstruction
E. Cartilage implantation, lobule transfer, posterior skin graft, tragus
reconstruction

5. Which of the following is true for stage 1 microtia repair?


A. Donor site incision is made on the ipsilateral chest.
B. Cartilage from a floating rib is used to create the helix.
C. The perichondrium is removed from the cartilage framework to
improve helical and antihelical details.
D. A thick recipient pocket flap is desirable to cover and protect the
cartilage framework.
E. The lobule is elevated and trimmed until it is symmetric with the
contralateral ear.

CHAPTER 199A
Reconstruction Surgery of the Ear: Microtia Reconstruction

1. C

2. A

3. D

4. E

5. B

549
BY DR. MOHAMMED ATIAA KAREEM ALNASHY
-------------------------------------------------------------------
CHAPTER ONE HUNDRED AND NINETY-NINE B: RECONSTRUCTION
SURGERY OF THE EAR: AUDITORY CANAL AND TYMPANUM
--------------------------------------------------------------------------------------------

1. Which of the following is not a major malformation that would


preclude atresiaplasty?
A. Poor pneumatization
B. Malleus/incus complex fixation
C. Abnormal course of the facial nerve
D. Inner ear deformity

2. In atresiaplasty surgery, landmarks routinely identified for drilling


the new ear canal include all of the following except
A. The glenoid fossa
B. The tegmen
C. Lateral semicircular canal
D. Malleus/incus complex

3. The most common complication following atresiaplasty is


A. Facial nerve injury
B. Sensorineural hearing loss
C. Devascularization of cartilage graft used for microtia repair
D. Tympanic membrane graft lateralization

4. The stapes footplate is derived from which embryologic structure (s)?


A. Meckel's cartilage (first branchial arch)
B. Reichert's cartilage (second branchial arch)
C. Otic capsule
D. A and B
E. B and C

5. The most common cause of inoperability in congenital aural atresia is


A. Poor pneumatization
B. Abnormal facial nerve
C. Inner ear deformity
D. Absence of the oval window

550
BY DR. MOHAMMED ATIAA KAREEM ALNASHY
-------------------------------------------------------------------
CHAPTER 199B
Reconstruction Surgery of the Ear: Auditory Canal and Tympanuum

1B
The malleus/incus complex is deformed and fixed in cases of congenital aural
atresia, but this does not preclude successful atresiaplasty. Most often, the
malleus/incus complex can be mobilized, and the tympanic membrane graft is
placed directly on the ossicular mass. In exceptional cases, a severe deformity or
fixation prevents use of the malleus/incus complex, and a partial ossicular
reconstruction prosthesis can be used with a degree of success. Major
malformations that signify poor candidacy for atresiaplasty include poor
pneumatization, abnormal or absent oval window/footplate, abnormal course of the
facial nerve, abnormalities of the inner ear.

2. C
The superior landmark for drilling the ear canal is the tegmen, the anterior
landmark is the glenoid fossa, and the medial landmark is the malleus/incus
complex in the epitympanum. The lateral semicircular canal is not identified in a
standard surgical approach until the epitympanum and ossicular mass have been
localized.

3D
Tympanic membrane graft lateralization is the most common complication
of atresiaplasty, occurring in up to 22% of cases. Sensorineural hearing loss occurs
in 2% or fewer cases, and with the routine use of preoperative high-resolution
temporal bone CT scanning and facial nerve monitoring, injury to the facial nerve
is rare. Auricular reconstruction precedes atresiaplasty because of the demand for
an excellent blood supply for the rib cartilage graft. Devascularization of cartilage
grafts used for auricular reconstruction rarely occurs after atresiaplasty.

4. E
The stapes footplate derives from both the second branchial arch and from the otic
capsule. This dual embryologic origin may account for the fact that in most cases
of con-gential aural atresia, the footplate is normal and mobile. Meckel's cartilage
gives rise to the neck and head of the malleus and the body of the incus, whereas
Riechert's cartilage forms the long processes of the malleus and incus and the stapes
superstructure.

5. A
Poor pneumatization is the primary cause for inoperability in congenital atresia;
however, most patients have a well-pneumatized tympanic cavity/mastoid air cell
system. Inner ear, facial nerve, and oval window/footplate anomalies are seen less
frequently. A high-resolution temporal bone CT scan identifies these significant
abnormalities that would preclude surgery and is necessary for all elective cases of
atresiaplasty.

551
BY DR. MOHAMMED ATIAA KAREEM ALNASHY
-------------------------------------------------------------------
CHAPTER TWO HUNDRED: ACUTE OTITIS MEDIA AND OTITIS
MEDIA WITH EFFUSION
--------------------------------------------------------------------------------------------

1. Which of the following statements regarding the epidemiology of OM is false?


A. The highest prevalence of all forms of OM occurs in the first 2 years of life.
B. In the United States, a large majority of children will have at least one bout
of AOM in their childhood, but less than 10% will be otitis-prone.
C. Day-care attendance is the most important risk factor for OM.
D. There are racial differences in the incidence of OM.
E. OME is uncommon in newborns.

2. With regard to the sequelae of chronic OME, which of the following are true?
A. Studies have clearly shown that the hearing loss associated with chronic
OME has adverse effects on speech, language, and cognitive development.
B. Studies have clearly shown that interventions with hearing-sparing
therapies such as tympanostomy tubes greatly mitigate adverse effects of
chronic OME-related hearing loss on speech, language, and cognitive
development.
C. Children with frequent upper respiratory infections not complicated by
OME show less cognitive delays than similarly afflicted children with OME.
D. There is a clear-cut relationship between socioeconomic status and the effect
of chronic OME on cognitive development.
E. None of the above

3. With regard to antibiotic therapy and AOM, which are true?


A. With a mixed age group and a diagnosis of AOM made on clinical grounds,
approximately 80% of patients not treated with antibiotics and 94% of patients
treated with antibiotics will be symptom free at 7 to 14 days.
B. Children younger than 2 years old are more likely to benefit from antibiotic
therapy than older children.
C. The use of broad-spectrum antibiotics clearly improves the response rates
when treating AOM.
D. Response rates are improved with the use of longer courses of antibiotics.
E. The risk of withholding antibiotics clearly outweigh the risks of
administering antibiotics in the treatment of AOM.

552
BY DR. MOHAMMED ATIAA KAREEM ALNASHY
-------------------------------------------------------------------
4. Regarding tympanostomy tubes, which of the following statements are false?
A. The rate of recurrent AOM is roughly halved for the duration the tubes
remain in situ.
B. The hearing loss produced by OME is effectively reversed by the insertion of
tubes in most instances.
C. The benefit to hearing and reduced rate of infection may extend for a period
of months after extrusion of the tubes.
D. Nearly half of all children who undergo tympanostomy tube placement will
have at least one bout of otorrhea at some time after the postsurgical period
while the tubes are in place.
E. The perforation rate for grommet-style tubes is less than 5%, and greater with
T-shaped tubes.

5. Which of the following are relative indications for tympanostomy tube


placement?
A. Cleft palate
B. Eustachian tube dysfunction and underlying sensorineural hearing loss
C. AOM complicated by facial paralysis
D. Severe tympanic membrane retraction
E. Hyperbaric therapy in patients not requiring airway support
F. All of the above

553
BY DR. MOHAMMED ATIAA KAREEM ALNASHY
-------------------------------------------------------------------
CHAPTER 200
Acute Otitis Media and Otitis Media with Effusion

1. B
Studies suggest that more than 80% of children will have at least one bout of OM in
their childhood, and approximately 25% will have six or more bouts. Although OM
can occur at any age, it is far more prevalent in children younger than 2 who attend
daycare. It is more common in Native Americans and rare in newborns.

2. E
The effect of the hearing loss produced by OME on speech, language, and cognitive
development has been hotly debated. Studies show conflicting results and long-term
adverse effects in the typical patient are likely to be subtle at most. Furthermore,
whether these differences, if present, can be attributed to the hearing loss, or to
other factors, such as frequent coexistent illness, has not been settled. In this
setting, any effect by tympanostomy tubes would clearly be difficult to tease out.

3. A, B
In general, most patients will have a spontaneous resolution of the bulk of their
symptoms within 2 to 3 days of the onset whether they receive antibiotics or not.
The minority of patients who do not improve is greatest in younger children (2
years old or younger) with culture-proven disease found by tympanocentesis. In this
subset, more than 40% may have pain and fever lasting over 7 days, and these
children are most likely to be helped by antibiotic therapy. In studies using less-
rigorous inclusion criteria, there seems to be little improvement in the naturally
good outcome by using broad-spectrum antibiotics or longer courses of treatment,
and initial therapy with analgesics alone may be considered in older children.

4. C
Tympanostomy tubes confer no protection after they are extruded. The remainder
of the statements are true.

5. F
All of the answers are factors that may warrant intervention with tympanostomy
tubes. Patients with cleft palate frequently have long-standing eustachian tube
dysfunction, and long-lasting tympanostomy tubes should be considered for the
initial insertion. The hearing loss associated with eustachian tube dysfunction may
greatly complicate rehabilitative efforts for patients with an underlying
sensorineural hearing loss. Reliable drainage of the middle ear by tympanostomy
tube placement may simplify management of suppurative complications of AOM.
Tympanostomy tube placement may be an alternative to or may augment
tympanoplasty in patients with severe TM retractions with impending
cholesteatoma of the pars tensa. Approximately half of conscious patients
undergoing hyperbaric oxygen therapy will have middle ear complications,
including severe pain, hemotympanum, and OME; these patients are candidates for
ear tube placement.

554
BY DR. MOHAMMED ATIAA KAREEM ALNASHY
-------------------------------------------------------------------
CHAPTER 201: GENETIC SENSORINEURAL HEARING LOSS
--------------------------------------------------------------------------------------------

1. With mitochondrial inheritance


A. Transmission from father to daughter is common.
B. The mtDNA is present in the nuclear envelope of the cell.
C. Only maternal transmission is possible.
D. Homoplasmy accounts for variability in expression.
E. Congenital hearing impairment is never seen.

2. In Branchio-oto-renal syndrome, otologic abnormalities include


A. Purely conductive hearing loss in most cases
B. Rarely preauricular pits
C. Enlarged vestibular aqueducts
D. Thyroglossal duct cysts
E. Michel aplasia

3. Waardenburg syndrome type I is characterized by all of the following except


A. Heterochromic irides
B. An association with Hirschsprung's disease
C. A white forelock
D. Hearing loss that is often profound, bilateral, and stable over time
E. Synophrys

4. In evaluating an individual for genetic hearing loss, which of the following


is not true:
A. An electrocardiogram (ECG) is necessary to rule out long-QT intervals in
Jervell and Lange-Nielsen syndrome.
B. Genetic counseling affords the opportunity to provide risk estimates and
appropriate interpretation to the family and patient.
C. Computed tomography of the temporal bones should be performed if GJB2-
related deafness is diagnosed.
D. Consultation with an ophthalmologist should be obtained in patients
suspected of having Usher syndrome.
E. Urinalysis can be useful to screen for hematuric syndromes associated with
hearing loss.

5. Nonsyndromic hearing impairment:


A. Accounts for 70% of genetic deafness
B. Can be due to mutations in GJB2 that result in loss of hair cell stability
C. Can be caused by aminoglycosides in persons with specific GJB2 mutations
D. Is never congenital
E. Is rarely a cause of late-onset deafness

555
BY DR. MOHAMMED ATIAA KAREEM ALNASHY
-------------------------------------------------------------------
CHAPTER 201
Genetic Sensorineural Hearing Loss

1. C
Mitochondrial mutations account for 1% to 2% of genetic deafness. Maternal transmission
is the hallmark of mitochondrial diseases, because sperm cells do not donate
mitochondria to the fertilized egg. Homoplasmy refers to the presence of all abnormal
mtDNA; heteroplasmy refers to the presence of a mix of normal and abnormal mtDNA.
Random transmission to progeny cells accounts for the variable expression that is often
seen in mitochondrial diseases.

2. C
The hearing impairment in bran-chiootorenal syndrome (BOR) is conductive in 30%,
sensorineural in 20%, and mixed in 50% of individuals. Outer ear abnormalities include
preauricular pits (82%), preauricular tags, microtia, and stenotic external auditory canals.
Enlarged vestibular aqueducts, cochlear dysplasia, and hypoplasia of the lateral
semicircular canal may be seen on GT of the temporal bone. Branchial cleft fistula and
renal anomalies are also part of this autosomal-dominant syndrome.

3. B
Waardenburg syndrome type I is characterized by sensorineural hearing loss, pigmentary
disturbances (white forelock, heterochromia irides), and dystopia canthorum. Synophrys;
broad nasal root and patent metopic suture are other findings that may be present. In type
II, dystopia canthorum is absent, although there is a greater likelihood of sensorineural
hearing loss. Type III is also known as Klein-Waardenburg syndrome and is similar to
type I, along with the presence of upper limb abnormalities. Type IV Waardenburg
syndrome includes Hirschsprung's disease and is also known as Waardenburg-Shah
syndrome. The hearing impairment is often profound, bilateral, and stable over time.

4. C
Long QT syndrome is found in Jervell and Lange-Nielsen syndrome. The long QT
interval may be visualized on EGG. Treatment, if instituted early, can significantly
reduce the risk of sudden death caused by cardiac arrhythmias. Genetic counseling allows
the opportunity to inform the family and patient about recurrence chances, data
interpretation, and treatment options. If a diagnosis of DFNB1 (GJ52-related deafness)
is made by genetic testing, no further investigations are necessary, because there are no
other comorbidities associated with this form of deafness. Usher syndrome includes
sensorineural hearing loss, variable vestibular dysfunction, and retinitis pigmentosa.
Consultation with ophthalmology is appropriate for the early detection and follow-up of
eye disease in patients suspected of having Usher syndrome. Alport syndrome may present
with a positive urinalysis that demonstrates the presence of red blood cells.

5A
Genetic hearing impairment accounts for 50% of childhood deafness and is nonsyndromic
in 70% of individuals. Connexins oligomerize to form a connexon that docks to a
neighboring connexon, thereby forming a gap junction. Aminoglycoside susceptibility is
caused by an mtDNA. Nonsyndromic can be either congenital or late onset.

556
BY DR. MOHAMMED ATIAA KAREEM ALNASHY
-------------------------------------------------------------------
CHAPTER TWO HUNDRED AND TWO: PEDIATRIC FACIAL FRACTURES
--------------------------------------------------------------------------------------------

1. Which is most important when operating on nasoethmoid fractures in children?


A. The use of absorbable plates and screws
B. Undercorrection of the fracture core
C. Overcorrection of the fracture core
D. Never to use preexisting lacerations for exposure
E. Setting the intercanthal distance wider than anticipated

2. The best imaging study for nasoethmoid complex fractures is


A. Axial computed tomography (CT) scan with 1.5- to 3.0-mm cuts
B. Panorex
C. Coronal CT scan with 1.5- to 3.0-mm cuts
D. Townes view
E. Both A and C

3. Absorbable plating systems in children are currently not indicated for


A. Frontal sinus fractures
B. Currently contraindicated in the pediatric population
C. Load-bearing fractures
D. Zygomatic fractures
E. Infraorbital rim fractures

4. Which of the following statements is true regarding orbital roof fractures


in children?
A. Orbital roof fractures occur primarily in younger children as a consequence of the
proportionally larger cranium and the lack of frontal sinus pneumatization.
B. Are rarely associated with concomitant intracranial injuries
C. Are unsafe to manage nonsurgically
D. If left unprepared have a high rate of orbital mucocele
E. None of the above

5. Which of the following statements regarding maxillomandibular fixation


in children is false?
A. Two to three weeks of mandibular immobilization in children younger than
12 is adequate.
B. Deciduous molars should be used for arch bars or capping when present.
C. In general after age 10, the development of permanent teeth provides for safe
anchors.
D. In children younger than 2 and between 5 and 9, immobilization requires
unconventional fixation techniques, because the dentition will not support
arch bars.
E. Should not be used in condylar fractures

557
BY DR. MOHAMMED ATIAA KAREEM ALNASHY
-------------------------------------------------------------------
CHAPTER 202
Pediatric Facial Fractures

1. C
Nasoethmoid fractures are relatively uncommon in the pediatric population.
The most important component of the repair is to over-correct the fracture
core. The use of absorbable plates is often difficult in this region, and
consequently plates and wires are often more practical. For cosmesis, it is
important to set the intercanthal distance narrower than anticipated.
Exposure is best obtained by preexisting lacerations or coronal incisions, with
mobilization of the globes along the orbital roof to expose the nasal dorsum.

2. E
Computed tomography scans have revolutionized the care of NOE fractures.
With both the coronal and axial CT cuts, it is possible to develop a three-
dimensional understanding of the fracture. This allows the surgeon to decide
whether surgical intervention is warranted and, if indicated, a surgical plan
of repair. Townes view does not give enough information on the fractured
area. The Panorex is helpful to search for a concomitant mandible fracture
but does not directly give suitable information on the NOE complex.

3. C
The available resorbable plating systems (1.5- and 2.0-mm screw diameters)
provide flexural and tensile strength comparable to the microplate titanium
systems (1.0 to 1.3 mm diameter screws). Definitive long-term studies in facial
trauma using resorbable plates have yet to answer whether any patient
experienced problems with growth restriction. At this time, absorbable plates
are not recommended for all types of pediatric facial fractures. The use of
absorbable plates in the mandible and aload-bearing" bone is still
investigational in children, and long-term results are limited. At this time,
the indications for the use of absorbable systems in pediatric trauma are on
non-load-bearing regions in the upper and middle third of the craniofacial
skeleton.

558
BY DR. MOHAMMED ATIAA KAREEM ALNASHY
-------------------------------------------------------------------
4. A
The pattern of orbital fractures changes from roof fractures to the
lower orbit around age 7. Up to 86% of orbital roof fractures are associated
with intracranial injury. The orbit and globe rarely sustain long-term
damage, and thus surgery is rarely necessary. Orbital encephaloceles have
been reported as a late, but uncommon, sequela.

5. E
In prepubescent children, the frequent absence of teeth and the poor retentive
shape of the deciduous teeth make the use of arch bars and interdental wiring
for maxillomandibular fixation (MMF) unfeasible to apply. Fortunately, 2
to 3 weeks of mandibular immobilization in children younger than 12 is
adequate. To obtain MMF, one must consider the age and development of the
teeth. In children younger than 2 and between 5 and 9, immobilization
requires unconventional fixation techniques, because the dentition will not
support arch bars. One approach is the use of an overlay acrylic mandibular
splint held in place by circummandibular and transnasal wires. Another
approach pioneered and described by Eppley takes advantage of resorbable
screws. Between age 2 and 5, the deciduous incisors have firm roots, and if
the deciduous molars have formed, then they can be used for cap splints or
arch bars. In general, after age 10, the development of permanent teeth
provides for safe anchors. However, children develop at different rates, and
the strength of the teeth should be carefully examined before the placement
of any type of MMF. Condylar fractures presenting with an open bite,
mandibular retrusion, or movement limitation are best treated with 2 to 3
weeks of immobilization.

‫الحمد هلل رب العالمين‬

559
BY DR. MOHAMMED ATIAA KAREEM ALNASHY
-------------------------------------------------------------------

560

Вам также может понравиться